DHA Exam Material For Pharmacists.

You might also like

Download as pdf or txt
Download as pdf or txt
You are on page 1of 495

TOTAL

Questions
2000 +

Features
Features 91+8077381582
8077381582 Chapter- wise
Chapter- wise
Chapter- wise
Students-friendly
• MCQ Type questions & Theory ebook Students-friendly
Students-friendly
• Based on the latest Exam Questions Synopsis for
Synopsis
Synopsis for
for quick-
• Chapter wise quick-and-easy
• Designed by DHA exam Experts and-easy
quick-and-easy
• Importants For DHA Exam revision
Preparation revision
revision
• Practice sets with Answer Sheet &
Very Important MCQ Sets.

Boost Your DHA Score


Www.arifpharma.com
MCQ e-Book
Scoring in DHA EXAM MADE EASY

DHA आपकी
Chapter-wise student friendly
synopsis for quick-and-easy
revision

Boost Your DHA Score


Essential for DHA Examination

by

Arif Pharma CLINIC


Content

DHA EXAM SYLLABUS FOR PHARMACY (ALL SUBJECT)

1. Pharmaceutical Care & Disease State Management..................... 1- 56


2. Pharmacy Calculation & Compounding......................................57 - 115
3. Pharmacokinetics.......................................................................116 .- 192
4. Pharmacy law And Ethics................................................... 193..- 222
5. Toxicology................................................................................. 223.- 257
6. Medication Safety....................................................................… 258 .. - 280
7. Pregnancy & Lactation............................................................... 281 .. – 305
8. Over The Counter Medication (OTC) ............................................ 306..– 321
9. .. – 334
Geriatrics..................................................................................... 322
10. Pediatrics............................................................................. 335 – 347

DRUG & ANTIDOTES


..
1. Important Drugs & Antidotes.....................................................348.. - 348
..
IMPORTANT DRUGS NOTES
1. Mechanism Of Drug Action ........................................................ 349 – 354
2. .. - 357
Intrinsic Sympathomimetic Activity Drugs & Abbreviation Sing..355

IMPORTANT MCQ & ONE LINER QUESTIONS


1. Important MCQ Questions Set – (1) ........................................... 358 – 412
2. Important One Liner Questions Set (2) .......................................... 413 .. – 423
3. Important One Liner Questions Set (3) ......................................... 424 ...– 428
...
PRACTICE SETS WITH ANSWER SHEET ..
...
1. DHA Exam Practice Set (1) ............................................................. 429 - 451
2. DHA Exam Practice Set (2) .............................................................. 452 – 491
INTERVIEW TEST QUESTIONS ..
..
1. DHA Basic Interview Questions ....................................................... 492 . - 492
.
Page 1

Total 261
Questions
in 2 Part

1- A 82-year-old woman is taking ferrous sulfate to treat iron deficiency anemia. Changes in
which hone of the following pharmacokinetic properties associated with aging can most
affect this agent?

A. ABSORPTION.
B. Distribution.
C. Metabolism.
D. Renal elimination

2- All the following patients are seeing their pediatrician today and are due for immunizations on
the basis of the routine schedule. For which one of the following patients would it be best to
recommend deferring immunizations until later?

A. 12-MONTH-OLD BOY WHO RECENTLY COMPLETED A CYCLE OF CHEMOTHERAPY FOR ACUTE


LYMPHOCYTIC LEUKEMIA.
B. 6-month-old girl receiving amoxicillin for otitis media.

C . A 12-month-old HIV-positive boy whose most recent CD4 count was greater than1000.

D. 12-year-old girl completing a prednisone “burst” (1 mg/kg/day for 5 days) for

asthma exacerbation

3- 18 month-old baby with a history of premature birth and chronic lung disease is admitted to the
pediatric intensive care unit with respiratory distress requiring intubation; fever; and a 3-day
historyof cold-like symptoms. A nasal swab is positive for RSV. Which one of the following is the
best intervention?

A. Palivizumab.
B. Corticosteroids
C. Cefuroxime.

D. INTRAVENOUS FLUIDS AND SUPPORTIVE CARE

4- A young girl has seizures and tremors, as the physician prescribed. Oxycarbamazepin after 2 weeks
of administration, redness, and pruritic rash.
Page 2

A- shift to carbamazepine
B- Shift to ethosuximide
C- Shift to clozapine
D- Keep using oxy

N.B: Ethsuoxmide is the drug of choice for petit mal epilepsy which is usually associated with children

6. A woman is taking oxcarbazepine, and after 2 weeks of administration, redness, and


flushingappear. What is the best choice for her?
A- Shift to phenytoin
B- Shift to carbamazepine
C- Take clozapine

D/ KEEP USING OXYCARBAMAZIPINE

N.B: It is a normal side effect but in other question (young girl)

The answer is to shift to ethosuximide

7. FDA Fast Track what this means:

A- For a drug that shows promising results for life-threatening disease with other drugs available can
treat it

B- A DRUG THAT SHOWS PROMISING RESULTS FOR LIFE-THREATENING DISEASE WITH NO


OTHERS AVAILABLE CAN DO THAT
c-A drug that shows cost-effectiveness

d-A drug with side effects

8. A 9-year-old boy has a new diagnosis of ADHD. At school, he is disruptive, talks when the
teacher is talking, and runs around the classroom. His parents report extreme difficulty in getting
him to do his homework after school. Which one of the following is best for his initial drug
therapy?

a. METHYLPHENIDATE EXTENDED RELEASE GIVEN ONCE DAILY.


b. Methylphenidate immediate release has been given 2 times/day with doses
administered 4 hours apart.
c. Guanfacine given at bedtime.

d- Methylphenidate is given 2 times/day with doses administered 4 hours apart

N.B

ARIF PHARMA CLINIC Copyright@arifpharma.com Www.arifpharma.com


Page 3

ADHD = Attention Deficit Hyperactivity Disorder

9. F.A. is a 55-year-old woman with rheumatoid arthritis. On diagnosis 1 year ago, F.A. had an RF
titer of 1:64 signs and symptoms of inflammation in the joints of both hands, and about 45
minutes of morning stiffness. She began therapy with methotrexate and is receiving 15 mg every
week, folicacid 2 mg/day, ibuprofen 800 mg 3 times/day, and omeprazole 20 mg/day. At today’s
clinic visit,
F.A. reports a recurrence of her symptoms. A radiographic evaluation of her hand joints shows the
progression of joint space narrowing and bone erosion. Which one of the following is the best next
step in therapy for F.A.?

A. ADMINISTER ETANERCEPT. (TRADE NAME ENBREL)

B. Switch to hydroxychloroquine.
C. Add prednisone bridge therapy.
D. Change to leflunomide.

N.B:
OSTEOPOROSIS …. ALENDRONATE NA
- Osteoporosis + Methotrexate …. Leflunomide
OSTEOARTHRITIS…ETANERCEPT
-Osteoarthritis + Methotrexate …. Etanercept is the first choice,

if not exist choose leflunomide

10. A 75-year-old woman reports urinary urgency, frequency, and loss of urine when she
cannotmake it to the bathroom in time. She also wears a pad night that she changes 2 or 3
times because of incontinence. Her medical history is significant for Alzheimer’s disease
(MMSE 23),osteoarthritis, and hypothyroidism. A urinalysis is negative, her examination is
normal, and postvoid residual (PVR) is normal (less than 100 mL).

Which one of the following interventions would be best at this time?

A. Bethanechol.
B. Pelvic floor exercises plus estrogen vaginal cream.

C. DARIFENACIN.
D. Oxybutynin.

ARIF PHARMA CLINIC Copyright@arifpharma.com Www.arifpharma.com


Page 4

11. A hypertensive and diabetic woman, after continuing medication of pioglitazone, it's blood
glucose level returns to normal and in lab reading, the protein appears, she will take which
drugto treat hypertension:

A. LISINOPRIL
B. Amiloride

N.B: Due to the presence of proteinuria we will choose ACEI drugs such as lisinopril as ACE Is reduce
proteinuria

12. A pregnant woman in her third week takes levothyroxine 100 mcg, you advise her:
a) Stop the medication immediately

B) INCREASE THE DOSE OF LEVOTHYROXINE


c) Ask the prescriber to change the medication
d) Continue using the medication as prescribed
N.B:

LEVOTHYROXINE IS CONSIDERED THE TREATMENT OF CHOICE FOR THE CONTROL OF


HYPOTHYROIDISM DURING PREGNANCY. DUE TO ALTERATIONS OF ENDOGENOUS MATERNAL
THYROID HORMONES, THE LEVOTHYROXINE DOSE MAY NEED TO BE INCREASED DURING
PREGNANCY AND THEDOSE USUALLY NEEDS TO BE DECREASED AFTER DELIVERY.

13- A 2-year-old child came to the clinic for taking the hepatitis A vaccine, we know that he took
the pneumonia vaccine a month ago, so we should:

A) GIVE HIM THE VACCINE IMMEDIATELY


b) Wait for 3 months
c) Wait for 6 months
d) Wait for 1 year

14- A patient comes to you taking alendronate, and you advise him:

a) Take alendronate 1/2 hour before breakfast and stand upright for 1/2 hour
b) Take alendronate 1/2 hour before breakfast and rest
c) Take alendronate 1/2 hour before breakfast with water and stand upright for 1/2 hour
Page 5

15- A nurse was giving medication to a hepatitis B patient when she got infected with his blood,
when she made an analysis the result was HBSAG negative and HBSAB negative, so she should be
treated with:

a) Give her the hepatitis B vaccine immediately


b) She needs no treatment
c) Give her immunoglobulin only

D) GIVE HER IMMUNOGLOBULIN + HEPATITIS B VACCINE

16- A pregnant woman has hepatitis B when she delivers we must give her

baby:A- Hepatitis B vaccine only

B-Hepatitis B vaccine with interferon

C- HEPATITIS B VACCINE WITH IMMUNOGLOBULIN HEPATITIS.


D-Hepatitis B immunoglobulin with lamivudine.

17-A Case about a Patient with Erectile dysfunction (ED) caused by long-lasting Diabetes… souse:

A-SILDENAFIL
B-Testosterone patch

C-yohimbine

18-A Kidney failure patient with hyperkalemia which of the following should be used

A-insulin

B-thiazide diuretic

C-CALCIUM GLUCONATE (OR CA CARBONATE)


D-spironolactone

N.B:

Due to the drug of


choice in case of
Page 6

renal failure is loop


diuretic, not
thiazides

- Calcium is used as prophylaxis for arrhythmia due to hyperkalemia


"Calcium gluconate or Calcium carbonate" both are right

19- A 2-year-old child has taken the Hepatitis A vaccine and came to take MMR

vaccine:

A) SHOULD TAKE MMR VACCINE IMMEDIATELY


b) MMR vaccine should be delayed 3 months
c) MMR vaccine should be delayed 6 months
d) MMR vaccine should be delayed 12 months

20- A non-smoker patient suffers from wheezing and chest tightness, what medication he should use?

A- B2 agonist

B-CORTICOSTEROIDS
C-omalizumab

N.B:1- Acute asthma …. B2 agonist "e.g.: Salbutamol"

(Short-acting selective β2 agonist: as Salbutamol, terbutaline, albuterol,

pirbuterol)

2 CHORIONIC ASTHMA …. CORTICOSTEROIDS (ANTI-INFLAMMATORY)

(Beclomethasone, prednisone, hydrocortisone)

Inhibit Ag-Ab reaction, inhibit the release of inflammatory mediator

3 Moderate to severe asthma …. Omalizumab

21 - A woman suffers from an acute asthma attack, what is the best choice for a heart attack

A) SALBUTAMOL
b) salmeterol
c) corticosteroids
Page 7

N.B:

Bronchodilators are:
I. Mixed agonist ((α1 "blood vessels constriction", β1 "stimulation of cardiac muscle" &
β2"dilatation of lung smooth muscles"): e.g. adrenaline "epinephrine"
II. B2 Agonist:
1 Non selective β agonist (β1, β2) as: isoprenaline
2 Short-acting selective β2 agonists: as Salbutamol, terbutaline, albuterol, pirbuterol
3 Long-acting selective β2 agonist: as Salmeterol, for moterolI
4 II- cholinergic antagonists: e.g. ipratropium, tiotropium
IV- Xanthenes: e.g.theophylline.

22. Pregnant woman in her 43th week began her labor, the contractions were going well for 12
hours but in the last hour, they decreased. So what medication do you give her?

a-Ritodrine

b-ergonovine

c-oxytocin

d-saline infusion

N.B:

- Oxytocin is a uterine stimulant used to induce labor in women with problems


- Ritodrine is used to stop premature labor
ergonovine used for the prevention and treatment of postpartum and post-abortion
hemorrhage

23. A 2-year-old girl has otitis media, which medication is the best for her?

A- A high dose of ampicillin


b-azithromycin

c- erythromycin
d-co-triazole
Page 8

N.B:
A high dose of AMOXICILLIN is the antibiotic of choice in this case, not ampicillin

if amoxicillin is not an option … choose azithromycin

24- A Man came to ER with vomiting and dizziness after he ingested a toxic dose of a certain drug,
what is the most important step?

A-decrease the amount of drug absorbed

B-Increase elimination of drug

C-WATCH THE VITAL SIGNS AND MAKE THEM NORMAL


D-Compensate the neurological symptoms of the toxicity

25 -A Woman came to the ER with a pointed pupil, vomiting, and dizziness...Which drug is responsible
for that?

A-HEROIN
B-Cocaine

N.B:
- Heroin because it is an opioid agonist and causes miosis (pinpoint pupil)).
- As morphine
- Cocaine is indirectly sympathomimetic and causes mydriasis
26- Which of the following vaccine should be taken although you don’t need it:

A- INFLUENZA VACCINE
b- Mmr vaccine

c-Dap vaccine

27 The summary of a long case is that a patient on clopidogrel (Plavix) treatment makes an
accident. He is admitted to ICU and has a catheter. He was taking omeprazole, what is the best
intervention for him to reduce gastric secretion:

a. Reduce omeprazole dose


b. Stop omeprazole
c. Do not change omeprazole

d. MOVE TO PANTOPRAZOLE IV

ARIF PHARMA CLINIC Copyright@arifpharma.com Www.arifpharma.com


Page 9

28- A patient came with multiple fractures of his bones and ribs from an accident. He has brain
trauma and he is on NGT (Nasogastric Tube) in ICU. Which of the following is best used for the
prophylaxis of stress-induced ulcers that can be happened to him:

a-Sucralfate

b- Misoprostol

C- IV PANTOPRAZOLE
Does not need this treatment

29- A patient suffers from poly urination and dizziness so he does lab tests and his results are
as follows:
- glycosuria +ve
-capillary blood glucose = 15 normal up to 6

-lbw = 28

30- What is the best medication for this patient?

a. glargine
b. Glipizide
c. Metformin
d. Diet & exercise

N.B:1- Poly urination: The production of an abnormally large amount of urine: one symptom of
diabetes

- glucosuria is the excretion of glucose into the urine

31-A pregnant woman in her 9th month, she has hyperproteinemia and

hypertension... What is the recommended medication for her?

Magnesium sulfate "Mgso4"

N.B:

This case is called "Eclampsia" characterized by HTN + hyperproteinemia

- Eclampsia is the development of seizures in a woman which is life-threatening


- Mgso4 used to treat these seizures, not for HTN

- to treat HTN in this case we use by order


1. methyl dopa
2. labetalol

ARIF PHARMA CLINIC Copyright@arifpharma.com Www.arifpharma.com


Page 10

3. hydralazine (for emergency or urgency HTN)

32- Diabetic patient uses insulin daily, but he forgot to take his insulin dose someday... he did
lab tests and his results were normal except, for high glucose, potassium (hyperkalemia)
what do you recommend for this case?

A-RESTART HIS DAILY INSULIN DOSE


b-use 0.9% NaCl "normal saline"

c-use 3% - 5% NaCl "hypertonic solution"d-use


0.33% NaCl "hypotonic solution"

N.B:

- hyperkalemia most commonly occurs in uncontrolled hyperglycemia (diabetic ketoacidosis)

due to lake of insulin

- The acidosis and high glucose levels in the blood work together to cause fluid and potassium to
moveout of the cells into the blood circulation

33- The summary of a long case that a pregnant woman in the hospital with deep vein
thrombosis"DVT" takes...?

A. HEPARIN (ENOXAPARIN OR CLEXANE)


b. warfarin
N.B: - Warfarin is category X

- Enoxaparin ((Clexane)) is a low molecular weight heparin

34- A post-menopause old woman is suffering from facial flushing and vaginaldrying. She has
done a hysterectomy procedure... Which drug of the following should she use?

A-ESTROGEN
B- PROGESTERONE
N.B:1- Women who have both the uterus and ovaries removed usually just get

estrogen replacement therapy (ERT) alone.

ARIF PHARMA CLINIC Copyright@arifpharma.com Www.arifpharma.com


Page 11

2-But women who have only the ovaries removed need both estrogen and progestin. That's because
estrogen alone can increase the risk of cancer in the uterus. Adding progestin removes this risk.

35 -A 39 years old what are oral birth control pills appropriate for her

A. Levonorgestrel

B. Ethinyl estradiol / mestranol (combined oral contraceptive)

C- lynestrenol / norethisterone (a progestogen hormone)

D-medroxyprogesterone (progestogen depot contraceptive)

N.B: This ques. may be incomplete … if there is any of the following

contraindications of COCP the answer will be Levonorgestrel.

Take the minipills of progestin if

• Older than age 35 + smoke


• Older than age 35 + migraine headache
• Older than age 35 + obese
• Older than age 50
• Breast-feeding
• Diabetes mellitus with vascular disease
• Risk of DVT or history of thromboembolism "blood clots"
• History of uncontrolled HTN or heart problems
• Breast or endometrial cancer

36- A woman had DVT and was treated a year ago... She wants to use oral contraceptive pills. Which is
more suitable for her?

A- LEVONORGESTREL
b-Ethinyl estradiol/mestranol

c-lynestrenol/ norethisterone (Lynestrenol norethisterone) area progestogen


hormone

d-medroxyprogesterone (parental depot contraceptive Synthetic Progestin).

ARIF PHARMA CLINIC Copyright@arifpharma.com Www.arifpharma.com


Page 12

37- A pregnant woman taking valproic acid went to the physician with tonic-colonic seizures...
Whichof the following is true?

a. stop valporic and use another drug


b. use valporic with iron supplementation
C- USE VALPORIC WITH FOLIC ACID
d- use valporic with multivitamins

38- A Pregnant woman with G6PD deficiency has. G-ve. M.o and UTI... which is the drug of choice
totreat her UTI?

a- Vancomycine

b-Nitrofurantoin

c- Cefuroxime
d- Sulphamethoxaole

N.B:

1Sulfa compounds will cause hemolysis in this patient… Vancomycin is the last choice &
Nitrofurantoin is used in pregnancy with caution "not used in G6PD" so the answer is Cefuroxime
2 N.B: G -ve M.O = Gram-negative microorganism

39-A Pregnant woman has. G-ve. M.o. and UTI, which. Is the drug of choice to treat her UTI?
A-ciprofloxacin

B-tetracycline

C-Sulphamethoxizole

D-NITROFURANTOIN

40- A Hyperglycemic patient his blood glucose level given by moles and his

HBA1C was high ((more than 10%) …What is the best medication for him?

A-Metformin

B-INSULIN 70/30
C- Glargine
Page 13

N.B: (HBA1C or HGBA1C): is a form of hemoglobin that is measured primarily to identify the average
plasma glucose concentration over prolonged periods of time… type 2 diabetes start with metformin

If HBA1C is getting higher even if 1% we increase metformin If HbA1c increased


by more than 10% we give insulin Hba1c is the same as a1c

normal HbA1c less than 6.5 – 7

41- A Diabetic patient takes metformin twice a day, he did a blood glucose analysis and there were
4results all were normal except one result was high and HbA1c was. What he should do?

A- Increase the dose of metformin


B- Increase the dose of metformin
C- don’t anything

42- A patient takes 4 drugs... He did a kidney function test and the result was high serum cr and
high BUN (blood urea nitrogen) so which drug should be stopped

A-METFORMIN
B- Tigcyciln

C- Insulin

D-metoprolol

N.B: METFORMIN IS CONTRAINDICATED WITH RENAL FAILURE

43- A 22 years woman wants to become pregnant, she is taking metformin and pioglitazone. She
has a history of hypoglycemia and she prefers oral therapy...What will be the best approach in her
case?

a - stop pioglitazone and titrate metformin

b - stop both of them and start insulin therapy

c- stop metformin and titrate pioglitazone D.do nothing

ARIF PHARMA CLINIC Copyright@arifpharma.com Www.arifpharma.com


Page 14

44- A elderly man around 60 years old complains of polyuria, dry mouth…There is no family
history of diabetes… He has done lab tests and the results were positive for
diabetes…Initialtreatment should be:

a- Insulin

b- Metformine

c- Tigecycline

N B: The answer is Metformin … as being overweight is an indication of type 2 diabetes

45- A diabetic woman is taking 850 mg of metformin… her results are 7.5... 5.5…6..5 and her
HbA1c is 7.5 … what's your advice?

A-stop metformin and use another medication

B-INCREASE METFORMIN DOSE


C-do nothing

D-ask her doctor

46– A Diabetic patient with Glycated Hemoglobin Hb 9 What is recommended to give him as a
treatment?

A- METFORMIN
B- Glipizide

C- C-Insulin

47-A 54 years woman with polyuria for 3 months before analysis.


A-Give Metformin 3 times a daily

B-Glyburide 10mg 2 times daily

C-Glargine

48. A diabetic patient takes metformin with glipizide and pioglitazone was added. Which test
should be done regularly?

A-Bun and Serum Creatinine

ARIF PHARMA CLINIC Copyright@arifpharma.com Www.arifpharma.com


Page 15

B. LIVER FUNCTION.

49-A 48 woman suffers from fatigue, weakness, and poly urination, and lab tests proved that she is
diabetic… A Doctor prescribed her 850 mg metformin twice a day and so her blood sugar become
normal... but after some time, she did another lab test and the result was that her
hemoglubinated sugar increased by 1% more than the last result what will you advise her?

A- INCREASE THE METFORMIN DOSE


b. decrease the metformin dose
c. do nothing
d. shift to another type of insulin

50-A 12 years child has diabetes type1 which drug can take?

A-METFORMIN
B-Glipzide

C-Pioglitazone

D- Glibenclamide

N.B:

Insulin is the drug of choice for Type 1 diabetes If insulin is not an option choose metformin

51. An Obese patient what is the effect of obesity on the absorption of lipid-soluble drugs?

A- Decrease

B- B-increase

C- NO EFFECT INCREASE THEN DECREASE

N.B: no effect on absorption but increase the volume of distribution

52- A woman with septic shock… Came to emergency who take ABC

HR = 122
BP = 90/70 mmHg Serum
creatinine = 6

ARIF PHARMA CLINIC Copyright@arifpharma.com Www.arifpharma.com


Page 16

Na & Cl... Very high than normal range and high


pulse

what should she take?

A- ALBUMIN 5% BOLUS
B- Nacl saline

C- Furosemide "Lasix"

D- D-dopamine

N.B: ABC = Airway, Breathing & Circulation

53-A Patient has septic shock and his BP is 70/40 mmHg, with a slight increase in K level and Na
within normal range… which drug should be recommended for him?

a. dopamine
b. Lasix
C-SALINE
d- albumin

N.B:

- For severe septic shock, we recommend intravenous fluids first… then


1. Norepinephrine
2. dopamine (is the DOC in case of septic shock + kidney injury)

3.Epinephrine (adrenaline)
respectively

- CARDIAC SHOCK TREATED BY DOPAMINE


- Anaphylactic shock treated by Epinephrine (adrenaline)

54- Losartan is better than captopril because…

A- Not teratogenic

B-MORE EFFECT ON ANGIOTENSIN 2 RECEPTORS

ARIF PHARMA CLINIC Copyright@arifpharma.com Www.arifpharma.com


Page 17

N.B: less side effects (don’t cause dry cough as captopril) is a better answer if found

55- Appropriate counseling and follow-up for PQ with the initiation of levothyroxine includes all
of the following, EXCEPT:

A -Separate the levothyroxine dose from calcium tablets for several hours.

B- TAKE LEVOTHYROXINE ON A FULL STOMACH FOR GREATER ABSORPTION.


C- Improved control of thyroid levels may also improve her glycemic control.

D- A Physician follow-up is needed every 6-8 weeks to have her thyroid function tests

repeated.

E-closer monitoring of her angina should be done during dosage titration of levothyroxine.

N.B: Levothyroxine is taken on an empty stomach approximately half an hour to an hour before meals

56- A First-Pass Effect

A-BLOOD PERFUSING VIRTUALLY ALL THE GASTROINTESTINAL TISSUES PASSES THROUGH THE
LIVER BY MEANS OF THE HEPATIC PORTAL VEIN.
B- Fifty percent of the rectal blood supply bypasses the liver (middle and inferior hemorrhoidal veins).

C- Drugs absorbed in the buccal cavity bypass the liver.

D- Drugs affected most by the first-pass effect are those with a high hepatic extraction ratio.

57- PQ is a 75-year-old patient who has just been diagnosed with hypothyroidism. Her past
medical history is significant for congestive heart failure, type 2 diabetes mellitus, osteoporosis,
and chronic stable angina, all of which are well-controlled. Her medications include:

Metoprolol 25 mg bid

Calcium carbonate 1250 mg bid Vitamin


D 1000 IU daily Glyburide 2.5 mg bid

Enalapril 10 mg bid Furosemide 40 mg


daily Nitroglycerin SL spray prn

PQ should be started on a low dose of levothyroxine because of her:

A. AGE.

B. Gender.

C. Diabetes.

ARIF PHARMA CLINIC Copyright@arifpharma.com Www.arifpharma.com


Page 18

D.Metoprolol use.
E. Nitroglycerin use

58- Which of the following parameters is the most appropriate for PQ's self-evaluation of the
effectiveness of levothyroxine therapy?

A- INCREASED ENERGY
b. Weight loss
c. Improved vision
d. less frequent angina
e. less frequent urination

59- Metabolism in the intestine

a-Hydrolysis

b-Reduction

60- African American women who take lisinopril and another hypertensive drug suffer from
noseswelling and other symptoms that make her more suspected of this reaction

a- age

b - Gender
C- drug combination

d- ethnicity

N.B: - if no ethnicity in choices… Choose Drug combination

- Black patients have low renin than normal


- ACEI (lisinopril) causes angioedema more in African Americans

ARIF PHARMA CLINIC Copyright@arifpharma.com Www.arifpharma.com


Page 19

61- A 35-year-old patient with megaloblastic anemia which of the following is appropriate
for treatment?

A-Ferrous gluconate

B-Vit. C

C-folic acid

D-FOLIC ACID & VIT. B12


N.B:

Iron deficiency anemia … lack of iron Megaloblastic anemia …


lack of vit b12 &folic acid Pernicious anemia … lack of vit b12
Anemia due to CRF … lack of RBCs and treated by Epoetin In aplastic anemia.
The body's bone marrow doesn't make enough new blood cells Hemolytic
anemia redblood cells are destroyed and removed from the bloodstream before their normal lifespan is up

62. A pregnant woman is sensitive to amoxicillin which is the Drug of Choice for her disease what will
be the suitable alternative anti-biotic for her?

A- ERYTHROMYCIN
B- Sulfacetamide

C- Cefixime

N.B: antibiotics that are allowed during pregnancy:

1-Penicillins family 2-Macrolides 3- Cephalosporin

63- The summary of a case of a woman who suffers from pain in menses and during intercourse
which is finally found that she had something like a tumor or a solid mass …What is the best
medication to fast relieve her pain?

A-ORAL CONTRACEPTIVES
c. Surgical

d. Danazol

e. Estrogen

N.B:

- hormone therapy “e.g ................ Oral contraceptive pills" are used to treat

ARIF PHARMA CLINIC Copyright@arifpharma.com Www.arifpharma.com


Page 20

endometriosis-associated pain and they are effective.

-Danazol can be used but it isn't the first choice because it can cause serious side effects and can be
harmful to the baby if the patient becomes pregnant while taking this medication.
- Progestin has a more favorable side effect profile than danazol.
- Surgery is the last resort and is recommended if the patient planning for pregnancy

64- The Summary of a case the patient has taken an unknown amount of paracetamol
(acetaminophen) since 8 hours ago… and you have shown some lab results of his
tests
what is the suitable choice for this case?

A-Charcoal

B-N-Acetyl cysteine

N.B:

- Activated charcoal can be used within 4 hours of ingestion


- N acetylcysteine is the antidote to acetaminophen

6 65. Anemic patient refuses to take injections so DOC +is:

A-Iron sucrose

B-FERROUS GLUCONATE
C Vit. b12

N.B:

-Ferrous gluconate is taken orally

-Iron sucrose… Injection

66. The summary of a case that shows lab results which were all normal or about to be except LDL was
6 very high… the answer is

INCREASE THE RISK OF ATHEROSCLEROSIS


Page 21

N.B:

LDL: Low-density lipoprotein

it’s the bad cholesterol that collects in the walls of blood vessels, causing the blockages of
atherosclerosis

6 67. A patient has low Cr. clearance which drug contraindicated in this case is?

A-SPIRONOLACTONE
B- Furusomide
C-Propanol d-Insulin

N.B:

- kidney damage= low Creatinine clearance (Crcl) = high serum creatinine


- aminoglycoside may exist instead of spironolactone
i.e.: aminoglycosides and any potassium-sparing diuretics (spironolactone, triamterene, and
amiloride) are C. I with any kidney diseases

68- A pregnant woman in 35 weeks and before 7 days from her labor she suffers from
severeheadaches... What is the DOC for her?

A-Ergotamine

B-ACETAMINOPHEN

N.B:

- Acetaminophen is the drug of choice


- Ergotamine is contraindicated in pregnancy category X

69- A 12 years old girl suffers from rashes after treating with Oxcarbazepine

A-SHIFT TO ETHOXSUMIDE
B-shift to topiramate
Page 22

70-A 14 years-obese girl comes to the clinic with a severe rash. She was initiated on oxcarbazepine
about 3 weeks ago for the management of partial seizures. Her medical history is significant only
forseizures. She has recently become sexually active and admits to inconsistent contraceptive use.
Which one of the following interventions is best for her?

A. Change to carbamazepine.
B. CHANGE TO TOPIRAMATE.
C. Change to valproic acid.
D. No change in therapy is necessary

71- A Child is on Oxacarbamazepine for epilepsy and suffers from a rash:

Change to ethosuximide

72- Which of the following prevents neural tube defect (NTDs) in the fetus

A- Vit D

B- Thiamin

C- Vit C

D- FOLIC ACID

73- A summary of a case that a diabetic girl whose lab results were mostly normal except for Glucose
27,Potassium is higher than normal by 1what do you recommend for her?

A- Calcium gluconate

B- Change to iv resin every 15 min.

C- INSULIN INFUSION 10 UNIT


D- bicarbonate

74- Analysis For a ma65year Renal Failure

ARIF PHARMA CLINIC Copyright@arifpharma.com Www.arifpharma.com


Page 23

Result.

Sodium. 110 Normal range ((135 – 145 mEq/L))

Potassium 9normal range ((3.5 to 5.5 mEq/L))

Urea. 54 Normal range ((35 – 40))

Serum creatinine. 10. Up to 3.5which of


the following?

A. 0.22% Nacl saline 500ml/hour

B. 0.9% NACL SALINE 500ML/ HOUR


C. Magnesium sulfate

75- A summary of a case that a pregnant woman who suffers from a disease related to genital
infections does not have a job and she needs cheap and fast medication… What is best for
her?

A. ACYCLOVIR 400 BID FOR 7 DAYS


B- Valacyclovir

76- A Young female takes lisinopril and will be conceiving-Still on

A- lisinopril

B-Take lisinopril w propranolol

C-Change to losartan

D-STOP LISINOPRIL W TAKE METHYLDOPA

N.B:

Only Methyldopa is used in ttt of hypertension in pregnancy

77- A girl with DM type 1 taking insulin… she didn't take insulin yesterday as she didn’t eat… She
becomes fatigued and has dizziness and nausea her BP is 80/50 with NO lactic acidosis … What
should be done for her?

A-take insulin

B- HALF A LITER 0.9% NACL EVERY 12 H.


Page 24

C-take sodium bicarbonate (for lactic acidosis)

78- A patient takes multivitamins and his lab results show high BUN and se.cr. What is the cause of
these results?

A-RENAL INSUFFICIENCY
B-his medication

C-Age

79- A woman found her child drinking an iron syrup bottle...she took him to the hospital and did some
rays. His body temperature was normal…What should he take?

DESFERRIOXAMINE

80-A pregnant woman with pyelonephritis went to the hospital given ceftriaxone IV what is an
antibiotic to go with

A- Ciprofloxacin
B- Doxycycline
C- Amoxicillin with clavulanic acid
D- Trimethoprim-sulfamethoxazole

N.B:

if "Ampicillin + sulbactam" is an option choose it

81-A summary of a case that an elderly patient’s lab results were all normal except for thigh K

HIGH SERUM CREATININE


high BUN.

what is the cause responsible for these results?

a. diabetes
b. hypertension
c. effect of taking more medication
d. renal insufficient

82- Which the following hormone suppress due to take oral contraception:

ARIF PHARMA CLINIC Copyright@arifpharma.com Www.arifpharma.com


Page 25

A) FSH
B) LH
C) GNRH

83- A menopausal woman takes Estrogen and progesterone derivative replacement


therapy… Which effect may be done due to this therapy?

A-decrease myocardial infarction risk

B-INCREASE THROMBOSIS RISK


C-increase fracture risk

84- A summary of a case that a patient suffers from headache, nausea, vomiting, and blurred
vision…He went to a hospital with alcohol toxicity ((methanol toxicity)) and did kidney and liver
analysis… His results and the normal range of the tests have shown to you… All results about to be
normal

the question is …What is your recommendation for methanol toxicity?

A- Vomiting and nausea

B- Vertigo and vomiting

C- Blurred vision

D- LAB RESULTS
N.B:
-If the ques. ask for recommendations or what should be done the answer is …
LAB RESULTS
- if the ques. asks for what you observe in this patient the answer is … Blurred
VISION

85-A diabetic woman takes metformin and glibenclamide… she went to the doctor and he increased
the dose of glibenclamide… Which analysis should be done regularly?

A-BUN & serum creatinine

B-LIVER FUNCTION
C-Potassium and phosphate in the blood

86-65 years old man BMI (of 28.1) and type 1 DM on the treatment of Glyburide, metformin, aspirin,
and hydrochlorothiazide come to the hospital for a routine check
Page 26

BP: 117/110

HR: normal
HbA1c: 7

and urine analysis for protein: + ve Proteinuria (normal negative) for


medical intervention in this case:

A: ADD LISINOPRIL
B: add Ca blocker

C: same medication no change

D: substitute glyburide and metformin with insulin

87- A Child is born to a Hepatitis B-positive mother:

A- first dose of the hepatitis B vaccine

B- One dose of the Hepatitis B Immune Globulin (HBIG).

C- Both of them

N.B: If a pregnant woman tests positive for hepatitis B, her Newborn child must be given two shots
in the delivery room:

1 The first dose of the hepatitis B vaccine and one dose of hepatitis B immune globulin
(HBIG).
2The infant will need additional doses of the hepatitis B vaccine at one and six months of
age to provide complete protection

88- A patient's cr clearance is 70% what should we do with the drug dose that which eliminated by the
kidney?

a-decrease the dose by 70%


b-decrease the dose by 30%

c-decrease the dose to 30%

d-do nothing

N.B: to 30% = by 70%


Page 27

89 -A girl 16 year has DM type 1 and she takes insulin in a specific medical schedule
She takes pioglitazone with insulin according to instructions from her doctor from 2months ago.
She complained of nausea and vomiting for two days as she escaped from the schedule… she
made Lab tests and you have shown her results in detail

some of her results:

HEART RATE110
Blood pressure 80/50

Initially you give her?

A. Began a new schedule


B. Following her schedule without changes
C.0.9 NORMAL SALINE IN 500 ML SOLUTION
D.225 normal saline in 500 ml solution

90. Sodium bicarbonate antacid makes an out elimination of

A. PKA=1.2, WEAK ACID


B. pka=5.1, weak acid

C. pka=7.8, weak base

D. pka= 9, weak base

91. Why B blockers are used to treat angina?

a. increase parasympathetic cardiac stimulation


b. increase sympathetic cardiac stimulation

C-DECREASE SYMPATHETIC CARDIAC STIMULATION

92. Drug make a complex with antacids?

TETRACYCLINE AND FLUOROQUINOLONES

ARIF PHARMA CLINIC Copyright@arifpharma.com Www.arifpharma.com


Page 28

N.B: Aluminum, magnesium in anti-acids, and calcium in dairy products, all of these cations make a
complex with fluoroquinolones and tetracycline antibiotics

93- A Drug acts on a non-receptor mechanism?

A-DEFEROXAMINE
B-Desloratadine

94-A Drug that may cause arrhythmia

DIGOXIN

95- Mechanism of action: albuterol inhaler?

BRONCHODILATOR SHORT-ACTING SELECTIVE Β2 AGONIST

96- Mechanism of action of dantrolene?

DANTROLENE DEPRESSES EXCITATION-CONTRACTION COUPLING IN SKELETAL MUSCLE BY BINDING


TO THE RYANODINE RECEPTOR AND DECREASING FREE INTRACELLULAR CALCIUM CONCENTRATION.
‫ ـ‬If the question is whether dantrolene has direct or indirect action .. the answer is:

DANTROLENE IS A DIRECT-ACTING SKELETAL MUSCLE RELAXANT. IT IS CURRENTLY THE


ONLY SPECIFIC AND EFFECTIVE TREATMENT FOR MALIGNANT HYPERTHERMIA

97 Solution differs from solid in?

EASY TO ADMINISTER.

98 KNOW ABOUT ANTIARRHYTHMIC DRUG CLASSES:

VERAPAMIL CLASS 4

ARIF PHARMA CLINIC Copyright@arifpharma.com Www.arifpharma.com


Page 29

PROPRANOLOL … CLASS 2

99- Category X:

Studies in animals or humans have demonstrated fetal abnormalities and/or there is positive evidence
of human fetal risk based on adverse reaction data from investigational or marketing experience, and
the risks involved in the use of the drug in pregnant women clearly outweigh any possible benefit. The
drug
Is contraindicated in women who are or may become pregnant.

100- A pregnant woman has a sulfa allergy, she suffers from vaginal itching
heranalysis shows positive E.coli
what is the antibiotic of choice for her infection?

A- Ciprofloxacin

B- Septazole

C- NITROFURANTOIN
D- Tetracycline

101- When should a woman change her pregnant pills?

A-if she forgets to use pills

B-if she forgets to use pills for 24 hrs.

C-if she forgets 48

D-IT IS NEEDED TO BE CHANGED EVEN I F SHE DOESN'T FORGET TO USE PILLS

102- A.W. is an 85-year-old man who presents to his physician with LUTS. A digital rectal
examination confirms the diagnosis of BPH, and the physician schedules a further workup including
a prostate ultrasound, which indicates his prostate volume is 31 g. A.W.'s score on the AUASI is 15.
His BP is 118/70 sitting, and 102/62 standing. Which one of the following therapies is best at this
time?
A. Terazosin.
B. Finasteride.
C. TAMSULOSIN.
D. Finasteride plus tamsulosin.
Page 30

N.B:1-Tamsulosin is an α-adrenergic blocker with more specific activity for the

GENITOURINARY SYSTEM.

2-A.W. already has orthostatic, tamsulosin would be preferred over terazosin for this patient.

Orthostatic hypotension can still occur with all α-adrenergic blockers, so patients should be monitored
when therapy is initiated.

3- Finasteride, an α-reductase inhibitor, and combination therapy with these agents are recommended
when there is evidence of large prostate size (greater than 40gm).

103-A patient with peptic ulcer due to H.pylori and has an allergy to B-lactam… what is the best
medication for him?

PPI+CLARITHROMYCIN+METRONIDAZOLE

104. A dyslipidemia patient has ischemic heart disease. Which of the following is contraindicated
with this case?

A-CELECOXIB
B-Simvastatin

N.B:

- dyslipidemia = elevated levels of LDL or low levels of HDL


- Celexocib is contraindicated with heart disease

105- W.F. is an 85-year-old man who presents to his physician with pain from hip OA
(osteoarthritis) He also has hypertension, coronary artery disease, and BPH. For his OA, W.F. has
been taking acetaminophen 650 mg 3 times/day. W.F. reports that acetaminophen helps, but he
still experiences pain that limits his ability to walk. Which one of the following is the best next step
in analgesic therapy for W.F.?

A. Change the analgesic to celecoxib.

ARIF PHARMA CLINIC Copyright@arifpharma.com Www.arifpharma.com


Page 31

B. ADD HYDROCODONE.
C. Change the analgesic to ibuprofen.
D. Add glucosamine.

N.B:

1- The AGS recommends treatment with opioids for OA when older patients
do not respond to initial therapy with acetaminophen.
2- The NSAIDs and COX-2 inhibitors are seldom considered when a thorough assessment of the
patient shows that the risk of treatment (gastrointestinal bleeding and renal disease) does not
outweigh the potential benefit

3Glucosamine can be added to this patient's medication regimen; however, if effective, it will not
provide immediate relief of pain.

106- A diabetic woman has hypertension and she is sensitive to sulpha compounds…Which of the
following is contraindicated with this case?

A-GLIPIZIDE

B-metformin

C-Amidrone

D-Enalopril

N.B: Glipizide is a sulphonylurea derivative


Page 32

Drx Arif khan

1. Drug can be used as an anti-inflammatory with long action.


a) Naproxen
b) Peroxicam
2. Summary case with women needing analgesic &anti-inflammatory with long action?
a) Naproxen
b) Peroxicam
3. 29 years man takes an overdose from a drug, then he takes an antidote, but still no effect. Why?
a) He has tolerance to the antidote.
b) Antidote does not work, because it was taken after 4hrs.
c) Antidote eliminated too fast from the body.
d) Antidote dose was small.
4. Biological substances can be stored at ----- 2-8 °C.

5. Wrong medication prescribed by the doctor, what pharmacist should do?


a) Inform the chief pharmacist.
b) Inform the prescriber’s doctor.
c) Inform the patient and tell how bad this drug is for him.
d) Inform the health ministry.
6. After dispensing the drug the pharmacist finds mistakes, what should do?
a. Refer to the prescriber’s doctor.
b. Head pharmacist.
c. Patient who received the drug.
7. Elderly man had hypertension, what should do?
a) Take a normal adult dose.
b) Start a small initial dose then increase.
c) Start a large initial dose then decrease.
8. Young girl has seizures and tremors while using oxycarbamazipine, what should do?
a) Shift to carbamazepine.
b) Shift to clozapine.
Page 33

C) Shift to topiramate.
d) No change in therapy.

9. A middle age man did a lab test Fasting blood glucose= 9.1, HbA1c= 9, and he is taking glipizide,
lisinopril, and pioglitazone.

a) Keep on these drugs.


b) Stop glipizide and take metformin.
c) Take insulin.

10. Normal range for HBA1C?

a. Nondiabetics (4---5.6)
b. High-risk (5.7--6.4)
c. Diabetic. (HbA1C equal to or above 6.5)

11. Heparin & warfarin contraindicated with


a) Dexamethasone
b) Indomethacin
c) Insulin

*According to the options, if Insulin not present the answer diclofenac

12. Decrease or increase dose by technician to the patient is error?


a. omission error
b. Prescription error
c. Dispensing error

d. Personal error
Page 34

13. Drug with Slow metabolism


a) Phenobarbital
b) Thiopental
c) Pentobarbital
d) Secobarbital
14. Amphotericin B for which infection disease.
a) Subcutaneous
b) Systemic
c) Cutaneous
d) Both A&B
15. Acyclovir is used for
Answer: Herpes simplex virus.

16. Drug with 500 units/ml, and the total unit needed is 2700 units, each vial containing 10 ml with
conc100 units /ml. How many vials will dispense

Answer: 3 vials

17. Metoclopramide MOA –

Answer: Metoclopramide is principally a dopamine D2 antagonist but also acts as an agonist

on serotonin 5-HT4 receptors and causes weak inhibition of 5-HT3 receptors.

18. Which committee rules out a drug from the hospital


Answer: Therapeutic evaluation committee?

19. Formula of the drug can be adjusted by


Answer: Pharmacy therapeutic committee

20. Which drug with a statin can increase myopathy?


a) Gemfibrozil
b) Gliclazide
c) Thyroxin sodium
d) Metformin
21. To determine the severity of myopathy caused by statin drugs
Answer: Serum protein kinase.
Page 35

22. Elderly patient, after taking statin drugs has myopathy. What analysis or test that he has to do?
a) lipid profile
b) Muscle thickness
c) Serum creatinine kinase level.
23. The treatment of Helicobacter pylori (H. pylori).
a) clarithromycin+pantoprazole
b) Clarithromycin+amoxicillin+pantoprazol

**Omeprazole & metronidazole not in the options.

24. Pt has gastritis due to H pylori, the ttt is

Answer: Amoxicillin + Clarithromycin + Omeprazole.

*Sensitivity test of penicillin done after: (range 30 days or more, so will the answers according to
choices.)

25. Sensitivity test of penicillin done after:


a) less20 days
b) More than 30 days

c) 25day exactly
d) May be 5 days or 3 days
26. Penicillin sensitive test is done
a) 10 - 15 days
b) 20 days or less
c) 25 days or more.

27. Sensitivity test of penicillin done after:

a) 20 days
b) More than 20 days
c) 2 weeks
d) 4 weeks.
Page 36

28. Penicillin sensitivity test for the intradermal stage which is a must
Answer: Skin scratch test

**Penicillin sensitivity test is done?? (Hours)

**Penicillin allergy skin testing is a relatively simple procedure completed in about one hour
via scratch and intradermal test in penicillin skin test should wait for (time?). 15-20mint.

29. Female Pt with pneumonia went to the hospital to get vaccine, which should give:
a) Pneumonia + meningitis.
b) Pneumonia + influenza.
c) Pneumonia + meningitis + influenza.
d) Only influenza.
30. Pregnant woman was admitted to the hospital suffering from upper respiratory disease, she
took a ceftriaxone IV dose; and stayed in the hospital for 3 days. So what the vaccine should
take?
a) Influenza.
b) Influenza + pneumonia.
c) Meningitis + influenza.
d) Pneumonia + meningitis
31. Patient has lung obstruction and he was not vaccinated, which vaccine should he take?
a) Influenza.
b) Pneumonia and influenza
c) Meningitis and pneumonia.

32. Rhabdomyolysis can be detected by: (All answers


true)
a) a) Creatine kinase level.
And can be detected by
a)
b) Serum Ca.
c) Myoglobin K.
d) Urine analysis.
Page 37

33. Pt. has involuntary movements, while taking antipsychotics, she gains weight, can’t sleep,
isolated from people, which antipsychotics must take:
a) Haloperidol then clozapine
b) fluoxetine then imipramine
c) clozapine then haloperidol
d) imipramine then fluoxetine
34. If we have a sustained-release drug, but we break it down into powder instead of using it as a
tablet, what will happen?

a) Toxicity will happen. (SR containing multi-dose)

b) Will give the same effect.


c) Will give a better effect.
d) Pharmacokinetic problems.

35. Laxative contraindicated in pregnancy


is:
a) Ethylene glycol.
b) Castor oil
c) Bisacodyl
d) Glycerin.
36. For a Newborn baby prescribed for his morphine, what is the best route of administration:
a) IV
b) IM
c) SC
d) Suppository.
37. Who is responsible for determining the best dosage form?
a) Prescriber doctor.
b) Pharmacist/Clinical pharmacist.
c) Nursing team.
d) Patient.
38. A person is addicted to medications, he entered the hospital suffering from cramps, gave him
diazepam, but there is no effect. So this patient is addicted to any of the following medications?
a) Cocaine
b) Opioid.
Page 38

39. NICOTINIC DRUGS ARE LABELLED IN HOSPITAL PHARMACIES:


a) A
b) I
c) O
d) N
40. In a large amount of parental medication we use
a) Bacteriostatic water
b) Sterile water.
c) Sterile water for injection.
41. In pediatric analgesics for tooth growth pain contraindications have a bad effect in ton brain.
a) Aspirin
b) acetic acid
c) antiseptic

42. Teething gel for children


contained
a) Lidocaine
b) aspirin
c) acetic acid
d) antiseptic
43. Tooth gels contain one of the following agents.
a) Antiseptic
b) Antifungal
c) sweetening agent
44. Drug must keep in a dark box used for pain of toothache and can cause more dangerous effects
on infants (seizure and toxicity)

d) Clove oil
e) Ibuprofen susp
f) Lidocaine
g) Viscous gel
Page 39

45. Neonate suffers from cardio coagulopathy, so vit k should be administered through?
a) Iv
b) im
c) supp
d) SC

46. Man with BMI 85% of his Age so he is?


a) Wt. loss
b) obese
c) highly obese
d) Very obese
47. Child 85% BMI of his age he had muscle and skin thickness test for...
a) edema
b) skin test
c) body fat
d) Vasculature.
48. Five-year-old child and a weight 80 % of his age, skin thickness and arm muscles arenormal,
this child is:
a) Risk of overweight
b) Overweight
c) Under normal.
d) Normal range.

BMI percentile & definitional categories childhood weight


Underweight ˂ 10 %

Normal 10 % to 84 %

Overweight 85 % to 94 %

Obese > 95 %

Xtreme obese > 120 %


Page 40

49. Antidote for tramadol


Answer: Naloxone

50. Which drug enhances the minimal alveolar concentration (MAC) of an anesthetic?
a) Halothane (Fluothane)
b) Isoflurane (Forane)
c) Enflurane (Ethrane)
d) Desflurane (Suprane)
e) Sevoflurane (Ultane)
f) Nitrous oxide (nitrous oxide)
51. Drugs decrease MAC
Answer: Barbiturates, Lidocaine, opioids, and chlorpromazine.
52. Drugs that increase MAC
Answer: L-dopa, amphetamine, and ephedrine.
53. In the elderly, what is the physiological effect of drug distribution?
a) Decrease water soluble and lipid soluble drug
b) Increase both
c) Water-soluble increase lipid decrease
d) Increase lipid sol. & decrease water sol drugs
54. In Pregnancy a woman takes naproxen what harm could to the baby?
a) Uncontrolled bleeding
b) GI reflux disease.
55. Pregnancy with otitis media?
a) Azithromycin
b) Cefixime

56. Mechanism of action of ciprofloxacin.


a) Cell wall synthesis
b) Inhibit. DNA gyrase
c) Inhibit. MRNA inhibits.
d) Protein synthesis inhibition.
Page 41

57. Drugs undergo phase 1 metabolism.


a) Diazepam,
b) Lorazepam.
58. Effects of benzodiazepine in breastfeeding ---
Answer: No effect.
59. Disease causes brain and liver damage while using aspirin in children.
a) Metabolic syndrome
b) Reye disease.
60. The prescription is valid to dispense within:
a) 72hr
b) 48hr
c) 24hr
d) 90 hr
61. Maximum days of use of narcotics?
Not more than one day’s medication may be administered at one time.
This treatment cannot last more than three days and may not be renewed or extended.
The nurses should replace the used ampoules from the pharmacy within 3 days from the date of the
prescription.

63. Controlled drug maximum prescription time?


a) 30
b) 45
c) 60
d) 90
*If there is no 90-day in choices. The answer will be 30day

64. Uncontrolled drug maximum prescription time?


a) 30 days
b) 45 days
c) 60 days
d) 90 days
65. Narcotic orders for inpatients:
a) 12hr
b) 24hr
Page 42

c) 36hr
d) 72hr
66. Narcotics continuous IV infusion with IV admixture.
a) 1-day
b) 7-day
c) 5 days
67. Non parenteral narcotics can be used for:
a) 15 days.
b) 30 days.
c) 45 days.
68. Non parenteral narcotics maximum prescription time
Answer 30 days.

69. Narcotics indication.


a) Gynecological
b) Obstetrics
c) Analgesic
d) Irritable bowel syndrome.
70. Narcotic drugs can be used for:
a) Cancer pain.
b) IBS.
c) RA
71. Cocaine overdose can be treated with? No methadone is present.
a) Naloxone
b) Opioid
* May be any of these Benzodiazepine, dexmedetomidine, and rimcazole
There is no officially approved specific antidote for a cocaine overdose

72. Gasoline fumes induce toxicity of:

a) Lead

b) Arsenic
c) Lithium
d) Magnesium
Page 43

73. Lead antidote:


Answer: Penicillamine or caEDTA

74. Convulsions due to lithium toxicity can be treated by (information)

a) Phenobarbital
b) Propanol
c) Benzodiazepines

75. Toxicity of lithium-treated by

Answer: Sodium bicarbonate. (Antidote of lithium)

76. Antiasthmatic used alone, can cause cardiovascular death.

a) Inhaled corticosteroids
b) Inhaled corticosteroids
c) Short-acting B agonist
d) Long-acting B agonist

77. Imposed oral Narcoticsgets addictive after?


a) 15day
b) 20day
c) 30 day

78. Drug Used for cytomegalovirus


a) abacavir
b) entecavir
c) zanamivir
d) Foscarnet.

**If there is no ganciclovir or valganciclovir in choices ................ then the answer is Foscarnet.

79. 65-year-old man suffering from heartburn and peptic ulcer. Which drug is not
recommended because of its unwanted effect on the central nervous system?
a) Famotidine
b) Cimetidine.
c) nizatidine

80. Which drug suppresses milk production in breastfeeding women?

* Estrogen has been shown to decrease the quantity and quality of human milk; use only if
clearly needed; monitor the growth of the infant closely.
Page 44

81. Which drug is contraindicated in breastfeeding:


a) Lithium.
b) Warfarin.
c) Methyldopa.

82. The best drug for insomnia:


a) Diazepam.
b) Buspirone
c) Zolpidem

83. What is the best hypnotic for geriatric?


a) Alprazolam
b) Diazepam.
c) Phenobarbitone.

** If there is no Zolpidem or Zaleplon in the other choices the answer Alprazolam will becorrect.

84. % of theophylline in aminophylline.


a) 75
b) 85
c) 90
d) 60
85. Drugs that are contraindicated during lactation (all answers true)
a) Anticancer
b) Aspirin
c) Iodine
d) Androgen
e) Bromocriptine

**I think Tetracycline is also contraindicated in lactation.

86. Which drug worsens the negative effect of statins on muscles?

a) Gemfibrozil.

b) Cholestyramine.
Page 45

87. What is the most commonly dispensed OTC in pharmacy?

a) Oral antiseptic.

b) Analgesic.

c) Antiallergic remedies.

88. Side effects of ciprofloxacin in children:

a) Phototoxicity.

b) Seizures.

c) Vertigo.

** If Phototoxcity not in options, choose seizures.

89. Drug has low androgenic activity:


a) Finasteride.
b) Flutamide
c) Mifepristone

**Suppose that the right answer if spironolactone is not present is mifepristone.

90. Which drug has anti-androgenic effects?


a) Tamoxifen
b) Mifepristone
c) Finasteride.
d) Anstrazol.
91. Patient came to the hospital suffering from pain in his leg after playing football.
Which analgesic should be used by injection?
a) Ketorolac.
b) Naproxen.
c) Ibuprofen.
d) Diclofenac.

**The answer is diclofenac. If there is no Diclofenac in the choices, then the answer will be Ketorolac.
Page 46
92. Nonaqueous preparation.

a) Ear and eye preparation.

b) Syrup and elixir.

c) Essence Colloid and elixir

93. Which drug has more effect as a skeletal muscle relaxant?


a- Tubucurarine
b- Doxacurarine.
c- Succinylcholine.
94. Which of the following drug cause nasal bone hypoplasia?
Answer: Warfarin.

95. Drug cause sedation for fetus in 3rd trimester?


a- Fluoxetine
b- benzodiazepine
c- TCA.
d- antiepileptic drugs
96. Pt has Benzodiazepines toxicity, flumazenil is given but the patient didn’t respond. Why?
Answer- Because the antidote was given to the patient after 4 hours.
97. Non-formulary drugs are dispensed to?
Answer: Inpatient (Indoor patients

* Non-Formulary Drugs shall be clearly documented, on a special Request Form and Direct
Purchase form, and properly labeled and stocked in a separate drawer in the inpatient pharmacy
dispensing area.

98. Drug waiting to be registered in the Saudi drug committee


Answer: Nonformulary drug?

99. Drugs to be registered in the Ministry of Health?

a- Formulary
b- Non-formulary
c- Controlled drugs
Page 47

100. Drug still waiting for approval by the therapeutic and pharmacy committee. What do we this
drug?
a- Formulary drug
b- Non-formulary drug
c- Controlled drug
d- Restricted drug
101. All these drugs are controlled drugs EXCEPT
a- Carbamazepine
b- Rivotril
c- Tylenol
d- Diazepam
e- Fosamax
102.Which one is an OTC drug?
a- Analgesics
b- Warfarin
c- Ciprofloxacin
d- Insulin
103. A pregnant woman has an infection. I don’t remember the m.o. and she is sensitive to penicillin.
And the m.o resistant to erythromycin and clindamycin. Which antibiotic is suitable for her?
a- Ciprofloxacin.
b- Vancomycin

104. How many ml syrups u prepare in the pharmacy?


The target of blood pressure for hypertension patients:

a) 117/70
b) 110/80
c) 140/ 90,

If the blood pressure increased to an abnormal range for any reason in a hypertensive patient,
the goal will be to decrease the blood pressure to 140/90 to protect the patient from organ damage.
Page 48

105. 20 years girl comes to take contraceptive piles which of the following can make oral
contraceptives fail?

a) Rifampicin
b) Amoxicillin
c) Floxacilline

106. Role of nicotine in abortion?

By FDA. Cigarette smoking is known to cause: information


a) Spontaneous abortion
b) Increased perinatal mortality.
c) Potent vasoconstriction
d) Decrease uterine & placenta blood flow... Risk of abortion

107. In the case of Sever state of Alzheimer, we use:


Answer: Memantine.

108. Mechanism of action of memantine treatment Alzheimer's:

a) Activated muscarinic receptors


b) Inhibit choline esterase enzyme
c) Partially activate central nicotinic receptors
d) Decreased enzyme may be this enzyme
(Glutamate NMDA enzymes receptor antagonist)

Memantine is an N-methyl-D-aspartate (NMDA) receptor antagonist.


The mechanism ofaction of memantine is (distinct) from those of the cholinergic agents:
it is proposed to be neuroprotective. Memantine is an uncompetitive antagonist
of the N-methyl-D-aspartate (NMDA) type of glutamate receptors.

109. Anti-diabetic FDA-approved for infants:


Answer: Metformin

110. Toxicity of TCA imipramine or amitriptyline CNS toxicity can be treated by:
a) Naloxone
b) Alpha agonist
c) Pyridostigmine
Page 49

111. Side effects and adverse drug reactions in geriatrics due to


a) Decreased elimination of the drug.
b) Polypharmacy.
c) Geriatric less sensitive to drug effects.
d) Good relations with health providers.
112. Mechanical error during filling the prescription:
a) Dispense the wrong drug.
b) Forget to ask about drug allergies.
c) Forget to counsel about drug-drug interaction.
d) Change the drug that is not present in the prescription.

113. Patient with hemodynamic shock treated with:


a) Saline.
b) Corticosteroid.

114. A pregnant woman has syphilis can be treated by:


a) Ciprofloxacin
b) Amoxicillin+clavulanic acid
c) Trimethoprim-sulfamethoxazole

*If there is no Benzathine penicillin as an option …choose Amoxicillin + Clavulanic acid.

115. Breastfeeding women need contraceptives which of these:


a) Estrogen.
b) Progesterone.
c) Monophasic pill.
d) Biphasic pills.

Mono and biphasic are techniques for the administration of combined oral contraceptives
(Estrogen and progesterone).

116. Drug used in treatment for Hypertension and migraine:


a) Clonidine
b) Hydralazine
c) Minoxidil
d) Reserpine

* If there is no propranolol as an option …choose clonidine.


Page 50

117. A breastfeeding woman has hypertension and the antihypertensive drug has a hazard to the
baby.
a) Propranolol
b) Atenolol
c) Verapamil
118. Is Propranolol similar in action?
a) Metoprolol
b) Carvedilol
c) pindolol
d) Sotolol
119. Who is responsible for the release of investigational drugs from the manufacturer to the pharmacy?
a) Chief pharmacist.
b) Physician.
c) Hospital owner.
d) Direct investigator.
120. Barbiturates side effects in elderly patients.
a) Ataxia
b) Agitation
c) Dementis (agitation)
** If there is no (excitement) choice... another choice, if both are not available, is (confusion).

121. Patient treated from amphetamine addiction: also has hypertension and depression,which
antihypertensive you give:

a) losartan
b) captopril
c) methyl dopa
d) clonidine
122. Mechanism of action of Zolpidem:
a) increase GABA release
b) Increase GABA Subtype
Page 51

123. GAD enzyme acting on glutamate produces


a) Amyl soluble protein
b) Glutamate.

**Glutamate decarboxylase or (GAD) is an enzyme that catalyzes.


The decarboxylation ofglutamate to GABA and CO2.

124. Drug of choice for nausea and vomiting in pregnant:


a) Domperidone.
b) Cyclizine.
c) Dimenhydrinate.
125. A pregnant woman is allergic to penicillin. Which antibiotic should be given to her as
prophylaxis against streptococci b4 delivery?
a) Penicillin G.
b) Cefazolin.
c) Vancomycin.
d) Clindamycin.
126. Mechanism of action of ramelteon:
a) Melatonin agonist.
b) Melatonin antagonist.
c) Increase Melatonin synthesis.
d) Decrease Melatonin synthesis.
e) Uterine vascular resistance
127. Why should finasteride be stopped if the woman wants to be pregnant?
a) Cause abortion.
b) Teratogenic effects.
c) Genital malformation to infant.
d) Cranial facial abnormality.

128. A man got to emergency after burns, toxicity dye to warfarin?


a) Cyanide.
b) Lead.
c) Mercury.
Page 52

Interaction between Warfarin and TCA (Tricyclic antidepressant) --- Increase bleeding risk or increase
the adverse effect of warfarin (Increase plasma level of warfarin)
129. Which anesthetic cause hepatotoxicity?
a) Ketamine.
b) Nitrous oxide.
c) Isoflurane
d) Propofol.
130. 12 years old taking oxcarbazepine, suffers from rash and redness. What should be done?
a) Lamotrigine.
b) Topiramate.
c) Carbamazepine.
**Ethosuximide is not in the choices.

131. Pt adult has vaccinated in 2010 against influenza, in 2011 against influenza, in 2012 against
meningococcal, in 2012 against influenza, 2013 against meningococcal, what is the vaccine
should take now?
a) Meningococcal
b) Hepatitis
c) Influenza
132. First baby milk named
Answer: colostrum

N: B Colostrum: A yellowish liquid, especially rich in immune factors, secreted by the mammarygland
of female mammals a few days before and after the birth of their young.
133. Pt has lower limb and trunk pain and used antipsychotic drug for 11years, what is his case?
a) Acute dystonia
b) Tardive dystonia
c) Acute dyskinesia
d) Tardive dyskinesia
134. Formulating of the drug is done by:
a) Saudi health
b) Drug therapeutic committee.
c) Pharmacist.
Page 53

135. Best antidepressants given to an 83 years old


a) Fluoxetine.
b) Alprazolam.

136. Lactulose used in hepatic encephalopathy patients to:


a) Acidify the colon to inhibit the absorption of ammonia.
b) Alkalization of the colon to inhibit the absorption of ammonia.

137. Anti-diabetic drug approved by FDA in gestation


Answer: Insulin

**Insulin is the drug of choice in gestational diabetes (diabetic pregnant)


** But glargine (long-acting insulin) is category C in pregnancy.
* If there is no Insulin in the choices choose Metformin NOT glargine.
*Drug should be stored in black container to make medical care staff
aware and it used forchildren for pain and it can cause brain injury? Aspirin

138. Interactions of CCB with alprazolam:

Only CCB has an antidepressant effect….I think answer will be (CCB increase effect of
antidepressant)

139. Criteria in formulary management system (information)


a) Safety may be in option
b) Economy
c) Aesthetic
140. Estrogen receptor positive cause breast cancer which can be treated by
Answer: Tamoxifen
141. What is the maximum prescription time for narcotic for patient admitted in emergency?
a) 24 hours
b) 72 hours
c) 45 hours
d) 30 hours

** 24 hrs. In case of emergency. 3 days (72hr) in case of outpatient.


Page 54

142. A patient admitted to emergency with cocaine toxicity, what will be the treatment?
a) Baclofen
b) Naloxone
c) Amitriptyline
d) Physiostigmine
1. Bisacodyl Mechanism of action?
a) Bulk laxative
b) Stimulant laxative
c) Stool softener
143. If a lactating mother has less milk production, what are the effects on nursing an infant?
a) constipation
b) loss of appetite
c) diminished weight gain
d) increased sleep
144. Iproniazid is given to a patient because it increases the secretion of noradrenaline n other
amines in the brain but its one adverse effect is it causes hypotension in the patient what is the
reason?
a) Increase excretion of nor adrenaline
b) Displacement of nor-adrenaline with dopamine in store.
c) It is highly bound to plasma protein.
d) It’s histaminic effect.
145. Iproniazid drug release dopamine, nor adrenaline and other amines but has hypotensive
effect due to
a) It is highly bound to plasma protein.
b) It’s histaminic effect.
c) It increases excretion of nor-adrenaline.
d) Displacement of nor-adrenaline with dopamine in store.
146. What are the benefits of patient orientation formulary system in hospitals?
a) Budget of medicine
b) Expensive medicine
c) Safe and effective medicines
d) Gut relation between the staff n medical officers
Page 55

147. What are the pharmacokinetic parameters studied for a drug before it to the patient and for
estimation of dose in DAT patient?
a) Bioequivalence studies
b) Gross estimation
c) Preclinical studies
d) Population means
148. A patient is suffering from insomnia, what should give him:
a) Phenobarbital
b) Zolpidem
c) Alprazolam
d) Nortriptyline
149. Patient is taking hypnotic Medication, he suffers from irritability and agitation this reaction
related to:
a) Diazepam.
b) Alprazolam
150. Patient stop medication hypnotic he suffered from irritability and Agitation this reaction

related to.

Answer: Side effect of barbiturates withdrawal symptoms.

151. Drug affect fetus in pregnancy cause renal dysfunction?


a) Sulfonyl urea.
b) An angiotensin-converting-enzyme inhibitor (ACE inhibitor) drugs.
152. According to Saudi ministry of health, which drug is anti-hyperglycemic?
a) Insulin
b) Metformin
c) Gliclazide

153. From OTC Multi-vitamin analogs


Answer: Vitamin D.
Page 56

154. 27 years old Pt has respiratory depression from anesthetic, which drug counteract post
anesthetic respiratory depression.

a) Morphine.
b) Thiopental.
c) Diazepam.
155. To be the medical center contains a highly efficient data must be available follows:
a) Cooperation between doctors and pharmacists.
b) To provide stocks of medicines.
c) Choose the appropriate medications and the least side effects.

BEST OF LUCK

Www.arifpharma.com
Page 57

Pharmacy Calculation & Compounding

Www.arifpharma.com

Arif Pharma Clinic Copyright@arifpharma.com arifpharma.com


Page 58

SYSTEMS OF MEASURE

COMMON METRIC

Apothecaries’ system Avoirdupois system


Length units
Weight
Unit volume

FUNDAMENTAL UNITS

Equivalents of Length meter (m)


1 inch = 2.54 cm
1 meter (m) = 39.37 in

Equivalents of Volume liter (l)


1 fluidounce (fl. oz.) = 29.57 mL

1 pint (16 fl. oz.) = 473 mL

1 quart (32 fl. oz.) = 946 mL

1 gallon, US (128 fl. oz.) = 3785 mL

1 gallon, UK = 4545 mL

Equivalents of Weight gram (g)

1 Pound (lb, Avoirdupois)


= 454 g
1 kilogram (kg) = 2.2 lb

Arif Pharma Clinic Copyright@gmail.com arifpharma.com


Page 59

The measure of length:

The meter (m) is the fundamental unit of length in the metric system

11 KKilometer
ilo meterter Km
Km Orr
O km 11000000 m meter
eteeter = 1-30
=r 10 0 --33
1meter
1 Hectometer
1 HeHecctoto meterter Hm
Hm Okrm
km
Or hhm
m m
1100 0etet0er mmeter
etetere=r10 -2
= 10 0 --22
1meter
11 Dec
Decameter
Decaa meterter Dm
D m m
110 0etetermeter
11 Dec
December
Decii meteterr Dm
Dm
Dm m
00.1 .ete1terem
r eteeter
meter 1 0 --11
= r10=-11meter
0
11 CCentimeter
entiti meterter Cm
Cm
Cm m
0 ete.rete0r 1meter
0.01 meteeter r =-210
= 10 1 0 --22
meter
11 MMillimeter
illi meterter Mm
Mm
Mm m e
00.001 e
te
t e
rr
. 0 0 1metermetere=ter -3
10= meter
11 mMicrometer
icro meterter µm
µm
µm 0103-. 0 0 0 0 0 1meter
10.000001
0 meteter = e10 r =-6 meter
1 nanometer nm 106-
10.000000001
0 meter = 10-9 meter

The measure of Volume:

The liter (l) is the fundamental unit of volume in the metric system

1 Kiloliter Kl Or kl 1000 liter = 10-3 liter


1 Hectoliter Hl Or hl 100 Liter = 10-2 liter
1 Decaliter Dl 10 liter
1 Deciliter Dl 0.1 liter = 10-1 liter
1 Centileter Cl 0.01 liter = 10-2 liter
1 Mililiter Ml 0.001 liter = 10-3 liter
1 Microliter µl 0.000001 = 10-6 liter

For example
Convert from micrometer to centimeter

1m = 100 cm 1 m = 1000,000

mcm

100 cm = 1000,000 mcm 1cm = 10,000 mcm

The measure of Weight:

Arif Pharma Clinic Copyright@arifpharma.com arifpharma.com


Page 60

The gram (g) is the fundamental unit of weight in the metric system

1 Kilogram Kg Or kg 1000 gram = 10-3 gram


1 Hectogra m Hg Or hg 100 gram = 10-2 gram
1 Decagra m Dg 10 ram =
1 Decigram Dg 0.1 gram = 10-1 gram
1 Centigra m Cg 0.01 gram = 10-2 gram
1 Miligram Mg 0.001gram = 10-3 gram
1 Microgra m µg 0.000001 gram = 10-6 gram
1 Nanogra m Ng 0.000000001 gram = 10-9 gram

For example:

Convert from micrometer to centimeter

1m = 100 cm 1 m = 1000,000

mcm

100 cm = 1000,000 mcm 1cm = 10,000 mcm

How many colchicine tablets containing 600 mcg may be prepared from 30 g of colchicine?

50,000 tablets
Reduce 1.256 g to micrograms, to milligrams, and to kilograms.
1,256,000 mcg
1256 mg
0.001256 kg

Arif Pharma Clinic Copyright@gmail.com arifpharma.com


Page 61

An intravenous solution contains 500 µg of a drug substance in each ML. how many
Mg of the drug would a patient receive from the intravenous infusion of a liter of the
solution?

Answer:
500 µg in 1ml
“X” mg in 1 liter

1 liter = 1000ml
500 µg 1ml
“X” µg 1000ml

“X” = 1000*500 = 500000 µg = ?? mg


1

1mg= 1000 µg

“X” = 500000 = 500mg


1000

The patient will receive 500mg of the drug from the intravenous infusion of a liter of the solution

The common system

(1) Apothecaries System (2) Avoirdupois System


The units in this system are small, so it is • The system is used to measure
used: large quantities such as quantities
*In Writing the prescription by the doctor sold by wholesalers and quantities
to the patients. used in pharmaceutical industries
* To dispense drugs to patients by the
pharmacists • It is not used in writing a
prescription
It includes units for both weight & volume It has units for weight only’ it does not
include units for volume

Arif Pharma Clinic Copyright@gmail.com arifpharma.com


Page 62

NB: The grain is the only unit that has the same value in both Apothecaries And
avoirdupois system other units which have the same name (the ounce and the
pound) have different values and we have to be able to differentiate between them

Arif Pharma Clinic Copyright@arifpharma.com arifpharma.com


Page 63

Apothecaries Measures of Avoirdupois measures of


weight (5 units) weight (3 units)

1. Grain (gr) = 0.065g 1. Grain (gr)= 00.065 g

2. Scruple

3. Drachm 2. Once (oz) = 437.5 gr

4. Apothecaries
ounce (Apot0h-oz) 3 Pound (ib)= 7000gr

5. Apothecaries pound
(Apoth, in)

Avoirdupois “measure of weight”

Commercial
Pound “Ib.” Ounce “oz.” Grain “gr.” uses
1 16 7000
1 437.5

Apothecaries “fluid measure”


Compoundin
g Prescription
Gallon“gal.” Quart Pint Fluidounce Fluidrachm“f ʒ” Minim“m.”
“qt.” “pt.” “f℥”

1 4 8 128 1024 61440

1 2 32 256 15360

1 16 128 7680

1 8 480

1 60

Arif Pharma Clinic Copyright@gmail.com arifpharma.com


Page 64

Pound Ounce Drachm“ʒ” Scruple Grain


“Ib.” “℥” “э” “gr.”

1 12 96 288 5760

1 8 24 480

1 3 60

1 20

Examples Reduce: f℥ iv fʒ iiss to fluidrachm

℥ iv = 4 x 8 fʒ = 32 f ʒ
fʒ iiss = 2.5 f ʒ
Reduce ℥ss ʒii 𝖲I to grain
℥ss= 0.5 X 480 = 250 gr.
ʒii = 2 x 60 =120 gr.
𝖲I = 1 x 20 = 20 gr
: 380 gr.

Arif Pharma Clinic Copyright@gmail.com arifpharma.com


Page 65

Numeral Roman

Arif Pharma Clinic Copyright@arifpharma.com arifpharma.com


Page 66

ARIF PHARMA CLINIC Copyright@arifpharma.com Www.arifpharma.com

Arif Pharma Clinic Copyright@gmail.com arifpharma.com


Page 67

Can you complete This hundred square? Remember, the Romans had no symbol for
zero.

I II III IV V VI VII VIII IX X

XI XII 13 XIV 15 16 17 18 19 XX

21 22 23 24 25 26 27 28 29 30

31 32 33 34 35 36 37 38 39 XL

XLI 42 43 44 45 46 47 48 49 L

51 52 53 54 55 56 57 58 LIX 60

61 62 63 64 65 66 67 68 69 70

71 72 73 74 75 76 77 78 79 80

81 82 83 84 85 86 87 88 89 90

91 92 93 94 95 96 97 98 99 100

Basics of roman numbers:

ss 0.5 i 1
V 5 X 10
L 50 C 100
D 500 M 1000

Arif Pharma Clinic Copyright@arifpharma.com arifpharma.com


Page 68

A) If the second number be bigger than the first one (subtract ) XL 50 - 10 = 40

B) If the second number is smaller than or equals the first one (add) XX 10 + 10

= 20 Xi 10 + 1 = 1B)

We have subtractor roman numbers; they are only used for subtraction

i 1
X 10
C 100

also every little subtract the next 2 Romans

ss 0.5 i 1
V 5 X 10
L 50 C 100
D 500 M 1000
To (add) we can
repeat the number up to 3 times Viii = 8 Lxx = 70

To (subtract) cannot repeat ever, used the number for one time = iX = 9

Example 2018 = MMXViii

2000 MM
2018 10 X
18
8 Viii

REDUCING AND ENLARGING FORMULAS

The following prescription for cold cream provides a 100-g quantity. What mass of each
ingredient is required to provide 1 lb (AV) of cream?
Arif Pharma Clinic Copyright@gmail.com arifpharma.com
Page 69

Rx
white wax 12.5 g
mineral oil 60.0 g
lanolin 2.5 g
sodium borate 1.0 g
rose water 24.0 g

1 lb = 454g

454 x 4.54
100

Multiplication factor to use in calculating the quantities of each ingredient)

12.5 g x 4.54 = 56.8 g of white wax


60.0 g x 4.54 = 272 g of mineral oil
2.5 g x 4.54 = 11.4 g of lanolin
1.0 g x 4.54 = 4.54 g of sodium borate
24.0 g x 4.54 = 109 g of rose wat

Arif Pharma Clinic Copyright@gmail.com arifpharma.com


Page 70

CONCENTRATION EXPRESSIONS

The term percent and its corresponding sign (%) mean ‘‘by the hundred’’ or ‘‘in a hundred,’’
and percentage means ‘‘rate per hundred’’; so 50 percent (or 50%) and a percentage of 50
are equivalent expressions.

50
50 % = = 0.5 = one half
100

Arif Pharma Clinic Copyright@arifpharma.com arifpharma.com


Page 71

Percent weight-in-volume Percent weight-in-volume Percent weight-in-weight


(w/v) (w/v) (w/w)

expresses the number of


expresses the number of expresses the number of grams of a constituent in
grams of a constituent in 100 milliliters of a constituent in 100 g of solution or
mL of solution or liquid 100 mL of solution or liquid preparation.
preparation and is used preparation.
regardless of whether
water or another liquid is
the solvent or vehicle.
Expressed as: % w/v. Expressed as: % v/v. Expressed as: % w/w.

How many grams of Peppermint spirit contains How many grams of a drug
dextrose are required to 10% v/v of peppermint oil. substance should be added to
prepare 4000 mL of a 5% What volume of the spirit 240 mL of water to make a 4%
solution? will contain 75 mL of (w/w) solution?
4000 mL represents 4000 g peppermint oil? 100% - 4% = 96% (by
of solution 5% = 0.05 weight)of water 240 mL of
4000gx 0.05 = 200 g, 10 ml 100 ml water weighs 240 g
answer. Or, solving by 75 ml x ml 96 % 240𝑔
dimensional analysis: 4% = 𝑥
75 𝑥 100 240 𝑥 4

5g 100ml 10 = 750 ml 96 = 10 g
Xg 4000 ml If 1500 g of a solution contains
75 g of a drug substance, what
5𝑔 is the percentage strength
100 𝑚𝑙 x 4000 mL= 200 g, (w/w) of the solution?
answer. 1500g 100%
75 g X%

75 𝑥 100%
=5%
1500
Ratio Strength Parts per Million (PPM)
Ratio strength is another way of expressing The strengths of very dilute solutions are
percentage strength. For example, a 1% commonly expressed in terms of parts per
w/v solution and a ratio strength of million(ppm). I e., the number of parts of the
1:100 w/v are equivalent. agent per
1 million
Express 1: 4000 as a percentage . For example, fluoridated drinking water in
strength.1 part 4000 parts which fluoride is added at levels of between 1
X % 100% to 4 parts per million (1:1,000,000 to
100
X= 4000 = 0.025 % 4:1,000,000) for the purpose of reducing
dental caries.

Arif Pharma Clinic Copyright@arifpharma.com arifpharma.com


Page 72

MOLARITY

Molarity is the number of moles of solute dissolved in one liter of


solution. Therefore, the equation to find the molarity (M) of a solution is:

Moles solute
Molarity =
liters solution

DILUTION AND CONCENTRATION

Dilution is the process of preparing a less concentrated solution from a


more concentrated one.

Arif Pharma Clinic Copyright@arifpharma.com arifpharma.com


Page 73

ALLEGATION ALTERNATE

Allegation alternate may be used to determine the proportion or quantities of two or more
components to combine in order to prepare a mixture of a desired strength.

For example, if a pharmacist wished to prepare a solution of a specified strength by


combining two or more other solutions of differing concentrations of the same ingredient,
theproportion or volumes of each solution to use may be determined by an allegation

alternate.

Arif Pharma Clinic Copyright@arifpharma.com arifpharma.com


Page 74

Prepair 100 ml of 70 % from two solutions that have conc. 80% and 30%.

4
𝑥 100 = 80 𝑚𝑙
80 40

70
30 10 1
𝑥 100 = 20𝑚𝑙

Subtract higher concentration from final conc. to give volume of


lower conc. and viceversa.

Then divided resulting parts on total parts and multiplication


with numbers of solutionmillimeter

So. We need to 80 ml of 80% and 20ml of 30% to prepare 100ml of 70% conc.

ALLIGATION MEDIAL

Result of mix (300 ml of 30%) and (500 ml of 70%)

Arif Pharma Clinic Copyright@arifpharma.com arifpharma.com


Page 75

(30% 𝑥 300𝑚𝑙) + (70% 𝑥 500𝑚𝑙)


𝐶𝑡 =
800

= 43.75%

Arif Pharma Clinic Copyright@gmail.com arifpharma.com


Page 76

RATE OF FLOW OF INTRAVENOUS FLUID

A physician prescribes a 5𝜇𝑔/𝑘𝑔/𝑚𝑖𝑛 IV drip of dopamine for a 175-lb.


patient,and the pharmacist adds an ampul of dopamine (200 mg/5 mL) to a
250-mL bottle of D5W. What drip rate should be run, in drops per minute,
using a minidrip set that delivers 60 drops/mL?

Answer: 30.45 or 30 drops/min

1) convert lb to kg 175 lb ÷ 2.2 = 79.5 kg

2) 1 kg 5 𝜇𝑔
79.5 x 𝜇𝑔
X= 397.7 𝜇𝑔/𝑚𝑖𝑛 ÷ 1000 =0.3977 𝑚𝑔/𝑚𝑖𝑛

3) 200 mg 250 + 5 = 255 ml


0.3977 x ml
X= 0.507 ml

4) 1 ml 60 dp
0.507 ml 30.4 dp ≈ 30 dp/min

Arif Pharma Clinic Copyright@gmail.com arifpharma.com


Page 77

CALCULATING DOSES
According to age

Arif Pharma Clinic Copyright@gmail.com arifpharma.com


Page 78

Arif Pharma Clinic Copyright@gmail.com arifpharma.com


Page 79

𝑑𝑜𝑠𝑒
Vd 𝑉𝑑 = Vd = Volume of distribution
𝐶𝑜
Co = Conc. of the drug in plasma at zero time
Loading dose = Vd x Css The loading dose is the dose needed to reach a steady
Or = Vd x (C2 - C1) state
Css = Concentration of the drug in blood at a steady state
C1 = Concentration of the drug in plasma
C2 = Concentration of the drug needed to add to C1 to
reach required conc.

(Tss) (Tss) = 4.5 or 5 t1/2 (Tss) Time required to reach a steady state
t1/2 t1 0.693 t1/2 = the time required for the concentration of a
2
ke substance in the body to decrease by half.

ln C1 ln C2 Ke = elimination rate consistent


ke
t t
1 2

(0693 𝑥 𝑉𝑑 )
CLEARANC E Cl = 𝑇 1/2
rate of elimination
Cl =
drug conc
Cl = Ke x Vd
Cl = renal Cl + nonrenal Cls
𝐴𝑈𝐶
Bioavailability F= AUC = Area Under Curve
𝐶𝑂𝑁𝐶
𝐴𝑈𝐶 ( 𝐸𝑋𝐶𝑇𝑅𝐴𝑉𝐴𝑆𝐶𝑈𝐿𝐴𝑅 )
F= 𝑥 100
𝐴𝑈𝐶 ( 𝐼𝑁𝑇𝑅𝐴𝑉𝐴𝑆𝐶𝑈𝐿𝐴𝑅)

CrCl Cockcroft-Gault equation


140 patient’s age in years Body weight in kg
CrCl
72 serum creatinine in mg/dl
For females: CrCl = 0.85 X CrCl determined using the formula for males
TI The therapeutic index The minimum effective dose, ED50, is the dose that
𝑻𝑫𝟓𝟎 produces the therapeutic effect in 50% of the
=
𝑬𝑪𝟓𝟎 population.
The toxic dose, TD50, is the dose that is toxic to
50% of the population. This is used in human
studies.

Arif Pharma Clinic Copyright@gmail.com arifpharma.com


Page 80

1. A liquid medicine is supplied in a concentration of 20 mg/5 mL. A patient requires 40 mg orally


three times daily for 5 days, then 20 mg three times daily for 5 days, then 20 mg twice daily for
5 days, and then 20 mg once daily for 5 days. Which of the following is the volume of liquid
medicine that you will need to dispense?

A. 600 mL
B. 200mL
C.300mL
D. 60 mL
E. 30 mL
Rationale Each 5 mL of the liquid medicine contains 20 mg drug. Therefore, there is 4 mg of the
drug in 1 mL. The total volume to dispense can thus be calculated: 40 mg three times daily for 5
days: 10 mL × 3 × 5 = 150 mL 20 mg three times daily for 5 days: 5 mL × 3 × 5 = 75 mL 20 mg twice
daily for 5 days: 5 mL × 2 × 5 = 50 mL 20 mg once daily for 5 days: 5 mL × 1 × 5 = 25 mL 150 mL + 75
mL + 50 mL + 25 mL = 300 mL.

2. You are required to make 350 g of a paste that contains 15% w/w zinc oxide. Which of the
following is the amount of zinc oxide required?

A. 5.25 g
B. 52.50 g
C. 35.00 g
D. 3.50 g
E. 15.00 g
Rationale If the paste is 15% w/w, then 100 g paste contains 15.0 g zinc oxide. The total amount of
zinc oxide can thus be calculated: 350/100 = 3.50 15% × 3.5 = 52.50

3. A 1 in 10 000 solutions of potassium permanganate contains which of the following concentrations?

A. 50.0 mg potassium permanganate in 500 mL solution


B. 1.0 mg potassium permanganate in 100 mL solution
C. 5.0 mg potassium permanganate in 500 mL solution
D. 1.0 mg potassium permanganate in 1000 mL solution
E. 3.0 mg potassium permanganate in 300 mL solution

Rationale A If the solution is 1 in 10 000, there is 1.0 g in 10 000 mL solution In 500 mL, there is
(1/10 000) × 500 = 0.05 g = 50.0 mg In 100 mL, there is (1/10 000) × 100 = 0.01 g = 10.0 mg In 1000
mL, there is (1/10 000)
× 1000 = 0.10 g = 100.0 mg In 300 mL, there is (1/10 000) × 300 = 0.03 g = 30.0 mg. By showing that
each of the statements B–E is untrue for a 1 in 10 000 solution

Arif Pharma Clinic Copyright@gmail.com arifpharma.com


Page 81

4. Which of the following volumes of an adrenaline 1 in 100 solutions would be given by


intramuscular injection to a 2-year-old child for treatment of anaphylaxis if the dose were
120 micrograms stat

A. 12.00 mL
B. 120.00 mL
C. 0.12 mL
D. 24.0 mL
E. 0.24 mL
Rationale A 1 in 1000 solution contains 1.0 g, or 1000 000 micrograms, in 1000 mL. This means that
there are 1000 micrograms in 1 mL. The total volume of solution required can thus be calculated: (1
mL/1000 micrograms) × 120 micrograms = 0.12 mL. Therefore, 120 micrograms are contained in
0.12 mL.

5. Which of the following amounts of copper sulfate is required to make 400 mL of an aqueous
stock solution, such that, when the stock solution is diluted 50 times with water, a final solution
of 0.1% w/vcopper sulfate is produced?

A. 0.2 g
B. 20.0 g
C. 0.4 g
D. 40.0 g
E. 50.0 g
Rationale (B) If we work backward from the final solution, we have 0.1% w/v, which equates to 0.1
g copper sulfate in 100 mL solution. Multiplying by 50 gives the concentration of the original stock
solution, which is, therefore, 5% w/v. This equates to 5.0 g in 100 mL. As we start with 400 mL stock
solution, we need 5.0 g × 4,which is equal to 20.0 g copper sulfate

6. A child requires a single oral daily dose of 7.0 mg/kg body weight of drug A. The child’s weight
is 8.0 kg. Which of the following oral daily doses of drug A is received by this child?

A. 0.82 mg
B. 8.20 mg
C. 82.00 mg
D. 5.60 mg
E. 56.00mg
Rationale The oral daily dose required = (7.0 mg/kg) × (8.0 kg) = 56.0 mg

Arif Pharma Clinic Copyright@arifpharma.com arifpharma.com


Page 82

7. A patient in one of the residential homes to which you supply medication is going on holiday and
needs her prescriptions made up for the 5 days that she will be away. If she usually takes
ranitidine 150 mg twicedaily and atenolol 50 mg in the morning, which of the following
combinations of Zantac syrup (75 mg ranitidine/5 mL) and Tenormin syrup (25 mg atenolol/5 mL)
would you supply?

A. 50 mL Zantac syrup and 50 mL Tenormin syrup


B. 100 mL Zantac syrup and 50 mL Tenormin syrup
C. 50 mL Zantac syrup and 100 mL Tenormin syrup
D. 150 mL Zantac syrup and 50 mL Tenormin syrup
E. 75 mL Zantac syrup and 25 mL Tenormin syrup

Rationale Zantac syrup contains 75 mg ranitidine/5 mL. Her ranitidine dose is 150 mg twice daily.
Therefore, she requires 20 mL daily. Accordingly, for the 5 days of her holiday, she will need 20 mL ×
5 = 100 mL Zantac syrup. Tenormin syrup contains 25 mg atenolol/5 mL. Her atenolol dose is 50 mg
in the morning. Therefore, she requires 10 mL daily. Accordingly, for the 5 days of her holiday, she
will need 10 mL × 5 = 50 mL Tenormin syrup.As a result, you will supply 100 mL Zantac syrup and 50
mL Tenormin syrup.

8. Potassium permanganate solution 1 in 8000 is prepared from a stock of 10 times this strength.
How much potassium permanganate will be needed to make sufficient stock solution if a
patient uses 200 mLof the diluted solution each day for 20 days?

A. 100 mg
B. 125 mg
C. 250 mg
D. 400mg
E. 500mg

Rationale (E) Of the diluted solution 4000 mL will be used in 20 days. If this solution has been
prepared by a 10-fold dilution of the stock solution, the volume of the stock solution required must
be 400 mL. As the stock solution is a 1 in 800 solutions, there would be 1 g potassium permanganate
in 800 mL. In 400 mL there must be 0.5 g or 500 mg of potassium permanganate,

Arif Pharma Clinic Copyright@gmail.com arifpharma.com


Page 83

9. What volume of phenytoin suspension 30 mg/5 mL is required to be added to a suitable


diluent to obtain 150 mL phenytoin suspension 20 mg/5 mL?
A. 75 Ml
B. 100 mL
C. 120 mL
D. 125 mL
E. 130 mL
Rationale A 20 mg/5 mL suspension contains 600 mg in 150 mL. A 30 mg/5 mL suspension contains
600 mg in100 mL, which is the volume that must be added to the diluent.

10. Given a 20% w/v solution of chlorhexidine gluconate, what volume is required to make 400
mL of a 2%w/solution?

A. 40 mL
B. 20 mL
C. 80 mL
D. 2 mL
E. 4 mL

Rationale A 2% w/v solution contains 2 g in 100 mL or 8 g in 400 mL. A 20% w/v solution contains 20 g in
100 mL or 1 g in 5 mL. Therefore, 8 g is found in 40 mL, which is the volume required to make the diluted
solution.

Arif Pharma Clinic Copyright@arifpharma.com arifpharma.com


Page 84

11. Which of the following shows the correct amounts of sodium chloride and anhydrous glucose
present in 500 mL of intravenous infusion containing sodium chloride 0.18% w/v and anhydrous
glucose 4.00% w/v?

A. Sodium chloride 0.18 g and anhydrous glucose


20.00 g
B. Sodium chloride 0.90 g and anhydrous glucose
20.00 g
C. Sodium chloride 0.90 g and anhydrous glucose 40.00 g
D. Sodium chloride 1.80 g and anhydrous glucose 20.00 g
E. Sodium chloride 1.80 g and anhydrous glucose 40.00 g

Rationale ( B) 0.18% w/v is equivalent to 0.18 g sodium chloride in 100.00 mL, or 0.90 g in 500.0 mL
4.00% w/vis equivalent to 4.00 g anhydrous glucose in 100.00 mL or 20.00 g in 500.0 mL.

12. You have presented with a prescription for allopurinol tablets 100 mg at a dose of 300 mg
each day for 14 days, reduced to 200 mg for a further 7 days. How many packs of 28 tablets
should you supply?

A. Two
B. Three
C. One
D. Four
E. One and a half

Rationale A Three tablets per day for 14 days = 42 tablets Two tablets per day
for 7 days = 14 tablets: 14 + 42= 56. Therefore, two 28-packs are required,

13. An injection solution contains 0.5% w/v of the active ingredient. How much of the active
ingredient is needed to prepare 500 L of solution?
A. 0.25 kg
B. 0.50 kg
C. 1.00kg
D. D. 2.50 kg
E. 5.00 kg

Rationale (D) 0.5% w/v is equivalent to 0.5 g in 100.0 mL, or 5.0 g in 1000 mL or 1.0 L. Multiplying by
500 givesthe amount of active ingredient in 500.0 L, which is 2500.0 g or 2.50 kg.

Arif Pharma Clinic Copyright@gmail.com arifpharma.com


Page 85

14. A patient taking 10.0 mL Erythroped suspension (250 mg/5 mL) qid will receive
how much Erythromycine each day?

A. 2.0 g
B. 20.0 g
C. 4.0 g
D. 40.0 g
E. 2.5 g
Rationale A 250.0 mg × 2 × 4 = 2000.0 mg or 2.0 g.

15. In your pharmacy you have a stock solution of drug F with a concentration of 25% w/v. Drug F
is used as a mouthwash at a concentration of 0.25% w/v. You are requested to supply 50 mL of a
solution of intermediate strength, such that the patient will dilute this solution 1 in 20 to get the
correct concentrationimmediately before use. Which of the following should be the
concentration of the intermediate solution?

A. 5% w/v
B. 10% w/v
C. 2.5% w/v
D. 0.5% w/v
E. 15% w/v

Rationale A Drug F is used as a mouthwash at a concentration of 0.25% w/v. If this has been
prepared from a solution that has been diluted 1 in 20, multiplication by 20 gives the concentration
of the intermediate solution, which is, therefore, 5% w/v.

16. You receive a prescription for phenindione tablets 50 mg of with the following instructions:
‘200 mg on day1,100 mg on day 2 and then 50 mg daily thereafter. Mitte: 56 days’ supply. Which
of the following is the correct quantity to supply?

A. 60 tablets
B. 84 tablets
C. 56 tablets
D. 120 tablets
E. 90 tablets
Rationale (A)Day 1: 4 tablets/day Day 2: 2 tablets/day Days 3–56: 1 tablet/day: 4 + 2 + 54 = 60.

Arif Pharma Clinic Copyright@arifpharma.com arifpharma.com


Page 86

17. A patient has been prescribed a reduced oral dose of prednisolone as follows:

A. Day 1: 10 mg
B. Day 2: 8 mg
C. Day 3: 6 mg
D. Day 4: 4 mg
E. Day 5: 3 mg
F. Day 6: 2 mg
G. Day 7: 1 mg

Rationale ( A) Day 1: 10 mg = 2 × 5 mg tablets Day 2: 8 mg = 1 × 5 mg and 3 × 1 mg Day 3: 6 mg


= 1 × 5 mg and 1 × 1 mg Day 4: 4 mg = 4 × 1 mg Day 5: 3 mg = 3 × 1 mg Day 6: 2 mg
= 2 × 1 mg Day 7: 1 mg = 1 × 1 mg. Therefore, it would be most appropriate to supply
4 × 5 mg tablets and 14 × 1 mg tablets in total.

18. Stock solution of drug G is available at 10%w/v. You need to dilute this with Syrup, and BP in
order to supply a patient with a solution containing 5 mg/mL of drug G. Assuming no volume
displacement effects, what is your formula for the preparation of 100 mL of the final solution?

A. 10 mL stock solution and 90 mL Syrup, BP


B. 80 mL stock solution and 20 mL Syrup, BP
C. 20 mL stock solution and 80 mL Syrup, BP
D. 95 mL stock solution and 5 mL Syrup, BP
E. 5 mL stock solution and 95 mL Syrup, BP

Rationale ( E) The stock solution = 10% w/v = 10 g in 100 mL = 10 000 mg in 100 mL = 100 mg/mL.
The solution supplied to the patient is 5 mg/mL and so a 1 in 20 dilutions must be performed; 5 mL
of the stock solution should be diluted to 100 mL with 95 mL Syrup, BP.

19. A patient is on a continuous intravenous drip of drug B. He needs to be dosed at a rate of 25 mg/h. The
drip is set to administer 10 drops of fluid/h, with 4 drops equalling 1 mL in volume.
Which of the following is the concentration of drug B in the intravenous fluid?

A.1 mg/Ml
B. 10 mg/mL
C. 5 mg/mL
D. 2.5 mg/mL
E. 25 mg/mL
Rationle The drip rate = 10 drops/h and 4 drops = 1 mL. Therefore, 2.5 mL are delivered in 1 h. The
patient requires 25 mg/h, so there must be 25 mg in 2.5 mL or 10 mg in 1 mL

Arif Pharma Clinic Copyright@gmail.com arifpharma.com


Page 87

20. A patient requires an intravenous infusion of 0.9%w/v sodium chloride. In your hospital
pharmacy department, you have Water for Injections, BP, and 4.5%w/v Sodium Chloride
Solution, BP. Assuming no volume displacement effects, which of the following volumes of
4.5% w/v Sodium Chloride Solution, needs to be added aseptically to an expandable PVC
infusion bag containing 100 mL Water for Injections, BP to produce the requisite sodium
chloride concentration?

A. 25 mL
B. 50 mL
C. 100 mL
D. 125 mL
E. 150 mL

Rationale Use of simple algebra is required to answer this question. If we call the volume of 4.5%
w/v Sodium Chloride Solution, BP added y, and remember that the amount of sodium chloride in y
mL of a 4.5% sodium chloride solution is 0.045y, then we have (0.045y/[100 + y]) × 100 = 0.9.
Multiplying out we have: 4.5y/ (100 + y) = 0.9. This can be rearranged to: 4.5y = 90 + 0.9y.
Subtracting 0.9y from both sides gives 3.6y =
90. y = 25 mL. The correct answer is, therefore,

21. A patient needs to use a 1 in 2500 chlorhexidine gluconate solution for wound washing. In
your pharmacy, you have a stock solution of 20%w/v chlorhexidine gluconate. Using this
solution you need to prepare an intermediate solution such that the patient will then dilute
this 20-fold to obtain a solution of the requisite concentration. Which of the following is the
correct strength of the intermediate solution?

A. 0.5% w/w
B. 0.2% w/w
C. 1.0% w/w
D. 0.4% w/w
E. 0.8% w/w

Rationale Final solution = 1 in 2500 Intermediate solution = (final concn) × (dilution factor) = (1 in
2500) × (20) = 1 in 125 1 in 125 = 1 g in 125 mL = 0.8 g in 100 mL = 0.8% w/v.

Arif Pharma Clinic Copyright@arifpharma.com arifpharma.com


Page 88

22. Which of the following is the correct volume of a 5% w/v solution required to supply 150
mg of theactive redient?
A. 30 Ml
B. 20 mL
C. 3 mL
D. 2 mL
E. 5 mL
Rationale C For a solution of 5% w/v, the concentration can also be expressed as 5 g/100 mL = 5000
mg/100 mL = 50 mg/mL. We need 150 mg, so the correct volume is 3 mL

23. According to an official formula for potassium citrate mixture, 300 mL double-strength
chloroform water is required per 1 L mixture. A 2-L bottle of the mixture is required. If the
double-strength chloroform water is prepared from concentrated chloroform water, which of
the following is the correct volume of concentrate required? (Double-strength chloroform
water is 2 parts concentrate to 38 parts water)
A. 10 Ml
B. 20 mL
C. 3 mL
D. 30 mL
E.15 mL
Rationale D For the 2 L mixture we need 600 mL of double-strength chloroform water. Double
strength chloroformwater is 2 concentrated rate to 38 mL water. We can thus calculate the volume
of the concentrate required: (600 mL/40 parts) × 2 parts = 30 mL

24. Mrs. A is currently taking Mucogel suspension at a daily dose of 10 mL after three meals
and at bedtime. How much magnesium hydroxide will Mrs. A has taken after 5 days of
compliant use of Mucogel? (Mucogel contains magnesium hydroxide 195 mg and dried
aluminum hydroxide 220 mg/5 mL.)
A. 7.8 mg
B. 780 mg
C. 3900 mg
D. 5.85 g
E. 7.8 g
Rationale (E) Each dose of Mucogel suspension is 10 mL, so it contains 2 × 195 mg magnesium
hydroxide Eachday Mrs. A takes four doses, so she will have taken 20 doses in 5 days Therefore she
will have taken (20 × 2 × 195) mg = 7800 mg = 7.8 g

Arif Pharma Clinic Copyright@gmail.com arifpharma.com


Page 89

25. A 10-year-old boy (weighing 30 kg) has been prescribed Rimactane 150 mg capsules (rifampicin)
for themanagement of brucellosis at a dose of 10 mg/kg twice daily for 4 weeks. How many of
these capsules should be dispensed for this patient to cover the 4 weeks?
A. 108 capsules
B. 110 capsules
C. 112 capsules
D. 114 capsules
E. 116 capsules
Rationale 10 mg/kg twice daily for a 30-kg patient means that he will take (10 × 30 × 2) mg daily
After 4 weeks (28 days) the patient will have taken (10 × 30 × 2 × 28) mg = 16 800 mg Each capsule
contains 150 mg, so the patient will use (16 800/150) capsules in 4 weeks = 112

26. Mr. B has been started on Cellcept suspension (mycophenolate mofetil 1 g/5 mL when
reconstituted with water) after heart transplantation. He is taking the medicine at a dose of 1.5 g
twice daily. How many complete days of compliant therapy will each 175 mL bottle of
reconstituted Cellcept suspension provide him?
A- 11 days
B- 12 days
C- 17 days
D- 23 days
E- 58 day
Rationale (A) Each day the patient will use 1.5 g twice daily = 3 g Suspension strength is 1 g/5 mL,
so the patient will use (3 × 5) mL daily Each bottle contains (175/[3 × 5])
days = 11.66667, i.e. 11 complete days.

27. The suggested initial dose of haloperidol for schizophrenia in elderly people is half the adult dose,
which is 1.5–3.0 mg two to three times daily. A local GP has decided to prescribe haloperidol for
an elderly man following this guidance for 3 days initially. Which of the following is NOT an
appropriate dose for the GP to include on an otherwise legally written prescription?

A- Three haloperidol 500 micrograms capsules three times daily × 3 days


B- Two haloperidol 500 micrograms capsules three times daily × 3 days
C- Six haloperidol 500 micrograms capsules three times daily × 3 days
D- 0.5 mL haloperidol 2 mg/mL twice daily × 3 days
E- One haloperidol 1.5 mg tablet three times daily × 3 days

Arif Pharma Clinic Copyright@arifpharma.com arifpharma.com


Page 90

Rationale (C) Adult dose: 1.5–3 mg two to three times daily Elderly dose: 0.75–1.5 mg two to three
times dailyA: (3 × 500) micrograms three times daily = 1.5 mg three times daily – appropriate B: (2 ×
500) micrograms three times daily = 1 mg three times daily – appropriate C: (6 × 500) micrograms
three times daily = 3 mg three times daily – not appropriate D: (0.5 mL × 2 mg/mL) = 1 mg twice
daily – appropriate E: 1.5 mg three times daily – appropriate.

28. Arnold, a 5-year-old boy (weight 18 kg) with epilepsy, currently takes Epanutin suspension
(phenytoin 30 mg/5 mL) at a dose of 5 mg/kg twice daily. How many milliliters of Epanutin
suspension will Arnold take during the month of October? You can assume that he is fully
compliant and that no spillages or medication loss occurs during the month of October.
A- 155 Ml
B- 450 ML
C- 465 ML
D- 900 ML
E- 930 ML
Rationale (E) 5 mg/kg twice daily for an 18-kg child equates to (5 × 18 × 2) mg daily = 180 mg
daily Epanutin suspension is 30 mg/5 mL, so the patient needs (180/30 × 5) mL daily = 30 mL
daily October has 31 days, so in October, the patient will use (31 × 30) mL = 930 mL.

29. A patient weighing 70 kg is prescribed drug C to be given intravenously at a dose of 4 mg/kg per h. Drug
Cis is available as a 10 mg/2 mL intravenous solution. Which of the following is a suitable flow rate for
administering drug C to this patient?
A- 1 mL/min
B- 2 mL/min
C- 14 mL/h
D- 28 mL/h
E- 56 mL/h

Rationale ( E) Drug C dose: 4 mg/kg per h for 70 kg patient rate is 4 × 70 mg/h = 280 mg/h Drug C is
available as10 mg/2 mL solution, the so rate is (280/10) × 2 mL/h = 56 mL/h.

30. A 7-year-old girl has been discharged from the hospital on Fucidin suspension (fusidic acid 250
mg/5 mL) for a staphylococcal infection. The girl has to take 500 mg of fusidic acid three times a
day for 10 days. How much Fusidic acid will she have taken after these 10 days of treatment?
A- 0.015 kg
B- 1.5 g
C- 150 mg
D- 1500 mg
E- 150 000 micrograms

Arif Pharma Clinic Copyright@gmail.com arifpharma.com


Page 91

Rationale (A) Fusidic acid dose: 500 mg three times daily × 10 days = 15 000 mg = 15 g = 0.015 kg.

31. Drug D has been prescribed for a 5-month-old baby with a body surface area of 0.4 m2. Drug D
should be given as a daily dose of 200 micrograms/m2 in two divided doses. Drug D is available
as an oral liquid with a concentration of 0.1 mg/mL. Which of the following is an appropriate
single dose for this baby?
A- 0.4 Ml
B- 0.8 mL
C- 4 mL
D- 8 mL
E- 80 mL
Rationale (A) Dose: 200 micrograms/m2 daily in two doses, so a single dose of 100 micrograms/m2.
For this patient this equates to 100 × 0.4 micrograms = 40 micrograms = 0.04 mg Drug D liquid
concentration is 0.1 mg/mL, so 0.04/0.1 mL = 0.4 mL.

32. Emma, a 10-kg 1-year-old girl, is to be administered drug M as an intravenous (IV) infusion 2
hours before surgery at a dose of 7.5 mg/kg. Drug M is available as a 5 mg/mL intravenous
infusion and should be administered at a rate of 5 mL/min. How long should Emma’s IV
infusion last?

A- 1.5 min
B- 3 min
C- 15 min
D- 150 s
E- 300 s
Rationale (B) Drug M dose is 7.5 mg/kg, so for a 10-kg patient dose this is 75 mg
Drug M is available as 5 mg/mL, So 75/5 mL = 15 mL Rate: 5 mL/min, so 15/5 min = 3 min.

33. A patient is to be administered 300 mg of fosphenytoin sodium by intravenous infusion. This


drug is available in a 10 mL vial of fosphenytoin sodium at a concentration of 75 mg/mL (Pro-
Epanutin), which is to be diluted to 25 mg/mL strength using glucose 5% before it can be
administered to a patient. How much fosphenytoin sodium and 5% glucose need to be used to
administer the correct dose to this patient?

A- 4 mL Pro-Epanutin made up to 12 mL with 5% glucose


B- 4 mL Pro-Epanutin and 12 mL 5% glucose
C- 10 mL Pro-Epanutin made up to 12 mL with 5% glucose
D- 4 mL Pro-Epanutin and 4 mL 5% glucose
E- 1 mL Pro-Epanutin made up to 4 mL with 5% glucose

Arif Pharma Clinic Copyright@arifpharma.com arifpharma.com


Page 92

Rationale A Vial concentration is 75 mg/mL and 10 mL volume, so each vial contains 750 mg /10
mL, so 75 mg in 1 mL, and 300 mg in 4 mL of 75 mg/mL Needs to be diluted to 25 mg/mL before
use, so a 1 in 3 dilutidilutions3 dilution using 4 mL of original means total volume =12 mL

34. A patient is to be administered 75 mg meptazinol hydrochloride intramuscularly every 4 hours


when required for severe pain. The nursing staff who will administer this opioid analgesic are
using U100 (100 units) insulin 1 mL syringes with needles. How many units should be
administered to this patient each timeof administration, given that meptazinol is manufactured
as 1 mL ampoules of strength 100 mg/mL (as hydrochloride)?
A- 7.5 units
B- 75 units
C- 50 units
D- 5 units
E- 150 units
Rationale (B)100 mg/mL, so 75 mg in 0.75 mL to be administered U100 syringes, so 100 units/mL,
so 0.75 mL delivered from (0.75 × 100) units = 75 unit

35. The recommended dosage for drug H is 3–5 mg/kg per day in four divided doses for children
between the ages of 1 and 5 years. Which of the following is a suitable dosage regimen for
drug H when being administered to an 18-month-old patient weighing 18 kg?
A- 10 mg four times daily
B- 10 mg twice daily
C- 15 mg four times daily
D- 25 mg four times daily
E- 54 mg four times daily

Rationale (C) A Has to be four times the daily dosage interval, so B is not a suitable answer straight
away Baby weighs 18 kg, so recommended dosage is (3 × 18)–(5 × 18) daily in four divided doses =
54–90 mg daily in four divided doses = 13.5–22.5 mg four times daily, so 15 mg four times daily
suitable.

Arif Pharma Clinic Copyright@gmail.com arifpharma.com


Page 93

36. A patient weighs 14 kg and requires drug A at a dose of 5 mg/kg per day. What is the total
daily dose for this patient?

A- 70 000 micrograms
B- 70 000 mg
C- 0.007 g
D- 70 g
E- 0.007 kg
Rationale 5 mg/kg per day for a patient weighing 14 kg = 5 × 14 mg/day =70 mg daily
= 70 000 micrograms.

37. Breda is 9 years of age and weighs 27 kg. She has been prescribed a suspension of drug C of
strength 40mg/5 mL at a dose of 4 mg/kg daily in three divided doses. How much suspension
should Breda’s mum give her for each dose?
A- 1.5 Ml
B- 4.5 mL
C- 5 mL
D- 6.75 mL
E- 13.5 mL
Rationale (B) Dose: 4 mg/kg daily in three divided doses. For this patient this means 4 × 27 mg daily
in three divided doses, so each individual dose is 36 mg Suspension strength is 40 mg/5 mL; 1 mg in
5/40 mL; 36 mg in (5 × 36/40) mL = 4.5 mL.

38. A patient weighing 80 kg requires an oral daily dose of 12 mg/kg of drug D for 14 days. Drug D
is available only as a suspension of 30 mg/2 mL. How much suspension will this person use
during this course of treatment? (You can assume that the patient is fully compliant and no
spillage or loss in any other way occurs.)
A- 32 Ml
B- 64 mL
C- 160 mL
D- 448 mL
E- 896 mL
Rationale Oral daily dose: 12 mg/kg for 14 days for 80-kg patient; this is 12 × 80 mg daily = 960 mg
daily Therefore over 14 days will use 960 × 14 mg = 13 440 mg Drug D suspension is 30 mg/2 mL, so
1 mg in (2/30)mL and 13 440 mg in (2 × 13 440/30) mL = 896 mL.

Arif Pharma Clinic Copyright@arifpharma.com arifpharma.com


Page 94

39. How much of a 4 mg/mL suspension would you supply to a patient who required 12 mg four
times dailyfor 30 days? (You can assume that no overage is supplied)
A- 0.036 L
B- 0.6 L
C- 90 mL
D- 360 Ml
E- 480 mL
Rationale ( D) 4 mg/mL suspension, so 12 mg in 3 mL Therefore each dose is 3 mL four times
daily Daily going

40. A junior house officer (JHO) asks for your advice about setting up an intravenous infusion of
dopexamine hydrochloride for a male patient, weighing 80 kg, in the cardiac ward. The JHO
wishes to administer the drug at a dose of 500 ng/kg per min. The drug is formulated as a
strong 10 mg/mL sterile solution but needs to be diluted to a concentration of 400
micrograms/mL with 5% glucose before intravenous administration can occur. Which of the
following is an appropriate administration flow rate for you to advise the JHO?
A- 0.1 mL/min
B- 1.25 mL/min
C- 4 mL/min
D- 8 mL/min
E- 0.1 L/min
Rationale (A) Dose: 500 ng/kg per min for an 80-kg patient means 500 × 80 ng/min = 40 000 ng/min
= 40 micrograms/min Administered at concentration of 400 micrograms/mL, so 40 micrograms in
0.1 mL Therefore, the IV rate should be 0.1 mL/min.

41. Drug E is available as a 5%w/v solution. Fiona, who weighs 6.25 kg, is to be given drug E at
a doseof 6mg/kg twice daily for 5 days. What volume of this solution of drug E should
Fiona be given daily?
A- 0.15 Ml
B- 0.75 mL
C- 1.5 Ml
D- 3.75 mL
E- 7.5 mL
Rationale Dose: 6 mg/kg as Fiona weighs 6.25 kg will be given (6 × 6.25) mg for each dose = 37.5 mg
Daily will receive 37.5 mg twice daily = 75 mg Solution concentration: 5% w/v, so 5 g drug E in 100
mL solution Therefore 1 g in 20 mL = 1000 mg in 20 mL = 1 mg in (20/1000) mL 75 mg required daily
which is in (75 × 20/1000) mL = 1.5 mL.

Arif Pharma Clinic Copyright@gmail.com arifpharma.com


Page 95

42. A 2-month-old baby needs to be given fluconazole for a candidal infection. The standard
dose of fluconazole for this indication is 3 mg/kg on the first day, then 3 mg/kg (max 100
mg) daily for 7 days. However, if the child has renal impairment, the following advice needs
to be heeded: usual initial dose, then halve subsequent doses if creatinine clearance < 50
mL/min per 1.73 m2. How much fluconazole will a 4.5 kg(0.28 m2) baby have been
administered after this course of therapy if the creatinine clearance is 5 mL/min?

A- 48.75 mg
B- 54 mg
C- 60.75 mg
D- 94.5 mg
E- 108 mg
Rationale (C) First of all we need to compare the renal function of this child against the limit for
changing the dose of the medication. The limit is 50 mL/min per 1.73 m2 For this child: he has a
clearance of 5 mL/min, which is per 0.28 m2 because we are told that this is the surface area for the
child Therefore, it is 5 mL/min per 0.28 m2, which equates to (5/0.28) × 1.73 mL/min per m2 = 30.89
mL/min per 1.73 m2 This is below the limit of 50 mL/min per 1.73 m2, so the child will receive the
drug following the alternative dosing regimen Dose received: initial dose of 3 mg/kg on day 1, then
1.5 mg/kg daily for 7 days For 4.5-kg patient = (3 × 4.5) + (1.5 × 4.5 × 7) mg = 60.75 mg.

43. A syringe driver contains 15 mL diamorphine hydrochloride 4 mg/mL solution. The length
of the syringe driver is 60 mm. What rate should the syringe driver be set at, so that the
patient receives 5 mg/h of diamorphine hydrochloride?
A- 3 mm/h
B- 4 mm/h
C- 5 mm/h
D- 6 mm/h
E- 7 mm/h
Rationale (C) Solution concentration is 4 mg/mL; 15 mL of this solution contains 4 × 15 mg = 60 mg
Syringe driver length is 60 mm, so going to deliver 60 mg over the length of 60 mm = 1 mg/mm Rate
required is 5 mg/h; this equates to 5 mm/h.

Arif Pharma Clinic Copyright@arifpharma.com arifpharma.com


Page 96

44. A 3-year-old girl has been prescribed ganciclovir (as the sodium salt) as maintenance therapy
at a doseof 6mg/kg daily for 5 days/week until she has an adequate recovery of immunity
after transplantation. She weighs 12 kg and has a normal renal function. For administration as
an intravenous infusion the ganciclovir (as the sodium salt) powder is reconstituted with water
for injections (500 mg/10 mL), then diluted to a concentration of 5 mg/mL with 0.9% sodium
chloride, and the infusion is then given over 1 hour. What flow rate is appropriate for this
patient and how much ganciclovir (as the sodium salt) will she have been administered after 1
week?

A- 0.24 mL/min and 360 mg


B- 7.2 mL/min and 504 mg
C- 18 mL/min and 360 mg
D- 30 mL/min and 360 mg
E- 30 mL/min and 504 mg

Rationale (A) Dose of 6 mg/kg daily for a 12-kg patient – 72 mg daily Rate of 72 mg/h = 72/60
mg/min = 1.2 mg/min Strength of 5 mg/mL = 1 mg in 1/5 mL or 0.2 mL, so 1.2 mg is in 1.2 × 0.2
mL/min = 0.24 mL/min Total amount in 1 week = 72 × 5 mg; as only 5 days admin given = 360 mg.

45. A patient is administered potassium chloride as a slow infusion over 150 min at a rate
of 0.1 mmol potassium/kg per h. If 15 mmol potassium is delivered during the infusion
what weight is thepatient?
A- 50 kg
B- 55 kg
C- 60 kg
D- 65 kg
E- 75 kg
Rationale ( C) 0.1 mmol/kg per h is the rate used. 150 min = 2.5 h Making the patient weight = x kg
0.1 × x × 2.5 = 15 0.25x = 15 x = 60 kg.

46. A 9-year-old patient with asthma is being transferred from terbutaline sulfate 1.5 mg/5 mL
syrup to terbutaline sulfate 5 mg tablets. The patient is currently taking 8 mL of syrup three
times a day. Which of the following is the most appropriate dosage as tablets for this
patient? The tablets are scored and not coated.

Arif Pharma Clinic Copyright@gmail.com arifpharma.com


Page 97

A- Half a tablet three times daily


B- Half a tablet twice daily
C- One tablet daily
D- One tablet three times daily
E- One tablet twice daily

Rationale ( A) 8 mL of 1.5 mg/5 mL contains 8 × 1.5/5 mg = 2.4 mg Daily dose is therefore 2.4 mg
three timesdaily 2.5 mg (1/2 tablet) three times daily is the closest dose of those suggested.

47. Miss A is given an intravenous dose of drug B and her peak serum level is found to be 20
mg/L. Given that 18 hours later her serum concentration is 2.5 mg/L, which of the
following is the elimination half-lifeof the drug Bin this patient? (You may assume that the
distribution is complete and that the elimination is described by a first-order process.)
A- 2 h
B- 4 h
C- 6 h+
D- 8 h
E- 10 h
Rationale (C) Peak is 20 mg/L, so after one half-life (t1 /2 ) concentration will be 10 mg/L, after
another reduces to 5 mg/L, and then after a third t1 /2 reduces to a serum concentration of 2.5
mg/L Therefore, 3 × t1/2 = 18 h And each t1 /2 = 6 h

48. Brian, aged 76 and weighing 72 kg, requires a loading dose of drug C. Which of the following
is the most suitable intravenous loading dose of drug C for Brian? (Volume of distribution
[VD] = 4 L/kg; therapeutic range3–5 ng/mL)
A- 500 micrograms
B- 750 micrograms
C- 1000 micrograms
D- 1250 micrograms
E- 1500 micrograms
Rationale Volume of distribution (VD) = 4 L/kg, so for a 72-kg patient = 4 × 72 L = 288 L Therapeutic
range = 3–4 ng/mL, so for a patient with a VD of 288 L want to have a dose of (3 × 288 × 1000)–(4
× 288 × 1000) ng = 864 000–1152 000 ng = 864–1152 micrograms.
49. A 54-year-old male patient with a body weight of 65 kg requires an oral loading dose of digoxin.
You know that the volume of distribution (VD) per kg body weight is 6 L, the salt factor (S) = 1,
and the bioavailability (F) = 0.7. What dose should be used to achieve a target plasma
concentration (Cp) of 1.25 micrograms/L?
Loading dose = (VD × Cp)/ (S × F)

Arif Pharma Clinic Copyright@arifpharma.com arifpharma.com


Page 98

A- 500 micrograms
B- 600 micrograms
C- 700 micrograms
D. 800 micrograms
E. 900 micrograms

Rationale (C) Loading dose (micrograms) = (6 L × 65 × 1.25 micrograms/L)/(1 × 0.7)


≈ 696.43 micrograms ≈ 700 micrograms.

50. A patient is given an intravenous dose of drug H and her peak serum level is found to
be 12 micrograms/mL; 12 hours later her serum concentration is 0.75 mg/L. What is the
elimination half-life (t1/2 ) of drug H in this patient? (You may assume that the
distribution is complete and that the elimination isdescribed by a first-order process.)
A- 2 h
B- 3 h
C- 4 h
D- 5
E- 6 h

Rationale (B)Peak serum level = 12 micrograms/mL After 12 h, serum level = 0.75 mg/L First
of all need to have both concentrations in the same units Peak serum level = 12
micrograms/mL After 12 h, serum level = 0.75/1000 mg/mL = 0.00075 mg/mL = 0.75
micrograms/mL 12 → 6 → 3 → 1.5 → 0.75 Therefore, four half-lives have passed for serum
concentration to reduce from 12 to 0.75 micrograms/mL These four half-lives have taken 12
h, so each t1 /2 lasts 3 h.

51. A 54-year-old male patient weighing 65 kg requires a loading dose of drug K to treat an infection.
Which is a suitable intravenous loading dose of drug K for this patient to achieve a concentration
between 7 and 9 mg/L? (Drug K volume of distribution [VD] = 0.25 L/kg.)
A- 75 mg
B- 100 mg
C- 125 mg
D- 150 mg
E- 175 mg
Rationale Drug K is being administered intravenously, so the bioavailability and salt factor of the
drug do not have to be considered. The volume of distribution = 0.25 L/kg; for this patient weighing
65 kg this equates to 0.25× 65 L = 16.25 L Target concentration = 7–9 mg/L As VD = 16.25 L, the

Arif Pharma Clinic Copyright@gmail.com arifpharma.com


Page 99

target concentration range is (7 × 16.25)–(9 × 16.25) mg = 113.75–146.25 mg 125 mg is the only


suggested answer within this range.

52. Doreen has been given a 600 mg dose of drug E while in the hospital. The elimination half-life
of drug E IS 8 and it follows first-order kinetics. How much of this drug will remain in her system 48
h after administration, assuming that complete absorption and distribution have occurred?
A- 937.5 × 10–10 kg
B- 937.5 × 10–9 kg
C- 937.5 × 10–8 kg
D- 937.5 × 10–7 kg
E- 937.5 × 10–6 kg

Rationale ( C) 48 h/8 h = 6, so six half-lives have occurred during


48 h: 600 mg → 300 mg → 150 mg → 75 mg → 37.5 mg → 18.75 mg → 9.375 mg
After 48 h the amount of drug E left is 9.375 mg = 0.009375 g
= 0.000009375 kg = 937.5 × 10–8 kg.

53. Mrs. K is to be given gentamicin for an infection and the target concentration that you, as the independent
prescriber, want to achieve is 8 mg/L. Mrs. K weighs 65 kg. Given the information below, what is a suitable
loading dose for you to prescribe? Loading dose = (VD × Cp)/(S × F) Volume of distribution (VD) = 0.25 L/kg
Bioavailability (F) = 1 Salt factor (S) = 1

A- 120 mg
B- 130 Mg
C- 140 mg
D- 150 mg
E- 160 mg
Rationale (B) Loading dose (mg) = (0.25 × 65 × 8)/(1 × 1) = 130 mg gentamicin.

54. Arnold has been started on a new drug to control his recent respiratory symptoms.
This drug is completely really excreted from the body. He is given a dose of 250 mg and,
When his urine is analyzed, it is found that 220 mg of the drug is present in the urine.
What is the best estimate of the bioavailability (F) of this drug given the information below?
Amount of drug reaching systemic circulation = F × S × dose
A. 0.79
B. 0.88
C. 0.97
D. 1.06
E. 1.14

Arif Pharma Clinic Copyright@arifpharma.com arifpharma.com


Page 100

Rationale (B) If the drug is completely eliminated by the kidneys will assume that 220 mg is the amount
of the 250 mg dose that reaches the systemic circulation Amount of drug reaching systemic circulation
= F × S × dose 220 mg = F × 1 × 250 mg F = 220/250 = 0.88.

55. While in the hospital a patient has been receiving ranitidine as an intravenous injection of
50 mg every 6h. This patient is being discharged from the hospital and the doctor has contacted
you in the pharmacy to recommend a bioequivalent dose of ranitidine 100 mg/5 mL oral solution.
Knowing that the bioavailability of ranitidine oral solution is 50%, which of the following is a
suitable dose for you to recommend.

A- 2 mL twice daily
B- 5 mL twice daily
C- 10 mL twice daily
D- 15 mL twice daily
E- 20 mL twice daily

Rationale (C) When ranitidine is given as an intravenous injection it is 100% bioavailable:


Intravenous dose: 50 mg every 6 h So 200 mg bioavailable within 24 h Therefore want 200 mg to be
bioavailable from the oral dose Bioavailability of oral solution is 50%, so need to take (200/50%) mg
= 400 mg Oral solution concentration is 100 mg/5 mL, so 400 mg available in 20 Ml

56. A manufacturer wishes to produce a batch of compressed tablets each containing 800 mg
active ingredient, with a mean table weight of 1.2 g. Which of the following is the weight of
the active ingredientthat will be required for a total batch size of 720 kg?

A- 600 kg
B- 400 kg
C- 250 kg
D- 480 kg
E- 420 kg
Rationale (D) Every 1200 mg tablet contains 800 mg active drug, so 800/1200 = 2/3 of each
tablet is a drug Accordingly, two-thirds of the batch size of 720 kg must be a drug. 720/3 = 240 240 × 2
= 480 kg

57. Your hospital pharmacy department has been asked to supply a 20 cm2 bioadhesive patch
containing 50 mg/cm2 of 5-aminolevulinic hydrochloride (ALA) for use in a clinical trial on a
named-patient basis. You know that 30 g aqueous gel containing ALA is required to prepare a
patch of this drug loading 100 cm2 in area. Which of the following is the concentration of ALA in
an aqueous gel used to prepare the 20 cm2 patch with an ALA loading of 50 mg/cm2?

Arif Pharma Clinic Copyright@gmail.com arifpharma.com


Page 101

A- 1000 mg ALA in 6 g gel


B- 500 mg ALA in 6 g gel
C- 50 mg ALA in 20 g gel
D- 50 mg ALA in 30 g gel
E- 2000 mg ALA in 6 g gel

Rationale A If the drug loading is 50 mg/cm2 and the patch area is 20 cm2, then 1000 mg ALA is
required If 30 g gel is required to make a patch 100 cm2 in area, then 6 g gel is required for a 20 cm
patch.

58. You have in your pharmacy an unopened 25 g tube of Metvix cream (16% w/w methylamino
levulinate). Which of the following is the amount of compatible diluent cream required todilute this
25 g cream to a level of 4% methyl amino levulinate?

A. 25 g
B. 50 g
C. 75 g
D. 100 g
E. 125 g

Rationale (C)Dilution factor = (initial concern)/(final concern) = (16% w/w)/(4% w/w) = 4 Therefore,
the original cream needs to be diluted 1 in 4, i.e. 1 part original cream and 3 parts diluent cream
Quantity of diluent cream required = 3 × 25 g = 75 g. ed. The correct answer is, therefore, A

59. A patient has been prescribed 60 g of 0.2% w/w glyceryl trinitrate ointment for an anal fissure. The
only strength glyceryl trinitrate ointment that you have available is 0.3% w/w. Which of the following is
the amount of the 0.3% w/w glyceryl trinitrate ointment that you would need to prepare the required
product?

A- 40 g
B- 50 g
C- 30 g
D- 20 g
E- 15 g

Arif Pharma Clinic Copyright@arifpharma.com arifpharma.com


Page 102

Rationale A 60 g of a 0.2% w/w ointment contains: (60/100) × 0.2 = 0.12 g glyceryl trinitrate The
0.3% w/w ointment contains 0.3 g glyceryl trinitrate in 100 g, or 0.03 g in 10 g 0.03 g × 4 = 0.12 g
Accordingly, 40 g of the 0.3% w/w ointment will be required.

60. Which of the following is the volume of Molipaxin liquid (trazodone hydrochloride 50 mg/5 mL)
required to be added to a suitable diluent to obtain 100 mL trazodone hydrochloride liquid 10 mg/5
mL?

A- 60 mL
B- 70 Ml
C- 50 mL
D- 20 mL
D. 40 mL

Rationale (D)100 mL of a 10 mg/5 mL syrup contains 200 mg trazodone hydrochloride Molipaxin


liquid contains 50 mg/5 mL of trazodone hydrochloride 200 mg/50 mg = 4 Hence, 4 × 5 mL aliquots
are required = 20 mL

61. Which of the following amounts of white soft paraffin is required to make 250 g of the product
below? Zinc oxide 12% w/w Salicylic acid 1% w/w Starch 15% w/w White soft paraffin to 100% w/w

A- 70 g
B- 100 g
C- 150g
D.180 g
E. 200 g

Rationale (D) First, add up all the other ingredients. This comes to 28%. Therefore, white soft paraffin
must constitute 72% of the final product. 250/100 × 72 = 180 g.

62. You are requested to supply 250 g of 1 in 4 dilutions of Eumovate (0.05% w/w clobetasone
butyrate) cream in aqueous cream. Which of the following are the correct amounts of the two
creams that you would need?
A- 112.5 g Eumovate plus 112.5 g aqueous cream
B- 75 g Eumovate plus 175 g aqueous cream
D- 50 g Eumovate plus 200 g aqueous cream

Arif Pharma Clinic Copyright@gmail.com arifpharma.com


Page 103

E- 100 g Eumovate plus 150 g aqueous cream


F- 62.5 g Eumovate plus 187.5 g aqueous cream

Rationale ( E) The concentration of clobetasone butyrate in the cream is irrelevant here. First, split 250 g
intofour parts, each of 62.5 g. One part (62.5 g) will be Eumovate and the other three parts (187.5 g) will
be aqueous cream.
63. Which of the following is the percentage strength (w/w) of a saturated aqueous solution of sodium
bicarbonate if it requires 20 mL water to just dissolve 2 g?

A- 4.54%
B- 9.09%
C- 10%
D- 10.9%
E- 11.1%
Rationale 2 g added to 20 mL water gives a solution of 22 g, as 1 mL water weighs 1 g (2/22) × 100 =
9.09% w/w.

64. Which of the following is the correct number of allopurinol 300 mg tablets required when preparing
the following prescription for a gout patient who is currently unable to swallow tablets? Allopurinol 50
mg/mL Cherry syrup ad 150 mL Signa (label) 10 mL in the morning after food
A. 20
B- 25
C- 10
D- 15
E- 40
Rationale Each 1 mL of the suspension contains 50 mg of allopurinol In 150 mL there is 50 mg × 150 =
7500 mgEach tablet contains 300 mg, so: 7500/300 = 25 tablets are required.

65. A manufacturer wishes to produce a batch of methylene blue gel for photodynamic antimicrobial
chemotherapy of wound infections. The gel contains 20% w/w methylene blue. Which of the following isthe
weight of methylene blue that will be required for a total batch size of 360 kg?
A- 36 kg
B- 72 kg
C- 50 kg
D- 20 kg
E- 80 kg
Rationale ( B) Every 100 kg of gel contains 20 kg methylene blue, so (20/100) × 360 = 72 kg.

Arif Pharma Clinic Copyright@arifpharma.com arifpharma.com


Page 104

66. Which of the following are the correct amounts of diclofenac sodium and misoprostol required
to prepare 25 tablets according to the formula for an individual tablet given below? Diclofenac
sodium 50 mgMisoprostol 200 micrograms Lactose q.s. (sufficient quantity)

A. 1250 mg diclofenac sodium and 50 mg misoprostol


B. 1250 mg diclofenac sodium and 0.5 mg misoprostol
C. 250 mg diclofenac sodium and 5 mg misoprostol
D. 125 mg diclofenac sodium and 0.5 mg misoprostol
E. 1250 mg diclofenac sodium and 5 mg misoprostol

Rationale (E) This is simply a case of multiplication, as follows: Diclofenac sodium: 50 mg × 25 =


1250 mg Misoprostol: 200 micrograms = 0.2 mg; 0.2 mg × 25 = 5 mg

67. Which of the following is the amount of erythromycin ethyl succinate in 60 mL of a 500 mg/5
mL oral liquid?

A- 3 g
B- 12
C- 6g
D. 60 g
E. 30 g
Rationale (C) The liquid contains 500 mg in 5 mL Multiplying by 12 gives the amount in 60 mL, which
is 6000 mg or 6 g.

68. You are asked to prepare and dispense six 4 g vaginal pessaries, each containing 500 mg
clotrimazole. Allowing for a 50% excess (i.e. calculating on the basis that a total of nine pessaries will
be prepared), which of the following are the amounts of suppository base and clotrimazole that will be
required for the correct formulation of these pessaries? The displacement value of clotrimazole is 1.5.

A- 0.5 g clotrimazole and 6.0 g pessary base


B- 4.5 g clotrimazole and 24 g pessary base
C- 3 g clotrimazole and 33 g pessary base
D- 3 g clotrimazole and 24 g pessary base
E- 4.5 g clotrimazole and 33 g pessary base

Arif Pharma Clinic Copyright@gmail.com arifpharma.com


Page 105

Rationale (E)A displacement value (DV) of 1.5 means that 1.5 g of the drug will displace 1 g pessary
base Clotrimazole has a DV of 1.5, i.e. 1.5 g clotrimazole displaces 1 g suppository base Amount of
base required =theoretical amount – displaced amount = (9 × 4 g) – (amount of drug/DV of drug) = (9
× 4 g) – (4.5 g/1.5) = 36.0 g – 3.0 g = 33.0 g Final formulation is: clotrimazole 4.5 g, pessary base 33.0 g.

69. Which of the following is the correct number of spironolactone 25 mg tablets required to
prepare 200 ML of a pediatric oral suspension containing 5 mg/mL spironolactone?

A- 40
B- 20
C- 30
D- 10
E- 50
F-
Rationale A 200 mL of a 5 mg/mL suspension contains 5 × 200 = 1000 mg spironolactone 1000/25 =
40 tablets required. The correct answer is A.

70. A rectal gel is prepared according to the formulation below: Lorazepam 80 mg Methylcellulose
2.5 gMethylparaben 100 mg Glycerol 5 g Water to 100 mL Which of the following is the
number of 1 mL ampoules of Ativan injection (lorazepam 4 mg/mL) required to prepare 25 g of
this gel correctly?

A- 2
B- 3
C- 4
D- 5
E. 6
Rationale (D) If there are 80 mg lorazepam in 100 mL gel, there must be 20 mg in 25 mL Each vial of
Ativan contains 4 mg lorazepam, so five vials are required.

71. Which of the following is the volume of Water for Injection, BP to be added to a vial containing
500 mgamoxicillin to produce a solution with a volume of 5 ML. The displacement volume of
amoxicillin is 0.1 mL for 125 mg.

A. 4.6 mL
B. 3.0 Ml

Arif Pharma Clinic Copyright@arifpharma.com arifpharma.com


Page 106

C. 3.6 mL
D. 5.0 mL
E. 4.5 M

Rationale (A) 125 mg amoxicillin displaces 0.1 mL, so 500 mg must displace 0.4 mL 5 mL – 0.4 mL = 4.6
mL water must be added.

72. Which of the following is the amount of codeine hydrochloride (solubility 1 in 20 of water)
that will dissolve in 150 mL water?

A- 15.5 g
B-8.5g
C.7.5 g
D. 5.5 g
E. 4.5 g

Rationale (C) This is a case of simple multiplication and division: 1 g will dissolve in 20 mL 150
ML/20 = 7.5 g will dissolve in 150 mL

73. Ergometrine maleate has a solubility of 1 in 40 in water. Which of the following is the amount of
water required to dissolve 2.5 g ergometrine maleate?

A- 25 mL
B- 80 mL
C- 50 Ml
D- 100 mL
E- 125 mL

Rationale ( D) This is, again, a case of simple multiplication and division: 1 g will dissolve in 40 mL
Multiplying by 2.5 tells us that 2.5 g will dissolve in 100 ML

74. The formula for a single powder for oral use is given as Aspirin 600 mg Aloxiprin 150 mg Caffeine
250 mg Which of the following is the concentration of caffeine in this powder?

A. 25.0% w/w
B. 12.5% w/w
C. 15.0% w/w
D. 10.0% w/w
E. 20.0% w/w

Arif Pharma Clinic Copyright@gmail.com arifpharma.com


Page 107

Rationale ( A) The ingredients add up to 1000 mg, of which 250 mg is caffeine, so the caffeine
concentration must be 25% w/w.

75. Which of the following is the volume of a 4 mg/mL suspension of drug G that you would
prepare for a patient who required 14 mg twice daily for 7 days?
A- 14 mL
B- 28 Ml
C- 50 mL

D- 49 mL
E. 15 mL
Rationale (D) We require 14 mg per dose, supplied as 4 mg/mL Volume per dose = (14 mg)/(4 mg/mL)
= 3.5 mL Total volume required = volume per dose × frequency × duration = 3.5 mL × 2 × 7 = 49 ML.

76. You add 90 mL of 4.5% w/v sodium chloride solution to a 500 mL infusion bag of 0.9%w/v
sodium chloride solution to obtain the correct level of NaCl for infusion to a patient. Which of
the following is thefinal concentration of the solution? Assume no volume displacement effects.

A. 1.45% w/v
B. 14.50% w/v
C. 4.50% w/v
D. 4.00% w/v
E. 1.00% w/v

Rationale (A) Final solution = 90 mL 4.5% sodium chloride solution + 500 mL 0.9%w/v sodium chloride
solution = 590 mL Sodium chloride from each solution: 0.9% w/v sodium chloride = 0.9 g in 100 mL =
(0.9 × 500)/100 = 4.5 g sodium chloride in 500 mL 4.5% w/v sodium chloride = 4.5 g in 100 mL = (4.5 ×
90)/100 = 4.05 g sodium chloride in 90 mL Total sodium chloride = 4.5 + 4.05 = 8.55 g Therefore, the
final concentration = 8.55 g in 590 mL = 1.45 g in 100 mL = 1.45% w/v.

77. The manufacturer of ferrous gluconate cannot supply it due to a fire in the main manufacturing plant.
Your patient normally takes two tablets once daily. They are now being given Plesmet syrup as an
alternative. They need to receive exactly the same amount of iron. Which of the following would be an
appropriate dose of this syrup? Ferrous gluconate tablets contain 35 mg iron per tablet. Plesmet syrup
contains 25 mg iron/5 mL.

A- 5 mL three times daily


B- 6 mL twice daily
C- 7 mL once daily
D-7 mL twice daily

Arif Pharma Clinic Copyright@arifpharma.com arifpharma.com


Page 108

E- 5 mL once daily

Rationale (D) Ferrous gluconate tablets contain 35 mg iron/tablet, so the total daily dose is 70 mg
Plesmet syrup contains 25 mg iron/5 mL = 5 mg/mL 70/5 = 14 mL or 7 mL twice daily.

78. How much active substance is required to manufacture a batch of granules for a compressed tablet
with a batch size of 420 kg, to produce tablets with a mean weight of 700 mg and an active
substance content of 600mg?

A- 400 kg
B- 380 kg
C- 378 kg
D-360 kg
F- 265 kg

Rationale (D) The fraction of active substance per tablet is 6/7 If we divide 420 kg by 7, we get 60 kg
Multiplying by 6 gives 360 kg, the amount of active substance required.

79. Given that the relative molecular mass (RMM) of sodium chloride is 58.5 g/mol, which of the
following amounts of sodium chloride powder would be required to prepare 300 mL of a solution
containing 50 mmol/L?

A. 0.878 g
B. 0.585 g
C. 1.75 g
D. 1.14 g
E. 1.5 g

Rationale A If there are 50 mmol/L, then there are 5 mmol in 100 mL and 15 mmol in 300 mL 1 mol
sodium chloride weighs 58.5 g so 1 mmol weighs 58.5/1000 = 0.0585 g 15 mmol weighs 0.0585 × 15 =
0.8775 g, which can be rounded to 0.878 g. The correct answer is A.

80. A tablet labelled to contain 350 mg active ingredient has acceptable limits of 90–110% of that
amount. Which of the following indicates the limits of content?

A. 300–400 mg
B. 310–390 mg
C. 315–385 mg
D. 320–380 mg
E. 340–360 mg

Arif Pharma Clinic Copyright@gmail.com arifpharma.com


Page 109

Rationale Calculate 10% of 350 mg, as 35 mg Then subtract this from 350 mg to get the lower limit,
315 mg, and add it to 350 mg to get 385 mg, the upper limit.

81. Ranitidine tablets are available as tablets containing ranitidine hydrochloride equivalent to 150
mg and 300 mg ranitidine. For ranitidine 150 mg tablets, which of the following amounts of
ranitidine hydrochloride is needed in each tablet? (RMM: ranitidine, C13H22N4O3S = 314.4 g/mol;
ranitidine hydrochloride, C13H22N4O3S.HCl = 350.9 g/mol.)

A. 134.55 mg
B. 167.41 mg
C. 172.35 mg
D. 122.98 mg
E. 150.00 mg

Rationale (B) 314.4:150 350.9:x Cross-multiplying, x = 167.41 167.41 mg ranitidine hydrochloride is


needed in each tablet.

82. Which of the following amounts of sodium ions does 50 mL sodium chloride solution 0.9%
w/v intravenous infusion contain? There are 150 mmol each of Na+ and Cl– /L of NaCl 0.9%
w/v infusions

A- 0.0075 mmol
B- 0.075 mmol
C- 7.5 mmol
D- 15 mmol
E- 0.15 mmol
Rationale (C) There are 150 mmol each of Na+ and Cl– /L of 0.9% w/v NaCl IV infusion So, in 100 mL,
you have 15 mmol. Therefore, in 50 mL you have 7.5 mmol.

83. Ferrous gluconate tablets are out of stock and your patient normally takes two tablets once
daily. TheGP has prescribed Galfer syrup as an alternative. The patient needs to receive the same
amount of iron. Which of the following would be an appropriate dose of Galfer syrup? Ferrous
gluconate contains 35 mg iron. Galfer syrup contains 45 mg iron/5 mL.
A- 15.6 mL twice daily
B- 7.8 mL once daily
C- 7.8 mL twice daily
D- 15.6 mL once daily

Arif Pharma Clinic Copyright@arifpharma.com arifpharma.com


Page 110

E- 15.6 mL three times daily

Rationale (B) Ferrous gluconate tablets each contain 35 mg iron, so two tablets contain 70 mg Galfer syrup
contains 45 mg iron/5 mL = 9 mg/mL 70/9 = 7.77 mL, which can be rounded to 7.8 mL.

84. Tablets containing 50 mg cortisone acetate have been formulated. Which of the following is the
amount of steroid in each tablet? (RMM: cortisone, C21H28O5 = 360.4 g/mol; cortisone acetate,
C23H30O6 = 402.5 g/mol.)
A- 36.04 g
B- 40.25 g
C- 44.77 g

D- 22.39 g
E- 45.00 g
Rationale ( C) Cortisone acetate 360.4:402.5 x:50 Cross-multiplying, we have (402.5) x
= 18 020 x = 44.77 g.

85. Which of the following is the amount of erythromycin lactobionate in a vial containing the
equivalent of 500 mg erythromycin for reconstitution? (RMM: erythromycin, C37H67NO13 =
733.9 g/mol; erythromycin lactobionate, C37H67NO13.C12H22O12 = 1092.2 g/mol.)

A. 604.12 mg
B. 500.00 mg
C. 1.49 g
D. 7.44 g
E. 744.11 mg

Rationale ( E) Erythromycin lactobionate 733.9:1092.2 500:x Cross-multiplying, we have 546 100 =


(733.9) x = 744.11 mg, which is the amount of erythromycin lactobionate in the vial.

86. An experimental antidepressant is found to undergo a first-order degradation reaction when


formulated as an aqueous solution. The first-order degradation reaction has a half-life of 1.98 days.
Given that the half-life, t1/2, of a first-order reaction is described by the equation t1 /2 = 0.693/k,
where k is the experimentally determined first-order rate constant, which of the following is the
rate constant for this reaction?

Arif Pharma Clinic Copyright@gmail.com arifpharma.com


Page 111

A- 1.37 days–1
B- 1972.80 min–1
C- 0.35 day–1
D- 0.35 mol day
E- 0.35 mol

Rationale t1 /2 = 0.693/k 1.98 = 0.693/k k = 0.35 day–1 k can also be expressed as 504 min–1 This is
obtained by multiplying the answer above by 24 h, then 60 min.

87. The enzyme-catalyzed breakdown of drug A (RMM = 470 g/mol) to yield degradation product
B (RMM = 235 g/mol) is known to follow zero-order kinetics. The concentration of B after 2 h of
reaction is 23.5 mg/L. The amount of product, x, formed by a zero-order reaction is given by x = kt,
where k is the experimentally determined zero-order rate constant and t is the time after the
start of the reaction. Which of the following is the zero-order rate constant for this reaction?

A- 11.75 g/L per h


B- 47.00 mg/L per h
C- 0.05 mmol/h
D- 0.20 mmol/h
E- 0.20 h
Rationale C x = kt 23.5 mg/L = k(2 h) k = 11.75 mg/L per h As we have the RMM of degradation
product B, we can convert the units to millimoles per hour 11.75 mg/L per h/235 000 mg/mol
= 5 × 10–5 mol/h, or 0.05 mmol/h The correct answer is, therefore,

88. Which of the following is the number of moles of 5-aminolevulinic acid hydrochloride in 50 mL
of a 1 mol/L solution?

A- 0.005 mol
B- 0.5 mol
C- 0.25 mol
D- 0.05 mol
E- 0.025
Rationale ( D) A 1 mol/L (molar) solution contains 1 mol in 1 L In 500 mL there are 0.5 mol and, in
50 mL, 0.05 mol.

89. Which of the following is the percentage of lithium in 200 mg lithium carbonate?
(RMM: lithium carbonate, Li2CO3 = 73.89 g/mol; lithium, Li = 6.941 g/mol.)

Arif Pharma Clinic Copyright@arifpharma.com arifpharma.com


Page 112

A. 18.79%
B. 9.39%
C. 0.94%
D. 1.88%
E. 2.05%
Rationale (B) Lithium carbonate: lithium 73.89:6.941 200:x Cross-multiplying, we have 1388.2 =
(73.89) x x =18.79 mg lithium in 200 mg lithium carbonate 18.79/200 × 100 = 9.39%.
The correct answer is B

90. Which of the following is the number of millimoles of water in 1000 mg calcium chloride
hexahydrate? (RMM: calcium chloride hexahydrate, CaCl2.6H2O = 219 g/mol; water: H2O =
18 g/mol.)

A- 18.6 mmol
B- 54.8 mmol

C- 13.7 mmol
D- 82.6 mmol
E- 27.4 mmol

Rationale 1000 mg = 1.0 g 1.0/219 = 4.57 × 10–3 mol calcium chloride hexahydrate As each mole of
calcium chloride hexahydrate contains 6 mol water, we multiply by 6 to calculate the number of moles
of water as: 6 × 4.57 × 10–3 = 0.0274 mol, or 27.40 mmol.

91. A reaction involved in the synthesis of an experimental anthelmintic requires heating to


350°F. However, The thermometer in the laboratory is marked out in degrees Celsius. Which of
the following is the correct temperature for the reaction in degrees Celsius?
A- 17.67°C
B- 176.67°C
C- 572.40°C
D- 190.02°C
E- 57.24°C
Rationale ( B) [°C] = ([°F] – 32) × 5/9 = (350 – 32) × 5/9 = (318) × 5/9 = 176.67°C.

92. For drug stability purposes, Daktacort cream requires storage at 2–8°C in a dispensary
refrigerator. Which of the following is this temperature range expressed in degrees
Fahrenheit
A. 35.6–46.4°F

Arif Pharma Clinic Copyright@gmail.com arifpharma.com


Page 113

B. 82.4–95.6°F
C. 3.6–4.6°F
D. 8.2–9.6°F
E. 20.0–80.0°F
Rationale (A) [°C] = ([°F] – 32) × 5/9 2 = (x – 32) × 5/9 Dividing across by 5/9, we have: 3.6 = x – 32 x =
35.6°F 8 = (x – 32) × 5/9 Dividing across by 5/9, we have: 14.4 = x – 32 x = 46.4°F.

93. A candidate antiretroviral drug is found to decompose spontaneously at 120°F. Which


of the following expresses in degrees centigrade this decomposition temperature?
A- A. 48.89°C
B- 80.12°C
C. 158.40°C
D. 8.01°C
E. 4.89°C

Rationale (A) [°C] = ([°F] – 32) × 5/9 = (120 – 32) × 5/9 = (88) × 5/9 = 48.89°C.

94. Which of the following is the boiling pint of water (100°C) expressed in degree Fahrenheit?
A- 100.00°F
B- 87.56°F
C- 300.00°F
D- 180.00°F
E- 212.00°F

Rationale (E) [°C] = ([°F] – 32) × 5/9 100 =(x – 32) × 5/9 Dividing across by 5/9, we have: 180 = x –
32 x = 212°F. The correct answer is E

95. Patient takes dose 20 mg/kg/day what is the dose if patient weight 60 pound?
Answer: 545 mg/day

Answer:
you have to know 1 kg = 2.2 pound (lb)

20 mg ------- -- 2.2 lb
X mg ------------ 60
X = 60 x 20 / 2.2 = 545.45 mg/day

Arif Pharma Clinic Copyright@arifpharma.com arifpharma.com


Page 114

96. A child was prescribed a drug with dose 65 mg/kg/hr. his body weight =35.2 pound Calculate the dose.

A- 1.040 gm
B- 10.40 gm
Answer:
35.2 Pound = 15.97 kg = about 16 kg 10.40 gm
65 mg … 1 kgX mg …
16 kg
X = 16 x 65 = 1040 mg = 1.040 gm

97. Fifty micrograms equals:

A- 50000 (nanogrames)
B- 0.05 (milligrams)
C- 0.0005 g
D- a and b
E- a and c
Note: mcg = 1000 nano-g ... milli-g = 1000 mc-g ... g = 1000 mg

98. 30gm of 1% hydrocortisone mixed with 40 gm 2.5% hydrocortisone, what is theconcentration of the
resulting solution?

A- 3%
B- 1.85%
C- 10%
D- None of the above

30gm 1% =0.3gm
40gm 2.5% =1gmSo, 70gm----------
---X% =1.3
So, the con. Result =1.3/70*100=1.857%

99. A solution is made by dissolving 17.52 g of NaCl exactly 2000 ml. What is themolarity of this solution?
A- 3.33
B- 0.15
C- 1.60 d-3.00 x 10 -4 e-1.6x10 -4

Molarity=mole/volume (L)
1 Mole=molecular weight of subs. In 1 gramsNo of Moles = wt /
Mwt

Arif Pharma Clinic Copyright@gmail.com arifpharma.com


Page 115

So, molecular weight of NACL=23+34=57So,


Mole=17.52/57=0.307
So, Morality=0.307/2=0.153

HCl= 36.4
NH4Cl= 53.5
NaCl= 58.5
Kcl= 74.5

MgCl2= 95.2
CaCl2= 111

Na=23,5 Mg 24.3 Cl= 35.4 K= 39.1

100. Fifty micrograms equals:


A- 50000 (nanogrames)?
B- 0.05 (milligrams)?
C- 0.0005 g
D- a and b
E- a and c

mc g : 1000 nano g milli g : 1000 mc g g : 1000 mg g : 1000000 mcg

Arif Pharma Clinic Copyright@arifpharma.com arifpharma.com


Page 116

Lippincott Pharmacokinetic

Total 175 Cases


Collected and Summarized By:

ARIF PHARMA CLINIC

LIPPINCOTT PHARMACOLOGY TEXTBOOK

DHA Exam
Important
Question

Arif Pharma Clinic


ARIF PHARMA CLINIC Copyright@arifpharma.com Www.arifpharma.com
Page 117

UNIT I: PRINCIPLES OF DRUG THERAPY


Chapter 1: Pharmacokinetics

1- An 18-year-old female patient is brought to the emergency department due to a drug


overdose. Which of the following routes of administration is the most desirable for
administering the antidote for the drug overdose?

A. Intramuscular.
B. Subcutaneous.
C. Transdermal.
D. Oral.
E. Intravenous.
Correct answer = E. The intravenous route of administration is the most desirablebecause it
results in rapidly achieving therapeutic plasma levels of the antidote.

2- A 40-year-old male patient (70 kg) was recently diagnosed with infant action involving
methicillin-resistant S. aureus. He received 2000 mg of vancomycin as an IV loading dose. The
peak plasma concentration of vancomycin was reported tobe 28.5 mg/L. The apparent
volume of distribution is:

A. 1 L/kg.
B. 10 L/kg.
C. 7 L/kg.
D. 70 L/kg.
E. 14 L/kg.
Correct answer = A. Vd = dose/C = 2000 mg/28.5 mg/L = 70.1 L. Because the patient is 70 kg,
the apparent volume of distribution in L/kg will be approximately1 L/kg (70.1 L/70 kg).

3- A 65-year-old female patient (60 kg) with a history of ischemic stroke was prescribed
clopidogrel for stroke prevention. She was hospitalized again after 6 months due to
recurrent ischemic stroke. Which of the following is a likely reasonshe did not respond to
clopidogrel therapy?
She is a:
A. Poor CYP2D6 metabolizer.
B. Fast CYP1A2 metabolizer.
C. Poor CYP2E1 metabolizer.
D. Fast CYP3A4 metabolizer.
E. Poor CYP2C19 metabolizer.

ARIF PHARMA CLINIC Copyright@arifpharma.com Www.arifpharma.com


Page 118

Correct answer = E. Clopidogrel is a prodrug, and it is activated by CYP2C19,which is a


cytochrome P450 (CYP450) enzyme. Thus, patients who are poor CYP2C19 metabolizers have
a higher incidence of cardiovascular events (for example, stroke or myocardial infarction)
when taking clopidogrel.

4- A 55-year-old male patient (70 kg) is going to be treated with an experimental


drug,Drug X, for an irregular heart rhythm. If the Vd is 1 L/kg and the desired steady-
state plasma concentration is 2.5 mg/L, which of the following is themost
appropriatee intravenous loading dose for Drug X?
A. 175 mg.
B. 70 mg.
C. 28 mg.
D. 10 mg.
E. 1 mg.

Correct answer = A. For IV infusion, Loading dose = (Vd) × (desired steady-stateplasma


concentration). The Vd in this case corrected to the patient’s weight is 70 L. Thus, Loading
dose = 70 L × 2.5 mg/L = 175 mg.

UNIT II: DRUGS AFFECTING THE AUTONOMIC NERVOUS SYSTEM


Chapter 3: The Autonomic Nervous System

5- An elderly man was brought to the emergency room after he ingested a large number of
carvedilol tablets, a drug that blocks α1, β1, and β2 adrenergic receptors, which mainly
mediate the cardiovascular effects of epinephrine and norepinephrine in the body. Which of
the following symptoms would you expect in this patient?

A. Increased heart rate (tachycardia).


B. Reduced heart rate (bradycardia).
C. Dilation of the pupil (mydriasis).
D. Increased blood pressure.

Correct answer = B. Activation of α1 receptors causes mydriasis, vasoconstriction, and


an increase in blood pressure. Activation of β1 receptors increases heart rate,
contractility of the heart, and blood pressure. Activation of β2 receptors causes dilation
of bronchioles and relaxation of skeletal muscle vessels. Thus, inhibition of these
receptors will cause vasorelaxation (α1 blockade), reduction in heart rate (β1 blockade),
reduction in contractility of the heart (β1 blockade), reduction in blood pressure,
bronchoconstriction (β2 blockade), and constriction ofblood vesselss supplying skeletal
muscles (β2 blockade).

ARIF PHARMA CLINIC Copyright@arifpharma.com Www.arifpharma.com


Page 119

Chapter 4: Cholinergic Agonists

6- An elderly female who lives in a farmhouse was brought to the emergency room in
serious condition after ingesting a liquid from an unlabeled bottle found near her bed,
apparently in a suicide attempt. She presented with diarrhea, frequenturination,
convulsions, breathing difficulties, constricted pupils (miosis), and excessive salivation.
Which of the following is correct regarding this patient?

A. She most likely consumed an organophosphate pesticide.


B. The symptoms are consistent with sympathetic activation.
C. Her symptoms can be treated using an anticholinesterase agent.
D. Her symptoms can be treated using a cholinergic agonist.

Correct answer = A. The symptoms are consistent with that of a cholinergic crisis. Since the
elderly female lives on a farm and since the symptoms are consistent with that of a cholinergic
crisis (usually caused by cholinesterase inhibitors), it may be assumed that she has consumed
an organophosphate pesticide (irreversible cholinesterase inhibitor). Assuming that the
symptoms are caused by organophosphate poisoning, administering an anticholinesterase
agent or a cholinergic agonist will worsen the condition. The symptoms are not consistent with
that of sympathetic activation, as sympathetic activation will cause symptomsopposite to that
of the cholinergic crisis seen in this patient.

Chapter 5: Cholinergic Antagonists

7- During an ophthalmic surgical procedure, the surgeon wanted to constrict the pupil of the
patient using a miotic drug. However, he accidentally used another drug that caused dilation of
the pupil (mydriasis) instead. Most likely, which of thefollowing drugs did he use?

A. Acetylcholine.
B. Pilocarpine.
C. Tropicamide.
D. Phentolamine.
E. Bethanechol.

Correct answer = C. Muscarinic agonists such as ACH, pilocarpine, and bethanechol contract
the circular muscles of the iris sphincter and cause constriction of the pupil (miosis), whereas

ARIF PHARMA CLINIC Copyright@arifpharma.com Www.arifpharma.com


Page 120

muscarinic antagonists such as atropine and ropicamide prevent the contraction of the circular
muscles of the iris and cause dilation of thepupil (mydriasis). α-Adrenergic antagonists such as
phentolamine relax the radialmuscles of the iris and cause miosis.

8- A patient with chronic obstructive pulmonary disease (COPD) was prescribed a β2 agonist
for the relief of bronchospasm. However, the patient did not respond to this treatment. Which
of the following drugs or classes of drugs would you suggest for this patient as the next option?

A. β1 Agonist.
B. Muscarinic agonist.
C. Physostigmine.
D. Ipratropium.
E. Phentolamine.
Correct answer = D. Major receptors present in the bronchial tissues are muscarinic and
adrenergic-β2 receptors. Muscarinic activation causes bronchoconstriction, andβ2 receptor
activation causes bronchodilation. Therefore, direct or indirect (physostigmine) muscarinic
agonists will worsen bronchospasm. Ipratropium is a muscarinic antagonist that can relax
bronchial smooth muscles and relieve bronchospasm in patients who are not responsive to β2
agonists. α1 and β1 receptors are not commonly present in bronchial tissues and, therefore, β1
agonists or α antagonists (phentolamine) do not have any significant effects on bronchospasm.

9- A patient was administered a neuromuscular blocker (NMB) prior to a surgical procedure


to produce skeletal muscle paralysis. This NMB drug affected small, rapidly contracting
muscles of the face and eyes first and diaphragm muscles last . The effect of this drug was
easily reversed with neostigmine. Which of the following neuromuscular blockers was most
likely administered to this patient?

A. Rocuronium.
B. Succinylcholine.
C. Diazepam.
D. Tubocurarine.

Correct answer = A. There are two types of NMBs: depolarizing and nondepolarizing NMBs.
Depolarizing NMBs are agonists at the nicotinic receptors, whereas nondepolarizing NMBs
are antagonists at the nicotinic receptors. Both types of NMBs affect the rapidly
contracting muscles (face, eye, etc.) first and the diaphragm muscles last. However,
cholinesterase inhibitors such as neostigmine increase ACH levels in the NMJ and reverse
the effects of nondepolarizing NMBs, but not those of depolarizing NMBs. Therefore, the
NMB administered to this patient is most probably rocuronium, which is a nondepolarizing
NMB. Tubocurarine is also a nondepolarizing NMB, but it is not used in practice.
Succinylcholine is a depolarizing NMB, and diazepam is a benzodiazepine that does not
cause paralysis of skeletal muscles.
ARIF PHARMA CLINIC Copyright@arifpharma.com Www.arifpharma.com
Page 121

10- A patient was administered a neuromuscular blocker (NMB) prior to a surgical


procedureto produce skeletal
muscle paralysis. This NMB drug caused initial skeletal muscle fasciculations before the
onset of paralysis. The effect of this drug could not be reversed with neostigmine. Which of
the following neuromuscular blockers was most likely administered to this patient?

A. Cisatracurium.
B. Succinylcholine.
C. Diazepam.
D. Tubocurarine.

Correct answer = B. Depolarizing NMBs causes muscle fasciculations before causing paralysis,
and their effects cannot be reversed using cholinesterase inhibitors such as neostigmine.
Nondepolarizing NMBs do not cause muscle fasciculations, and their effects can be reversed
using cholinesterase inhibitors. Therefore, the NMB used in this patient is succinylcholine,
which is a depolarizing NMB. Cisatracurium and tubocurarine are nondepolarizing NMBs, and
diazepam does not cause paralysis of skeletal muscles.

Chapter 6: Adrenergic Agonists

11- A hypertensive patient was accidentally given an α2 agonist instead of an α1blocker.


Which of the following is correct in this situation?

A. α2 Agonists can increase the release of norepinephrine from sympathetic nerve terminals.
B. α2 Agonists can reduce blood pressure in this patient.
C. α2 Agonists can increase blood pressure in this patient.
D. α2 Agonists will not affect blood pressure in this patient.

Correct answer = B. α2 Agonists activate α2 receptors located in the presynaptic terminal of


sympathetic neurons and cause a reduction in the release of norepinephrine from
sympathetic nerve terminals. This leads to a reduction in blood pressure. α2 Agonists such as
clonidine and methyldopa are therefore used as antihypertensive agents.

12- An asthma patient was given a nonselective β agonist to relieve bronchoconstriction.


Which of the following adverse effects would you expect to see in this patient?
A. Bradycardia.
B. Tachycardia.
ARIF PHARMA CLINIC Copyright@arifpharma.com Www.arifpharma.com
Page 122

C. Hypotension (reduction in blood pressure).


D. Worsening bronchoconstriction.

Correct answer = B. A nonselective β agonist activates both β1 as well as β2 receptors. β1


activation causes an increase in heart rate (tachycardia), contractility, and subsequent increase
in blood pressure. It relieves bronchoconstriction
because of the β2 receptor activation.

13- A 12-year-old boy who is allergic to peanuts was brought to the emergency room
after accidentally consuming peanuts contained in fast food. He is in anaphylactic shock.
Which of the following drugs would be most appropriate to treat this patient?

A. Norepinephrine.
B. Phenylephrine.
C. Dobutamine.
D. Epinephrine.

Correct answer = D. Norepinephrine has more α agonistic effects and activates mainly α1,
α2, and β1 receptors. Epinephrine has more β agonistic effects and activates mainly α1, α2,
β1, and β2 receptors. Phenylephrine has predominantlyα effects and activates mainly α1
receptors. Dobutamine mainly activates β1 receptors and has no significant effects on β2
receptors. Thus, epinephrine is the drug of choice in anaphylactic shock that can both
stimulate the
heart (β1 activation) and dilate bronchioles (β2 activation).

14- A 70-year-old patient was brought to the emergency room with a blood pressure of
76/60 mm Hg, tachycardia, and low cardiac output. He was diagnosed with acute heart
failure. Which of the following drugs would be the most appropriate to improve his cardiac
function?

A. Epinephrine.
B. Fenoldopam.
C. Dobutamine.
D. Isoproterenol.
Correct answer = C. Among the choices, the ideal drug to increase the contractility of the heart
in acute heart failure is dobutamine, since it is a selective β1-adrenergic agonist. Fenoldopam
is a dopamine agonist used to treat severe hypertension.
Other drugs are nonselective adrenergic agonists that could cause unwanted side effects.

15. One of your patients who is hypertensive and gets mild asthma attacks occasionally
bought an herbal remedy online to help with his asthma. He is not on any asthma
medications currently but is receiving a β1-selective blocker for his hypertension. The
herbalremedy seems to relieve his asthma attacks, but his blood pressure seems to
increase despitethe β-blocker therapy. Which of the following drugs is most likely present
in the herbal
ARIF PHARMA CLINIC Copyright@arifpharma.com Www.arifpharma.com
Page 123

remedy he is taking?

A. Phenylephrine.
B. Norepinephrine.
C. Dobutamine.
D. Ephedrine.
E. Salmeterol.

Correct answer = D. Two drugs among the choices that could relieve asthma are ephedrine and
salmeterol, as they activate β2 receptors in the bronchioles and cause bronchodilation.
However, salmeterol is a selective β2 agonist and should not cause an increase in blood
pressure. Ephedrine on the other hand stimulates the release of norepinephrine and acts as
adirect agonist at α- and β-adrenergic receptors, thus causing an increase in blood pressure.
Phenylephrine (a nonselective α agonist) does not cause bronchodilation. Norepinephrine is a
nonselective adrenergic agonist that does not have any stimulatory effects on β2 receptors.
Also, norepinephrine is not active when given orally.

Chapter 7: Adrenergic Antagonists

15- A 30-year-old male patient was brought to the ER with an amphetamine overdose.
Hepresented with high blood pressure and arrhythmia. Which of the following is correct
regarding this patient?

A. Amphetamine can activate all types of adrenergic receptors.


B. β-Blockers are the ideal antidotes for amphetamine poisoning.
C. α-Blockers can normalize the blood pressure in this patient.
D. Miosis could be a possible symptom of amphetamine poisoning.

Correct answer = A. Amphetamine is an indirect adrenergic agonist that mainly enhances the
release of norepinephrine from peripheral sympathetic neurons. Therefore, it activates all types
of adrenergic receptors (that is, α and β receptors) and causes an increase in blood pressure.
Since both α and β receptors are activated by amphetamine, α-blockers or β-blockers alone
cannot relieve the symptoms of amphetamine poisoning. Since amphetamine causes
sympathetic activation, it causes mydriasis, not miosis.

16- A β-blocker was prescribed for hypertension in a female asthma patient. After about a
week of treatment, the
asthma attacks got worse, and the patient was asked to stop taking the β-blocker.

ARIF PHARMA CLINIC Copyright@arifpharma.com Www.arifpharma.com


Page 124

Which of the following


β-blockers would you suggest as an alternative in this patient that is less likely to worsen
her asthma?

A. Propranolol.
B. Metoprolol.
C. Labetalol.
D. Carvedilol.

Correct answer = B. The patient was most likely given a nonselective β-blocker (antagonizes
both β1 and β2 receptors) that made her asthma worse due to β2 antagonism. An alternative is
to prescribe a cardioselective (antagonizes only β1) β-blocker that does not antagonize β2
receptors in the bronchioles. Metoprolol is a cardioselective β-blocker. Propranolol, labetalol,
and carvedilol are nonselective β-blockers and could worsen asthma.

18 A 70-year-old male needs to be treated with an α-blocker for overflow incontinence due to
his enlarged prostate. Which of the following drugs would you suggest in this patent that will
not affect his blood pressure significantly?

A. Prazosin.
B. Doxazosin.
C. Phentolamine.
D. Tamsulosin.
E. Terazosin.

Correct answer = D. Tamsulosin is an α1 antagonist that is more selective to the α1receptor subtype
(α1A) present in the prostate and less selective to the α1 receptor subtype (α1B) present in the
blood vessels. Therefore, tamsulosin does not affect blood pressure significantly. Prazosin,
doxazosin, terazosin, and phentolamine antagonize both these subtypes and cause significant
hypotension as a side effect

19- A 50-year-old male was brought to the emergency room after being stung by a hornett.
The patient was found to be in anaphylactic shock, and the medical team tried to reverse the
bronchoconstriction and hypotension using epinephrine.
However, the patient did not fully respond to the epinephrine treatment. The patient’s
wife mentioned that he is taking prescription medication for his blood pressure, the
nameof which she does not remember. Which of the following medications is he most
likely taking that could have prevented the effects of epinephrine?

A. Doxazosin.
B. Propranolol.
C. Metoprolol.
D. Acebutolol.
ARIF PHARMA CLINIC Copyright@arifpharma.com Www.arifpharma.com
Page 125

Correct answer = B. Epinephrine reverses hypotension by activating β1 receptors and relieves


bronchoconstriction by activating β2 receptors in anaphylaxis. Since epinephrine was not
effective in reversing hypotension or bronchoconstriction
in this patient, it could be assumed that the patient was on a nonselective β-blocker
(propranolol). Doxazosin (α1-blocker), metoprolol, or acebutolol (both β1- selective blockers)
would not have completely prevented the effects of epinephrine.

UNIT III: DRUGSAFFECTING THE CENTRALNERVOUS SYSTEM

Chapter 8: Drugs for Neurodegenerative Diseases

20- Peripheral adverse effects of levodopa, including nausea, hypotension,


and cardiac arrhythmias, can be diminished by including which of the following
drugs in the therapy?

A. Amantadine.
B. Ropinirole.
C. Carbidopa.
D. Tolcapone.
E. Pramipexole.

Correct answer = C. Carbidopa inhibits the peripheral decarboxylation of levodopa to


dopamine, thereby diminishing the gastrointestinal and cardiovascular side effects of levodopa.
The other agents listed do not ameliorate the adverse effects of levodopa.

Chapter 9: Anxiolytic and Hypnotic Drugs

21 A 45-year-old man who has been injured in a car accident is brought into the emergency
room. His blood alcohol level on admission is 275 mg/dL. Hospital records show a prior
hospitalization for alcohol-related seizures. His wife confirmed that he has been
drinking heavily for 3 weeks. What treatment should be provided to the patient if he
goes into withdrawal?

A. None.
B. Lorazepam.

ARIF PHARMA CLINIC Copyright@arifpharma.com Www.arifpharma.com


Page 126

C. Pentobarbital.
D. Phenytoin.
E. Buspirone.

Correct answer = B. It is important to treat the seizures associated with alcohol withdrawal.
Benzodiazepines, such as chlordiazepoxide, diazepam, or the shorter-acting lorazepam, are
effective in controlling this problem. They are less sedating than pentobarbital or phenytoin.

22 Which agent is best used in the Emergency Room setting for patients who are believed to
have received too much of a benzodiazepine drug or taken an overdose of
benzodiazepines?

A. Diazepam.
B. Ramelteon.
C. Flumazenil.
D. Doxepin.
E. Naloxone.

Correct answer = C. Flumazenil is only indicated to reverse the effects of benzodiazepines via
antagonizing the benzodiazepine receptor. It should be used with caution due to the risk of
seizures if the patient has been a long-time recipient of benzodiazepines or if the overdose
attempt was with mixed drugs. Naloxone is an opioid receptor antagonist. The other
agents are not efficacious in reversing the effects of benzodiazepines.

Chapter 10: Antidepressants

23 A 55-year-old teacher began to experience changes in mood. He was losing interest in his
work and lacked the desire to play his daily tennis match. He was preoccupied with
feelings of guilt, worthlessness, and hopelessness. In addition to the psychiatric
symptoms,the patient complained of muscle aches throughout his body. Physical and
laboratory testswere unremarkable. After 6 weeks of therapy with fluoxetine, his
symptoms resolved. However, the patient complains of sexual dysfunction. Which of the
following drugs might be useful in this patient?

A. Fluvoxamine.
B. Sertraline.
C. Citalopram.
D. Mirtazapine.
E. Lithium.
ARIF PHARMA CLINIC Copyright@arifpharma.com Www.arifpharma.com
Page 127

Correct answer = D. Mirtazapine is largely free from sexual side effects. However, sexual
dysfunction commonly occurs with SSRIs (fluvoxamine, sertraline, and citalopram), as well as
with TCAs, and SNRIs. Lithium is used for the treatment of mania and bipolar disorder.

24 A 25-year-old woman has a long history of depressive symptoms accompanied by body


aches and pain secondary to a car accident 2 years earlier. Physical and laboratory tests are
unremarkable. Which of the following drugs might be useful for this patient?

A. Fluoxetine.
B. Sertraline.
C. Phenelzine.
D. Mirtazapine.
E. Duloxetine.

Correct answer = E. Duloxetine is an SNRI that can be used for depression accompanied by
symptoms of pain. SSRIs (fluoxetine and sertraline), MAOIs (phenelzine), and atypical
antidepressants (mirtazapine) have little activity against pain syndromes.

25 A 51-year-old woman with symptoms of major depression also has angle-closure


glaucoma. Which of the following antidepressants should be avoided in this
patient?

A. Amitriptyline.
B. Sertraline.
C. Bupropion.
D. Mirtazapine.
E. Fluvoxamine.

Correct answer = A. Because of its potent antimuscarinic activity, amitriptyline should not be
given to patients with glaucoma because of the risk of acute increases in intraocular pressure.
The other antidepressants all lack antagonist activity at in the muscarinic receptor Correct
answer =
A. Because of its potent antimuscarinic activity, amitriptyline should not be given to patients with
glaucoma because of the risk of acute increases in intraocular pressure. The other
antidepressants all lack antagonist activity at the muscarinic receptor.

ARIF PHARMA CLINIC Copyright@arifpharma.com Www.arifpharma.com


Page 128

26 A 36-year-old man presents with symptoms of compulsive behavior. If anything


isout of order, he feels that “work will not be accomplished effectively or
efficiently.” He realizes that his behavior is interfering with his ability to
accomplish his daily tasks but cannot seem to stophimself. Which of the
following drugs would be most helpful too this patient?

A. Imipramine.
B. Fluvoxamine.
C. Amitriptyline.
D. Tranylcypromine.
E. Lithium.

Correct answer = B. SSRIs are particularly effective in treating an obsessive-compulsive


disorder, and fluvoxamine is approved for this condition. The other drugs are less effective in
the treat mobsessive–compulsivesive disorder.

27 Which agent would be a poor choice in a 70-year-old elderly female with depressive
symptoms due to the drug having significant α1 receptor antagonism and thus a
higherrisk for falls due to orthostatic hypotension?

A. Lithium.
B. Bupropion.
C. Escitalopram.
D. Imipramine.
E. Sertraline.

Correct answer = D. Lithium should not be used for depression in an elderly patient without
first trying first-line antidepressants, and even then, it is used as an adjunct. Bupropion,
sertraline, and escitalopram have very little effect on blood pressure (no α1 receptor
antagonism) and are considered acceptable choices for the treatment of depression in the
elderly. Imipramine is associated with a high risk for orthostasis in the elderly and should be
avoided due to its adverse effect proris kdrisk for falls.

ARIF PHARMA CLINIC Copyright@arifpharma.com Www.arifpharma.com


Page 129

Chapter 11: Antipsychotic Drugs

28 A 21-year-old male has recently begun pimozide therapy for Tourette’s disorder. His parents
bring him to the emergency department. They describe that he has been having different-
appearing tics” than before, such as prolonged contraction of the facial muscles. While
beingexamined, he experiences opisthotonos a (type of extrapyramidal spasm of the body
in whichthe head and heels are bent backward and the body is bowed forward). Which of
the following drugs would be beneficial in reducing these symptoms?

A. Benztropine.
B. Bromocriptine.
C. Lithium.
D. Prochlorperazine.
E. Risperidone.
Correct answer = A. The patient is experiencing EPS due to pimozide, and a muscarinic
antagonist such as benztropine would be effective in reducing the symptoms. The other
drugs would have no effect or, in the case of prochlorperazine and risperidone, might
increase the symptoms.

29 A 28-year-old woman with schizoaffective disorder (combination of mood and psychotic


symptoms) reports difficulty falling asleep. Which of the following would be most beneficial
for this patient?

A. Lithium.
B. Chlorpromazine.
C. Haloperidol.
D. Paliperidone.
E. Ziprasidone.

Correct answer = D. Paliperidone is the only agent that is FDA approved for schizoaffective
disorder. Chlorpromazine has significant sedative activity as well as antipsychotic properties
and is the drug most likely to alleviate this patient’s major complaint of insomnia. Although
other antipsychotics may benefit this patient’s disorder, paliperidone has the indication for
this disorder, and if the underlying disorder is improved, then the symptom of insomnia may
also improve without risking other, unwanted adverse effects, such as the anticholinergic
effects of chlorpromazine.

ARIF PHARMA CLINIC Copyright@arifpharma.com Www.arifpharma.com


Page 130

30 A 30-year-old male patient who is treated with haloperidol for his diagnosis of
schizophrenia is considered to be well-managed symptomatically for his psychotic
symptoms. However, he is reporting restlessness, and the inability to sit still at the dinner
table and his family notices that he is pacing up and down the hallway frequently. Of the
following, which is the best medication to treat this antipsychotic-induced akathisia?

A. Benztropine.
B. Dantrolene.
C. Amoxapine.
D. Bromocriptine.
E. Propranolol.

Correct answer = E. Propranolol, a β-blocker, is considered the drug of choice for the
management of antipsychotic-induced akathisia. Benztropine is more effective for pseudo
parkinsonism and acute dystonias. Amoxapine is an antidepressant that has been associated
with EPS. Bromocriptine is more effective for Parkinson-like symptoms, and dantrolene is a
muscle relaxant that is best reserved for managing some symptoms of the neuroleptic
malignantsyndrome.

Chapter 12: Drugs for Epilepsy

31 A 9-year-old boy is sent for neurologic evaluation because of episodes of apparent


inattention. Over the past year, the child has experienced episodes during which he
develops a blank look on his face and his eyes blink for 15 seconds. He immediately
resumes his previous activity. Which one of the following best describes this
patient’s seizures?

A. Simple partial.
B. Complex partial.
C. Tonic–clonic.
D. Absence.
E. Myoclonic.
Correct answer = D. The patient is experiencing episodes of absence seizures. Consciousness is
impaired briefly and they generally begin in children aged 4 to 12years. Diagnosis includes
obtaining an EEG that shows generalized 3-Hz waves.

ARIF PHARMA CLINIC Copyright@arifpharma.com Www.arifpharma.com


Page 131

32 A child is experiencing absence seizures that interrupt his ability to pay attention during
school and activities. Which of the following therapies would be most appropriate for
thispatient?

A. Ethosuximide.
B. Carbamazepine.
C. Diazepam.
D. Carbamazepine plus primidone.
E. Watchful waiting.

Correct answer = A. The patient has had many seizures that interrupt his ability to pay
attention during school and activities, so therapy is justified. Monotherapy
with primary agents is preferred for most patients. The advantages of monotherapy include
reduced frequency of adverse effects, fewer interactions between antiepileptic drugs, lower cost,
and improved compliance. Carbamazepine and diazepam are not indicated for absence seizures.

33 A 25-year-old woman with myoclonic seizures is well-controlled on valproate. She


indicatesthat she is interested in becoming pregnant in the next year. With respect to her
antiepilepsy medication, which of the following should be considered?

A. Leave her on her current therapy.


B. Consider switching to lamotrigine.
C. Consider adding a second antiepilepsy medication.
D. Decrease her valproate dose.

Correct answer = B. Valproate is a poor choicforin woman of child-bearing age. A review of the
medication history of this patient is warranted. If she has not tried any other antiepilepsy
medication, then consideration of another antiepilepsy medication may be beneficial. Studies
show that valproate taken during pregnancy can have a detrimental effect on cognitive abilities
in children.

34 A woman with myoclonic seizures is well controlled with lamotrigine. She becomes
pregnant and begins to have breakthrough seizures. What is most likely happening?

A. Her epilepsy is getting worse.


B. Lamotrigine concentrations are increasing.
C. Lamotrigine concentrations are decreasing.
D. Lamotrigine is no longer efficacious for this patient.

ARIF PHARMA CLINIC Copyright@arifpharma.com Www.arifpharma.com


Page 132

Correct answer = C. Pregnancy alters the pharmacokinetics of lamotrigine. As pregnancy


progresses, most women require increased dosages to maintain blood concentrations and
seizure control.

35 A 42-year-old man undergoes a neurologic evaluation because of episodes of apparent


confusion. Over the past year, the man has experienced episodes during which he
develops a blank look on his face and fails to respond to questions. Moreover, it appears
to take several minutes before the man recovers from the episodes. Which one of the
following best describes this type of seizure?

A. Focal (simple partial).


B. Focal (complex partial).
C. Tonic–clonic.
D. Absence.
E. Myoclonic.

Correct answer = B. The patient is experiencing episodes of complex partial seizures.


Complex partial seizures impair consciousness and can occur in all age groups. Typically,
staring is accompanied by impaired consciousness and
recall. If asked a question, the patient might respond with an inappropriate or unintelligible
answer. Automatic movements are associated with most complex partial seizures and involve
the mouth and face (lip-smacking, chewing, tasting, and swallowing movements), upper
extremities (fumbling, picking, tapping, or clasping movements), vocal apparatus (grunts or
repetition of words and phrases), as are complex acts (such as walking or mixing foods in a
bowl).

36 A 52-year-old man has had several focal complex partial seizures over the last year.
Whichone of the following therapies would be the most appropriate initial therapy for
this patient?

A. Ethosuximide.
B. Levetiracetam.
C. Diazepam.
D. Carbamazepine plus primidone.
E. Watchful waiting.

ARIF PHARMA CLINIC Copyright@arifpharma.com Www.arifpharma.com


Page 133

Correct answer = B. The patient has had many seizures, and the risks of not starting drug
therapy would be substantially greater than the risks of treating his seizures. Because the
patient has impaired consciousness during the seizure,
he is at risk for injury during an attack. Monotherapy with primary agents is preferred for
most patients. The advantages of monotherapy include reduced frequency of adverse
effects, absence of interactions between antiepileptic drugs, lower cost, and improved
compliance. Ethosuximide and diazepam are not indicated for complex partial seizures.

37 A patient with focal complex partial seizures has been treated for 6 months with
carbamazepine but, recently, has been experiencing breakthrough seizures on a more
frequent basis. You are considering adding a second drug to the antiseizureregi men.
Whichof the following drugs is least likely to have a pharmacokinetic interaction with
carbamazepine?

A. Topiramate.
B. Tiagabine.
C. Levetiracetam.
D. Lamotrigine.
E. Zonisamide.

Correct answer = C. Of the drugs listed, all of which are approved as an adjunct therapy for
refractory focal complex partial seizures, only levetiracetam do not affect the
pharmacokinetics of other antiepileptic drugs, and other drugs
do not significantly alter its pharmacokinetics. However, any of the listed drugs could be added
depending on the plan and the patient’s characteristics. Treatment of epilepsy is complex, and
diagnosis is based on history and may need to be reevaluated when drug therapy fails or seizures
increase.

38 A 75-year-old woman had a stroke approximately 1 month ago. She is continuing to


have small focal seizures where she fails to respond appropriately while talking.
Whichof the following is the most appropriate treatment for this individual?

A. Phenytoin.
B. Oxcarbazepine.
C. Levetiracetam.
D. Phenobarbital.

Correct answer = C. Levetiracetam is renally cleared and prone to very few drug interactions.
Elderly patients usually have more comorbidities and are taking more medications than younger
patients. Oxcarbazepine may cause hyponatremia, which is more symptomatic in the elderly.

ARIF PHARMA CLINIC Copyright@arifpharma.com Www.arifpharma.com


Page 134

Phenytoin and phenobarbital have many drug interactions and a side effect profile that may be
especially troublesome in the elderly age group including dizziness that may lead to falls, cognitive
issues, and bone health issues

Chapter 13: Anesthetics

39 A patient with heart failure and significantly reduced cardiac output requires surgical
anesthesia. Which of the following would you expect to see in this patient?

A. Slower induction time with IV anesthetics.


B. Need for increased dosage of IV anesthetics.
C. Slower induction time with inhaled anesthetics.
D. Enhanced removal of inhaled anesthetics to peripheral tissues.

Correct answer = A. When cardiac output is reduced, the body compensates by diverting
more cardiac output to the cerebral circulation. A greater proportion of the IV anesthetic
enters the cerebral circulation under these circumstances.
Therefore, the dose of the IV drug must be reduced (not increased). Also, with reduced cardiac
output, it takes a longer time for an IV induction drug to reach the brain, resulting in a slower
induction time. For inhaled anesthetics, lower cardiac output removes anesthetic from the
alveoli to the peripheral tissues more slowly and thus enhances the rate of rise in alveolar
concentration of gas. Therefore, the gas reaches equilibrium between the alveoli and the site
of action in the brain more quickly.

40 An 80-year-old patient with asthma and low blood pressure requires anesthesia for an
emergency surgical procedure. Which of the following agents would be most appropriate
for inducing anesthesia in this patient?

A. Desflurane.
B. Ketamine.
C. Propofol.
D. Thiopental.

Correct answer = B. Ketamine may be beneficial since it is a potent bronchodilator and may not
lower blood pressure like other agents. Desflurane is an inhaled anesthetic that may stimulate
respiratory reflexes. It is used for maintenance,
not induction, and may lower blood pressure. Propofol may also decrease blood pressure.
Thiopental is a short-acting barbiturate that can cause bronchospasm.

ARIF PHARMA CLINIC Copyright@arifpharma.com Www.arifpharma.com


Page 135

41 A 52-year-old woman will be undergoing sedation with propofol for a brief diagnostic
procedure. Which of the following is an advantage of propofol for this patient?

A. Rapid analgesia.
B. Sustained duration.
C. Decreased incidence of nausea and vomiting.
D. Less pain at the injection site.

Correct answer = C. Propofol has some antiemetic effect, so it does not cause postoperative
nausea and vomiting. It has a short duration of action (which makes it good for the brief
proceure, but does not produce analgesia. Pain at the injection site is common.

42 A 32-year-old woman requests an epidural to ease labor pains. She reports that she had an
allergic reaction to Novocaine (procaine) at the dentist’s office. Which of the following
local anesthetics would be appropriate for use in an epidural for this patient?

A. Chloroprocaine.
B. Mepivacaine.
C. Ropivacaine.
D. Tetracaine.

Correct answer = C. Procaine is an ester local anesthetic. Since this patient has an allergy to
procaine, other ester anesthetics (chloroprocaine, tetracaine) should not be used.
Mepivacaine, an amide local anesthetic, should not be used due to the potential for increased
toxicity to the neonate. Ropivacaine is an amide anesthetic.

Chapter 14: Opioids

43 A young woman is brought into the emergency room. She is unconscious, and she has
pupillary constriction and depressed respiration. Based on reports, an opioid overdose is
almost certain. Which of the listed phenanthrene opioids will exhibit a full and immediate
response to treatment with naloxone?

A. Meperidine.
B. Morphine.
C. Buprenorphine.
D. Fentanyl.

Correct answer = B. A morphine overdose can be effectively treated with naloxone and

ARIF PHARMA CLINIC Copyright@arifpharma.com Www.arifpharma.com


Page 136

morphine phenanthrene. Naloxone antagonizes the opioid by displacing it from the receptor,
but there are cases in which naloxone is not effective. Meperidine is a phenylpiperidine, not a
phenanthrene, and the active metabolite, normeperidine, is not reversible by naloxone. The
effects of up renorphine are only partially reversible by naloxone. Naloxone is effective for
fentanyl overdoses; however, fentanyl is a phenylpiperidine, and not a phenanthrene.

44 A 76-year-old female with renal insufficiency presents to the clinic with severe pain
secondary to a compression fracture in the lumbar spine. She reports that the pain has been
uncontrolled with tramadol, and it is decided to start treatment with an opioid. Which of
the following is the best opioid for this patient?

A. Meperidine.
B. Fentanyl transdermal patch.
C. Hydrocodone.
D. Morphine.

Correct answer = C. Hydrocodone would be the best choice of the opioid given in this case. It will
be very important to use a low dose and monitor closely for proper pain control and any side
effects. Meperidine should not be used for chronic pain, nor should it be used in a patient with
renal insufficiency. The transdermal patch is not a good option, since at this time, her pain would
be considered acute and she is opioid naïve. Morphine also is not the best choice in this case due
to the active metabolites that can accumulate in renal insufficiency.

45 A 56-year-old patient who has suffered from severe chronic pain with radiculopathy
secondary to spinal stenosis for years presents to the clinic for pain management. Over
the years, this patient has failed to receive relief from the neuropathic pain from
radiculopathy with traditional agents such as tricyclicsor anticonvulsants. Based on the
mechanism of action, which opioid might be beneficial in this patient to treat both
nociceptive and neuropathic pain?

A. Meperidine.
B. Oxymorphone.
C. Morphine.
D. Methadone.

Correct answer = D. Methadone has a unique mechanism of action in comparison to the


otherchoices given. Methadone is a μ agonist, but it also exhibits NMDA receptor antagonism
that isthought to aid in the treatment of neuropathic pain and could also aid in the prevention
of opioid tolerance. All other μ agonists could help manage neuropathic pain, but in some
situations, higher doses of opioids are needed to achieve efficacy. It is much better to consider
adjuvants such as tricyclics or certain anticonvulsants in the treatment of neuropathic pain.

ARIF PHARMA CLINIC Copyright@arifpharma.com Www.arifpharma.com


Page 137

46 A 64-year-old male is preparing for a total knee replacement. He is taking many


medications that are metabolized by the CYP450 enzyme system and is worried about
drug interactions with the pain medication that will be used following his surgery.
Which of the following opioids would have the lowest chance of interacting with his
medications that are metabolized by the CYP450enzyme system?

A. Methadone.
B. Oxymorphone.
C. Oxycodone.
D. Hydrocodone.

Correct answer = B. Oxymorphone is metabolized via glucuronidation and has not been shown
tohave any drug interactions associated with the CYP enzyme family.
All other opioids listed are metabolized by one or more CYP enzymes and increase the risk of
drug interactions.

47 KM is a 64-year-old male who has been hospitalized following a car accident in which he
sustained a broken leg and broken arm. He has been converted to oral morphine in
anticipation of his discharge. What other medication should he receive with his
morphineupon discharge?

A. Diphenhydramine.
B. Methylphenidate.
C. Docusate sodium with Senna.
D. Docusate sodium.

Correct answer = C. A bowel regimen should be prescribed with the initiation of the opioid.
Docusate and senna include both a stool softener and a stimulant, which is recommended
foropioid-induced constipation. Treatment with docusate sodium only is ineffective.
Constipation is very common with all opioids, and tolerancedoes not occur. Diphenhydramine
can be used for urticaria that might occurr with the initiation ofan opioid, but this is not
reported in this case.
Methylphenidate has been used for opioid-induced sedation in certain situations but is not
an issue in this case.

48 AN is a 57-year-old male who has been treated with oxycodone for chronic
nonmalignant pain for over 2 years. He is now reporting increased pain in the afternoon
while at work. Which of the following opioids is a short-acting opioid and is the best
choice for this patient’s breakthrough pain?

A. Methadone.
B. Pentazocine.
C. Hydrocodone.
ARIF PHARMA CLINIC Copyright@arifpharma.com Www.arifpharma.com
Page 138

D. Nalbuphine.

Correct answer = C. Hydrocodone is a commonly used short-acting agent that is


commercially available in combination form with either acetaminophen or ibuprofen.
Methadone should not routinely be used for breakthrough
pain due to the unique pharmacokinetics and should be reserved for practitioners who have
experience with this agent and understand the variables associated with this drug.
Pentazocine and nalbuphine are mixed agonist/antagonist
analgesics that could precipitate withdrawal in patients who are currently taking a full μ
agonist such as oxycodone.

Chapter 15: Drugs of Abuse

49 A 22-year-old HIV patient has been told that marijuana may benefit him should he start
using the substance. Which of the following adverse effects has been associated with
marijuana usage and may be a reason for this patient to avoid use of marijuana?

A. Hyperphagia.
B. Hyperthermia.
C. Hepatitis.
D. Progression of HIV.
E. Hyponatremia.

Correct answer = D. Although hyperphagia is a side effect observed with marijuana usage,
this may be of benefit for some HIV patients. Hyperthermia, hepatitis, and hyponatremia
have not been associated with marijuana use.
Progression of HIV has been linked to marijuana use and is a serious consideration for anyone
with this disease.

50 A 21-year-old college student is curious about the effects of LSD. She asks what type of risks
may be involved with using the drug for the first time. Which of the following is a correct
response to her question?
A. Exaggerated hallucinations.
B. Cardiomyopathy.
C. Hyperphagia.
D. Bronchitis.

Correct answer = A. Exaggerated hallucinations, sometimes known as “bad trips,” may occur,
even in first-time users. These hallucinations can lead to extreme panic, which has caused
individuals to react in a manner very uncharacteristic

ARIF PHARMA CLINIC Copyright@arifpharma.com Www.arifpharma.com


Page 139

of their typical behavior.

51 A 58-year-old male is brought into the emergency department following an automobile


accident. His blood alcohol level on admission is 280 mg/dL. He has been treated in the
past for seizures related to alcohol abuse, and he confirms that he has been drinking
heavily over the past month since losing his job. What treatment should be given to this
patient if he begins to go in to withdrawl while hospitalized?

A. None.
B. Lorazepam.
C. C-Acamprosate
D. Naltrexone.
E. Disulfiram.
Correct answer = B. Should this patient go into alcohol withdrawal,
he will likely also have seizures associated with it, given his past history. Benzodiazepines are
used to treat seizures associated with alcohol withdrawal. Acamprosate, naltrexone, and
disulfiram may be considered at a later time to treat the dependence, but would not be useful
in the acute withdrawal setting.

52 A 35-year-old man has been abusing cocaine and is agitated, tachycardic,


hypertensive, and hyperthermic. Which of the following is correct regarding
treatment in this situation?

A. This patient should undergo gastric lavage; that is, he should have his stomach
pumped immediately.
B. Cocaine toxicity commonly involves CNS depression that can be reversed with atropine.
C. Benzodiazepines would be a good choice, as they should help calm the patient down,
decrease heart rate, decrease blood pressure, and decrease body temperature.
D. Phenobarbital should be the first choice as an anticonvulsant.

Correct answer = C. Benzodiazepines such as lorazepam have anxiolytic properties and can calm
a cocaine-toxic patient down, thereby decreasing heart rate and blood pressure. As the patient
becomes less agitated, he/she decreases movement, and his her body temperature drops. In
addition, the use of benzodiazepines decreases the chance of the patient experiencing a
convulsion and would be the first choice to treat cocaine-induced convulsions.

53. A 22-year-old man with a history of substance abuse arrives in the emergency
department hypertensive, hyperthermic, and tachycardic, with altered mental status
and hyperreflexia. His friends say he has been snorting “bath salts.” Which of the
following is correct regarding this patient?

ARIF PHARMA CLINIC Copyright@arifpharma.com Www.arifpharma.com


Page 140

A. This patient’s clinical presentation is consistent with opioid toxicity and he


should receive an opioid antagonist such as naloxone immediately.
B. “Bath salts” are often labeled as “not for human consumption” and sold with an
unstated understanding that they contain synthetic cathinone, which are amphetamine-
like compounds.
C. Treatment with a serotonin agonist might be beneficial.
D. Along with cooling measures, antihypertensives, β-blockers, and monoamine oxidase
inhibitors would be reasonable options for the treatment.

Correct answer = B: “Bath salts” often contain synthetic cathinone and are labeled,
marketed, and sold as something “not for human consumption” to avoid law enforcement
and prosecution. In addition, they are usually not detected on urine toxicology screening.
These products can cause an amphetamine-like sympathomimetic toxidrome, as well as
serotonin syndrome, which would be treated with symptomatic/ supportive care and
possibly a serotonin antagonist (not a serotonin agonist) such as cyproheptadine. The
combination of an amphetamine or amphetamine-like substance and a monoamine oxidase
inhibitor(MAO inhibitor) can precipitate serotonin syndrome and should be avoided in a
hyperdynamic patient such as this.

Chapter 16: CNS Stimulants

54 A young male was brought to the emergency room by the police due to severe agitation.
PsA psychiatric examination revealed that he had injected dextroamphetamine several
times in the past few days, the last time being 10 hours previously. He was given a drug
thatsedated him, and he fell asleep. Which of the following drugs was most likely used to
counter this patient’s apparent symptoms of dextroamphetamine withdrawal?

A. Phenobarbital.
B. Lorazepam.
C. Cocaine.
D. Hydroxyzine.
E. Fluoxetine.

Correct answer = B. The anxiolytic properties of benzodiazepines, such as lorazepam, make


them the drugs of choice in treating the anxiety and agitation of amphetamine or cocaine

ARIF PHARMA CLINIC Copyright@arifpharma.com Www.arifpharma.com


Page 141

abuse. Lorazepam also has hypnotic properties.


Phenobarbital has hypnotic properties, but its anxiolytic properties are inferior to those of
benzodiazepines. Hydroxyzine, an antihistamine, is effective as a hypnotic, and it is
sometimes used to deal with anxiety, especially if
emesis is a problem. Fluoxetine is an antidepressant with no immediate effects on anxiety or
agitation.

55 JM is a 10-year-old male who is sent to a pediatric neurologist for an evaluation due


to receiving poor grades in class. JM’s parents have recently received complaints
from his teacher that he is performing poorly in school and he is repeatedly caught
not paying attention in class. Several times a day during class, JM is noted to be
getting out of his chair and socializing with other students. He has also been getting
into fights with some children, as he is being in gled out by others and teased. JM is
given a diagnosis of ADHD with impulsivity and irritability. Which of the following is
the most appropriatere commendation for management of the ADHD?

A. Clonidine.
B. Caffeine.
C. Dextroamphetamine.
D. Haloperidol.
E. Buspirone.

Correct answer = C. Dextroamphetamine is the only stimulant medication in the list that is
approved for ADHD. Certain symptoms like fighting may improve with haloperidol and
hyperactivity may improve with clonidine, but these agents
would not improve the patient’s academic performance and the under lying problems.

56 JM is a 10-year-old male with ADHD. His symptoms are currently controlled with an oral
psychostimulant. However, he and his family wish to avoid having to give a second dose
ofmedication at school. They are looking for an alternative treatment option that could
be implemented in the morning and last the entire day. Which treatment option would
be best for JM’s needs?

A. Mixed amphetamine salts in immediate-release oral tablet formulation.


B. Methylphenidate in a transdermal delivery system.
C. Nicotine in a chewing gum formulation for buccal absorption.
D. Methylphenidate in immediate-release pills.
Correct answer = B. Methylphenidate is also a psychostimulant, and the transdermal (patch)
formulation is designed for once-per-day use to avoid middle of the day dosing. Immediate-
release formulations require dosing at least

ARIF PHARMA CLINIC Copyright@arifpharma.com Www.arifpharma.com


Page 142

twice daily. Nicotine is not indicated for ADHD.

57 TT is a 35-year-old male who is interested in quitting smoking. In previous quit attempts, he


has tried nicotine gum, the nicotine patch, and the “cold turkey” method. He has been
unsuccessful in each of these attempts and usually resumed smoking within 4 to 6 weeks.
Which of the following may be useful to assist TT in his attempt to quit smoking?

A. Varenicline.
B. Dextroamphetamine.
C. Lorazepam.
D. Methylphenidate.

Correct answer = A. Varenicline is FDA approved as an adjunctive treatment option for the
management of nicotine dependence. It is believed to attenuate the withdrawal symptoms
of smoking cessation, though continued observation
is needed to monitor for changes in psychiatric status, including suicidal ideation. The use of
dextroamphetamine, lorazepam, and methylphenidate will bring the risk of addiction to another
substance with abuse potential.

UNIT IV: DRUGS AFFECTING THE CARDIOVASCULAR SYSTEM

Chapter 17: Antihypertensives

58 A 45-year-old man was just started on therapy for hypertension and developeda persistent,
dry cough. Which is most likely responsible for this side effect?
A. Enalapril.
B. Losartan.
C. Nifedipine.
D. Prazosin.
E. Propranolol.
Correct answer = A. The cough is most likely an adverse effect of the ACE inhibitor enalapril.
Losartan is an ARB that has the same beneficial effects as anACE inhibitor but is less likely to
produce a cough. Nifedipine, prazosin, and propranolol does not cause this side effect.

59- A 48-year-old hypertensive patient has been successfully treated with a thiazide diuretic
for the last 5 years. Over the last 3 months, his diastolic pressure has steadily increased,

ARIF PHARMA CLINIC Copyright@arifpharma.com Www.arifpharma.com


Page
59 143

and he was started on an additional antihypertensive agent. He complains of several


instances of being unable to achieve an erection and not being able to complete three sets
of tennis as he once did. Which is the likely second antihypertensive medication?

A. Captopril.
B. Losartan.
C. Metoprolol.
D. Minoxidil.
E. Nifedipine.

Correct answer = C. The side effect profile of β-blockers, such as metoprolol, is


characterizedby interference with sexual performance and decreased exercise tolerance.
None of the other drugs is likely to produce this combination of side effects.

60 A 40-year-old male has recently been diagnosed with hypertension due to pressure readings
of 163/102 and 165/100 mm Hg. He also has diabetes that is well controlled with oral
hypoglycemic medications. Which is the best initial treatment regimen for treatment of
hypertension in this patient?

A. Felodipine.
B. Furosemide.
C. Lisinopril.
D. Lisinopril and hydrochlorothiazide.
E. Metoprolol.

Correct answer = D. Because the systolic blood pressure is more than 20 mm Hg above goal
(10 mm Hg above goal diastolic), treatment with two different medications is preferred.
Because the patient is diabetic, he also has a compelling indication for an ACE inhibitor or ARB.

61 A 60-year-old white female has not reached her blood pressure goal after 1month of
treatment with a low dose of lisinopril. All of the following would be appropriate next
steps in the treatment of her hypertension except:

A. Increase dose of lisinopril.


B. Add a diuretic medication.
C. Add on a calcium channel blocker medication.
D. Add on an ARB medication.

Correct answer = D. Increasing the dose of lisinopril or adding a second medication from a
different class (such as a calcium channel blocker or diuretic) would be appropriate steps to

ARIF PHARMA CLINIC Copyright@arifpharma.com Www.arifpharma.com


Page
60 144

control the blood pressure. Adding an ARB as the second medication is not recommended.
ARBs have a similar mechanism of action to ACE inhibitors, and combination therapy may
increase the risk of adverse effects.

62 A patient returns to her health care provider for routine monitoring 3 months after
herhypertension regimen was modified. Labs reveal elevated serum potassium.
Which is likely responsible for this hyperkalemia?

A. Chlorthalidone.
B. Clonidine.
C. Furosemide.
D. Losartan.
E. Nifedipine.

Correct answer = D. Losartan, an ARB, can cause an increase in serum potassium similar to
ACE inhibitors. Furosemide and chlorthalidone can cause a decrease in serum potassium.
Nifedipine and clonidine do not affect potassium levels.

63 A 58-year-old female reports that she recently stopped taking her blood pressure
medications because of swelling in her feet that began shortly after she started
treatment. Which is most likely to cause peripheral edema?

A. Atenolol.
B. Clonidine.
C. Felodipine.
D. Hydralazine.
E. Prazosin.

Correct answer = C. Peripheral edema is one of the most common side effects of
calcium channel blockers. None of the other agents commonly cause peripheral
edema.

64- DD is a 50-year-old male with newly diagnosed hypertension. His comorbidities include
diabetes and chronic hepatitis C infection with moderate liver impairment. He requires
two drugs for initial treatment of his hypertension. Which should be prescribed in
combination with a thiazide diuretic?

A. Lisinopril.
B. Spironolactone.

ARIF PHARMA CLINIC Copyright@arifpharma.com Www.arifpharma.com


Page 145

C. Fosinopril.
D. Furosemide.
E. Hydralazine.

Correct answer = A. Because DD has diabetes, he has a compelling indication for an ACE
inhibitor or ARB for the treatment of his hypertension and prevention of diabetic nephropathy.
However, most ACE inhibitors undergo hepatic conversion to active metabolites, so his hepatic
impairment is of concern. Because lisinopril is one of the two ACE inhibitors that does not
undergo hepatic conversion to active metabolites, it is the best choice. Fosinopril is the only
ACE inhibitor that is not eliminated primarily by the kidneys but does undergo hepatic
conversion. An additional diuretic like spironolactone or furosemide is not indicated. DD does
not have a compelling indication for hydralazine.

Chapter 18: Diuretics

65 An elderly patient with a history of heart disease is brought to the emergency room with
difficulty breathing. Examination reveals that she has pulmonary edema. Which
treatment is indicated?

A. Acetazolamide.
B. Chlorthalidone.
C. Furosemide.
D. Hydrochlorothiazide.
E. Spironolactone.

Correct answer = C. This is a potentially fatal situation. It is important to administer a diuretic


that will reduce fluid accumulation in the lungs and, thus, improve oxygenation and heart
function. The Loop diuretics are most effective in removing large fluid volumes from the body
and are the treatment of choice in this situation. In this situation, furosemide should be
administered intravenously. The other choices are inappropriate.

66- A group of college students is planning a mountain climbing trip to the Andes. Which
wouldbe appropriate for them to take to prevent mountain sickness?

A- A thiazide diuretic such as hydrochlorothiazide.


B- An anticholinergic such as atropine.

ARIF PHARMA CLINIC Copyright@arifpharma.com Www.arifpharma.com


Page 146

C- A carbonic anhydrase inhibitor such as acetazolamide.


D- A loop diuretic such as furosemide.
E- A β-blocker such as metoprolol.

Correct answer = C. Acetazolamide is used prophylactically for several days before an


ascent above 10,000 feet. This treatment prevents the cerebral and pulmonary problems
associated with the syndrome as well as other difficulties, such as nausea.

67 An alcoholic male has developed hepatic cirrhosis. To control the ascites and
edema, which should be prescribed?

A. Acetazolamide.
B. Chlorthalidone.
C. Furosemide.
D. Hydrochlorothiazide.
E. Spironolactone.

Correct answer = E. Spironolactone is very effective in the treatment of hepatic edema. These
patients are frequently resistant to the diuretic action of loop diuretics, although a combination
with spironolactone may be beneficial. The other agents are not indicated.

68 A 55-year-old male with kidney stones has been placed on a diuretic to decrease calcium
excretion. However, after a few weeks, he develops an attack of gout. Which diuretic
washe taking?

A. Furosemide.
B. Hydrochlorothiazide.
C. Spironolactone.
D. Triamterene.
E. Urea.

Correct answer = B. Hydrochlorothiazide is effective in increasing calcium reabsorption, thus


decreasing the amount of calcium excreted, and decreasing the formation of kidney stones
that contain calcium phosphate or calcium oxalate.
However, hydrochlorothiazide can also inhibit the excretion of uric acid and cause its
accumulation, leading to an attack of gout in some individuals. Furosemide increases the
excretion of calcium, whereas the K+-sparing osmotic diuretics, spironolactone and
triamterene, and urea do not have an effect.

69. A 75-year-old woman with hypertension is being treated with a thiazide. Her
bloodpressure responds well and reads at 120/76 mm Hg. After several months
on the

ARIF PHARMA CLINIC Copyright@arifpharma.com Www.arifpharma.com


Page 147

medication, she complains of being tired and weak. An analysis of the blood indicates low
values for which of the following?

A. Calcium.
B. Glucose.
C. Potassium.
D. Sodium.
E. Uric acid.

Correct answer = C. Hypokalemia is a common adverse effect of thiazides and causes fatigue
and lethargy in the patient. Supplementation with potassium chloride or foods high in K+
corrects the problem. Alternatively, a potassium-sparing diuretic, such as spironolactone, may
be added. Calcium, uric acid, and glucose are usually elevated by thiazide diuretics.
Sodium loss would not weaken the patient.

70 A male patient is placed on a new medication and notes that his breasts have
becomeenlarged and tender to the touch. Which medication is he most likely taking?

A. Chlorthalidone.
B. Furosemide.
C. Hydrochlorothiazide.
D. Spironolactone.
E. Triamterene.

Correct answer = D. An adverse drug reaction to spironolactone is gynecomastia due to its


effects on androgens and progesterone in the body. Eplerenone may be a suitable alternative
ifthe patient is in need of an aldosterone antagonist but has a history of gynecomastia.

71 A patient presents to the emergency department with an extreme headache. After a


thorough workup, the attending physician concludes that the pain is due to
increased intracranial pressure. Which diuretic would work best to reduce this
pressure?

A. Acetazolamide.
B. Indapamide.
C. Furosemide.
D. Hydrochlorothiazide.
E. Mannitol.
Correct answer = E. Osmotic diuretics, such as mannitol, are a mainstay of treatment for
patients with increased intracranial pressure or acute renal failure due to shock, drug toxicities,
and trauma.

ARIF PHARMA CLINIC Copyright@arifpharma.com Www.arifpharma.com


Page 148

Chapter 19: Heart Failure

72 BC is a 70-year-old female who is diagnosed with HFrEF. Her past medical history is
significant for hypertension and atrial fibrillation. She is taking hydrochlorothiazide,
lisinopril, metoprolol tartrate, and warfarin. BC says she is feeling “good” and has no cough,
shortness of breath, or edema. Which is the most appropriate medication change to make?

A. Discontinue hydrochlorothiazide.
B. Change lisinopril to losartan.
C. Decrease warfarin dose.
D. Change metoprolol tartrate to metoprolol succinate.

Correct answer = D. Metoprolol succinate should be used in HF, given that there is mortality
benefit shown with metoprolol succinate in landmark HF trials.
Hydrochlorothiazide and warfarin are appropriate based on the information given; there is no
reason to change to an ARB since the patient has no cough or history of angioedema.

73 SC is a 75-year-old white male who has HF. He is seen in clinic today, reporting shortness
of breath, increased pitting edema, and a 5-pound weight gain over the last 2 days. His
current medication regimen includes losartan and metoprolol succinate. SC has no chest
pain and is deemed stable for outpatient treatment. Which of the following is the best
recommendation?

A. Increase the dose of metoprolol succinate.


B. Start hydrochlorothiazide.
C. Start furosemide.
D. Discontinue losartan.

Correct answer = C. As it is possible that SC is having a HF exacerbation, increasing the dose of


the β-blocker is not indicated at this time. There is no reason to stop losartan, based on the
information we have. Loop diuretics are preferred over thiazide diuretics when patients require
diuresis immediately.

ARIF PHARMA CLINIC Copyright@arifpharma.com Www.arifpharma.com


Page 149

Chapter 20: Antiarrhythmics

74 A 60-year-old woman had a myocardial infarction. Which of the following should


beused to prevent life-threatening arrhythmias that can occur post– myocardial
infarction in this patient?

A. Digoxin.
B. Flecainide.
C. Metoprolol.
D. Procainamide.
E. Quinidine.

Correct answer = C. β-Blockers such as metoprolol prevent arrhythmias that occur subsequent
to a myocardial infarction. None of the other drugs has been shown to be effective in
preventing postinfarct arrhythmias. Flecainide should be avoided inpatients with structural
heart disease.

75 A 57-year-old man is being treated for an atrial arrhythmia. He complains of dry mouth,
blurred vision, and urinary hesitancy. Which antiarrhythmic drug is hemostly like taking?

A. Metoprolol.
B. Disopyramide.
C. Dronedarone.
D. Sotalol.

Correct answer = B. The clustered symptoms of dry mouth, blurred vision, and urinary
hesitancy are characteristic of anticholinergic adverse effects which are caused by class IA
agents (in this case, disopyramide). The other drugs do not cause anticholinergic effects.

76 A 58-year-old woman is being treated for chronic suppression of a ventriculararrhythmia.


After 1 week of therapy, she complains about feeling severe upset stomach and
heartburn. Which antiarrhythmic drug is the likely cause of these symptoms?

A. Amiodarone.
B. Digoxin.
C. Mexiletine.
D. Propranolol.
E. Quinidine.

ARIF PHARMA CLINIC Copyright@arifpharma.com Www.arifpharma.com


Page 150

Correct answer = C. The patient is exhibiting a classic adverse effect of mexiletine. None of the
other agents listed are likely to cause dyspepsia.

77 A 78-year-old woman has been newly diagnosed with atrial fibrillation. She is not
currently having symptoms of palpitations or fatigue. Which is appropriate to initiate for
rate controlas an outpatient?

A. Amiodarone.
B. Dronedarone.
C. Esmolol.
D. Flecainide.
E. Metoprolol.
Correct answer = E. Only C and E are options to control rate. The other options are used for
rhythm control in patients with atrial fibrillation. Since esmolol is IV only, the only option to start
as an outpatient is metoprolol.

78 A clinician would like to initiate a drug for rhythm control of atrial fibrillation. Which of
thefollowing coexisting conditions would allow for initiation of flecainide?

A. Hypertension.
B. Left ventricular hypertrophy.
C. Coronary artery disease.
D. Heart failure.

Correct answer = A. Since flecainide can increase the risk of sudden cardiac death in those with
a history of structural heart disease, only A will allow for flecainide initiation. Structural heart
disease includes left ventricular hypertrophy, heart failure, and atherosclerotic heart disease.

Chapter 21: Antianginal Drugs

79- A 72-year-old male presents to the primary care clinic complaining of chest tightness and
pressure that is increasing in severity and frequency. His current medications include
atenolol, lisinopril, and nitroglycerin. Which intervention is most appropriate at this
time?

ARIF PHARMA CLINIC Copyright@arifpharma.com Www.arifpharma.com


Page 151

A. Add amlodipine.
B. Initiate isosorbide mononitrate.
C. Initiate ranolazine.
D. Refer the patient to the nearest emergency room for evaluation.

Correct answer = D. Crescendo angina is indicative of unstable angina that requires further
workup.

80 A 62-year-old patient with a history of asthma and vasospastic angina states that he gets
chest pain both with exertion and at rest, about ten times per week. One sublingual
nitroglycerin tablet always relieves his symptoms, but this medication gives him an awful
headache every time he takes it. Which is the best option for improving his angina?

A. Change to sublingual nitroglycerin spray.


B. Add amlodipine.
C. Add propranolol.
D. Replace nitroglycerin with ranolazine.

Correct answer = B. Calcium channel blockers are preferred for vasospastic angina. β-
Blockers can actually worsen vasospastic angina; furthermore, nonselective β-blockers
should be avoided in patients with asthma. The nitroglycerin spray would also be expected
to cause headache, so this is not the best choice. Ranolazine is not indicated for immediate
relief of an angina attack, not is it a first-line option.

81 A 65-year-old male experiences uncontrolled angina attacks that limit his ability to do
household chores. He is adherent to a maximized dose of β-blocker with a low heart rate
and low blood pressure. He was unable to tolerate an increase in isosorbide
mononitratedue to headache. Which is the most appropriate addition to his antianginal
therapy?

A. Amlodipine.
B. Aspirin.
C. Ranolazine.
D. Verapamil.

Correct answer = C. Ranolazine is the best answer. The patient’s blood pressure is low, so
verapamil and amlodipine may drop blood pressure further. Verapamil may also decrease heart
rate. Ranolazine can be used when other agents are maximized, especially when blood pressure
is well controlled. The patient will need a baseline ECG and lab work to ensure safe use of this
medication.

ARIF PHARMA CLINIC Copyright@arifpharma.com Www.arifpharma.com


Page 152

82 A 68-year-old male with a history of angina had a MI last month, and an echocardiogram
reveals heart failure with reduced ejection fraction. He was continued on his previous home
medications (diltiazem, enalapril, and nitroglycerin), and atenolol was added at discharge.
He has only had a few sporadic episodes of stable angina that are relieved with
nitroglycerin or rest. What are eventual goals for optimizing this medication regimen?

A. Add isosorbide mononitrate.


B. Increase atenolol.
C. Stop atenolol and increase diltiazem.
D. Stop diltiazem and change atenolol to bisoprolol.

Correct answer = D. nondihydropyridine calcium channel blockers such as diltiazem should


be avoided in patients with heart failure with reduced ejection fraction. Patients should be
treated with one of three β-blockers approved for heart failure with reduced ejection
fraction (bisoprolol, metoprolol succinate, or carvedilol). It sounds like his angina
symptoms are well managed with his current therapy so adding isosorbide mononitrate
would not be necessary. These symptoms may become even less frequent as his new β-
blocker is titrated.

83 A patient whose angina was previously well controlled with once-daily


isosorbide mononitrate states that recently he has been taking isosorbide on
ocitrate twice a day to control angina symptoms that are occurring more
frequently during early morning hours. Which of the following is the best option
for this patient?

A.Continue once-daily administration of isosorbide mononitrate but advise the patient


totake this medication in the evening.
B.Advise continuation of isosorbide mononitrate twice daily for full 24-hour coverage of
anginal symptoms.
C.Switch to isosorbide dinitrate, as this has a longer duration of action than the
mononitrate.
D.Switch to a nitroglycerin patch for consistent drug delivery and advise him to wear
thepatch around the clock.

Correct answer = A. It is important to maintain a nitrate-free period to prevent the


development of tolerance to nitrate therapy. The mononitrate formulation has a longer
half-life. The nitroglycerin patch should be taken off for 10 to 12 hours daily to allow
fornitrate-free intervals.

ARIF PHARMA CLINIC Copyright@arifpharma.com Www.arifpharma.com


Page 153

Chapter 22: Anticoagulants and Antiplatelet Agent

84 A 70-year-old female is diagnosed with nonvalvular atrial fibrillation. Her past


medical history is significant for chronic kidney disease, and her renal function is
moderately diminished. All of the following anticoagulants
would be expected to require a reduced dosage in this patient except:

A. Apixaban.
B. Dabigatran.
C. Rivaroxaban.
D. Warfarin.

Correct answer = D. Warfarin does not require dosage adjustment in renal dysfunction. The
INR is monitored and dosage adjustments are made on the basis of this information. All of the
other agents arereallyy cleared to some extent and require dosage adjustments in renal
dysfunction.

85- An 80-year-old male is taking warfarin indefinitely for the prevention of deep venous
thrombosis. He is a compliant patient with a stable INR and has no issues with bleeding or
bruising. He is diagnosed with a urinary tract infection and is prescribed
sulfamethoxazole/ trimethoprim. What effect will this have on his warfarin therapy?

A. Sulfamethoxazole/trimethoprim will decrease the anticoagulant effect of warfarin.


B. Sulfamethoxazole/trimethoprim will increase the anticoagulant effect of warfarin.
C. Sulfamethoxazole/trimethoprim will activate platelet activity.
D. Sulfamethoxazole/trimethoprim will not change anticoagulation status.

Correct answer = B. Sulfamethoxazole/trimethoprim has a significant drug interaction with


warfarin, such that it will inhibit warfarin metabolism. Therefore, sulfamethoxazole/
trimethoprim will cause increased anticoagulation, and the patient will need to have his
warfarin dose decreased and INR checked frequently while he is on this antibiotic.

86- A 56-year-old man presents to the emergency room with complaints of swelling,
redness, and pain in his right leg. The patient is diagnosed with acute DVT and
requirestreatment with an anticoagulant. All of the following are approved for the
treatment of this patient’s DVT except:

ARIF PHARMA CLINIC Copyright@arifpharma.com Www.arifpharma.com


Page 154

A. Rivaroxaban.
B. Dabigatran.
C. Enoxaparin.
D. Heparin.

Correct answer = B. Dabigatran is only approved for the prevention of stroke in


nonvalvular atrial fibrillation; it is not approved for the treatment of acute DVT. All of the
other options are approved for treatment of acute DVT.

87 A 62-year-old male taking warfarin for stroke prevention in atrial fibrillation presents to
his primary care physician with an elevated INR of 10.5 without bleeding. He is instructed
to hold his warfarin dose and given 2.5 mg of oral vitamin K1. When would the effects of
vitamin K on the INR most likely be noted in this patient?

A. 1 hour.
B. 6 hours.
C. 24 hours.
D. 72 hours.

Correct answer = C. Vitamin K1 takes about 24 hours to see a reduction in the INR. This is
due to the time required for the body to synthesize new coagulation factors.

E. Rivaroxaban.
F. Dabigatran.
G. Enoxaparin.
H. Heparin.

Correct answer = B. Dabigatran is only approved for the prevention of stroke in


nonvalvular atrial fibrillation; it is not approved for the treatment of acute DVT .All of the
other options are approved for treatment of acute DVT.

88 A 62-year-old male taking warfarin for stroke prevention in atrial fibrillation presents to
his primary care physician with an elevated INR of 10.5 without bleeding. He is instructed
to hold his warfarin dose and given 2.5 mg of oral vitamin K1. When would the effects of
vitamin K on the INR most likely be noted in this patient?

E. 1 hour.
F. 6 hours.
G. 24 hours.
H. 72 hours.

Correct answer = C. Vitamin K1 takes about 24 hours to see a reduction in the INR. This is
due to the time required for the body to synthesize new coagulation factors.

ARIF PHARMA CLINIC Copyright@arifpharma.com Www.arifpharma.com


Page 155

89 A 58-year-old man receives intravenous alteplase treatment for acute stroke.


Five minutes following the completion of alteplase infusion, he develops
orolingualangioedema. Which of the following drugs may have increased the
risk of developing orolingual angioedema in this patient?

A. ACE inhibitor.
B. GP IIb/IIIa receptor antagonist.
C. Phosphodiesterase inhibitor.
D. Thiazide diuretic.

Correct answer = A. ACE inhibitors, aspirin, and prasugrel all have possible adverse
effects including orolingual angioedema. In the setting of alteplase administration,
ACE inhibitors have been associated with an increased risk of developing orolingual
angioedema with concomitant use.

Chapter 23: Drugs for Hyperlipidemia

90 JS is a 65-year-old man who presents to his physician for the management of


hyperlipidemia. His most recent lipid panel reveals an LDL cholesterol level of
165 mg/ dL. His physician wishes to begin treatment to lower his
LDL cholesterol levels. Which of the following therapies is the best option to
lower JS’s LDL cholesterol levels?

A. Fenofibrate.
B. Colesevelam.
C. Niacin.
D. Simvastatin.
E. Ezetimibe.

Correct answer = D. Simvastatin, an HMG CoA reductase inhibitor (statin), is the most
effective option for lowering LDL cholesterol, achieving reductions of 30% to 41%
from baseline levels. Fenofibrate and niacin are more effective at lowering
triglyceride levels or raising HDL levels (niacin). Colesevelam can reduce LDL levels
but not as effectively as statins. Ezetimibe lowers LDL levels modestly compared to
the LDL reduction achieved by statins.
Page 156

91- WW is a 62-year-old female with hyperlipidemia and hypothyroidism. Her current


medications include cholestyramine and levothyroxine (thyroid hormone). What
advice would you give to WW to avoid a drug interaction between her
cholestyramine and levothyroxine?

A- Stop taking levothyroxine as it can interact with cholestyramine.


B- Take levothyroxine 1 hour before cholestyramine on an empty stomach.
C- Switch cholestyramine to colesevelam as this will eliminate the interaction.
D- Switch cholestyramine to colestipol as this will eliminate the interaction.
E- Take levothyroxine and cholestyramine at the same time to minimize the
interaction.

Correct answer = B. Cholestyramine and the bile acid resins can bind several medications
causing decreased absorption. Cholestyramine can decrease the absorption of
medications such as levothyroxine. Taking levothyroxine 1 hour before or 4 to 6 hours
after cholestyramine can help to avoid this interaction. Choices C and D are incorrect, as
all bile acid resins cause this interaction. Choice A is incorrect, as this patient should not
stop her thyroid medication. Choice E will worsen this drug interaction.

92AJ is a 42-year-old man who was started on niacin sustained-release tablets 2


weeks ago for elevated triglycerides and low HDL levels. He is complaining of an
uncomfortable flushing and itchy feeling that he thinks is related to the niacin.
Which of the following options can help AJ manage this adverse effect of niacin
therapy?

A. Administer aspirin 30 minutes prior to taking niacin.


B. Administer aspirin 30 minutes after taking niacin.
C. Increase the dose of niacin SR to 1000 mg.
D. Continue the current dose of niacin.
E. Change the sustained-release niacin to immediate-release niacin.

Correct answer = A. Flushing associated with niacin is prostaglandin mediated;


therefore, use of aspirin (a prostaglandin inhibitor) can help to minimize this adverse
effect. It must be administered 30 minutes prior to the dose of
the niacin; therefore, choice B is incorrect. Increasing the dose of niacin is likely to
increase these complaints; therefore, choice C is incorrect. Continuing the current
dose is unlikely to relieve these complaints, which are bothersome to AJ. The
sustained-release formulation of niacin has less incidence of flushing versus that of the
immediate release; therefore, choice E is incorrect.
Page 157

93 CN is a 72-year-old male who is treated for hyperlipidemia with high-dose


atorvastatin for the past 6 months. He also has a history of renal insufficiency. His
most recent lipid panel shows an LDL cholesterol level of 131 mg/dL, triglycerides
of510 mg/dL, and HDL cholesterol of 32 mg/dL. His physician wishes to add an
additional agent for his hyperlipidemia. Which of the following choices is the best
option to address CN’s dyslipidemia?

A. Fenofibrate.
B. Niacin.
C. Colesevelam.
D. Gemfibrozil.
E. Ezetimibe.
Correct answer = B. This patient has significantly elevated triglycerides and low
HDL. Niacin can lower triglycerides by 35% to 50% and also raise HDL levels. The
fibrates (fenofibrate and gemfibrozil) should not be used due to CN’s history of renal
insufficiency The usese of colesevelam is contraindicated because triglycerides are
greater than 400 mg/dL. Ezetimibe can further lower LDL cholesterol but has
modest effects on triglycerides versus niacin.

UNIT V: DRUGS AFFECTING THE ENDOCRINE SYSTEM

Chapter 24: Pituitary and Thyroid

94 A 40-year-old female is undergoing infertility treatments. Which of the


following drugs might be included in her treatment regimen?

A. Cabergoline.
B. Follitropin.
C. Methimazole.
D. Vasopressin.

Correct answer = B. Follitropin is a recombinant version of FSH that causes ovarian


follicular growth and maturation. Cabergoline is a dopamine agonist that is used for
hyperprolactinemia. Methimazole is the treatment of choice for hyperthyroidism.
Vasopressin is an antidiuretic hormone.
Page 158

Chapter 25: Drugs for Diabetes

95 DW is a patient with type 2 diabetes who has a blood glucose of 400 mg/dL today
at his office visit. The physician would like to give them some insulin to bring the
glucosedown before he leaves the office. Which of the following would lower the
glucose in the quickest manner in DW?

A. Insulin aspart.
B. Insulin glargine.
C. NPH insulin.
D. Regular insulin.

Correct answer = A. Insulin aspart is a rapid-acting insulin that has an onset of action
within 15 to 20 minutes. Insulin glargine is a long-acting insulin that is used for basal
control. NPH insulin is an intermediate-acting insulin that is used for basal control.
Although regular insulin can be used to bring glucose down, its onset is not as
quick as insulin aspart. The onset of regular insulin is about 30 to 60 minutes.

96 A 64-year-old woman with a history of type 2 diabetes is diagnosed with heart


failure. Which of the following medications would be a poor choice for controlling
her diabetes?

A. Exenatide.
B. Glyburide.
C. Nateglinide.
D. Pioglitazone.
E. Sitagliptin.

Correct answer = D. The TZDs (pioglitazone and rosiglitazone) can cause fluid
retention and lead to a worsening of heart failure. They should be used with
caution and dose reduction, if at all, in patients with heart failure. Exenatide,
glyburide, nateglinide, and sitagliptin do not have precautions for use in heart
failure patients.

97 KD is a 69-year-old male with type 2 diabetes and advanced chronic kidney


disease. Which of the following diabetes medications is contraindicated in this
patient?

A. Glipizide.
B. Insulin lispro.
C. Metformin.
D. Saxagliptin.
Page 159

Correct answer = C. Metformin should not be used in patients with kidney disease
due to the possibility of lactic acidosis. Glipizide can be used safely in patients with
CrCl as low as 10 mL/min. Insulin is not contraindicated in renal dysfunction,
although the dosage may need to be adjusted. While the dose of the DPP-4 inhibitor
saxagliptin may need to be reduced in renal dysfunction, it is not contraindicated.

98 A patient with type 2 diabetes is taking metformin. The fasting glucose levels are
in range, but the postprandial glucose is uncontrolled. All of the following drugs
would be appropriate to add to metformin to target postprandial glucose except:

A. Acarbose.
B. Exenatide.
C. Insulin aspart.
D. Pramlintide.

The correct answer = D. Although all of these drugs target postprandial glucose,
pramlintide should only be used in conjunction with mealtime insulin. Since this
patient is not on insulin, pramlintide is not indicated.

Chapter 26: Estrogens and Androgens

99 A 53-year-old woman has severe vasomotor symptoms (hot flushes) associated


with menopause. She has no pertinent past medical or surgical history. Which of
the following would be most appropriate for her symptoms?

A. Conjugated estrogens vaginal cream.


B. Estradiol transdermal patch.
C. Oral estradiol and medroxyprogesterone acetate.
D. Injectable medroxyprogesterone acetate.

Correct answer = C. Estrogen vaginal cream only treats vaginal symptoms of


menopause such as vaginal atrophy and does not treat hot flushes. Since this
patient has an intact uterus, a progestin such as medroxyprogesterone needs to be
used along with estrogen to prevent the development of endometrial hyperplasia.
Unopposed estrogen (for example, the estradiol transdermal patch) should not be
used. Injectable medroxyprogesterone acetate is used for contraception.

ARIF PHARMA CLINIC Copyright@arifpharma.com Www.arifpharma.com


Page 160

100- A 70-year-old woman is being treated with raloxifene for osteoporosis. Which
of the following is a concern with this therapy?

A. Breast cancer.
B. Endometrial cancer.
C. Venous thrombosis.
D. Hypercholesterolemia.

Correct answer = C. Raloxifene can increase the risk of venous thromboembolism.


Unlike estrogen and tamoxifen, raloxifene does not result in an increased incidence of
endometrial cancer. Raloxifene lowers the risk of breast cancer
in high-risk women, and it also lowers LDL cholesterol.

100-A 26-year-old female is using injectable medroxyprogesterone acetate as a


method of contraception. Which of the following adverse effects is a concern if
she wishes to use this therapy long-term?

A- Hyperkalemia.
B- Male pattern baldness.
C- Osteoporosis.
D- Weight loss.

Correct answer = C. Medroxyprogesterone acetate may contribute to boneloss and


predispose patients to osteoporosis and/or fractures. Therefore, thedrug should not
be continued for more than 2 years if possible. The drug often causes weight gain, not
weight loss. The other adverse effects are notassociated with medroxyprogesterone.

Chapter 27: Adrenal Hormones

101-A child with severe asthma is being treated with high doses of inhaled
corticosteroids. Which of the following adverse effects is of particular
concern?

A. Hypoglycemia.
B. Hirsutism.
C. Growth suppression.
D. Cushing syndrome.
E. Cataract formation.
Page 161

Correct answer = C. Corticosteroids may retard bone growth. Chronic treatment with
the medication therefore may lead to growth suppression, so linear growth should be
monitored periodically. Hyperglycemia, not hypoglycemia, is a possible adverse effect.
Hirsutism, Cushing syndrome, and cataract formation are unlikely with the dose that
the child would receive by inhalation.

102-A patient with Addison disease is being treated with hydrocortisone but is still
having problems with dehydration and hyponatremia. Which of the following
drugs would be best to add to the patient’s therapy?

A. Dexamethasone.
B. Fludrocortisone.
C. Prednisone.
D. Triamcinolone.

Correct answer = B. To combat dehydration and hyponatremia, a corticosteroid


with high mineralocorticoid activity is
needed. Fludrocortisone has the greatest mineralocorticoid activity of the agents
provided. The other drugs have little or
no mineralocorticoid activity.

Chapter 28: Drugs for Obesity

103-45-year-old female presents seeking treatment for weight loss. She has tried
several fad diets in the past with very little success. She exercises twice weekly
at the gym for 30 minutes and tries to watch what she eats. Her BMI is 31 and
she has diabetes and uncontrolled hypertension. Which of the following
medications would be most appropriate to treat her obesity?

A. Phentermine.
B. Phentermine/topiramate.
C. Orlistat.
D. Diethylpropion.

Correct answer = C. Orlistat is the only medication of those listed that does not
increase heart rate and blood pressure. Since this patient’s blood pressure is
currently uncontrolled, choosing a drug that does not affect blood pressure would be
best at this time.
104-A 38-year-old obese male with depression is considering a weight loss
medication following several failed attempts with diet and exercise. Which of
the followingg medications should be avoided in this individual?
Page 162

A. Phentermine.
B. Phentermine/topiramate.
C. Orlistat.
D. Diethylpropion.
E. Lorcaserin.
Correct answer = E. Lorcaserin may cause suicidal ideation and would not be
advisable for an individual with depression. Also, he is likely on a medication that may
increase serotonin levels. The addition of lorcaserin, a serotonin receptor agonist,
could lead to serotonin syndrome. Therefore, avoidance of the combination is
advisable.

105-A 27-year-old recently married female is asking about treatment options for
her obesity. She recently stopped taking her birth control medications, as she
felt these were contributing to her weight gain. Which of the following
medications should be avoided in this patient?

A. Phentermine.
B. Phentermine/topiramate.
C. Orlistat.
D. Diethylpropion.
E. Lorcaserin.

Correct answer = B. The topiramate component of this medication is contraindicated


in pregnancy. Since this patient stopped her birth control, she is at risk of becoming
pregnant and her fetus is at risk of developing birth defects if she is taking this
medication.

UNIT VI: DRUGS FOR OTHER DISORDERS

Chapter 29: Drugs for Disorders of the Respiratory System

106-A 12-year-old girl with a childhood history of asthma complained of cough,


dyspnea, and wheezing after visiting a riding stable. Her symptoms became so
severe that her parents brought her to the emergency room. Which of the
following is the most appropriate drug to rapidly reverse her
bronchoconstriction?

A. Inhaled fluticasone.
B. Inhaled beclomethasone.
C. Inhaled albuterol.
D. Intravenous propranolol.
E. Oral theophylline.
Page 163

Correct answer = C. Inhalation of a rapid-acting β2 agonist, such as albuterol,


usually provides immediate bronchodilation. An acute asthmatic crisis often
requires intravenous corticosteroids, such as methylprednisolone. Inhaled
beclomethasone and fluticasone treat chronic airway inflammation but will not
provide any immediate effect. Propranolol is a nonselective β-blocker and would
aggravate the patient’s bronchoconstriction. Theophylline has been largely
replaced with β2 agonists and is no longer recommended for acute bronchospasm.

107-A 9-year-old girl has severe asthma, which required three hospitalizations in
the last year. She is now receiving therapy that has greatly reduced the
frequency of these severe attacks. Which of the following therapies is most
likely responsible for this benefit?

A. Inhaled albuterol.
B. Inhaled ipratropium.
C. Inhaled fluticasone.
D. Oral theophylline.
E. Oral zafirlukast.

Correct answer = C. Administration of a corticosteroid directly to the lung significantly


reduces the frequency of severe asthma attacks. This benefit is accomplished with
minimal risk of the severe systemic adverse effects of oral corticosteroid therapy.
Albuterol is used only to treat acute asthmatic episodes. The other agents may reduce
the severity of attacks, but not to the same degree or consistency as fluticasone (or
other corticosteroids).

108-A 68-year-old male has COPD with moderate airway obstruction. Despite
using salmeterol twice daily as prescribed, he reports continued symptoms of
shortness of breath with mild exertion. Which one of the following agents
would be an appropriate addition to his current therapy?

A. Systemic corticosteroids.
B. Albuterol.
C. Tiotropium.
D. Roflumilast.
E. Theophylline.

Correct answer = C. The addition of an anticholinergic bronchodilator to the LABA


salmeterol would be appropriate and provide additional therapeutic benefits.
Systemic corticosteroids are used to treat exacerbations in patients with COPD,
but not
recommended for chronic use. The addition of a SABA (albuterol) is less likely to
provide additional benefit since the patient is already using medication
Page 164

with the same mechanism of action. Roflumilast is not indicated as the patient only
has moderate airway obstruction. Theophylline is an oral bronchodilator that is
beneficial to some patients with stable COPD. However, because of its toxic potential,
its use is not routinely recommended.

109-A 58-year-old female ceramics worker with a COPD exacerbation has recently
been discharged from the hospital. This is the third hospitalization in the past
year for this condition, although the patient reports only mild symptoms in
between exacerbations. The patient is currently still on the samedrug regimen
prior to her admission of salmeterol inhalation twice daily and tiotropium
inhalation once daily. Her current FEV1 is below 60%. Which of thefollowing
would be an appropriate change in her medication regimen?

A. Chronic systemic corticosteroids.


B. Discontinue the tiotropium.
C. Discontinue the salmeterol.
D.Change the salmeterol to a combination product that includes both a LABA and
an inhaled corticosteroid (for example, salmeterol/fluticasone DPI).
E. Theophylline.

Correct answer = D. The addition of an inhaled corticosteroid may provide additional


benefit since the patient has significant airway obstruction and frequent
exacerbations requiring hospitalization. Systemic corticosteroids are used
on a short-term basis to treat exacerbations in patients with COPD but are not
recommended for chronic use. It is not routinely recommended to discontinue a
long-acting bronchodilator unless the patient experiences an adverse effect or
experiences no therapeutic benefit. In this case, the patient reports only mild
symptoms in between exacerbations, suggesting she may be benefiting from both
bronchodilators. Theophylline is an oral bronchodilator that is beneficial to some
patients with stable COPD. However, because of its toxic potential, its use is not
routinely recommended.

110-A 32-year-old male with a history of opioid addiction presents with


symptoms of an upper respiratory system infection for the past 5 days. It
is determined to be viral in nature, and no treatment of the underlying
infection
is appropriate. Which of the following is appropriate symptomatic treatment
for this patient’s cough?

A. Guaifenesin/dextromethorphan.
B. Guaifenesin/codeine.
C. Cromolyn.
D. Benzonatate.
E. Montelukast.
Page 165

Correct answer = D. Benzonatate suppresses the cough reflex through peripheral action
and has no abuse potential. Dextromethorphan, an opioid derivative, and codeine, an
opioid, both have abuse potential. Neither cromolyn nor montelukast is indicated for
cough suppression.

Chapter 30: Antihistamines

111-A 43-year-old heavy machine operator complains of seasonal allergies.


Which one of the following medications would be most appropriate for
management of his allergy symptoms?

A. Cyclizine.
B. Doxylamine.
C. Hydroxyzine.
D. Fexofenadine.
Correct answer = D. The use of first-generation H1 antihistamines is contraindicated
in the treatment of pilots and others who must remain alert. Because of its lower
potential to induce drowsiness, fexofenadine may be recommended for individuals
working in jobs in which wakefulness is critical.

112-A passenger sitting next to you on a plane boasts that he was a famous
biochemist. He said he carboxylated a sedating antihistamine, and it is now
only partially sedating and is a very well-known drug on the market.
Which drug is he talking about?

A. Hydroxyzine.
B. Cetirizine.
C. Diphenhydramine.
D. Doxylamine.
E. Cyproheptadine.

Correct answer = B. Choices A, C, D, and E are first-generation antihistamines and are


known to cross the blood–brain barrier. Cetirizine is carboxylated hydroxyzine.

113-Your neighbor said she used an H1 antihistamine that was available over-the-
counter (OTC), and it caused her marked drowsiness and dry mouth and she
slept quite longer than usual. Which is the most possible drug that she used?

A. Loratadine.
B. Levocetirizine.
C. Diphenhydramine.
D. Fexofenadine.
Page 166

E. Desloratadine.

Correct answer = C. The only first-generation drug in the list is diphenhydramine.


Diphenhydramine and doxylamine, another first-generation antihistamine, are
common ingredients in OTC sleep products.

114- A patient is going on a deep-sea fishing trip and is worried about motion
sickness. Which of the following would be the most appropriate?

A. Dimenhydrinate 1 hour prior to departure.


B. Desloratadine 1 hour prior to departure.
C. Doxylamine 1 hour prior to departure.
D. Meclizine at the onset of symptoms.

Correct answer = A. Dimenhydrinate and meclizine are both useful for preventing the
symptoms of motion sickness. However, they are much more effective in preventing
symptoms than treating symptoms once they have started.
Therefore, they should be taken prior to expected travel/ boating, etc.
Desloratadine and doxylamine are not useful for motion sickness.

115- A patient has a severe ear infection that is associated with significant vertigo.
Which of the following might be helpful?

A. Azelastine.
B. Brompheniramine.
C. Meclizine.
D. Olopatadine.

Correct answer = C. Meclizine is useful for the treatment of vertigo associated with
vestibular disorders. Azelastine and olopatadine are ophthalmic or intranasal
antihistamines, but they are not useful for symptoms of ear infection.
Brompheniramine is a first-generation antihistamine that is mainly used for allergy
symptoms.

Chapter 31: Gastrointestinal and Antiemetic Drugs

116-A 68-year-old patient with cardiac failure is diagnosed with ovarian cancer. She
begins using cisplatin but becomes nauseous and suffers from severe vomiting.
Which of the following medications would be most effective to counteract the
emesis in this patient without exacerbating her cardiac problem?
Page 167

A. Droperidol.
B. Dolasetron.
C. Prochlorperazine.
D. Dronabinol.
E. Palonosetron.

Correct answer = E. Palonosetron is a 5-HT3 antagonist that is effective against drugs


with high emetogenic activity, such as cisplatin. Although dolasetron is also in this
category, its propensity to affect the heart makes it a poor choice for this patient.
Droperidol also affects the heart and now is generally a second-line drug used in
combination with opioids or benzodiazepines. The antiemetic effect of
prochlorperazine, a phenothiazine, is most beneficial against anticancer drugs with
moderate to low emetogenic properties.

117-A 45-year-old woman complains of persistent heartburn and an unpleasant,


acid-like taste in her mouth. The clinician suspects that she has
gastroesophageal reflux disease and advises her to raise the head of her bed 6
to 8 inches, not to eat for several hours before retiring, and to eat smaller
meals. Two weeks later, she returns and says the symptoms have subsided
slightly but still are a concern. The clinician will likely prescribe which one of
the following drugs?

A. An antacid such as aluminum hydroxide.


B. Dicyclomine.
C. An antianxiety agent such as alprazolam.
D. Esomeprazole.

Correct answer = D. It is appropriate to treat this patient with a proton-pump


inhibitor (PPI) to reduce acid production and promote healing. An H2-receptor
antagonist might also be effective, but the PPIs are preferred. An antacid would
decrease gastric acid, but its effects are short-lived compared to those of the PPIs
and H2-receptor inhibitors. Dicyclomine is an antimuscarinic drug and would
decrease acid production, but it is not as effective as the PPIs or the H2-receptor
inhibitors. An antianxiety agent might have antiemetic action but would have no
effect on acid production.

118- A couple celebrating their 40th wedding anniversary are given a trip to
Peru to visit Machu Picchu. Due to past experiences while traveling, they ask
their doctor to prescribe an agent for diarrhea. Which of the following would
be effective?
Page 168

A. Omeprazole.
B. Loperamide.
C. Famotidine.
D. Lorazepam.

Correct answer = B. Loperamide is the only drug in this set that has antidiarrheal
activity. Omeprazole is a proton-pump inhibitor, famotidine antagonizes the H2
receptor, and lorazepam is a benzodiazepine that is a sedative and an anxiolytic
agent.

119-A 27-year-old woman who is 34 weeks pregnant is on bed rest and visits her
obstetrician. During the visit, she informs her physician that she has been
experiencing mild constipation. Which of the following medications will most
likely be recommended to her?

A- Castor oil.
B- Mineral oil.
C- Loperamide.

Correct answer = B. Although its effects are not immediate, docusate may be used
for mild constipation and is generally considered safe in pregnancy. Castor oil
should not be used in pregnancy because of its ability to cause uterine
contractions. Mineral oil should not be used in bedridden patients due to the
possibility of aspiration. Loperamide is used for diarrhea, not constipation.

120 A patient is receiving treatment with lorazepam prior to chemotherapy to help


reduce her anticipatory nausea and vomiting. Which of the following should
generally be avoided in this patient?

A. Docusate.
B. Ondansetron.
C. Polyethylene glycol.
D. Ethanol.

Correct answer = D. Ethanol combined with benzodiazepines, particularly at high


doses, may produce unconsciousness, respiratory depression, and even death. The
other drugs listed here have not shown a specific drug interaction with
benzodiazepines.
Page 169

Chapter 32: Drugs for Urologic Disorders

121- A patient who is taking a PDE-5 inhibitor for ED is diagnosed with angina.
Which of the following antianginal medications would be of particular concern in
this patient?

A. Metoprolol.
B. Diltiazem.
C. Amlodipine.
D. Nitroglycerin.

Correct answer = D. Nitrates, when taken with PDE-5 inhibitors, can cause life-
threatening hypotension. While metoprolol, diltiazem, and amlodipine may all
lower blood pressure, the interaction with PDE-5 inhibitors is not relevant.

122- A patient is worried about starting terazosin because he is very sensitive to


the side effects of medications. Which of the following adverse effects would be
mostexpected in this patient?

A. Erectile dysfunction.
B. Gynecomastia.
C. Dizziness.
D. Vomiting.

Correct answer = C. Because of the α-blocking properties, terazosin commonly


causes dizziness (this may be related to orthostatic hypotension). ED and
gynecomastia would be unexpected with α-blockers. While most any drug may
cause nausea and vomiting, terazosin is much more likely to cause dizziness.

123- A 70-year-old male with BPH and an enlarged prostate continues to have
urinary symptoms after an adequate trial of tamsulosin. Dutasteride is added to his
therapy. In addition to tamsulosin, he is also taking hydrochlorothiazide,
testosterone, and vardenafil as needed prior to intercourse. Which of his
medications could have an interaction with dutasteride?

A. Hydrochlorothiazide.
B. Tamsulosin.
C. Testosterone.
D. Vardenafil.

Correct answer = C. Because dutasteride prevents the conversion of testosterone to


Page 170

the more active form, DHT, these medications have an interaction. Essentially,
dutasteride prevents testosterone from “working.” Hydrochlorothiazide
does not interfere with the metabolism of dutasteride, and dutasteride does not have
any effect on the blood pressure-lowering effects of hydrochlorothiazide. Tamsulosin
is appropriate in combination with a 5-α reductase inhibitor when the prostate is
enlarged. Vardenafil is only prescribed as needed, and the two drugs do not have a
pharmacokinetic interaction.

Chapter 33: Drugs for Anemia

124- A 56-year-old female is discovered to have megaloblastic anemia. Her past


medical history is significant for alcoholism. Which of the following would be the
best treatment option for this patient?

A. Oral vitamin B12.


B. Parenteral vitamin B12.
C. Oral folate.
D. Oral vitamin B12 with oral folate.

Correct answer = D. The patient has a history of alcoholism, which would suggest folic
acid deficiency anemia. However, folic acid administration alone reverses the
hematologic abnormality and masks possible vitamin B12 deficiency, which can then
proceed to severe neurologic dysfunction and disease. The cause of megaloblastic
anemia needs to be determined in order to be specific in terms of treatment.
Therefore, megaloblastic anemia should not be treated with folic acid alone but,
rather, with a combination of folate and vitamin B12.

125- A 60-year-old female presents to her primary care physician complaining of


dizziness and fatigue. Following laboratory testing, the patient is diagnosed with
iron deficiency anemia, and oral iron supplementation is needed. Which of the
following would be the most appropriate dosing regimen for the patient?

A. Ferrous fumarate 325 mg once daily.


B. Ferrous gluconate 256 mg once daily.
C. Polysaccharide–iron complex 150 mg two to three times daily.
D. Ferrous sulfate 325 mg two to three times daily.

Correct answer = D. The recommended dose of iron supplementation in iron


deficiency anemia is typically about 150 mg of elemental iron in two to three divided
doses. Extended-release formulations (such as polysaccharide– iron complex) may be
dosed once daily. Ferrous sulfate 325 mg contains approximately65 mg of elemental
Page 171

iron, ferrous fumarate 325 mg contains about 107 mg elemental iron, ferrous
gluconate 256 mg contains approximately 30 mg of elemental iron, and polysaccharide–iron complex 150 mg
contains 150 of mg elemental iron.

126. A 63-year-old female patient with anemia secondary to chronic kidney disease
and a hemoglobin level of 8.6 g/dL is treated with epoetin alfa. Eight days after the
initial dose of epoetin alfa, the patient’s hemoglobin is 11.3 mg/dL. Why is it
appropriate to discontinue treatment with epoetin alfa?

A- Treatment goals of hemoglobin greater than 12 g/dL and a rise in hemoglobin of greater
than 1 g/dL in a 2-week period are associated with cardiovascular events and decreased
survival.

B- The patient has not responded to the epoetin alfa and therefore requires
treatment with a different agent for her anemia.

C- Epoetin alfa is less effective than darbepoetin alfa, and treatment with epoetin alfa
should be transitioned to darbepoetin to receive maximum benefit.

D- Epoetin alfa is not indicated for treatment of anemia secondary to chronic kidney
disease.

Correct answer = A. Answer B is incorrect because the patient has responded to the
epoetin alfa, as the patient’s hemoglobin has increased following its administration.
Answer C is incorrect because there is no clear evidence
to claim that either agent is more effective than the other in treatment of anemia.
Answer D is incorrect because epoetin alfa is indicated for the treatment of anemia
secondary to chronic kidney disease.

Chapter 34: Drugs for Dermatologic Disorders

127. A 3-year-old boy has contracted scabies from his playmate at the daycare
center. Which of the following would be the most appropriate treatment?

A. Azelaic acid.
B. Mupirocin.
C. Permethrin.
D. Triple antibiotic ointment.
Correct answer = C. Permethrin is a topical scabicide that is preferred due to its lower
risk of neurotoxicity. Azelaic acid is a topical treatment for acne. Mupirocin and triple
antibiotic ointment are used for the treatment of bacterial infections and would not
Page 172

be appropriate for scabies.


Chapter 35: Drugs for Bone Disorders

128- OP is a 65-year-old female who has been diagnosed with postmenopausal


osteoporosis. She has no history of fractures and no other pertinent medical
conditions. Which of the following would be most appropriate for management of
her osteoporosis?

A. Alendronate.
B. Calcitonin.
C. Denosumab.
D. Raloxifene.
E. Teriparatide.
Correct answer = A. Bisphosphonates are first-line therapy for osteoporosis in
postmenopausal women without contraindications. Calcitonin and raloxifene are
alternatives but may be less efficacious (especially for nonvertebral fractures).
Teriparatide and denosumab should be reserved for patients at high risk or those
who fail other therapies.

130 TT is a 55-year-old female who has been diagnosed with postmenopausal


osteoporosis. She has a past medical history of ethanol abuse, alcoholic liver disease,
erosive esophagitis, and hypothyroidism. Which of the following would be the
primary reason oral bisphosphonates should be used with caution in this patient?

A. Age.
B. Erosive esophagitis.
C. Liver disease.
D. Thyroid disease.

Correct answer = B. Bisphosphonates are known to cause esophageal irritation and


should be used with caution in a patient with a history of erosive esophagitis. Age is
not a factor for consideration in bisphosphonate use. Liver disease is not a
contraindication to bisphosphonate use, since bisphosphonates are mainly cleared via
the kidney. Thyroid disease is not a contraindication to bisphosphonate use, although
overaggressive replacement of thyroid may contribute to osteoporosis.

131- VS is a 70-year-old female who is being started on ibandronate once monthly


for the treatment of osteoporosis. Which of the following is important to
communicate to this patient?
Page 173

A. Take this medication with orange juice to increase absorption.


B. Take this medication after meals to minimize stomach upset.
C. Remain upright for at least 60 minutes after taking this medication.
D. Adverse effects may include blood clots and leg cramps.

Correct answer = C. Patients need to remain upright for 60 minutes after


ibandronate (30 minutes for other bisphosphonates). Ibandronate should be given
on an empty stomach with plain water only. Bisphosphonates, unlike raloxifene, are
not associated with blood clots and leg cramps.

Chapter 36: Anti-inflammatory, Antipyretic, and Analgesic Agents

132- A 64-year-old male presents with mild to moderate musculoskeletal back pain
after playing golf. He states he has tried acetaminophen and that it did not help. His
past medical history includes diabetes, hypertension, hyperlipidemia, gastric ulcer
(resolved), and coronary artery disease. Which of the following is the most
appropriate NSAID regimen to treat this patient’s pain?

A. Celecoxib.
B. Indomethacin and omeprazole.
C. Naproxen and omeprazole.
D. Naproxen.

Correct answer = C. This patient is at high risk of future ulcers, due to the historyof
gastric ulcer. Therefore, using a regimen that includes an agent that is more COX-2
selective or a proton pump inhibitor is warranted. Therefore,
D is incorrect. Choices A and B are incorrect because this patient has significant
cardiovascular risk and a history of coronary artery disease. Naproxen is thought ofas
the safest NSAID regarding cardiovascular disease, though it still can present risks.
Therefore, C is correct as it uses the first-choice NSAID with the GI protection of a
proton pump inhibitor.

133- A 64-year-old male presents with signs and symptoms of an acute gouty flare.
His doctor wishes to treat him accordingly to improve his symptoms. Which of the
following strategies would be the LEAST likely to acutely improve his gout symptoms
and pain?

A. Naproxen.
B. Colchicine.
C. Probenecid.
D. Prednisone.
Page 174

Correct answer = C. Probenecid is a uricosuric agent indicated to lower serum urate


levels to prevent gout attacks. It is not indicated during acute gout flares and should
not be started until after the resolution of an acute attack. Naproxen, colchicine, and
prednisone all represent viable treatment options that acutely reduce pain and
inflammation associated with acute gout attacks.

UNIT VII: CHEMOTHERAPEUTIC


Chapter 37: Principles of Antimicrobial Therapy

134- A 24-year-old pregnant female presents to the urgent care clinic with fever,
frequency, and urgency. She is diagnosed with a urinary tract infection (UTI).
Based on potential harm to the fetus, which of the following medications should be
avoided in treating her UTI?

A. Nitrofurantoin.
B. Amoxicillin.
C. Cephalexin.
D. Tobramycin.
Correct answer = D. Tobramycin (an aminoglycoside) is considered a pregnancyrisk
category D drug which mean there is chance for potential harm to the fetus.
Nitrofurantoin, amoxicillin (a penicillin), and cephalexin (a cephalosporin) are
considered category B.

135- A 58-year-old male with a history of hepatitis C, cirrhosis, and ascites


presents with spontaneous bacterial peritonitis. Which of the following antibiotics
requires close monitoring and dosing adjustment in this patient given his liver
disease?

A. Penicillin G.
B. Tobramycin.
C. Erythromycin.
D. Vancomycin.

Correct answer = C. Erythromycin is metabolized by the liver and should be used


with caution in patients with hepatic impairment. Penicillin G, tobramycin, and
vancomycin are primarily eliminated by the kidney
Page 175

136- A 72-year-old male presents with fever, cough, malaise, and shortness of
breath. His chest x-ray shows bilateral infiltrates consistent with pneumonia.
Bronchial wash cultures reveal Pseudomonas aeruginosa sensitive to cefepime.
Which of the following is the best dosing scheme for cefepime based on the drug’s
time dependent bactericidal activity?

A. 1 g every 6 hours given over 30 minutes.


B. 2 g every 12 hours given over 3 hours.
C. 4 g every 24 hours given over 30 minutes.
D. 4 g given as continuous infusion over 24 hours.

Correct answer = D. The clinical efficacy of cefepime is based on the percentage of time
that the drug concentration remains above the MIC. A continuous infusion would
allow for the greatest amount of time above the MIC compare to intermittent (30
minutes) and prolonged infusions (3 to 4 hours).

Chapter 38: Cell Wall Inhibitors

137- A 45-year-old male presented to the hospital 3 days ago with severe cellulitis
and a large abscess on his left leg. Incision and drainage were performed on the
abscess, and cultures revealed methicillin-resistant Staphylococcus aureus. Which
ofthe following would be the most appropriate treatment option for once daily
outpatient intravenous therapy?

A. Ertapenem.
B. Ceftaroline.
C. Daptomycin.
D. Piperacillin/tazobactam.
Correct answer = C. Daptomycin is approved for skin and skin structure infections
caused by MRSA and is given once daily. A and D are incorrect because they do not
cover MRSA. Ceftaroline covers MRSA, but it must be given twice daily.

138- A 72-year-old male is admitted to the hospital from a nursing home with
severe pneumonia. He was recently discharged from the hospital 1 week ago after
open heart surgery. The patient has no known allergies. Which of the following
regimens is most appropriate for empiric coverage of methicillinresistant
Staphylococcus aureus and Pseudomonas aeruginosa in this patient?
Page 176

A. Vancomycin + cefepime + ciprofloxacin.


B. Vancomycin + cefazolin + ciprofloxacin.
C. Telavancin + cefepime + ciprofloxacin.
D. Daptomycin + cefepime + ciprofloxacin.

Correct answer = A. Vancomycin provides adequate coverage against MRSA, and


cefepime and ciprofloxacin provide adequate empiric coverage of Pseudomonas. Bis
incorrect because cefazolin does not have activity against Pseudomonas. C is incorrect
because telavancin should be avoided if possible with drugs that prolong the QTc
interval, in this case ciprofloxacin. Daptomycin is inactivated by pulmonary surfactant
and should not be used for pneumonia.

139- A 23-year-old male presents with acute appendicitis that ruptures shortly after
admission. He is taken to the operating room for surgery, and postsurgical cultures
reveal Escherichia coli and Bacteroides fragilis, susceptibilities pending. Which of the
following provides adequate empiric coverage of these two
pathogens?

A. Cefepime.
B. Piperacillin/tazobactam.
C. Aztreonam.
D. Ceftaroline.

Correct answer = B. While all of these agents cover most strains of E. coli,
piperacillin/tazobactam is the only drug on this list that provides coverage against
Bacteroides species.

140- A 68-year-old male presents from a nursing home with fever, increased
urinary frequency and urgency, and mental status changes. He has a penicillin
allergy of anaphylaxis. Which of the following β-lactams is the most appropriate
choice for gram-negative coverage of this patient’s urinary tract infection?

A. Cefepime.
B. Ertapenem.
C. Aztreonam.
D. Ceftaroline.

Correct answer = C. Based on the severity of the allergic reaction, aztreonam is the
choice of all the β-lactams. Although cross-reactivity with cephalosporins and
Page 177

carbapenems is low, the risk rarely outweighs the benefit in these case
141- A 25-year-old male presents to the urgent care center with a painless sore on
his genitals that started 1 to 2 weeks ago. He reports unprotected sex with a new
partner about a month ago. A blood test confirms the patient has Treponema
pallidum. Which of the following is the drug of choice for the treatment of this
patient’s infection as a single dose?

A. Benzathine penicillin G.
B. Ceftriaxone.
C. Aztreonam.
D. Vancomycin

Correct answer = A. A single treatment with penicillin is curative for primary and
secondary syphilis. No antibiotic resistance has been reported, and it remains the
drug of choice unless the patient has a severe allergic reaction.

142- A 20-year-old female presents to the emergency room with a headache,


stiff neck, and fever for 2 days and is diagnosed with meningitis. Which of the
following agents is the best choice for the treatment of meningitis in this patient?

A. Cefazolin.
B. Cefdinir.
C. Cefotaxime.
D. Cefuroxime axetil.

Correct answer = C. Cefotaxime is the only drug on this list with adequate CSF
penetration to treat meningitis. Cefdinir and cefuroxime axetil are only available
orally, and cefazolin CSF penetration and spectrum of coverage against S.
pneumoniae are not likely adequate to treat meningitis.

143- An 18-year-old female presents to the urgent care clinic with urinary
frequency, urgency, and fever for the past 3 days. Based on symptoms and a
urinalysis, she is diagnosed with a urinary tract infection. Cultures reveal
Enterococcus faecalis that is pan sensitive. Which of the following is an
appropriate oral option to treat the urinary tract infection in this patient?

A. Cephalexin.
B. Vancomycin.
C. Cefdinir.
D. Amoxicillin.

Correct answer = D. Option A and C are incorrect because enterococci are


inherently resistant to all cephalosporins. Option B is incorrect because oral
vancomycin is not absorbed and would not reach the urinary tract in sufficient
quantities to treat a urinary tract infection. Option D is the best choice, as
Page 178

amoxicillin is well absorbed orally and concentrates in the urine.


Chapter 39: Protein Synthesis Inhibitors

144- Children younger than 8 years of age should not receive tetracyclines because
these agents:

A. Cause rupture of tendons.


B. Deposit in tissues undergoing calcification.
C. Do not cross into the cerebrospinal fluid.
D. Can cause aplastic anemia.

Correct answer = B. Tetracyclines is contraindicated in this age group because they


are deposited in tissues undergoing calcification, such as teeth and bone, and can
stunt growth. Ciprofloxacin can interfere in cartilage formation and cause rupture of
tendons and is also contraindicated in children, but it is a fluoroquinolone.
Tetracyclines can cross into the cerebrospinal fluid. They do not cause aplastic anemia,
a property usually associated with chloramphenicol.

145- A 30-year-old pregnant female has cellulitis caused by MRSA. Which of the
following antibiotics would be the most appropriate option for outpatient therapy?

A. Doxycycline.
B. Clindamycin.
C. Quinupristin/dalfopristin.
D. Tigecycline.

Correct answer = B. Clindamycin is the safest option for the treatment of MRSA in a
pregnant patients. Doxycycline and tigecycline can cross the placenta and can cause
harm to the fetus. Moreover, quinupristin/dalfopristin and tigecycline are only
available intravenously and would not be appropriate for home antibiotic therapy for
the given indication.

146- A patient is being discharged from the hospital on a 3-week course of


clindamycin. Which of the following potential adverse effects should be discussed
with her?

A. Hyperbilirubinemia.
B. Nephrotoxicity.
C. Clostridium difficile diarrhea.
D. Pseudotumor cerebri.

Correct answer = C. Clindamycin, among other antibiotics, is associated with the


development of C. difficile and pseudomembranous colitis due to disruption of
Page 179

normal gut flora, particularly with prolonged therapy. Hyperbilirubinemia is


associated with quinupristin/dalfopristin, nephrotoxicity is associated with
aminoglycosides, and pseudotumor cerebri can occur with tetracyclines

Chapter 40: Quinolones, Folic Acid Antagonists, and Urinary TractAntiseptics

147- A 32-year-old male presents to an outpatient clinic with a 5-day history of


productive cough, purulent sputum, and shortness of breath. He is diagnosed with
community acquired pneumonia (CAP). It is noted that this patient has a severe
ampicillin allergy (anaphylaxis). Which of the following would be an acceptable
treatment for this patient?

A. Levofloxacin.
B. Ciprofloxacin.
C. Penicillin VK.
D. Nitrofurantoin.

Correct answer = A. Streptococcus pneumoniae is a common cause of CAP, and the


respiratory fluoroquinolones levofloxacin and moxifloxacin provide good coverage.
Ciprofloxacin does not cover S. pneumoniae well and is a
poor choice for treatment of CAP. Penicillin would be a poor choice due to allergy.
Nitrofurantoin has no clinical utility for respiratory tract infections.

148- A 22-year-old female presents with a 2-day history of dysuria with increased
urinary frequency and urgency. A urine culture and urinalysis are done. She is
diagnosed with a urinary tract infection (UTI) caused by E. coli. All of the following
would be considered appropriate therapy for this patient except:

A. Levofloxacin.
B. Cotrimoxazole.
C. Moxifloxacin.
D. Nitrofurantoin.

Correct answer = C. Moxifloxacin does not concentrate in the urine and would be
ineffective for treatment of a UTI. All other answers are viable alternatives, and the
resistance profile for the E. coli can be utilized to direct therapy.
Page 180

Chapter 41: Antimycobacterial Drugs

149- A 35-year-old male, formerly a heroin abuser, has been on methadone


maintenance for the last 13 months. Two weeks ago, he had a positive tuberculosis
skin test (PPD test), and a chest radiograph showed evidence of right upper lobe
infection. He was started on standard four-drug antimycobacterial therapy. He has
come to the emergency department complaining of “withdrawal symptoms.” Which
of the following antimycobacterial drugs is likely to have caused this patient’s acute
withdrawal reaction?

A. Ethambutol.
B. Isoniazid.
C. Pyrazinamide.
D. Rifampin.
E. Streptomycin.

Correct answer = D. Rifampin is a potent inducer of cytochrome P450–dependent


drug-metabolizing enzymes. The duration of action of methadone is dependent
upon hepatic clearance, so enhanced drug metabolism will shorten the duration
and increase the risk of withdrawal symptoms in individuals on methadone
maintenance. None of the other drugs listed induce cytochrome P450 enzymes.

150- A 42-year-old male HIV patient was recently diagnosed with active
tuberculosis. Currently, he is on a stable HIV regimen consisting of two protease
inhibitors and two nucleoside reverse transcriptase inhibitors (NRTIs). What is the
most appropriate regimen to use for treatment of his tuberculosis?

A. Rifampin + isoniazid + pyrazinamide + ethambutol.


B. Rifabutin + isoniazid + pyrazinamide + ethambutol.
C. Rifapentine + isoniazid + pyrazinamide + ethambutol.
D. Rifampin + moxifloxacin + pyrazinamide + ethambutol.
E. Amikacin + moxifloxacin + cycloserine + streptomycin.

Correct answer = B. Rifabutin is recommended in place of rifampin in patients


coinfected with HIV, since it is a less potent inducer of CYP enzymes than
rifampin. However, rifabutin is a CYP3A4 substrate and “bidirectional”
interactions may result. Other medications, such as the protease inhibitors, may
affect the concentration of rifabutin, requiring a dose adjustment. Answer E is
incorrect as these are not first-line agents.
Page 181

151- A 24-year-old male has returned to the clinic for his 1-month check-up after
starting treatment for tuberculosis. He is receiving isoniazid, rifampin,
pyrazinamide, and ethambutol. He states he feels fine, but now is having difficulty
reading his morning newspaper and feels he may need to get glasses. Which of the
following drugs may be causing his decline in vision?

A. Isoniazid.
B. Rifampin.
C. Pyrazinamide.
D. Ethambutol.

Correct answer = D. Optic neuritis, exhibited as a decrease in visual acuity or loss


ofcolor discrimination, is the most important side effect associated with
ethambutol. Visual disturbances generally are dose related and more common
in patients with reduced renal function. They are reversible (weeks to months) if
ethambutol is discontinued promptly.

Chapter 42: Antifungal Drugs


152- A 55-year-old female presents to the hospital with shortness of breath, fever,
and malaise. She has a history of breast cancer, which was diagnosed 3 months ago,
and has been treated with chemotherapy. Her chest x-ray shows possible
pneumonia, and respiratory cultures are positive for Aspergillus fumigatus. Which
of the following is the MOST appropriate choice for treatment?

A. Voriconazole.
B. Fluconazole.
C. Flucytosine.
D. Ketoconazole.
Correct answer = A. Voriconazole is the drug of choice for aspergillosis. Studies have
found it to be superior to other regimens including amphotericin B. Fluconazole,
flucytosine and ketoconazole do not have reliable in vitro activity and are therefore not
recommended.

153- A 56-year-old female with diabetes presents for routine foot evaluation with
her podiatrist. The patient complains of thickening of the nail of the right big toe
and a change in color (yellow). The podiatrist diagnoses the patient with
onychomycosis of the toenails. Which of the following is the most appropriate
choice for treating this infection?

A. Terbinafine.
B. Micafungin.
Page 182

C. Itraconazole.
D. Griseofulvin.

Correct answer = A. Terbinafine is better tolerated, requires a shorter duration of


therapy, and is more effective than either itraconazole or griseofulvin. Micafunginis
not active for this type of infection.

Chapter 43: Antiprotozoal Drugs

154- A group of college students are traveling to a chloroquine-resistant malaria


area for a mission trip. Which of the following medications can be used for both
prevention and treatment of malaria in these students?

A. Pyrimethamine.
B. Artemisinin.
C. Atovaquone–proguanil.
D. Melarsoprol.

Correct answer = C. The combination of atovaquone–proguanil has been used for


both prevention and treatment of malaria in chloroquine-resistant areas.
Pyrimethamine is not recommended for prophylaxis of malaria. Artemisinin
and its derivatives are not used for prophylaxis, only treatment of malaria.
Melarsoprol is used for the treatment of African sleeping sickness.

155- An 18-year-old male is diagnosed with Chagas disease. Which medication


would be the best for this patient?

A. Nifurtimox.
B. Suramin.
C. Sodium stibogluconate.
D. Metronidazole.

Correct answer = A. Nifurtimox is indicated for the treatment of American


trypanosomiasis (Chagas disease) caused by T. cruzi. Suramin is used for the
treatment of first-stage African trypanosomiasis due to T. brucei rhodesiense. Sodium
stibogluconate is used for the treatment of leishmaniasis. Metronidazole is used for
the treatment of amebiasis and giardiasis.
Page 183

Chapter 44: Anthelmintic Drugs


156- A 48-year-old immigrant from Mexico presents with seizures and other
neurologic symptoms. Eggs of T. solium are found upon examination of a stool
specimen. A magnetic resonance image of the brain shows many cysts, some of
which are calcified. Which one of the following drugs would be of benefit to this
individual?

A. Ivermectin.
B. Pyrantel pamoate.
C. Albendazole.
D. Diethylcarbamazine.
E. Niclosamide.

Correct answer = C. The symptoms and other findings for this patient are consistent
with neurocysticercosis. Albendazole is the drug of choice for the treatment of this
infestation. The other drugs are not effective against the larvalforms of tapeworms.

157- A 56-year-old man from South America is found to be parasitized by both


schistosomes and T. solium—the pork tapeworm. Which of the following
anthelmintic drugs would be effective for both infestations?

A. Albendazole.
B. Ivermectin.
C. Mebendazole.
D. Praziquantel.
Correct answer = D. Praziquantel is a primary drug for the treatment of trematode and
cestode infestations. Although albendazole is effective in cysticercosis, it is not active
against flukes, and this patient has no evidence of cysticercosis. Ivermectin and
mebendazole treat nematode infestations.

Chapter 45: Antiviral Drugs

158- A 30-year-old male patient with human immunodeficiency virus infection is


being treated with a HAART (highly active antiretroviral therapy) regimen. Four
weeks after initiating therapy, he presents to the emergency department
complaining of fever, rash, and gastrointestinal upset. Which one of the following
drugs is most likely the cause of his symptoms?
Page 184

A. Zidovudine.
B. Nelfinavir.
C. Abacavir.
D. Efavirenz.
E. Darunavir.

Correct answer = C. The abacavir hypersensitivity reaction is characterized by


fever, rash, and gastrointestinal upset. The patient must stop therapy and not be
rechallenged.

159- A 75-year-old man with chronic obstructive pulmonary disease is diagnosed


with suspected influenza based on his complaints of flu-like symptoms that began
24 hours ago. Which of the following agents is most appropriate to initiate for the
treatment of influenza?

A. Ribavirin.
B. Oseltamivir.
C. Zanamivir.
D. Rimantadine.
E. Amantadine.
Correct answer = B. Oseltamivir is the best choice since it is administered orally and
not associated with resistance. Zanamivir is administered via inhalation and is
not recommended for patients with underlying COPD. High rates of resistance
have developed to adamantanes (amantadine, rimantadine), and these drugs are
infrequently indicated. Ribavirin is not indicated for treatment of influenza.

160- A 24-year-old female is diagnosed with genital herpes simplex virus


infection. Which of the following agents is indicated for use in this diagnosis?

A. Valacyclovir.
B. Cidofovir.
C. Ganciclovir.
D. Zanamivir.
E. Lamivudine.

Correct answer = A. Valacyclovir, famciclovir, penciclovir, and acyclovir are all


indicated for herpes simplex virus infection. Cidofovir and ganciclovir are used for CMV
retinitis. Zanamivir is indicated for influenza. Lamivudine is indicated for HIV and
hepatitis B.
Page 185

161- A female patient who is being treated for chronic hepatitis B develops
nephrotoxicity while on treatment. Which is the most likely medication she is
taking for HBV treatment?

A. Entecavir.
B. Telbivudine.
C. Lamivudine.
D. Adefovir.
Correct answer = D. Nephrotoxicity is the most commonly seen with adefovir.

Chapter 46: Anticancer Drugs

162. A patient is about to undergo three cycles of chemotherapy prior to surgery


for bladder cancer. Which of the following best describes chemotherapy in this
setting?

A. Adjuvant.
B. Neoadjuvant.
C. Palliative.
D. Maintenance.

Correct answer = B. Since the chemotherapy is being given before the surgery, it is
considered neoadjuvant. Chemotherapy is indicated when neoplasms are
disseminated and are not amenable to surgery (palliative). Chemotherapy is also used
as a supplemental treatment to attack micrometastases following surgery and
radiation treatment, in which case it is called adjuvant chemotherapy.
Chemotherapy given prior to the surgical procedure in an attempt to shrink the
cancer is referred to as neoadjuvant chemotherapy, and chemotherapy given in
lower doses to assist in prolonging a remission is known as maintenance
chemotherapy.

163- A 45-year-old male patient is being treated with ABV chemotherapy for
Hodgkin lymphoma. He presents for cycle 4 of a planned 6 cycles with a new-
onset cough. He states it started a week ago and he also feels like he has a little
trouble catching his breath. Which drug in the ABVD regimen is the most likely
cause of his pulmonary toxicity?

A. Doxorubicin (Adriamycin).
B. Bleomycin.
C. Vinblastine.
D. Dacarbazine.
Page 186

Correct answer = B. Pulmonary toxicity is the most serious adverse effect of bleomycin,
progressing from rales, cough, and infiltrate to potentially fatal fibrosis.The pulmonary
fibrosis that is caused by bleomycin is often referred as “bleomycinlung.”

164- FL is a 64-year-old male about to undergo therapy for rhabdomyosarcoma. His


chemotherapy includes if osfamide. Which of the following is most appropriate to
include in chemotherapy orders for this patient?

A. IV hydration, mesna, and frequent urinalyses.


B. Leucovorin and frequent urinalyses.
C. Allopurinol and frequent urinalyses.
D. IV hydration, prophylactic antibiotics, and frequent urinalyses.

Correct answer = A. A unique toxicity of if osfamide is hemorrhagic cystitis. This


bladdertoxicity has been attributed t toxic metabolites of if osfamide. Adequate
hydration as well as IV injection of mesna (sodium 2-mercaptoethane sulfonate),
which neutralizes the toxic metabolites, can minimize this problem. Frequent
urinalyses to monitor for red blood cells should be ordered. Leucovorin is used with
methotrexate or 5-FU (not ifosfamide). Allopurinol has a drug interaction with
ifosfamide and is not an agent that prevents hemorrhagic cystitis. Prophylactic
antibiotics are not needed.

Chapter 47: Immunosuppressants

165- A 45-year-old male who received a renal transplant 3 months previously and
is being maintained on prednisone, cyclosporine, and mycophenolate mofetil is
found to have increased creatinine levels and a kidney biopsy indicating severe
rejection. Which of the following courses of therapy would be appropriate?

A. Increased dose of prednisone.


B. Hemodialysis.
C. Treatment with rabbit antithymocyte globulin.
D. Treatment with sirolimus.
E. Treatment with azathioprine.
Correct answer = C. This patient is apparently undergoing an acute rejection of the
kidney. The most effective treatment would be administration of an antibody.
Increasing the dose of prednisone may have some effect but would not be
enough to treat the rejection. Sirolimus is used prophylactically with cyclosporine to
prevent renal rejection but is less effective when an episode is occurring.
Furthermore, the combination of cyclosporine and sirolimus is more nephrotoxic
than cyclosporine alone. Azathioprine has no benefit over mycophenolate.
Page 187

UNIT VIII: TOXICOLOGY


Chapters: Clinical Toxicology

166- A 3-year-old boy is brought to the emergency department by his mother, who
reports that he has been crying continuously and “does not want to play or eat” for
the last few days. She also states that he has not had regular bowel movements,
with mostly constipation and occasional diarrhea, and frequently complains of
abdominal pain. The child now has an altered level of consciousness, is difficult to
arouse, and begins to seize. The clinician rules out infection and other medical
causes. Upon questioning, the mother states that the house is in an older
neighborhood, that her house has not been remodeled or repainted since the
1940s,and that the paint is chipping around the windows and doors. The child is
otherwise breathing on his own and urinating normally. Which toxin would you
expect to be producing such severe effects in this child?

A. Iron.
B. Lead.
C. Carbon monoxide.
D. Cyanide.
E. Ethylene glycol.

Correct answer = B. Lead poisoning is common among children in older homes


painted before lead was removed from paint. Paint chips with lead are easily
ingested by toddlers, and excessively high lead levels can lead to the signs and
symptoms described plus clumsiness, confusion, headaches, coma, constipation,
intestinal spasms, and anemia. Death is rare when chelation therapy is instituted.
Iron can produce abdominal pain, but more often would cause diarrhea, vomiting,
and volume loss. If he had cyanide poisoning, death would have occurred quickly
following respiratory arrest of oxidative phosphorylation and production of
adenosine triphosphate, but this child has been exhibiting symptoms over several
days. Carbon monoxide would affect the entire household, depending on the source.
Clinical effects from carbon monoxide would include headache, nausea, and CNS
depression. Ethylene glycol is sweet and may be ingested by a toddler. The
presentation of ethylene glycol toxicity would include initial appearance of
intoxication, which was not mentioned.

167- A 41-year-old male pocket watch maker presents to the emergency


department after he was found unconscious on the floor of the shop by a coworker.
The coworker states that the patient complained of being cold this morning around8
am (the central heat was broken, and the outdoor temperature was 34°F) and that
since noon, he had been complaining of headache, drowsiness, confusion, and
nausea. The clinician notices that he has cherry red skin. What is the most likely
toxin causing his signs and symptoms?
Page 188

A. Ethylene glycol.
B. Cyanide.
C. Acetaminophen.
D. Carbon monoxide.
E. Methanol.

Correct answer = D. Although watchmakers and other professionals who use


electroplating may be at higher risk for cyanide exposure because many plating baths
use cyanide containing ingredients (for example, potassium cyanide), this patient
shows signs of carbon monoxide poisoning, such as cherry red skin, headache,
confusion, nausea, and drowsiness leading to unconsciousness. The history also leads
us to believe that this person may have been using a space heater to stay warm,
whichwould be consistent with the description. A carboxyhemoglobin level should be
obtained to confirm the exposure. Cyanide in low doses from such an occupational
exposure can present with loss of consciousness, flushing, headache, and confusion.
Chronically, workers may develop a rash after handling cyanide solutions. Also, an
odor of bitter almonds may be present. An arterial blood gas and a venous blood gas
could be obtained and compared to determine if cyanide is present (a lack of oxygen
extraction would be present on the venous side). Ethylene glycol and methanol toxicity
may cause alterations in mental status, but the history did not include anything
suggesting toxic alcohol ingestion. Acetaminophen toxicity is not consistent with this
presentation.

168- A 50-year-old migrant worker comes to the emergency department from the
field he was working in and complains of diarrhea, tearing, nausea and vomiting,
and sweating. The clinician notices that he looks generally anxious and has fine
fasciculations in the muscles of the upper chest as well as pinpoint pupils.
Which antidote should he receive first?

A. N-acetylcysteine.
B. Sodium nitrite.
C. Deferoxamine.
D. Atropine.
E. Fomepizole.

Correct answer = D. Atropine is appropriate for this patient, who has symptoms
consistent with organophosphate (insecticide) poisoning. The mnemonic DUMBBELS
(diarrhea, urination, miosis, bronchorrhea/bradycardia, emesis, lacrimation,
salivation) can be used to remember the signs and symptoms of cholinergic toxicity.
An anticholinergic antidote, atropine, controls these muscarinic symptoms, whereas
the antidote pralidoxime treats the nicotinic symptoms like fasciculations (involuntary
muscle quivering or twitching). N- acetylcysteine is the antidote for acetaminophen
overdose and acts as a sulfhydryl donor. Sodium nitrite is one of the antidotes
included in the old cyanide antidote kit (sodium nitrite and sodium thiosulfate).
Deferoxamine is the chelating agent for iron. Fomepizole is the antidote for methanol
Page 189

and ethylene glycol.

169- A 45-year-old male presented to the emergency department 18 hours after


ingesting an unknown product. On presentation, he is tachycardic, hypertensive,
tachypneic, and complaining of flank pain. A metabolic panel is obtained, and the
patient has a large anion gap acidosis, an increased creatinine, and hypocalcemia.
Which substance was most likely ingested?

A. Methanol.
B. Acetaminophen.
C. Ethylene glycol.
D. Iron.
E. Opioids.

Correct answer = C. Ethylene glycol produces a metabolic acidosis from the toxic
metabolites. The formation of calcium oxalate crystals, which can be found on
urinalysis, leads to hypocalcemia and renal failure. The treatment regimen for
this patient would include intravenous fomepizole, if some of the parent compound
was still present, and hemodialysis. Thiamine and pyridoxine are the cofactors
involved in the metabolism of ethylene glycol. Methanol may produce a metabolic
acidosis as well, but its target organ of toxicity is the eyes instead of the kidneys as
with ethylene glycol. Acetaminophen toxicity may produce upper quadrant pain within
the first 24 hours, but vital sign abnormalities are not usually found during this time
frame. Iron toxicity may also produce a metabolic acidosis and tachycardia. However,
hypocalcemia does not occur. Opioid toxicity, as mentionedin Chapter 14, usually
presents with CNS and respiratory depression, not tachycardia and hypertension.

170- A 27-year-old female presents to the emergency department 6 hours after


reportedly ingesting 20 tablets of acetaminophen 500 mg. An acetaminophen level
is drawn, but it has to be sent out to another lab an will not return for another 6
hours. What is the most appropriate next step in management of this patient?

A. Administer a dose (50 g) of activated charcoal.


B. Empirically start N-acetylcysteine therapy.
C. Administer a dose of intravenous naloxone.
D. Wait for the level to return and then decide what to do.
E. Draw a NAPQI level.

Correct answer = B. N-acetylcysteine should be started empirically on the basis of the


history, and then, once the level returns and is plotted on the Rumack-Matthew
nomogram, a final decision on whether to continue therapy can be made. Activated
charcoal would not be of any benefit 6 hours post–acetaminophen ingestion. Naloxone
is utilized for opioid toxicity, not acetaminophen toxicity. The optimaltime frame to give
N-acetylcysteine is within 8 to 10 hours postingestion. So, waiting on the level to return
Page 190

would pu the patient more than 12 hours postingestion. Therefore, initiation of N-


acetylcysteine therapy should happen, if possible, during the optimal time frame.
Clinicians are unable to draw a NAPQI level and therefore cannot utilize this to guide
therapy.

171- A 4-year-old female presents to the emergency department with CNS


depression. Her vital signs
indicate that she is slightly bradycardic and slightly hypotensive for her age. Upon
further questioning, the mother admits that there are two clonidine 0.2 mg tablets
missing from the home. Which of the following antidotes might be beneficialfor this
patient?

A. Flumazenil.
B. Atropine.
C. Deferoxamine.
D. Naloxone.
E. Succimer.

Correct answer = D. Naloxone has a reversal rate of the CNS effects of


approximately 50% in clonidine ingestions. Flumazenil reverses benzodiazepinesand
has no effect on clonidine. Atropine is an anticholinergic agent and would not
improve the CNS depression. Deferoxamine is the chelator for iron, and succimer is
a lead chelator.

172- A 40-year-old male presents to the emergency department with a complaint of


abdominal pain. The patient appears intoxicated, but an ethanol level returns as
negative and his basic metabolic panel is unremarkable. Which of these substances
did he probably ingest?

A. Isopropyl alcohol.
B. Methanol.
C. Ethylene glycol.
D. Ethanol.
E. Organophosphates.

Correct answer = A. Isopropyl alcohol produces twice as much CNS depression as


ethanol and is known to cause GI distress. Isopropyl alcohol is metabolized to acetone,
so a metabolic acidosis does not result (which is in contrast to the acid osisgenerated
by methanol and ethylene glycol). The ethanol level was negative, eliminating ethanol
as an ingestion. Organophosphate toxicity yields nicotinic and muscarinic effects,
which are not described in the history.
Page 191

173- A 5-year-old male is brought in to the health care facility for being irritable
and failure to thrive. He is alert, and his vital signs are normal. The doctor
diagnoses him with lead toxicity when the blood lead level returns as 50 μg/dL.
Which chelator regimen should be started?
A. Dimercaprol.
B. Calcium disodium edetate.
C. Both dimercaprol and calcium disodium edetate.
D. Succimer.
E. Deferoxamine.

Correct answer = D. Succimer (dimercaptosuccinic acid, DMSA) is utilized when the


lead level is greater than 45 μg/dL, without encephalopathy. If encephalopathyis
present, or the lead level is greater than 70 μg/dL in a child, then dual parenteral
therapy with dimercaprol and calcium disodium edetate is indicated. Dimercaprol
intramuscular therapy is initiated 4 hours prior to the intravenous administration of
calcium disodium edetate when both medications are required. Deferoxamine is not
indicated since it is the chelator for iron.

174- A 3-year-old healthy female ingested one of her mother’s 1 mg alprazolam


tablets 45 minutes ago. The child presented to the emergency department with
CNS depression but a normal heart rate and blood pressure. Her bedside
glucose check is also normal. Which of the following antidotes might be helpful?

A. Flumazenil.
B. Naloxone.
C. Physostigmine.
D. Atropine.
E. Fomepizole.

Correct answer = A. Flumazenil is a competitive benzodiazepine antagonist that


reverses the CNS depression from benzodiazepines such as alprazolam. After
flumazenil administration, resedation usually occurs, since the
duration of the benzodiazepine is longer than that of the flumazenil. Naloxone
reverses the effects from opioids and clonidine, not benzodiazepines.
Physostigmine is the antidote for anticholinergic toxicity, and atropine is an
anticholinergic agent. Fomepizole is the antidote for methanol or ethylene glycol
toxicity.

175- A 34-year-old male with a history of a seizure disorder, maintained on


phenytoin and phenobarbital, presented to the emergency department for CNS
depression. The phenobarbital level was 70 mg/L (15 to 40 mg/L therapeutic range)
and the phenytoin level was 15 mg/L (10 to 20 mg/L therapeutic range). Hedenies
any acute ingestion. What therapy can be considered to enhance the elimination of
phenobarbital without impacting the phenytoin?
Page 192

A. Multiple doses of activated charcoal.


B. Gastric lavage
C. Urinary alkalinization.
D. Whole bowel irrigation.
E. Urinary acidification.

Correct answer = C. Urinary alkalinization enhances the elimination of the


phenobarbital but does not affect the therapeutic phenytoin level. Sodium
bicarbonate, 1 mEq/kg, is administered intravenously initially and then a sodium
bicarbonate continuous infusion is titrated to maintain a urine pH of 7.5 to 8, without
exceeding a serum pH of 7.55. Multiple doses of activated charcoal would lower the
concentration of both medications, rendering the phenytoin subtherapeutic. Gastric
lavage is a GI decontamination technique employed usually with in the first hour after
an acute ingestion of a life-threatening amount, to remove approximately 30% of the
product in the stomach. Whole bowel irrigation is another GI decontamination
modality involving administration of large quantities (up to 2 L/hour in adults) of a
polyethylene glycol–balanced electrolyte solution via a nasogastric tube until the
patient generates clear rectal effluent. Urinary acidification is no longer performed for
substances such as amphetaminesand quinidine.
Page 193

Ethics Law
• Unenforceable norms and values guide behavior • Values are written into enforceable standards of
• There are no specific laws behavior
• Laws are enforced by the justice system.

• Main ethical principles (Basic principles of ethics) → Moral rules ◆


v◆
: :

1- Autonomy • The right of individuals to self-rule and make decisions about:


1- what will happen to their bodies.
2- what choices will be made among competing options
3- what they choose to take, or not take, into their bodies
4- choice among health care providers
5- choice of refusing medical treatment.
• Exceptions to the theory of autonomy:
– weak paternalism: if one lacks the ability to make an autonomous decision, then it is
up to the healthcare provider
– the harm principle: making the wrong decision or a decision that will cause harm to
themselves.
Privacy: another rule within the principle of autonomy which means the right of the
individual to control his or her affairs without interference.
2- Beneficence • To do good (Benefit the patient).
3- Non-maleficence • Prevention of harm and the removal of harmful conditions.
4- Justice • Treat all patients fairly (Fairness and equity to all)
• Example: Ask the patient to stand in a queue.
• Principles of professionalism: ◆
v◆
: :
v

1- Accountability • Activities, responsibilities, and results


2- Altruism • Do the best for the best interest of the patient. (Not self-interest)
3- Duty • Commitment to the service
4- Honor • Highest standard of behavior and good conduct.
5- Integrity • Adherence to ethical principles and refusal to violate personal codes.
• Common characteristics of professionalism
1- Knowledge
2- Ethics
3- Social sanction = Public trust
• Competence, Trustworthiness, and Caring
• Pharmacists should be aware of the basic moral responsibilities that all healthcare practitioners have toward
their patients. There are three characteristics that a pharmacist should possess:
1- Pharmacists must be competent.
• They must possess the knowledge base that at least minimally allows them to carry out their functions as
reliable therapeutic experts.
2- Pharmacists must be trustworthy.
• Patients must know that they can seek the confidential advice and assistance of their pharmacist and that their
wishes will be carried out.
3- Pharmacists must care for and about their patients.
• As the 1995 American Pharmaceutical (now Pharmacists) Association (APhA) Code of Ethics directs,
• -Apharmacist places concern for the well-being of the patient at the center of professional practice."
• Pharmacists who do spend time with their patients and attempt to understand their concerns are much more
likely to be viewed as caring.

Arif Pharma Clinic Copyright@arifpharma.com arifpharma.com


Page 194

❖ Ethical principles and moral rules:

Veracity • Pharmacists should be honest in their dealings with patients. (Telling the truth)

:
v • violation of veracity may be ethically justifiable (as with the use of placebos)
• patients have a right to expect that pharmacists will be frank in dealings with them.
Fidelity ❖ it means that the pharmacists demonstrate loyalty to their patients, regardless of

:
v the length of the professional relationship
❖ Trust and keep promises.
1- Covenantal fidelity:
• is often described as an intimate and spiritual commitment between individuals.
• Examples would include the fidelity of marriage and the fidelity between a member
of the clergy and his or her congregation.
2- Contractual fidelity
• It does not involve a level of commitment beyond that owed another as the result
of a binding agreement.
• An example of this form of fidelity would be the relationship one might have with a
contractor such as a plumber or an electrician.
Informed Consent • What and how much information about medication should be given to a patient

:
v • patients must be fully informed about the benefits and risks of their
participation in a clinical trial, taking medication, or electing to have surgery,
and this disclosure must be followed by their autonomous consent.
• Informed consent is obtained:
Formally Informally
For legal and ethical reasons Whenever a pharmacist counsels a
Ex: clinical trials, research, surgery patient and dispenses medication to
a patient, a type of informal occurs.
The patient is informed about the
benefits and any risks of the drug,
and then
decides whether to take it or not.
Confidentiality
◆◆
:
v :
v • Revealing information about a patient's medications to members of the family
• Medical confidentiality need not be requested by patients; all medical information
Refuse or give is considered confidential unless the patient grants approval for its release.
privileged information Ethical situations:
– Members of the health care team may have access to confidential medical records
without the consent of the patient.
– A patient who expresses a desire not to have information revealed to a member of
thehealth care team.
– Exceptions: Weak paternalism, the Harm principle
Full disclosure • Inform the patients about Benefits and risks then decide what to do.
Patient-centered care • Pre-vision information about the patient:
• Culture competition – Confidentiality – Full disclosure
Patient adherence • Help the patient to stick to the drug dose and time (Help not to force)
Respect for persons • Duty to the welfare of the individual, particularly described in religion.
Excellence • Efforts and commitment to life-long learning and ongoing professional development
Humanism • Respect and compassion for others
Distributive justice • Equal distribution of the benefits and burdens among all members of society.
• Pharmacists do not always provide care with equal dedication to all patients.
• Patient's socioeconomic status often impacts the level and intensity of care
providedby healthcare professionals.
• Medicaid patients are sometimes provided a much lower quality of care than a
patientwho is a cash-paying customer or who has a full coverage drug benefits
plan.
• Justice demands that the focus is on patients and their medical needs, not on the
financial impact on the health care professional.

Arif Pharma Clinic Copyright@arifpharma.com arifpharma.com


Page 195

Macro ethical issues Micro situations


issues that are not specific to a given pharmacist, but issues that may confront individual pharmacists in the
rather are those that must be addressed by all course of their daily practice.
pharmacists and by society in general.
• Abortion • The use of placebos
• Assisted suicide • Patient confidentiality
• Genetic engineering • Informed consent
• Rationing of and access to health care
• Organ transplantation
• In vitro fertilization.

Sometimes, macro issues are manifested in micro situations. This is especially true with socially controversial issues.
For example, a pharmacist may receive a prescription for a drug and know that it is intended for use in assisted
suicide. Not only must the pharmacist deal with the legal issues involved, but also with the ethical responsibility as
ahealth care professional. A further complication in such situations is the influence of the pharmacist's personal
beliefsin choosing the course of action.

❖ Ethical conflicts and issues in health care

Law and Ethics


Example Example
1 2
what should a pharmacist do when a what if the medication is a controlled
patient's prescription for heart medicine has substance used for pain control in a terminally
been depleted, no refills remain, and the ill patient?
prescriber is unavailable?
Most pharmacists would do the ethical thing The potential for legal action from drug
and provide such patients with a few doses to enforcement authorities might make a
hold them over until a new prescription can pharmacist reluctant to dispense extra doses,
be obtained, even though this course of even though the patient might be in just as
action is much
Assisted Suicide • illegal
Medical euthanasia (mercy killing) has long been
need.an ethical issue
• Legally it is not set yet whether to support or to prohibit.
• From an ethical perspective, the key issue remains whether assisted suicide violates the
Hippocratic responsibilities of health care practitioners to do no harm.
• Those who advocate its availability to patients suggest that allowing a patient to continue
to experience unrelenting pain is doing harm.
• They suggest that patients have the right to make an autonomous decision to end their
life; their opponents worry that legal assisted suicide would be abused.
Human Drug • Two important ethical aspects of human drug experimentation are
Experimentation 1- The role of the institutional review board ORB v
◆◆
: :
v
• Review protocol for ethical purposes, clinical experiments, or before animal studies
• Making sure that the rights and welfare of the patient-subject are protected
• evaluate and approve informed consent forms used in conjunction with the research
2- The use of placebos.
• placebos are agents devoid of pharmacologic activity and have served as a point of
comparison for determining therapeutic efficacy
• Patients do not receive any benefits (beneficence)

Arif Pharma Clinic Copyright@arifpharma.com arifpharma.com


Page 196

1. The Drug Enforcement Administration (DEA) regulations require pharmacies to keep control
substances records, including prescriptions for at least ……….
a- two years
b- three years
c- four years
d- five years.
2. … ....... are issues that are not specific to a given pharmacist, but rather are those that must be
addressed by all pharmacists and by society in general.
a. Macro ethical issues
b. Micro ethical issues
c. Micro situations
d. Macro situation

3. revealing information about a patient's medications to members of the family is an example of:
a. Patient confidentiality
b. informed consent
c. informed refusal
d. express consent
4. Pharmacists must possess the knowledge base that at least minimally allows them to carry out their
Functions as reliable therapeutic experts. This is called:
a. Competency.
b) Caring
c) Trustworthiness.
d) Knowledge.
5. Is the right of individuals to make decisions about what will happen to their bodies,
what choices will be made among competing options, and what they choose to take,
or not take, in to their bodies, the choice among health care providers, and the choice of
refusing medical treatment.
a. Autonomy
b. Beneficence
c. Nonmaleficence
d. justice
6. Indicates that you act in a manner to do good.
a. Autonomy
b. Beneficence
c. Nonmaleficence

Arif Pharma Clinic Copyright@arifpharma.com arifpharma.com


Page 197

d. justice
7. is sometimes used more broadly to include the prevention of harm and the removal of
harmful conditions.
a. Autonomy
b. Beneficence
c. Nonmaleficence
d. justice

8. This means that pharmacists demonstrate loyalty to their patients. Pharmacists have an
obligation of fidelity to all their patients, regardless of the length of the professional relationship.
a. Autonomy
b. Beneficence
c. Nonmaleficence
d. Fidelity

9. Is the ethical principle that instructs pharmacists to be honest in their dealings with
patients?
a. Autonomy
b. Beneficence
c. Nonmaleficence
d. veracity

10. Refers to the equal distribution of the benefits and burdens of society among all members of this
society.
a. Autonomy
b. Beneficence
c. Nonmaleficence
d. distributive justice

11. Euthanasia means:


a. Autonomy
b. Beneficence
c. Nonmaleficence
d. Mercy killing

12. If one lacks the ability to make an autonomous decision, then it is up to the health care provider,
this iscalled:
a. weak paternalism

Arif Pharma Clinic Copyright@arifpharma.com arifpharma.com


Page 198

b. strong paternalism
c. the harm principle
d. autonomy.

13. Making the wrong decision or a decision that will cause harm to themselves is called:
a. weak paternalism
b. strong paternalism
c. the harm principle
d. autonomy.

14. are unenforceable norms and values guide behavior


a. Ethics.
b. Morals.
c. Laws.
d. Rules.

15. Values are written into enforceable standards of behavior


a. Ethics.
b. Morals.
c. Laws.
d. Rules.

16. How much pseudoephedrine may a patient purchase in 1 day?

a. 3.6 g

b. 6.3 g

c. 9 g

d. 100 mg

17. Which act created Medicare Part D?

a. Medicare Prescription Drug Improvement and Modernization Act of 2003

b. OBRA’ 90

c. HIPAA

d. Prescription Drug Marketing Act

Arif Pharma Clinic Copyright@arifpharma.com arifpharma.com


Page 199

18. The CSA established how many schedules of controlled substances?

a. 3

b. 4

c. 5

d. 6

19. Which of the following forms is required to order C-II narcotics?

a. DEA 224
b. DEA 106
c. DEA 41
d. DEA 222

20. Which drug used to treat severe acne, requires the registration with the iPLEDGE program?

a. Clozapine
b. Thalidomide
c. Isotretinoin
d. Warfarin

21. Which of the following USP chapters outlines the requirements for sterile

compounding?

a. <795>

b. <797>

c. <61>

d. <1056>

22. Which of the following USP chapters outlines the requirements for nonsterile compounding?

a. <795>

b. <797>

c. <61>

d. <1056>
Arif Pharma Clinic Copyright@arifpharma.com arifpharma.com
Page 200

23. Which reference can be used to check therapeutic equivalency?

a. PDR

b. Drug Facts and Comparisons

c. FDA Orange Book

d. Trissel’s Handbook on Injectable Drugs

24. All the State Boards of Pharmacy together comprise which of the following?

a. FDA

b. DEA

c. BOP

d. NABP

25. Which organization can make decisions regarding licensure, certification, or continuing
education requirements for each state?

a. FDA
b. BOP
c. NABP
d. DEA

26. USP refers to which of the following?

a. United States Pharmacopeia


b. United States Pharmacists
c. Universal Sales Program
d. United Service Plan

27. DEA form 106 is used to report which of the following?

a. Order C-II medications

b. Document destruction of C-II medications

c. Document theft or loss of C-II medications

d. None of the above

Arif Pharma Clinic Copyright@arifpharma.com arifpharma.com


Page 201

28. Which of the following medications is in schedule II?

a. Diazepam

b. Buprenorphine

c. Midazolam

d. Methadone

e. Zolpidem

29. The Pure Food and Drug Act of 1906 required which of the following?

a. That drugs not be mislabeled


b. Industry purity and strength standards
c. Therapeutic benefit documentation
d. Both a and b

Rationale The Pure Food and Drug Act of 1906 required drugs not be mislabeled and set standards
for strength and purity.

30. Drugs are intended for use in which of the following?

a. Prevention of disease

b. Treatment of disease

c. Diagnosis of disease

d. All of the above


Ratioale Drugs are intended for the use of prevention, treatment, and diagnosis of disease.

31. Which of the following medications is with a low abuse potential and acceptable medical uses?

a. Schedule II

b. Schedule III

c. Schedule IV

d. OTC
Rationale Medications with a low abuse potential and acceptable medication use are in scheduleIV.

Arif Pharma Clinic Copyright@arifpharma.com arifpharma.com


Page 202

32. The Durham-Humphrey Amendment created which of the following?

a. Scheduled medications
b. OTC medication class and Rx medication class
c. Safety regulations
d. Purity laws
Rationale The Durham-Humphrey Amendment created 2 medication classes: OTC and prescription.

33. The Drug Listing Act required which of the following?

a. Tax incentives for orphan drugs


b. Labeling of dietary supplements
c. Is on all prescription medications
d. All of the above
Rationale The Drug Listing Act required each drug to be assigned a NDC number.

34. Pharmacies must make a reasonable effort to obtain which of the following?

a. Patient’s name
b. Patient’s physical address
c. NDC numbers for medications
d. Patient counseling
Rationale To accurately dispense a prescription, the pharmacy should obtain the patient’s name,
address, and age.

35. The Poison Prevention Act created which pharmacy dispensing standard?

a. Patient counseling
b. Child-resistant container use
c. Counting tray used
d. All of the above
Rationale The Poison Prevention Act created the standard requirement of child-resistant containers.

36. Which of the following medications is in schedule III?


a. Anabolic steroids
b. Lorazepam
c. Temazepam

Arif Pharma Clinic Copyright@arifpharma.com arifpharma.com


Page 203

d. Morphine
Rationale Anabolic steroids are in DEA schedule III.

37. Which of the following practices promotes the protection of PHI?

a. Faxing without a cover sheet


b. Asking other patrons to step away from the counter during a counseling session
c. Looking up a neighbor’s profile to “see what they are on”
d. Calling out a patient’s name, the drug that is ready, and reason for use

Rationale Asking other patrons to step away from the counter during a counseling session promotes
the protection of PHI.

38. The “Combat Methamphetamine Epidemic Act” restricts the sale of which OTC medication?
a. Acetaminophen
b. Ibuprofen
c. Guaifenesin
d. Pseudoephedrine
Rationale Pseudoephedrine is the main ingredient in methamphetamine.

39. The pharmacy must keep written or electronic records of ephedrine and
pseudoephedrine including which of the following?
a. Product name
b. Date and time of sale
c. Name and address of the purchaser
d. All of the above

Rationale The pharmacy must keep track of the name and address of the purchaser, date and timeof sale,
and the name of the product, in order to abide by the law.

40. The pharmacy must keep written or electronic records of ephedrine and
pseudoephedrine including which of the following?

a. Product name
b. Date and time of sale
c. Name and address of the purchaser
d. All of the above

Rationale The pharmacy must keep track of the name and address of the purchaser, date and time
of sale, and the name of the product, in order to abide by the law.

Arif Pharma Clinic Copyright@arifpharma.com arifpharma.com


Page 204

41. Which form is used to order C-II medication?

a. DEA 41

b. DEA 106

c. DEA 222
d. None of the above
Rationale The DEA 222 is used to order C-II medications.

42. Which of the following methods may be used to transmit a prescription to a pharmacy?

a. Facsimile

b. Telephone

c. E-Script (electronic transfer)

d. All of the above

Rationale Facsimile, telephone, and e-script may be used to transmit a prescription to thepharmacy.

43. Which of the following is not required on a valid prescription?


a. Date of issue
b. Name and address of the practitioner
c. Name of the patient
d. All of the above

Rationale A valid prescription requires date of issue, name and address of the practitioner, and the
name of the patient.

44. A prescription for a C-II substance requires which of the following?

a. Quantity in numerical form (30)

b. Quantity in alpha form (thirty)

c. Physician DEA number

d. All of the above

Rationale A C-II prescription requires the quantity written in numerical and alpha form, and a
physician’s DEA.

Arif Pharma Clinic Copyright@arifpharma.com arifpharma.com


Page 205

45. FDA refers to which of the following?

a. Federal Drug Administration

b. Food and Drug Administration

c. Food and Dental Agency

d. Federal Drug Agency

Rationale The Food and Drug Administration (FDA) protects the public by assuring safety,
effectiveness and control of drugs, cosmetics, food, dietary supplements, and other medical
products and devices

46. Medication labels must contain at the very least which of the following?

a. The name, address, and phone number of the pharmacy

b. Name of the patient

c. Date of next refill

d. a and b only

Rationale Medication labels must contain the name of the pharmacy, address, phone number, and
name of the patient.

47. When faced with different state and federal laws, which should be followed?
a. The more strict law
b. The state law
c. It is the pharmacists’ discretion
d. None of the above
Rationale When faced with different state and federal laws, the more strict law should be followed.

48. The Pure Food and Drug Act of 1906 required which of the following?

a. That drugs not be mislabeled

b. Industry purity and strength standards

c. The exact amount of active ingredient on the label

d. All of the above

Rationale The Pure Food and Drug Act of 1906 required that drugs not be mislabeled, be industrypure
and of standard strength, and contain the amount stated on the label

Arif Pharma Clinic Copyright@arifpharma.com arifpharma.com


Page 206

49. What did the Controlled Substances Act define?

a. Labeling requirements

b. Drug schedules

c. Privacy laws

d. Purity standards
Rationale The Controlled Substances Act defined drug schedules.

50. Which of the following medications is in schedule IV?

a. Morphine

b. Anabolic steroids

c. Clonazepam
d. Heroin
Rationale Clonazepam is a schedule IV medication.

51. Drugs are listed in which of the following?

a. USP
b. HIPAA
c. DEA
d. All of the above
Rationale Medications are listed in the USP.

52. Natural supplements are not drugs because they are not intended for use in which of
the following?

a. Prevention of disease

b. Treatment of disease

c. Diagnosis of disease

d. All of the above


Rationale Natural supplements are not drugs, because they cannot claim to prevent, treat, or
diagnose a disease.
53. Which is the most restrictive class of medications defined by the Controlled Substances Act?
a. C-V
b. C-III
c. C-IId. C-I

Arif Pharma Clinic Copyright@arifpharma.com arifpharma.com


Page 207

Rationale The most restrictive class is C-I.

54. Which of the following is not a factor when defining a medication schedule?

a. Abuse potential

b. Safety information

c. Pharmacokinetics
d. All of the above
Rationale Medication schedules are defined by abuse potential and safety information.

55. Which law created the OTC class of medications?

a. Durham-Humphrey Amendment
b. Controlled Substances Act
c. Poison Prevention Act
d. OBRA ′90
Rationale The Durham-Humphrey Amendment created 2 medication classes: OTC and prescription.

56. HIPAA refers to which of the following?

a. Health Information Privacy Administration and Agency


b. Hospital Infused Pure Antibiotic Administration
c. Health Insurance Portability and Accountability Act
d. Health Insurance Privacy and Accountability Act

Rationale The Health Insurance Portability and Accountability Act (HIPAA) protects the privacy of a
patients’ protected health information.
57. Which of the following is correct about “Caution: Federal law prohibits dispensing
without a prescription”?

a. Must never be shortened


b. Is the federal legend
c. Is on all OTC medications
d. All of the above
Rationale “Caution: Federal law prohibits dispensing without a prescription” is the federal legend.

Arif Pharma Clinic Copyright@arifpharma.com arifpharma.com


Page 208

58. Patient counseling became mandatory after which law was passed?

a. 1970 Poison Prevention Act


b. 1938 Food, Drug, and Cosmetic Act
c. 1951 Durham-Humphrey Amendment
d. 1990 Omnibus Budget Reconciliation Act
Rationale The Omnibus Budget Reconciliation Act of 1990 (OBRA’90) required patient counseling.

59. HIPAA sets standards to protect which of the following?

a. USP
b. PHI
c. NF
d. OTC
Rationale HIPAA protects PHI.

60. Which law sets standards for storing patient information?

a. Poison Prevention Act


b. Controlled Substances Act
c. HIPAA
d. All of the above
Rationale The HIPAA sets standards for storing patient information.

61. The “Combat Methamphetamine Epidemic Act” restricts the sale of which OTC medication?

a. Acetaminophen
b. Ibuprofen
c. Guaifenesin
d. None of the above

Rationale Pseudoephedrine is the main ingredient in methamphetamine.

Arif Pharma Clinic Copyright@arifpharma.com arifpharma.com


Page 209

62. What does the DEA enforce?

a. State-specific dispensing laws


b. Pharmacist licensing
c. Controlled substance laws of the United States
d. OTC product labeling
Rationale The Drug Enforcement Administration (DEA) enforces the controlled substance laws.

63. Who licenses pharmacists?

a. The DEA
b. The State Board of Pharmacy
c. The FDA
d. All of the above
Rationale The State Board of Pharmacy licenses pharmacists.

64. Which of the following medications is in schedule III?

a. Zolpidem
b. Lacosamide
c. Hydromorphone
d. Ketamine
Rationale Ketamine is a schedule III medication.

65. DEA refers to which of the following?

a. Drug Enforcement Administration


b. Drug Enforcement Agency
c. Drug Elimination Administration
d. Drug Evaluation Agency
Rationale The Drug Enforcement Administration (DEA) enforces controlled substance laws.

66. The Food, Drug, and Cosmetic Act of 1938 defined which of the following?

a. Officially defined drugs


b. Established policies to determine drug safety
c. Created childproof cap laws
d. Only a and b
Rationale The Food, Drug, and Cosmetic Act of 1938 officially defined drugs and established policiesto
determine drug safety.

Arif Pharma Clinic Copyright@arifpharma.com arifpharma.com


Page 210

67. Which of the following is an example of a medication that does not require child-
resistant packaging?

a. Birth control pills


b. Lisinopril
c. Hydrochlorothiazide
d. Atorvastatin
Rationale Birth control pills do not require child-resistant packaging.

68. Which of the following practices does not promote the protection of PHI?

a. Using a fax cover sheet


b. Asking other patrons to step away from the counter during a counseling session
c. Inviting more than one patient at a time into the counseling area
d. Keeping up with training every year
Rationale Inviting more than one patient at a time into the counseling area does not promote the
protection of PHI.

69. Which of the following strategies can help decrease handwriting errors?

a. Verbal orders
b. CPOE
c. Tall-man letters
d. Bar code
Rationale Computerized physician order entry (CPOE) decreases handwriting errors.

70. Which of the following abbreviations is on The Joint Commission’s (TJC’s) “Do Not Use” list?

a. IU
b. IV
c. IM
d. IO
Rationale IU should be spelled out to the international unit.

Arif Pharma Clinic Copyright@arifpharma.com arifpharma.com


Page 211

71. Which of the following abbreviations is on The Joint Commission’s “Do Not Use” list?

a. Gid
b. Bid
c. qod
d. tid
Rationale qod should be spelled out to every other day

72. What Health care professional is ultimately responsible for all dispensed prescriptions?

a. Physician
b. Pharmacy technician
c. Dispensing pharmacist
d. Pharmacist manager
Rationale The dispensing pharmacist is responsible for all dispensed medications.

73. What clinical information should be obtained on a pediatric patient to help prevent
medication errors?

a. Date of birth
b. Weight
c. Height
d. All of the above
Rationale Date of birth, height, and weight are all essential to prevent errors.

74. A pregnant patient presents a prescription for Accutane to the pharmacy technician. What
should the technician do?

a. Alert the pharmacist that the patient is pregnant and filling a category X medication
b. Contact the physician to discuss the patient’s pregnancy status
c. Return the patient’s prescription without filling
d. Fill the prescription without notifying the pharmacist
Rationale The pharmacist should discuss the clinical situation with the patient and physician.

75. Which of the following are high-alert medications?

a. Insulin, heparin
b. Insulin, citalopram
c. Heparin, citalopram
d. Morphine, pantoprazole

Arif Pharma Clinic Copyright@arifpharma.com arifpharma.com


Page 212

Rationale Insulin and heparin are both high-alert medications.

76. What Health care professional is ultimately responsible for all dispensed prescriptions?

c. Physician
d. Pharmacy technician
c. Dispensing pharmacist
d. Pharmacist manager
Rationale The dispensing pharmacist is responsible for all dispensed medications.

77. What clinical information should be obtained on a pediatric patient to help prevent
medication errors?

a. Date of birth
b. Weight
c. Height
d. All of the above

Rationale Date of birth, height, and weight are all essential to prevent errors

78. A pregnant patient presents a prescription for Accutane to the pharmacy technician. What
should the technician do?

a. Alert the pharmacist that the patient is pregnant and filling a category X medication
b. Contact the physician to discuss the patient’s pregnancy status
c. Return the patient’s prescription without filling
d. Fill the prescription without notifying the pharmacist

Rationale The pharmacist should discuss the clinical situation with the patient and physician.

79. Which of the following are high-alert medications?

a. Insulin, heparin
b. Insulin, citalopram
c. Heparin, citalopram
d. Morphine, pantoprazole
Rationale Insulin and heparin are both high-alert medications.

Arif Pharma Clinic Copyright@arifpharma.com arifpharma.com


Page 213

80. Which of the following should not be used during prescribing in order to minimize errors?

a. mg
b. grain
c. mL
d. gm
Rationale The apothecary system (grain) should be avoided to minimize errors.

81. What can contribute to a medication error during a verbal order?

a. Poor cell phone connection


b. Accent
c. Similar-sounding medications
d. All of the above
Rationale Many distractions can occur during a verbal order including cell phone connection issues,
accent, and similar sounding medications.

82. Which of the following strategies can be utilized during verbal orders to help
minimize medication errors?

a. Read back the order to the prescriber


b. Guess at the prescriber’s order
c. Multitask during verbal orders
d. All of the above
Rationale Reading the verbal order back to the prescriber helps to verify the order.

83. Which of the following patients are at the highest risk of a calculation error?

a. A 7 week old
b. A 14 year old
c. A 28 year old
d. A 56 year old
Rationale Infants are at the highest risk of a calculation error due to their small size.

84. Which of the following prevention strategies can be used to help prevent look-alike-sound
a like errors?
a. Computerized physician order entry
b. Tall-man lettering

Arif Pharma Clinic Copyright@arifpharma.com arifpharma.com


Page 214

c. Shelf dividers
d. Adequate lighting
Rationale Tall-man lettering can help distinguish between look-alike sound a like medications.

85. Which of the following organizations is devoted entirely to medication error prevention?

a. Institute for Safe Medication Practices


b. National Patient Safety Foundation
c. The Joint Commission
d. The American Heart Association

Rationale The ISMP is dedicated solely to medication error prevention.

86. The use of shelf dividers can help prevent which of the following type of error?

a. Calculation error
b. Decimal point error
c. Look-alike packaging error
d. High-alert medication error
Rationale: Shelf dividers can help prevent look-alike packaging errors.

87. Which of the following is a high-alert medication?

a. Insulin
b. Heparin
c. Morphine
d. All of the above

Rationale Insulin, heparin, and morphine are all high-alert medications.

88. Why are high-alert medications more dangerous than other medication categories?

a. Commonly prescribed
b. Uncommonly prescribed
c. High cost
d. Devastating adverse effects
Rationale High-alert medications have the potential to cause devastating adverse effects.

Arif Pharma Clinic Copyright@arifpharma.com arifpharma.com


Page 215

89. Which of the following systems helps decrease medication errors when stocking medications?

a. Sorting medications by brand name A to Z


b. Sorting medications by generic name A to Z
c. Sorting medications by dosage form A to Z
d. Sorting medications by fast movers

Rationale Medications should be sorted regardless of dosage form by generic name A to Z to


minimize errors.

90. Which of the following should be dispensed with a prescription for amoxicillin of 250 mg/5
mL—give 500 mg PO bid × 10 days?

a. 2.5 mL syringe
b. 5 mL dosing spoon
c. 5 mL syringe
d. 10 mL dosing spoon
Rationale The dose will be 2 mL, thus a 2.5-mL syringe is closest in size.

91. Which of the following pharmacy personnel can counsel a patient on how to use a
medication device?

a. Cashier
b. Pharmacy technician
c. Certified pharmacy technician
d. Pharmacist
Rationale Only a pharmacist can counsel a patient on medication use.

92. Which of the following contributes to medication errors?

a. Poor lighting
b. Cluttered work space
c. Loud noise
d. All of the above

Rationale There are many factors to medication errors including poor lighting, clutter, and noise.

Arif Pharma Clinic Copyright@arifpharma.com arifpharma.com


Page 216

93. Which of the following prevention strategies can be implemented to decrease medication
errors?

a. Adequate lighting
b. Dim lighting to save on electricity
c. Spotlight each work area
d. None of the above

Rationale Adequate lighting can help decrease medication errors.

94. Which committee develops a formulary for an institution?

a. BOP
b. FDA
c. P&T
d. TJC
Rationale The Pharmacy and Therapeutics (P&T) Committee develops a formulary.

95. Which of the following is the best method to reduce medication errors?

a. Education of pharmacy staff


b. Redesign systems and processes that lead to errors
c. Correct individuals who make errors
d. Overlook errors

Rationale Redesigning systems and processes that lead to errors is the best way to decrease errors.

Arif Pharma Clinic Copyright@arifpharma.com arifpharma.com


Page 217

96. When should a pharmacy technician alert a pharmacist to make a clinical decision?

a. Therapeutic interchange
b. Drug utilization review
c. Medication misuse
d. All of the above
Rationale A pharmacy technician should always alert a pharmacist to make a clinical decision.

97. The OBRA ’90 required pharmacies to do which of the following?

a. Perform a prospective drug utilization review on all prescriptions


b. Provide drive-through service
c. Therapeutically substitute brand to generic drugs
d. Protect private information

Rationale OBRA ′90 required pharmacies to perform a prospective drug utilization review.

98. Which of the following numbers could lead to a medication error due to a decimal point
error?
a. 1.00
b. 1.01
c. 0.11
d. 0.01
Rationale Trailing zeros can cause medication errors

99. Which of the following is required during a prospective drug utilization review?

a. Therapeutic duplication
b. Drug-disease contraindications
c. Incorrect drug dosage
d. All of the above

Ratioanle There are many sections to a drug utilization review.

100. Which of the following committees manages the formulary system?


a. Medical Executive Committee
b. Nursing Council
c. Pharmacy and Therapeutics Committee
d. Pharmacy Administration Board

Arif Pharma Clinic Copyright@arifpharma.com arifpharma.com


Page 218

Rationale The Pharmacy and Therapeutics (P&T) Committee develops a formulary.

101. Which of the following is an example of a generic substitution?

a. Norvasc-Amlodipine
b. Lopressor-Toprol XL
c. Protonix-Prevacid
d. Prozac-Zoloft
Rationale Norvasc-amlodipine is the only example of a brand-generic substitution.

102. Which of the following is an example of the therapeutic substitution?

a. Protonix-Prevacid
b. Norvasc-Amlodipine
c. Prozac-Fluoxetine
d. Ventolin-Albuterol
Rationale Protonix-Prevacid is the only example of a therapeutic substitution.

103. If a medication guide is required to be dispensed with a medication, when should it be given
to the patient?

a. First time only


b. Refills only
c. Every time dispensed
d. Only if the patient is younger than 18 years
Rationale Required medication guides must be dispensed with every dispensed prescription.

104. Mr. Smith is asking for an OTC recommendation. Who may answer his question?

a. Pharmacy technician
b. Certified pharmacy technician
c. Pharmacist
d. All of the above
Rationale Only a pharmacist can make a clinical decision for an OTC recommendation.

105. Which of the following activities may be performed by a certified pharmacy technician?

a. Patient counseling
b. Generic substitution
c. Prospective drug utilization review
d. OTC medication recommendation

Arif Pharma Clinic Copyright@arifpharma.com arifpharma.com


Page 219

Rationale Pharmacy technicians may perform generic substitutions whare allowed by law.

106. Which of the following abbreviations is on TJC’s “Do Not Use” list?
a. Gd
b. Daily
c. Bid
d. Twice daily
Rationale Gd should be spelled out as daily or once daily.

107. Which law requires pharmacies to perform a prospective drug utilization review on
all prescriptions?

a. Food, Drug, and Cosmetic Act of 1936


b. Durham-Humphrey Act of 1950
c. Occupational Safety and Health Act of 1970
d. Omnibus Budget Reconciliation Act of 1990
Rationale The OBRA ′90 requires a prospective drug utilization review.

108. What does the FDA refer to?

a. Free Drug Assistance


b. Food and Drug Association
c. Food and Drug Abuse
d. Food and Drug Administration
Rationale FDA stands for Food and Drug Administration.

109. Which of the following is not a task completed by pharmacy technicians?

a. Refilling prescriptions
b. Performing inventory
c. Counseling patients
d. Ordering medications
Rationale Pharmacy technicians may not counsel patients.

110. Pharmacy technicians may perform all of the tasks except which of the following?

a. Drug utilization review


b. Inventory review

Arif Pharma Clinic Copyright@arifpharma.com arifpharma.com


Page 220

c. Laminar flow hood cleaning


d. Repackaging medications
Rationale Pharmacy technicians may not perform drug utilization review

111. What should a certified pharmacy technician do when encountering an incorrect drug
dosage during a prospective drug utilization review?

a. Evaluate drug dose


b. Contact the physician to change the dose
c. Alert the pharmacist
d. Return the prescription to the patient
Rationale A pharmacy technician should alert the pharmacist when encountering an alert during
aprospective drug utilization review.

112. Who may recommend an OTC medication to a patient?

a. Pharmacy technician
b. Certified pharmacy technician
c. Certified pharmacy technician with OTC training
d. None of the above
Rationale Only a pharmacist may make recommendation to a patient.

113. For which of the following reasons tall-man lettering is being used on medications?

a. Easier to read for patients


b. Reduces errors from look-alike drugs
c. Enlarged to fit the size of the prescription bottle
d. None of the above
Rationale Tall-man lettering helps decrease errors from look-alike drugs.

114. Which of the following best defines a formulary?


a. A set of rules for nurses on how to administer medications
b. A list of medications that are approved by the medical council
c. A list of medications that are approved by the P&T Committee
c. A set of rules for the department of pharmacy

Rationale A formulary is a list of medications that is approved by the P&T Committee.

Arif Pharma Clinic Copyright@arifpharma.com arifpharma.com


Page 221

115. The requirement for pharmacists to counsel patients on a medication they have not taken
before is listed under which law?

a. Prescription Drug Marketing Act


b. Kefauver-Harris Amendment
c. OBRA ’90
d. Durham-Humphrey Amendment
Rationale The OBRA ′90 requires a pharmacist to offer to counsel a patient.

116. Which of the following prevention strategies should be utilized to decrease medication errors?

a. Two-person independent check of medication calculation


b. Tall-man lettering
c. Shelf separators
d. All of the above
Rationale Multiple strategies may be employed to decrease medication errors.

117. Which of the following strategies have been shown to decrease medication errors?

a. Punitive strategies
b. Staff education
c. Redesign process
d. All of the above
Rationale Redesigning processes have shown to decrease medication errors.

118. Which of the following strategies can help prevent illegible handwriting errors?

a. Verbal orders
b. Telephone orders
c. Computerized physician order entry
d. Handwriting Classes
Rationale CPOE can help prevent illegible handwriting errors.

119. Which of the following medications is a high-alert medication and can cause significant
patient harm?

a. Acetaminophen
b. Citalopram
c. Warfarin
c. Fluticasone

Arif Pharma Clinic Copyright@arifpharma.com arifpharma.com


Page 222

Rationale Warfarin is a high-alert medication.

120. Which of the following medications has a narrow therapeutic range?

a. Fluticasone
b. Acetaminophen
c. Phenytoin
d. Metoprolol
Rationale Phenytoin has a narrow therapeutic range.

Arif Pharma Clinic Copyright@arifpharma.com arifpharma.com


Page 223

DHA EXAM
TOTAL 120
QUESTIONS
General Management:

1. Skin decontamination:
• Don’t neutral as naturalization is exothermic.
• Put plenty of water.
2. Gastric Management:
a) Emesis:
• Using ipecac syrup
• Acting after 30 & make 2 vomiting episodes in 1 hour.
• Emesis C/I in:
1- Corrosive
2- Sharp objective
3- Children < 6 months
4- 4- Seizure & coma
5- Already vomiting
6- Fast acting
b) Gastric lavage:
• Using it in case emesis is C/I
c) Activated charcoal:
• Universal antidote
• 3 cases charcoal ineffective:
1- Iron, Ca, Cl, Li (Metals)
2- Strong acrid or base HCL, H2SO4, NaOH, KOH
3- 3- Alc.
4- Methanol, Ethylene glycol

d) Whole bowel irrigation:


• Used when charcoal is not available or not effective
• Through anus
e) Forced Diuresis:
• Acidic drugs excreted in alkaline urine & vice versa
• Urine alkalizer: acetazolamide NaHCO3
• Urine acidifier: NH4CL ascorbic acid (Vit.C)

ARIF PHARMA CLINIC Copyright@arifpharma.com Www. arifpharma.com


Page 224

f) Dialysis:
• Done if the drug is absorbed “reach blood”
• Peritoneal dialysis
• Hemodialysis
• Drug must:
1- M.wt
2- Vd. Blood + toxin
3- Binding protein
g) Hemoperfusion: Charcoal
• Thrombocytopenia
• Hypoglycemia
• Hypocalcemia
• Leukopenia
• Used in theophylline toxicity

Blood

ARIF PHARMA CLINIC Copyright@arifpharma.com Www. arifpharma.com


Page 225

Management of specific ingestion

Acetaminophen
N –Acetylcysteine (ORAL/I.V)
Vomiting Hepatectomy +
Metoclopramide
Toxic dose > 4 gm/day

Ethylene glycol -Ethanol Alc. acetic acid “in urine”


Dehydrogenase
OR
Alcohol - Alcohol Dehydrogenase inhibitor
Antifreeze
dehydrogenase e.g disulfiram, fomepizole
Oxalic acid (toxin) OR
Vit.B6/B1 Form
non oxalate metabolite
Methanol -Ethanol or fomepizole or disulfiram

Alc. -Vit.B9 Form


dehydrogena non metabolite
se
Formaldehyde
“Blindness”
TCA -Physostigmine ( Ach)
-Phenytoin or BDZ for seizure
Anticholinergic
S.E

Seizure
SSRI Gastric lavage & supportive treatment
Heparin Protamine sulfate I.V , S.C. 1mg
Protamine neutralizes 100 I.V. heparin
Warfarin Vit.K. (oral, paretral)
Benzodiazepine FLumazenil “short acting”Need
multiple daily dose Activated
charcoal
Cathartic ( Laxative)

B. Blocker -Epi
-Glucagon hormone S.C, I.V, I.M
Hypoglycemia
-Gastric Lavage & Charcoal

ARIF PHARMA CLINIC Copyright@arifpharma.com Www. arifpharma.com


Page 226

C.C.B. -Ca+gluconate
-CaCl2 (I.V)
Hypoglycemia -Glucagon
Cocaine BDZ for seizure
Labetalol for HT
C. N. S stimulant
Corrosive -Avoid neutralization
-Milk + uncooked egg
Strong acid or base
Cyanide Cyanide KIT Amyl
nitrite +Na+ (thiosulfate)Hb
CN blind to Hb & stop respiration Amyl
Nitrite Met.Hb
CN
Na thiosulfate
Met.Hb.CN
Met.CN Low respiration
+ Hb Excretion

Digoxin K+ , Digibind , ipecia ,


Phenytoin for seizure
Charcoal
+2 CaCl2 CaCl2 ,
Mg
SPS ,
K+
(See in renal Failure)
Iron Deferoxamine (chelating agent)

Overdose diarrhea Make red urine


Ipecac, whole bowl irrigation
Isoniazid Vit.B6 Avoid
S.E neurotoxicity Vit.B6 vomiting
Seizure

Lead (Pb+2) EDTA (IM / I.V)


Dimercaprol ( I.M.)
(chelating agent)

SPS, HemodialysisNo
Li+ charcoal

ARIF PHARMA CLINIC Copyright@arifpharma.com Www. arifpharma.com


Page 227

Opioids Opioid antagonist:


Downer Naloxone Short-acting
Naltrexone
Long-acting
Nalmefene

Organophosphorus c.p.d Atropine


• Pralidoxime (2- PAM)
(Irreversible anticholinesterase)
( Ach) cholinesterase reactivator

Salicylate “ aspirin” urine alkalizer ( NaHCO3,


acetazolamide)

Theophylline Hemoperfusion
Adenosine (specific antidote)
Hemodialysis

ARIF PHARMA CLINIC Copyright@arifpharma.com Www. arifpharma.com


Page 228

DHA EXAM QUESTIONS


START
1. Which of the following case(s) can be categorized as toxicity?

A Development of chronic lung inflammation in smokers.


B Chronic rhinitis associated with influenza.
C Gout, which is inflammation of joints due to the accumulation of uric acid
crystals.
D Post-surgical pus formation in a wound on the forearm.
E All of the above

2. Liver is the main metabolizing organ in the body. Which of the following statement is true in
association with toxicant metabolism and reduction of toxicity?

A Liver alone metabolizes all the toxicants and does not need any further assistance
B Kidney plays an essential role in the elimination of metabolized toxicants
through urine.
C Cardiovascular system plays an important role in the detoxification of chemicals.
D All the chemicals are removed from the body through bile, after metabolism in the
liver.
E Gastrointestinal tract controls the flow of chemicals into the liver.

3. Which of the following is not a natural route of exposure to chemicals?

A An Inhalation
B Oral/Gastrointestinal tract
C Topical/Dermal
D Intraperitoneal
E None of the above

Answer: (D) Intraperitoneal route of administration is used in various experimental and clinical
administrations where a test substance/drug is delivered into the peritoneal cavity with the help
of a syringe and needle.

4. Which one of the following is most accurate in the case of biotransformation of the
chemicals/toxicants?

A It is a synonym for metabolism.


B Biotransformation always results in the activation of a neutral chemical.
C Biotransformation is the transformation of one chemical into another
irrespective of its activity.

ARIF PHARMA CLINIC Copyright@arifpharma.com Www. arifpharma.com


Page 229

D Biotransformation reduces toxicity by transforming chemicals into neutral


ones.
E Biotransformation does not change the chemicals but alters their activities in a
biological system.

5. What is bioaccumulation?

A An Accumulation of biological entities in a geographical area.


B Accumulation of fossils of plants and animals underneath several layers of mud, rock,
and sand.

C Accumulation of fluid in a body part after a toxic exposure.


D Accumulation of chemicals in the body mainly in fat tissue and bones. The cumulative
effect of several

6. Bioactivation encompasses the following.

A A less harmful substance is converted into a more harmful one.


B It is a process that activates the metabolizing enzymes in the liver.
C Bioactivation is a part of the biotransformation process.
D B&C
E A&C

7. Which of the following enzymes play a crucial role in th e detoxification of the chemicals?

A Digestive enzyme in the gastrointestinal tract destroys all the chemicals entering into
the body.
B Cytochrome p450 oxidases, UDP- glucuronyltransferases, and glutathione S-
transferases

C Superoxide dismutase, catalase, glutathione peroxidase


D B&C
E All of the above

8. Chemicals have specific targets in the body.

A True
B Few do have targets, others are nonspecific
C False

ARIF PHARMA CLINIC Copyright@arifpharma.com Www. arifpharma.com


Page 230

D Depends on the route of exposure


E Chemicals decide after entering into the body.

9. Which is an acute event of toxicity?

A Lung inflammation after two days of inhalation of metal dust.


B Liver toxicity after three daily doses of 325 mg of paracetamol for one week only.
C Liver toxicity of three doses of 500 mg of paracetamol for one day only.
D A toxic event occurring suddenly without an unknown cause.
E A&C

10. Which is true regarding chronic exposure?

A Repeated exposure occurs for a period of three months.


B Repeated exposure occurring for a period of one month.
C Repeated exposure occurring for a period of more than three months.
D Repeated exposure occurring for a period of one day.

E It is not categorized as chronic if no toxicity occurs.

11. Following phases are included in the biotransformation of the chemicals.

A Phase 0, Phase I, Phase II


B Phase I, Phase II, and sometimes Phase III
C Phase 0, Phase I, Phase II, Phase III and Phase IV
D Phase 0, Phase I,
E Phase 0, Phase I, Phase II, Phase III

12. Chemicals are toxic at any dose level.

A A few chemicals, which are lethal to humans, can be defined like that.
B No, dose determines whether they are toxic.
C There is no dose-response relationship when we talk about highly toxic
chemicals.
D The statement can be applied to inhalant toxicants, because of the lungs’ high
sensitivity.

ARIF PHARMA CLINIC Copyright@arifpharma.com Www. arifpharma.com


Page 231

E This is true for all kinds of pesticides.

13. Which of the following will have the most damaging effects?

A A hydrophilic chemical via the inhalation route.


B A lipophilic chemical via dermal route.
C A lipophilic chemical via the inhalation route.
D A hydrophilic chemical via the oral route.
E A hydrophilic chemical via dermal route.

14. Which of the following will have the least damaging effects?

A A hydrophilic chemical via the inhalation route.


B A lipophilic chemical via dermal route.
C A lipophilic chemical via the inhalation route.
D A hydrophilic chemical via the oral route.
E A hydrophilic chemical via dermal route.

15. Most of the chemicals are excreted out by the kidneys through urine. Which are the other
organs or modes involved in the excretion of chemicals?

A Lungs through the expiration of volatile compounds.


B Sweat glands excrete a number of metabolites out of the body.
C Through bile.
D Mammary glands
E All of the above but A, B, and C are important

Answer: (E) Theoretically toxicants/metabolites can be eliminated along with any secretion from
the body.

16. What is the biological half-life (t1/2) of a chemical?

A A chemical reduces the life of an individual to half of the average life.


B The time in which the systemic concentration of a chemical reduces to half of its
initial concentration.
C Half of the time taken to reach the maximum concentration of chemicals in the
system.

ARIF PHARMA CLINIC Copyright@arifpharma.com Www. arifpharma.com


Page 232

D Half of the dose of a chemical can affect the quality of life.


E None of the above

17. Which information can be obtained from an acute toxicity study?

A Median toxic dose (TD50)


B Median lethal dose (LD50)
C No Observed Adverse Effect Level (NOEL)
D Target organ
E All of the above

Answer: (E)– In an acute toxicity study any endpoint parameter such as death, brain damage, liver
damage, etc. can be used to obtain information regarding a particular toxicant.

18. A particular dose of a substance X is minimally toxic to animals. Substance Y is


also minimal toxic to the animals at the same dose, but when both substances are
administered together they show toxicity several orders of magnitude higher
than compared with individual administrations. This is an example of:

A Potentiation
B Synergism
C Additivity
D Acute Toxicity
E Agonism

Answer:–(B) In potentiation, a non-toxic substance increases the toxicity of a relatively less


toxic substance. In synergism a substance shows unusually high toxicity in presence of a less
toxic substance, it is much greater than the additive effect. In agonism a substance (agonist)
mimics the mechanism of action of another substance, its opposite is known as antagonism.

19. 1 ppm is equivalent to:

A 1 g/kg
B 1 μg/kg
C 1 mg/kg
D 1 mg/100g
E None of the above

ARIF PHARMA CLINIC Copyright@arifpharma.com Www. arifpharma.com


Page 233

Answer: (C) 1 mg is the one-millionth part of a kilogram.

20. Which is true about the LD50?

A Dose of a substance to which 50% of animals do not show any response.


B Dose of a substance which kills 50% of animals exposed.
C 50% of the dose of a substance which can kill an animal.
D Dose of a substance which can kill 50 animals.
E 50 mg/kg dose of a substance to test the toxic responses in animals.

21. Acceptable daily intake (ADI)

A An estimate of the amount of food taken daily.


B Amount of nutrients required on daily basis.
C A&B
D An estimate of the amount of substance in the food that can be ingested daily over a
lifetime by humans without appreciable health risk.
E All of the above

22 Descriptive toxicology is concerned directly with

A Description of toxicology-related research fields.


B Origin of toxicants in nature.
C Toxicity testing, which provides information for safety evaluation and regulatory
requirements?
D Describing toxicology as a science and art to laymen and unrelated scientific
fields.
E A and D

23. The term ‘toxin’ generally refers to toxic substances that are

A Any kind of poison.


B Produced by biological systems such as plants, animals, fungi, or bacteria.
C Toxicants released as industrial effluents.
D Toxic elements of inorganic origin such as mercury, lead, arsenic etc.
E All substances except gases.

ARIF PHARMA CLINIC Copyright@arifpharma.com Www. arifpharma.com


Page 234

24. The term ‘toxicant’ is used in speaking of toxic substances that are

A Specifically toxic to humans.


B Nonlethal in their effects.
C produced by organisms.
D produced by anthropogenic activities.
E Toxic to organisms such as plants, animals, fungi, or bacteria.

25. What is a chemical allergy?

A The allergy is described in terms of chemistry.


B An immunologically mediated adverse reaction to a chemical resulting from previous
sensitization to that chemical or to a structurally similar one.
C When an individual exhibit adverse immunological response against all chemicals.
D Immunological response where chemicals mediate the process instead of antibodies.
E A and D

26. Chemical idiosyncrasy

A Extreme sensitivity to low doses or extreme insensitivity to high doses of the


chemical.
B Attribute of chemicals where they can elicit a low or high response that solely
depends on their chemistry.
C Abnormal responses of individuals towards chemicals which is determined by the
genetic constitution.
D Abnormal responses of individuals towards chemicals which is determined by
environmental factors.
E A and C

27. Threshold dose

A The dose level at which mortality starts occurring.


B A threshold for a lethal dose.
C The dose below which no effects appear.
D A and B
E The dose at which 50% animals exhibit toxicity signs

ARIF PHARMA CLINIC Copyright@arifpharma.com Www. arifpharma.com


Page 235

28. No observed adverse effect level (NOAEL)

A. The condition of experimental animals when no adverse effect can be observed.


B. The maximum dose at which the toxicant show no signs of toxicity.
C. A condition where adverse effects exist, but are hard to observe.
D. A and C
E. The maximum dose of a chemical which caused the death, hence no other adverse
effect got observed.

29. Which of the following can be a source of toxicants?


A Car
B Fruits
C Cosmetics
D Medicines
E All of the above

30. Which one of the following elements are required by our body for normal functioning of some
enzymes?

A Mercury (Hg)
B Zinc (Zn)
C Lead (Pb)
D Antimony (Sb)
E Scandium (Sc)

31. Which one of the elements is not required by our body?

A. Selenium (Se)
B. Potassium (K)
C. Copper (Cu)
D. Arsenic (As)
E. Phosphorus (P)

Answer: D Arsenic is one of the most toxic substances and people are exposed to it through
contaminated water and soil.

32. What is a reference dose (RfD)?


A Dose of a standard chemical to test the toxic doses of other chemicals.
B An estimate of exposure to an agent for a long period without any appreciable risk to
life.

ARIF PHARMA CLINIC Copyright@arifpharma.com Www. arifpharma.com


Page 236

C Dose of an agent during a previous exposure to estimate the risks of a latest exposure
to the same agent.
D All of the above
E None of the above

33. Where in the body do toxicants/chemicals get stored?

A. Plasma proteins
B. Body fat
C. Liver and kidneys
D. Bones
E. All of the above

Answer: (E) – Storage of toxicants within the body depends on their affinity towards the different
kinds of tissues and their components. For example, lipophilic substances are most likely to be
stored in fat tissue.

34. The mode of excretion of xenobiotics from the body. Select which applies.

A Urinary
B Fecal (nonabsorbed, biliary excretion)
C Exhalation
D Sweat, saliva and milk
E All of the above

35. Egg shell thinning is caused by the pesticides DDT and DDE. The mode of exposure, in this
case, is the following:

A. Direct exposure of birds to DDT and DDE.


B. Nesting of birds in DDT and DDE exposed agricultural fields.
C. Movement of DDT and DDE in the food chain.
D. DDT and DDE don’t cause egg shell thinning.
E. Direct exposure of eggs to DDT and DDE.

36. Which one of the following does not elicit toxicological interests?

A Forest fire
B Volcanic eruption
C Earthquake
D Acid rain
E Sand storm

ARIF PHARMA CLINIC Copyright@arifpharma.com Www. arifpharma.com


Page 237

37. What does IDLH stands for?

A International Directory For Long-Term Health Effects


B Immediately Dangerous To Lung Or Heart
C International Directory For Lung And Heart Diseases
D Immediate Or Delayed Effects On Lung Or Heart
E Immediately Dangerous To Life Or Health

Answer: (E) – Represents the levels of a particular hazardous substance.

38. Which one of the following are the main targets of lead toxicity?

A Liver and kidneys


B Nervous system and hematopoietic system
C Heart and lung
D Bones and muscles
E Skin and hair

Answer: (B) – Lead targets the developing nervous system and shows severe effects
in children. In adults mainly the hematopoietic system is targeted.

39. Organic mercury targets whereas inorganic mercury primarily target .

A. Bones, ligaments
B. Liver, hematopoietic system
C. Nervous system, kidneys
D. Hematopoietic system, nervous system
E. Liver, nervous system

40. ‘Itai-Itai disease is caused by

A Cadmium
B Mercury
C Lead
D Copper
E Chromium

ARIF PHARMA CLINIC Copyright@arifpharma.com Www. arifpharma.com


Page 238

Answer: (A) – Itai itai is a cadmium poisoning disaster that occurred around 1912 in Japan.
The disease affected kidneys and bones causing severe pain in joints and the spine,
hence the name itai itai (“Ouch, Ouch” or “It hurts, it hurts”).

41. ‘Black foot’ disease is caused by

A Chromium
B Mercury
C Arsenic
D Lead
E Copper

42. Which of the following is not a pesticide?

A Chloropicrin
B Fluoroacetamide
C Malachite green
D Nicotine
E Malathion

Answer: (C) - Malachite green is used as a dye in leather and paper industries.

43. Which of the following toxins is not produced by bacteria?

A. Cholera toxin
B. Aflatoxin
C. Botulinum toxin
D. Tetanus toxin
E. Diphtheria toxin

44. Ergotism is associated with toxin produced by

A Bacteria
B Plant
C Fungus
D Virus
E Mycoplasma

ARIF PHARMA CLINIC Copyright@arifpharma.com Www. arifpharma.com


Page 239

45. Aspergillus species produce aflatoxins. Aflatoxin B1, one of the aflatoxins, causes the
following toxic effect/s in humans:

A Severe liver toxicity and Carcinogenesis


B It is less toxic due to its natural origin
C Severe renal effects resulting in kidney failure
D Cardiotoxicity
E Respiratory collapse

46. Tetrodotoxin, a deadly toxin, is found in

A. Rattlesnake
B. Shellfish
C. Mussels
D. Pufferfish
E. Scorpion

Answer: (D) – The liver, intestine, and skin of puffer fish are known to be a source of tetrodotoxin,
a potent neurotoxin, which is produced by symbiotic bacteria Pseudoalteromonas tetraodonis and
species of Vibrio and Pseudomonas.

47. Which is the most likely toxic effect of cosmetics?

A Skin corrosion
B Allergic contact dermatitis
C Local muscular degeneration
D Local neuronal damage
E Major risk of systemic toxicity after absorption of chemicals through the skin.

48. Gastrointestinal tract does not have any profound effect on the nature of ingested
chemicals

A True
B False
C Maybe
D Gut-microflora plays important role in biotransformation of ingested
chemicals.

ARIF PHARMA CLINIC Copyright@arifpharma.com Www. arifpharma.com


Page 240

E It does not matter whether the gastrointestinal tract has any effects on
chemicals

49. Which of the following is the main way of transportation of a lipid soluble toxicant within
body?

A. Filtration
B. Special transport
C. Endocytosis
D. D A and C
E. E Passive diffusion

50. What is the most common toxicity target of ethanol (beverage alcohol) in humans?

A Brain
B Fetus
C Liver
D Kidneys
E Heart

51. What are the common targets of ethanol toxicity in humans?

A Liver, brain, heart, and kidneys


B Liver, lungs, and intestine
C Liver, brain, and fetus
D Spleen, liver, brain, and thymus
E All of the above

Answer: (C) After the liver and brain, the ethanol causes fetal toxicity
known as Alcohol Fetal Syndrome (AFS) causing severe mental and physical
damage in developing babies.

52. Methanol consumption is associated with permanent blindness. The following comment is
true regarding methanol toxicity.

A Methanol, unlike ethanol, causes direct toxicity to optical nerves after absorption.
B Apart from blindness, methanol is less toxic than ethanol.
C Alcohol dehydrogenase converts methanol into formaldehyde which causes
blindness.
D Aldehyde dehydrogenase converts methanol into formaldehyde which causes
ARIF PHARMA CLINIC Copyright@arifpharma.com Www. arifpharma.com
Page 241

blindness.
E Alcohol dehydrogenase and aldehyde dehydrogenase converts methanol into
formic acid which causes blindness.

53. Which part of the body is primarily affected by caffeine intoxication?

A. Heart
B. Brain
C. Liver
D. Kidneys
E. Gastrointestinal tract

54. Leaves from the following plants are the significant source of caffeine.

A Coffea arabica
B Erythroxylum coca
C Cola acuminata
D Camellia sinensis
E Theobroma cacao

55. Who first of all described the association between chemical exposure and cancer?

A. Percival Pott
B. Paracelsus
C. Mathieu Orfila
D. Bernardino Ramazzini
E. Oswald Schmiedeberg

56. Who is the father of forensic toxicology?

A Louis Lewin
B Paracelsus
C Mathieu Orfila
D Bernardino Ramazzini
E Oswald Schmiedeberg

ARIF PHARMA CLINIC Copyright@arifpharma.com Www. arifpharma.com


Page 242

57. Who is the father of occupational medicine?

A. Louis Lewin
B. Paracelsus
C. Mathieu Orfila
D. Bernardino Ramazzini
E. Oswald Schmiedeberg

58. Exposure to is associated with occupation.

A Aflatoxins
B Cigarette smoke
C Formaldehyde
D Ethanol
E Acetaminophen

Answer: C – Formaldehyde is used in shoe industry and has been associated with nasal cancer.

59. Exposure to is associated with lifestyle.

A. Chromium
B. Benzene
C. Benzidine
D. Nicotine
E. Asbestos

60. Mesothelioma (cancer of lining covering internal organs) is associated with exposure to

A Benzene
B Nickel
C Mercury
D Arsenic
E Asbestos

61. Benzene is known to cause cancer.

A. Bone marrow
B. Cervical
C. Ovarian
D. Prostate
E. Breast

ARIF PHARMA CLINIC Copyright@arifpharma.com Www. arifpharma.com


Page 243

62. Thalidomide is known for its effects.

A Teratogenic
B Carcinogenic
C Mutagenic
D Neurotoxic
E Hepatotoxic

.
63. The very well reported birth defect of thalidomide is

A. Cleft palate
B. Congenital heart disease
C. Ectrodactyly
D. Club foot
E. Phocomelia

Answer: E – Phocomelia, a condition where long limb bones are not formed in fetuses. Arms without
long bones appear like flippers hence the condition is called ‘Flipper Arms’ a common condition
caused by thalidomide.

64. What is the expression of dose of a substance?

A mg/lbs
B g/kg
C μg/100g
D μg/kg
E mg/kg

Answer: E – The dose of a substance is always measured in mg/kg body weight. Where the
measurement in case of toxic gases or fumes the exposure level is measured as a concentration
in ppm/m3.

65. Ppm (parts per million) is equivalent to


A. ml/L
B. mg/kg
C. μg/kg
D. mg/L
E. μg/g

ARIF PHARMA CLINIC Copyright@arifpharma.com Www. arifpharma.com


Page 244

66. Which of the following pesticides is acetylcholinesterase inhibitor?

A Formamidines
B DDT and DDE
C Nicotine
D Pyrethroids
E Organophosphates

67. Which one is an antidote for organophosphate poisoning?

A. Activated charcoal
B. EDTA
C. Ipecac
D. Atropine
E. BAL

68.Dose is defined as the

A Amount of substance which is released into the environment.


B Amount of substance which reaches the target site in the body.
C Amount of substance which is converted into active metabolite to exert a toxic
effect.
D Amount of substance which enters into the body.
E Amount of substance is bound to the plasma proteins.

69. Which one of the following is true regarding Dose-response relationship?

A. Response is directly proportional to the dose of a substance.


B. B Dose exhibits ‘all or none response.
C. The dose which elicitst a toxic response.
D. The dose which does not induce any response.
E. None of the above.

70. What is a quantal dose-response?

A Continuous responses in an individual against varying doses of a substance.


B The responses which can be quantified.
C The most probable responses in experimental animals after toxicant exposure.
D The responses which cannot be quantified.
E Individual responses distributed in a population to different doses of a substance.
ARIF PHARMA CLINIC Copyright@arifpharma.com Www. arifpharma.com
Page 245

71. Following is an example of quantal dose-response observation.

A Toxic effects in response to increasing doses of a chemical.


B Effect of a dietary supplement on body weight in a population.
C Observation of median lethal dose (LD50) in rats.
D Death of an individual after a snake bite.
E B and C

72. What is a graded dose-response?

A. Continuous responses in an individual against varying doses of a substance.


B. Responses to a single exposure at different time interval.
C. Effects other than the major one against a dose of a substance.
D. Individual responses distributed in a population to different doses of a substance.
E. A and C

73. What is the antidote for methanol poisoning?

A Atropine
B British anti lewisite
C Charcoal
D Ethanol
E Disodium EDTA

Answer: (D) – Ethanol is agonist for methanol and competes with it for metabolism by alcohol
dehydrogenase and aldehyde dehydrogenase, thus minimizes the formation of toxic metabolites
of methanol, formaldehyde and formic acid.

Alcohol Dehydrogenase Aldehyde Dehydrogenase


Methanol ---------------> Formaldehyde > Formic acid
(Causes blindness)

Alcohol Dehydrogenase Aldehyde Dehydrogenase


Ethanol ----------------> Acetaldehyde > Acetic acid

ARIF PHARMA CLINIC Copyright@arifpharma.com Www. arifpharma.com


Page 246

74. Which one of the following is not an air pollutant?


A. Ozone
B. Argon
C. Nitrogen oxides
D. Sulfur oxides
E. Hydrogen sulfide

75. What is the mode of action of carbon monoxide (CO)?

A It reduces the oxygen absorption by directly or indirectly damaging the alveoli.


B It destroys the red blood cells.
C Combines with hemoglobin to reduce oxygen carrying capacity of RBCs.
D It has a corrosive effect on airways, specially bronchi and bronchioles.
E It alters the structure of the hemoglobin.

Answer: (C) – Carbon monoxide has around 200 times greater affinity for hemoglobin than
oxygen.

76. Sulfur dioxide (SO2) affects the respiratory system by

A Forming sulfurous acid in respiratory tract, resulting in irritation.


B Reducing oxygen carrying capacity of blood.
C Causing allergy
D Carcinogenesis
E Respiratory collapse

Answer: (A) – SO2 reacts with aqueous layer on lung epithelium to form sulfurous acid.
SO2 + H2O → H2SO3

77. Which of the following participate in formation of photochemical smog?

A. Oxides of sulfur
B. Hydrogen
C. Ozone
D. Oxides of nitrogen
E. Carbon dioxide

ARIF PHARMA CLINIC Copyright@arifpharma.com Www. arifpharma.com


Page 247

78. What is the cause of acid rain?

A Over production of acids in factories.


B Increased environmental concentration of Oxides of nitrogen and sulfur due to air
pollution.
C Increased use of acid in household cleaning.
D Increased aviation activities.
E Carbon dioxide (CO2) and global warming.

Answer: B - Oxides of nitrogen and sulfur reacts with environmental moisture to form
nitric acid and sulfuric acid respectively.

79. Which of the following is not a possible route of exposure to a toxicant in the environment?

A Inhalation
B Ingestion
C Dermal absorption
D Transdermal
E None of the above

Answer: D – Transdermal route is used to deliver certain drugs in clinical set up.

80. What happens to DDT when it enters the body?

A It is stored in the bones.


B It is water soluble and easily excreted out from the body.
C It is fat soluble and stored in fat tissue.
D It is converted into an active metabolite.
E It bypasses the metabolism and excreted as such

81. Who is most likely to exhibit toxic effects of environmental tobacco smoke (ETS).

A. A person with asthma


B. An obese teenager
C. An athlete
D. A child in 2nd grade
E. A driver who smokes

ARIF PHARMA CLINIC Copyright@arifpharma.com Www. arifpharma.com


Page 248

Answer: A – Asthma can be triggered by tobacco smoke and pose severe damage. Children are also
prone to toxic insults caused by tobacco smoke. However, tobacco smoke is toxic for anyone
exposed.

82. Which of the following pneumocyte clears the particles deposited in the lungs?

A Type I pneumocyte
B Type II pneumocyte
C Clara cells
D Fibroblasts
E Alveolar macrophages

83. Emphysema is an inflammatory tissue damage, which can be caused by toxicants. Which organ
is affected in case of emphysema?

A. Kidneys
B. Heart
C. Lung
D. Skin
E. Nervous system

Answer: C – In emphysema lung alveoli are damaged, become enlarged and loose flexibility. It is
one of the several conditions collectively known as chronic obstructive pulmonary disease (COPD).

84. Which protein is mainly damaged in emphysema?

A Collagen
B Elastin
C Keratin
D Albumin
E Globuli

85. Cigarette smoking is the main cause of Chronic Obstructive Pulmonary Disease, which is a
combination of following ailments.

A Emphysema, chronic bronchitis, asthma


B Chronic bronchitis, bronchiolitis, edema
C Asthma, tracheitis, necrosis
D Emphysema, cough, sneezing
E Allergic cough, rhinitis, lower respiratory infections

ARIF PHARMA CLINIC Copyright@arifpharma.com Www. arifpharma.com


Page 249

86. Which of the following statements is true?

A. Light cigarettes are safer than the regular ones.


B. Electronic cigarettes are less harmful to the lungs and heart.
C. Filtered cigarettes are way safer than the non-filtered cigarettes.
D. Herbal cigarettes are natural hence harmless.
E. None of the above

87. What does PM10 indicate?

A Stage of a lung disease.


B Size of inhalant particles enough to reach and be deposited in the alveoli.
C Number of cigarettes per day to cause lung cancer in 10 years.
D Measure of pollutants in a unit volume of inhalant air.
E Degree of concentration of toxicants present in the air.

88. A plenty of free radicals and oxidants are released with cigarette smoke. What does these
oxidative species actually do to the lungs?

A They alter the secretion of surfactants which protect the lung epithelium.
B They block clearance of particulate matter by inhibiting cilia movement.
C They damage hemoglobin and reduce oxygen carrying capacity of the blood.
D They can damage cellular macromolecules like DNA, proteins and lipids and exert
severe cellular damages leading to several diseases.
E A and B

Answer: D – Oxidants exert their effect by damaging cellular macromolecules.

89. Four of the following are the sources of hazard and one is a hazard. Which one is a hazard?

A. Asbestos mining
B. Chemical factory
C. Cigarette
D. Metal fumes
E. Automobiles

ARIF PHARMA CLINIC Copyright@arifpharma.com Www. arifpharma.com


Page 250

90. Which of the following is not helpful in determining the dose of a toxic substance?

A Body weight
B Animal species (including human)
C Origin of toxicant
D Chemico-biological nature of the toxicant
E Body surface area

91. Following are the normal functions of the metals in the body except.

A Calcium in bone formation


B Phosphorus in ATP
C Iron in hemoglobin
D Lithium in sodium channels activities
E Magnesium in enzyme functions

Answer: D – Lithium is used as medicine in mental illnesses like depression, schizophrenia and
bipolar disorder.

92. What are the effects of cellular hypoxia in case of carbon monoxide poisoning?

A Alveolar collapse
B Lung edema
C Brain damage and cardiac dysrhythmia
D Disrupted glycolysis
E Disrupted hematopoiesis

93. Why activated charcoal is administered in case of oral poisoning?

A It deactivates toxicants
B It increases the metabolic processes of detoxification
C It chemically detoxifies the toxicants
D It chelates metal circulating in the blood
E It reduces absorption of toxicants by adsorbing them

ARIF PHARMA CLINIC Copyright@arifpharma.com Www. arifpharma.com


Page 251

94. Which of the following is associated with acetaminophen toxicity?

A Methanol
B N-acetyl-p-benzoquinone imine
C 3-(1-methylpyrrolidin-2-yl) pyridine
D Benzo(a)pyrene
E Diethyl nitrosamine

Answer: B - N-acetyl-p-benzoquinone imine is reactive metabolite of acetaminophen which is


responsible for liver damage and other associated toxic effects.

CYP 2E1 (Liver damage)


Acetaminophen N-acetyl-p-benzoquinone imine (NAPQI)

(Glutathione)
---------------> NAPQI-glutathione conjugates Excretion

95. Who is the most susceptible to hepatotoxicity by high doses of acetaminophen (paracetamol)?

A A child in second grade


B An old lady
C A person on fasting
D An alcoholic
E A factory worker

96. What kind of toxicity does ammonia cause?

A. Cardiotoxicity
B. Tissue corrosion
C. Neurotoxicity
D. Reduced gastric motility
E. Hepatotoxicity

97. Which one of the arsenic compounds causes hemolysis?

A Arsenic trioxide (As2O3)


B Arsenic pentoxide (As2O5)
C Arsenic pentafluoride (AsF5)
D Arsenic trisulfide (As2S3)
E Arsine (AsH3)

ARIF PHARMA CLINIC Copyright@arifpharma.com Www. arifpharma.com


Page 252

98. Asbestos is a group of naturally occurring silicates, which include .

A. Chrysotile and anthophyllite


B. Amosite and actinolite
C. Crocidolite
D. Tremolite
E. All of the above

99. Sodium azide (NaN3) which is used in car air bags and as preservative in laboratories has a
mechanism of toxicity similar to.

A Aconite
B Cyanide
C Nicotine
D Arsenic
E Lithium

100. Botulinum toxin causes death by .

A CNS depression and coma


B Cardiac failure
C Respiratory failure
D Severe hemolysis
E Renal failure

Answer: C – Botulinum causes paralysis of respiratory muscles.

101. Cadmium is a highly toxic metal that causes .

A Chemical pneumonitis
B Damage to renal tubules
C Cancer
D GI tract irritation
E All of the above

ARIF PHARMA CLINIC Copyright@arifpharma.com Www. arifpharma.com


Page 253

102. What is the main concern with the chlorinated hydrocarbon pesticide when compared with
other classes of pesticide?

A They are highly neurotoxic


B They can cause death
C They persist in the environment
D They damage the crops
E They can kill the cattle

103. Chlorine has toxic effects similar to .

A Arsine
B Phosgene
C Mustard gas
D Ammonia
E Carbon monoxide

Answer: D – Chlorine and ammonia, both have corrosive effects.

104. Which one of the following is not associated with sea food toxicity?

A Ciguatoxin
B Aflatoxin
C Tetrodotoxin
D Saxitoxin
E Okadaic acid

105. What are the most likely toxic effects of Iodine?

A Corrosive effects like that of chlorine


B Oxidative damage to cellular molecules
C Disturbed thyroid functions
D Adrenal gland disturbances
E Disturbed pancreatic gland functions

Answer:C – Aflatoxin is produced by a fugus , Aspergillus flavus, which usually contaminate peanuts.

ARIF PHARMA CLINIC Copyright@arifpharma.com Www. arifpharma.com


Page 254

106. Ipecac syrup is derived from Cephaline ipecacuanha plant and used for .

A Inducing emesis in case of oral poisoning


B Supporting cardiac function in case of pesticide toxicity
C To potentiate liver function
D lung inflammatory diseases
E GI tract cleaning

107. Which is the common target of toxicity of lithium, manganese and magnesium?

A Cardiac muscles
B Bones
C Skeletal muscles
D Central nervous system (CNS)
E Immune system

108. Common toxicity target of elemental mercury and methyl mercury is .

A Kidneys
B Liver
C Muscles
D Skin
E CNS

109. Common toxicity target of inorganic mercuric salts and organic mercury is .

A Kidneys
B Liver
C Muscles
D Skin
E CNS

110. Common toxicity target of elemental mercury, inorganic mercuric salts and organic
mercury is .

A Kidneys
B Liver
C Muscles
D Skin
E CNS
ARIF PHARMA CLINIC Copyright@arifpharma.com Www. arifpharma.com
Page 255

.
111. Major rout of exposure to elemental mercury is .

A Dermal
B Inhalation
C Ingestion
D Intravenous
E Intramuscular

112. ‘Metal fume fever’ is caused by inhalation of fumes of .

A Zinc oxide
B Elemental mercury
C Chromium oxide
D Ferric chloride
E Any of the metal oxides

113. Nitrate. What sort of toxicity is expected in that person?

A Reduced oxygen carrying capacity/hypoxia


B Acute hypotension
C Increased hypoxia and a little hypotension
D Increased hypotension and a little hypoxia
E Respiratory collapse

Answer: B – Nitrates cause vasodilation and reduces blood pressure.

114. Toxicity targets of nonsteroidal anti-inflammatory drugs (NSAIDs) include.

A CNS and muscles


B Musculoskeletal system and liver
C GI tract and kidneys
D Cardiovascular system
E Respiratory system

ARIF PHARMA CLINIC Copyright@arifpharma.com Www. arifpharma.com


Page 256

115. Morphine is obtained from the following plant.

A Nicotiana tabacum
B Erythroxylum coca
C Atropa belladonna
D Papaver somniferum
E Datura alba

116. Opioids like heroin and morphine can cause death by .

A CNS depression
B Cardiac failure
C Respiratory depression and failure
D Renal failure
E Hemolytic anemia

117. Target organ of the herbicide paraquat is .

A Liver
B Kidney
C Endocrine glands
D Lung
E Reproductive organs

118. Which of the following is not recommended in case of paraquat poisoning?

A Charcoal administration
B Oxygen administration
C Washing of exposed skin
D Maintenance of open airway
E Fluid administration

ARIF PHARMA CLINIC Copyright@arifpharma.com Www. arifpharma.com


Page 257

119. What are the effects, other than the systemic effects of the smoke?

A Thermal damage to the airways


B Irritation
C Asphyxia
D A, B and C
E B and C

120. Which of the following statement is associated with first order kinetic?

A Excretion can be increased by increasing pH


B Amount of excretion does not depend on the
concentration of the toxicant
C Amount of excretion can be increased by diuresis
D Amount of excretion depends on the concentration of the toxicant
E Excretion is not associated with the metabolism

Answer: D – In first order kinetics excretion is directly proportional to the concentration of the
substance.

ARIF PHARMA CLINIC Copyright@arifpharma.com Www. arifpharma.com


Page 258

1. Which of the following forms is used for ordering Schedule II controlled substances?

a. DEA form 41
b. DEA form 106
c. DEA form 222
d. DEA form 223

Rationale DEA form 41 is submitted to the DEA for the destruction of outdated or damaged
controlledsubstances. DEA form 106 is for theft and DEA form 223 is a duplicate certificate
request.

2. Which of the following are used to provide information about each chemical,
including flammability, Potential health hazards, and proper disposal?
a. OSHA
b. MSDS
c. FHSA
d. PPPA
Rationale OSHA is the Occupational Safety and Health Administration which helps
prevent workplacedisease and injury, FHSA is the Federal Hazardous Substance Act which
involves the use of disposal ofhazardous material, and PPPA is the Poison Prevention
Packaging Act which requires child-resistant packaging for most prescription drugs.

3. Of the following DEA numbers, which one is correct if the physician’s name is Dr. Tim Longley?

a. AL4317651
b. AT4317651
c. AL4317653
d. AT4317653

Rationale The DEA number for physicians consists of two letters, the first letter of
which is either an A, B, F, or M. The second letter is the first letter of the physician’s last name.
These letters are followed by seven numbers, thesequence of which can easily be
solved: 4 + 1 + 6 = 11; 3 + 7 + 5 = 15; (15)(2) = 30; 11 + 30 = 31

4. The FDA utilizes a system called to receive information about adverse


events or product problems.
a. Phase I Marketing
b. Phase II Marketing

Arif Pharma Clinic Copyright@arifpharma.com arifpharma.com


Page 259

c. Phase III Marketing


d. MedWatc

Rationale MedWatch is a voluntary program that is designed for healthcare professionals to report
any serious event that may be associated with the use of a specific drug or dietary supplement. The
FDA can thenutilize the information reported to track any problems with a medication that may
have not been apparent when it was first approved

5. Which of the following forms is used for ordering Schedule II controlled substances?

a. DEA form 41DEA form 106

b. DEA form 222

c. DEA form 223


Rationale DEA form 41 is submitted to the DEA for the destruction of outdated or damaged
controlledsubstances. DEA form 106 is for theft and DEA form 223 is a duplicate certificate request.

6. Which of the following are used to provide information about each chemical,
including flammability, potential health hazards, and proper disposal?
a. OSHA
b. MSDS
c. FHSA
d. PPPA
Rationale OSHA is the Occupational Safety and Health Administration which helps prevent
workplacedisease and injury, FHSA is the Federal Hazardous Substance Act which involves the
use of disposal ofhazardous material, and PPPA is the Poison Prevention Packaging Act which
requires child-resistant packaging for most prescription drugs.

7. Of the following DEA numbers, which one is correct if the physician’s name is Dr. Tim Longley?

a. AL4317651
b. AT4317651
c. AL4317653
d. AT4317653

Rationale The DEA number for physicians consists of two letters, the first letter of
which is either an A, B, F, or M. The second letter is the first letter of the physician’s last name.
These letters are followed by seven numbers, the sequence of which can easily be
solved: 4 + 1 + 6 = 11; 3 + 7 + 5 = 15; (15)(2) = 30; 11 + 30 = 31

Arif Pharma Clinic Copyright@arifpharma.com arifpharma.com


Page 260

8. The FDA utilizes a system called to receive information about adverse


events orproduct problems
a. Phase I Marketing
b. Phase II Marketing
c. Phase III Marketing
d. MedWatch
Rationale MedWatch is a voluntary program that is designed for healthcare professionals to
report any serious event that may be associated with the use of a specific drug or dietary
supplement. The FDA can thenutilize the information reported to track any problems with a
medication that may have not been apparent when it was first approved

9. Which of the following is a CII-controlled substance?


a. Valium
b. Oxybutynin
c. Percocet
d. Xanax
Rationale Percocet. Vicodins a schedule III drug; Oxybutnin is a noncontrolled drug; Xanax is a
schedule IV drug.

10. OSHA requires pharmacy workplace safety through which of the following measures?

a. All sharps disposed of in sharps containers


b. All hazardous materials disposed of in labeled biohazard bins
c. Eyewash stations are available for flushing hazardous material from the eye
d. All of the above

Rationale All of these are measures that OSHA requires for pharmacy workplace safety.

11. Of the following regulatory agencies, which one deals with the regulation of healthcare
organizations,such as a pharmacy in a hospital setting?
a. FDA
b. TJC
c. DEA
d. HIPPA
Rationale FDA deals with drug regulations, DEA deals with controlled drugs,
and HIPPA deals with patientconfidentiality. TJC regulates pharmacies in the hospital setting.

Arif Pharma Clinic Copyright@arifpharma.com arifpharma.com


Page 261

12. The portion of Medicare which covers prescription drugs is

a. Part A.
b. Part B.
c. Part C.
d. Part D.

Rationale Medicare Part A is hospital insurance, Medicare part B is outpatient or medical insurance,
and Medicarepart C allows private health insurance plans such as HMOs or PPOs to offer benefits
(Medicare Advantage Plans).

13. Of the following schedules of drugs, which one is considered to have no medicinal value?

a. Schedule I
b. Schedule II
c. Schedule III Schedule V

Rationale Schedule I drugs are considered to have no medicinal value and are not dispensed in the
pharmacy setting.
14. The requires the proper labeling or branding of products and also requires that the
The contents of a container cannot be changed from what they are labeled to be.
a. PPPA
b. FD&C Act
c. Hazardous Substance Labeling Act
d. Orphan Drug Act

Rationale PPPA deals with safety caps for prescription drugs. The Hazardous Substance Labeling
Act deals withthe use of a hazardous container for hazardous materials. Orphan Drug Act deals
with drugs that are used for rare disease states. The FD&C Act is the correct answer.

15. Which of the following is a main consideration of the pharmacy technician code of ethics?
a. Preventing the patient from dying
b. Prevention of communicable diseases
c. Health and safety of the patient
d. Patients’ ability to afford medications

Rationale The goal of the code is to ensure patient health and safety. Although the other items are
importantconsiderations, they are not directly covered in the code.

Arif Pharma Clinic Copyright@arifpharma.com arifpharma.com


Page 262

16. The PPPA requires

a. The use of safety caps for most prescription orders in a retail pharmacy.
b. patient counseling for all new prescription orders received in a retail pharmacy.
c. that all drug manufacturer containers be labeled correctly as to what is inside the container.
d. that drugs within a drug manufacturer’s container not be adulterated.

Rationale PPPA stands for the Poison Prevention Packaging Act, so the most appropriate answer is
the use ofsafety caps.

17. Lack of attention to detail or any distractions can contribute to which of the following situations?

a. Professional ethics violations


b. Conflicts of interest
c. Cocial problems
d. Errors and mistakes
Rationale Lack of attention will cause mistakes or errors.

18. Enacted in 2005, the Combat Methamphetamine Epidemic Act was intended to stop the use
of the illegal drug methamphetamine by limiting the sales of what drug?
a. Phenylpropanolamine
b. Pseudoephedrine
c. Caffeine
d. Ranitidine

Rationale Pseudoephedrine is the generic name for Sudafed®. Phenylpropanolamine was removed
from themarket due to addiction concerns. Caffeine is also of concern but will never be removed
from the market. Ranitidine is the generic name for Zantac®, which is used for GERD

19. At the pharmacy level, they must record the acknowledgment of the facility’s standards for
Each new patient.

a. OBRA
b. SARS
c. HIPPA
d. PPPA

Arif Pharma Clinic Copyright@arifpharma.com arifpharma.com


Page 263

Rationale HIPPA deals with patient confidentiality. All new customers in a pharmacy setting are
given aprotected health information (PHI) form that details the pharmacy’s stance on this issue.

20. According to CSA, schedule II drugs are not allowed any refills. What is the maximum number
of refills a CIII through CV can have, and how long is the prescription valid after the date of
issue?
a. 5 refills and six months
b. 11 refills and one year
c. 6 refills and one year
d. 11 refills and six months
Rationale (A) Schedule III to Schedule V drug is allowed to have 5 refills, with the prescription
being valid forsix months from the date of issue. Noncontrolled legend (prescription) drugs are
allowed a maximum of 11 refills, with the prescription being valid for one year from the date of
issue. Schedule II drugs cannot be refilled.

21. Which chapter in the USP sets standards for sterile compounding?

a. USP <795>
b. USP <797>
c. USP <800>
d. USP <900>

Rationale USP is the chapter for nonsterile compounding, and USP is for hazardous drugs.

22. Any questions or situations involving controlled drugs should be directed to which regulatory
agency?

a. FDA
b. BOP
c. DEA
d. HIPPA
Rationale FDA regulates the marketing of drugs, BOP regulates pharmacies within their state, and
HIPPA dealswith patient confidentiality. The DEA regulates controlled substances.

23. Which state regulatory agency is responsible for the administration of licensure
and/orcertification of pharmacy personnel?

a. FDA
b. BOP
c. DEA
d. HIPPA

Arif Pharma Clinic Copyright@arifpharma.com arifpharma.com


Page 264

Rationale BOP regulates the administration of licensure and/or certification of pharmacy


personnel. The FDA deals with getting drugs to the market that are safe and effective. The DEA

regulates scheduled drugs that arepart of the Controlled Substance Act. HIPPA is a federal law that
mandates patient confidentiality.
24. Of the following, which one accredits and certifies healthcare organizations in the United States?
a. OSHA
b. TJC
c. NABP
d. BOP
Rationale The Joint Commission accredits and certifies healthcare organizations in the United
States. OSHAdeals with safety requirements in all work settings. NABP is the National Association

Board of Pharmacies which regulates individual state boards of pharmacies. BOP is an individual
state board of pharmacy that regulates pharmacies within their state and administration of
licensure and/or certification of pharmacy personnel.

25. Comprehensive Drug Abuse Prevention and Control Act is also known as the
a. Poison Prevention Packaging Act.
b. Federal Drug and Cosmetic Act.
c. Controlled Substance Act.
d. Omnibus Budget Reconciliation Act.

Rationale The Controlled Substance Act. The PPPA deals with the mandated issuance of safety caps
in thepharmacy setting. The FD&C Act deals with labeling and adulteration of a drug, and OBRA
deals with the need to do DURs and the counseling of Medicaid patients.

26. Which of the following is a CIV-controlled substance?


a. Vicodin
b. Oxybutynin
c. Percocet
d. Ambien

Rationale Ambien Sleep aids are generally schedule IV drugs, as are anti-anxiety aids that belong
to thedrug classification called benzodiazepines

27. The Orphan Drug Act deals with drugs that are
a. Outdated.
b. Used to treat rare diseases.

Arif Pharma Clinic Copyright@arifpharma.com arifpharma.com


Page 265

c. Used to treat syndromes.


d. Used to treat type II diabetes mellitus.

Rationale The Orphan Drug Act offers financial incentives to develop medications for rare diseases.

28. Which chapter in the USP sets standards for the handling of hazardous drugs?

a. USP <795>
b. USP <797>
c. USP <800>
d. USP <900>

Rationale USP is the chapter for nonsterile compounding, and USP is the chapter for
sterile compounding.

29. In the case of Penicillins, there is a 1% chance that a patient who is allergic to Penicillins is
also allergic to cephalosporins. What is this called?

a. Adverse reaction
b. side effect
c. Cross-sensitivity
d. Hypersensitivity reaction

Rationale This is what we would call a cross-sensitivity.

30. Which of the following medications is antihyperlipidemic?


a. Atorvastatin
b. Sertraline
c. Digoxin
d. Plavix

Rationale Atorvastatin is antihyperlipidemic. Sertraline is for depression, digoxin is for arrhythmias,


and Plavix is for stroke prevention.

31. Which of the following medications is antihyperlipidemic?


a. Atorvastatin
b. Sertraline
c. Digoxin
d. Plavix
Rationale Atorvastatin is antihyperlipidemic. Sertraline is for depression, digoxin is for arrhythmias, and
Plavix® is for stroke prevention.

Arif Pharma Clinic Copyright@arifpharma.com arifpharma.com


Page 266

32. Which of the following medications is used to treat a bacterial infection?

a. Fluconazole
b. Amlodipine
c. Amoxicillin
d. Lorazepam

Rationale Amoxicillin treats bacterial infections. Fluconazole is for fungal infections, amlodipine is for
bloodpressure, and lorazepam is for anxiety

33. Which of the following medications is the generic version of Zestril®?

a. Enalapril
b. Alprazolam
c. Metoprolol
d. Lisinopril
Rationale Lisinopril is the generic name for Zestril®. Alprazolam is generic Xanax®, metoprolol is generic
Lopressor, and enalapril is generic Vasotec.

34. Which of the following is an HMG-CoA reductase inhibitor?

a. Esomeprazole
b. Glipizid
c. Simvastatin
d. Valsartan
Rationale An HMG-CoA Reductase Inhibitor is also known as a “statin.” Generally, the generic will
end with- statin. Esomeprazole is a PPI, glipizide is a sulfonylurea, and valsartan is an ARB.

35. Which of the following medications is the brand name for atenolol?

a. Toprol-XL
b. Lipitor
c. Tenormin
d. Zocor
Rationale Atenolol is also known as Tenormin. Toprol-XL is known as metoprolol succinate, Lipitor
is also known as atorvastatin, and Zocor is known as simvastatin.

36. Which of the following medications is a macrolide antibiotic?


Arif Pharma Clinic Copyright@arifpharma.com arifpharma.com
Page 267

a. Levaquin
b. Zithromax
c. cephalexin
d. Augmentin
Rationale Zithromax is a macrolide. Levaquin is a quinolone, cephalexin is a cephalosporin,
andAugmentin is a Penicillin.

37. Which of the following medications is the generic for Lasix?


a. HCTZ
b. Dyazide
c. furosemide
d. Nexium

Ratioanle Lasix is furosemide. HCTZ is hydrochlorothiazide, triamterene/HCTZ is Dyazide, and


esomperazole is Nexium

38. What is the MOA of ACE inhibitors?

a. They work by connecting A to C and then to E.


b. They work by inhibiting bacterial cell wall synthesis.
c. They work by preventing the attachment of A2 to receptor sites.
d. They work by inhibiting the conversion of angiotensin 1 into angiotensin2.

Rationale ACE inhibitors, or angiotensin-converting enzyme inhibitors, work by preventing


the conversion ofangiotensin 1 to angiotensin 2, which causes vasoconstriction and high blood pressure.

39. Which of the following medications is a beta-blocker?

a. Lexapro
b. zolpidem
c. Zocor
d. metoprolol succinate

Rationale Metoprolol succinate is the beta-blocker. Lexapro is an SSRI, Zocor is antihyperlipidemic,


and Zolpidem is hypnotic.

Arif Pharma Clinic Copyright@arifpharma.com arifpharma.com


Page 268

40. Which of the following medications is the generic name for Proventil®?

a. Salmeterol
b. Prednisone
c. Albuterol
d. Synthroid

Rationale Albuterol is Proventil. Salmeterol is Serevent, prednisone is Deltasone, and Synthroid is


levothyroxine.

41. Which of the following medications is the brand name for amlodipine?
a. Norvasc
b. Toprol-XL
c. Singulair
d. Flonase
Rationale Norvasc is amlodipine. Singulair is montelukast, Toprol-XL is metoprolol,
and Flonase isfluticasone.

42. Which of the following medications is used in patients with thyroid hormone deficiency?

a. Singulair
b. Synthroid
c. Plavix
d. Premarin
Rationale Synthroid. Singulair is for allergies, Plavix is for stroke prevention, and Premarin is for
menopause.

43. Which of the following medications are used for patients with hyperglycemia?

a. Xanax
b. Metformin
c. Predinsone
d. Celebrex
Rationale Metformin is used to treat hyperglycemia. Xanax is used for anxiety, prednisone is for
allergiesand inflammation, and Celebrex® is for inflammation.

44. Which of the following medications is an SSRI?


a. Enalapril
b. Allegra
c. Prevacid

Arif Pharma Clinic Copyright@arifpharma.com arifpharma.com


Page 269

d. Zoloft
Rationale Zoloft is an SSRI. Enalapril is an ACE inhibitor, Prevacid is a proton pump inhibitor, and
Allegra is an antihistamine.

45. What is the brand name of escitalopram?

a. Zoloft
b. Synthroid
c. Lexapro
d. Restoril

Rationale Lexapro is escitalopram. Zoloft is sertraline, Synthroid is levothyroxine, and Restoril is


temazepam

46. Which of the following medications is the trade name for hydrocodone/acetaminophen?
Tylenol No 3. Percocet.
a. Vicodin
b. Ibuprofen
Rationale Vicodin is hydrocodone with acetaminophen, Tylenol® No. 3 is acetaminophen
with 30 mgcodeine, Percocet® is oxycodone with acetaminophen, and Motrin® is ibuprofen.

47. Which of the following medications is a cephalosporin antibiotic?

a. Cephalexin
b. Augmentin
c. Amoxicillin
d. Zithromax
Rationale Cephalexin is a cephalosporin. Amoxicillin and Augmentin are Penicillins, and Zithromax
is amacrolide

48. Which of the following medications are used for insomnia?

a. Xanax
b. Metformin
c. Zolpidem
d. Atenolol
Rationale Zolpidem is used for insomnia. Xanax is for anxiety, metformin is for diabetes,
and atenolol is for hypertension.

Arif Pharma Clinic Copyright@arifpharma.com arifpharma.com


Page 270

49. Which of the following medications is a corticosteroid?

a. lorazepam
b. Lipitor
c. Norvasc
d. Prednisone
Ratioanle Prednisone is a corticosteroid. Lorazepam is a benzodiazepine, Norvasc is a beta-
blocker, andLipitor is an antihyperlipidemic.

50. Which of the following medications is the generic for Nexium?

a. Metformin
b. Esomeprazole
c. Ranitidine
d. Naproxen

Rationale Esomeprazole is the generic for Nexium. Ranitidine is the generic for Zantac, metformin is
thegeneric for Glucophage, and naproxen is the generic for Naprosyn

51. Which of the following medications is available OTC?

a. Ibuprofen
b. Lexapro
c. Dyazide
d. Flonase
Rationale Ibuprofen is available over-the-counter with different trade names such as Motrin.

52. Which of the following is an example of a chemotherapy drug?

a. Doxyrubicin
b. Atenolol
c. Lisinipril
d. Azithromycin
Rationale Doxyrubicin, or Adriamycin, would be an example of a chemotherapy drug used to treat
cancer.Other examples of chemo drugs would be 5-fluorouracil (5FU) and Cisplatin.

Arif Pharma Clinic Copyright@arifpharma.com arifpharma.com


Page 271

53. Which of the following medications is the brand name version of simvastatin?

a. Singulair
b. cephalexin
c. Zocor
d. Levaquin
Rationale Zocor is the brand name for simvastatin. Singulair is the brand name for montelukast,
Keflex® isthe brand name for cephalexin, and Levaquin® is the brand name for levofloxacin.

54. Which of the following medications is a beta-blocker?

a. Tramadol
b. Carvedilol
c. Gabapentin
d. Trazodone
Rationale Carvedilol is a beta-blocker. Tramadol is an opioid analgesic, gabapentin is an anticonvulsant,
And trazodone is an antidepressant.

55. Which of the following medications is used for depression?

a. Prozac
b. Flonase
c. Altace
d. Naproxen
Rationale: Prozac is an antidepressant. Altace is an ACE inhibitor, Flonase is a steroid, and naproxen
is an NSAID.

56. Which of the following medications is the brand name of lorazepam?

a. Allegra
b. Tramadol
c. Ativan
d. Levoxyl
Rationale Ativan is the brand name for lorazepam. Allegra is the brand name for fexofenadine,
Ultram® isthe brand name for tramadol, and Levoxyl is the brand name for levothyroxine.

57. Which of the following medications is the generic of Plavix?


a- Clopidogrel
b- Zolpidem

Arif Pharma Clinic Copyright@arifpharma.com arifpharma.com


Page 272

c- Metformin
d- Clonazepam
Rationale Clopidogrel is the generic version of Plavix. Metformin is the generic name for
Glucophage, zolpidem is the generic name for Ambien, and clonazepam is the generic name for
Klonopin.
58. Which of the following medications is a sulfonylurea?
a- Nasonex
b- Percocet
c- Glipizide
d- Cozaar

Rationale Glipizide is a sulfonylurea, Percocet® is an opiate analgesic, Nasonex® is a steroid, and


Cozaar® isan ARB

59. Which of the following medications is used to treat infection?


a. Lotrel
b. Advair
c. Augmentin
d. Enalapril

Rationale. Augmentin is used to treat infection. Lotrel® is for high blood pressure, Advair is for
asthma,and enalapril is for blood pressure.

60. Which of the following medications is the brand name for fluticasone/salmeterol?

a. Seroquel
b. Flonase
c. Levaquin
d. Advair
Rationale. Advair is the brand name for fluticasone/salmeterol. Seroquel is the brand name of
quietapine,Levaquin is the brand name of levofloxacin, and Flonase is the brand name of
fluticasone.

61. Which of the following medications is the generic for Fosamax®?

a. levothyroxine
b. alendronate
c. amitriptyline

Arif Pharma Clinic Copyright@arifpharma.com arifpharma.com


Page 273

d. Levaquin
Rationale Alendronate is the generic name for Fosamax®. Levothyroxine is the generic name for
Levoxyl, Amitriptyline is the generic name for Elavil®, and Levaquin® is the brand name of
levofloxacin.

62. Which of the following medications is not an SSRI?

a. Zoloft
b. Prozac
c. Effexor XR
d. Celexa
Rationale. Effexor XR is an SNRI; the other drugs are SSRIs.

63. Which of the following medications is used to prevent blood clots?


a. Coumadin
b. Paxil
c. Trazodone
d. Lotrel
Rationale. Coumadin prevents blood clots. Trazodone and Paxil® treat depression. Lotrel treats
hypertension.

64. Which of the following medications is the brand name for paroxetine?

a. Zyrtec
b. clonazepam
c. Paxil
d. Protonix
Rationale Paroxetine is the generic name for Paxil.

65. Which of the following medications is the generic name for Klonopin?

a- Premarin
b- Diovan
c- Levoxyl
d- Clonazepam
Rationale Klonopin is the brand for clonazepam. Premarin® is the brand name for conjugated
estrogens,Levoxyl® is the brand name for levothyroxine, and Diovan® is the brand name for
valsartan.

Arif Pharma Clinic Copyright@arifpharma.com arifpharma.com


Page 274

66. Which of the following medications is an H1 antagonist?


a- Naproxen
b- Zyrtec
c- Flonase
d- Tramadol

Rationale Zyrtec is an H1 antagonist, or blocker. Naproxen is an NSAID, Flonase is a steroid, and


tramadol is an analgesic opioid.

67. Which of the following medications is used to treat GERD?

a- Protonix
b- Trazodone
c- Lotrel
d- Enalapril
Rationale Protonix is used to treat GERD. Lotrel and enalapril treat hypertension, and trazodone
treatsdepression.

68. Which of the following is defined as the study of the physiological effects
that medication has on the body?

a. Pharmacodynamics
b. Metabolism
c. Absorption
d. Pharmacokinetics
Rationale The study of the physiological effects a medication has on the body is called
pharmacodynamics.Metabolism is how a medication is broken down, absorption is how a medication
makes it into the body, andpharmacokinetics is the path the medication takes in the body.

69. Which of the following is also referred to as the medication’s use?

a. Action
b. Classification
c. Indication
d. Interaction
Rationale Indication refers to a medication’s use. Action is what the medication does,
classification is thegrouping of medications, and interaction is when a medication clashes with
another.

Arif Pharma Clinic Copyright@arifpharma.com arifpharma.com


Page 275

70. What is a group of medications with similar characteristics called?

a. Generics
b. Drug classification
c. Indication
d. Action
Rationale A group of medications with similar characteristics is called a drug classification.
Generics are cost-effective medications, the indication is the use, and action is what the medication
does.

71. What is known as an undesired effect resulting from the use of a medication?

a. Interaction
b. Indication
c. Adverse reaction
d. Action
Rationale An undesired effect from a medication is known as an adverse reaction. Indication is use,
and action is how a medication works.

72. Two drugs are considered if they have the same clinical effect and safety profile, and the
Orange Book gives them an A rating.

a. Pharmacodynamics
b. Alternative medicine
c. SSRIs
d. Therapeutically equivalent
Rationale The two drugs are therapeutically equivalent.

73. A patient is on tranylcypromine, an MAO inhibitor. Patient counseling might include:


a. a discussion of the drug’s action
b. a warning to stay away from certain foods
c. a warning to see a physician if certain symptoms, such as rapid heart rate and restlessness, are exhibited
d. All of the above

Rationale Patient counseling involves a discussion of the drug action, adverse effects, food and drug
contraindications, and dosing. All of the answers are appropriate.

74. A medical student may prescribe drugs:


a. Only in the scope of his or her studies with in the hospital
b. Using his or her DEA number
c. Only in the hospital, using the hospital’s DEA number

Arif Pharma Clinic Copyright@arifpharma.com arifpharma.com


Page 276

d. both a and c are correct

Rationale Medical students may prescribe drugs only under the DEA number of the hospital and
undersupervision.

75. In October 2010, a customer brings in a prescription for Kefl ex. The prescription is
dated 2/10/2010.Legally, The prescription:

a. will only be valid until 2/10/2011


b. will only be good for two refi lls, instead of the three originally prescribed
c. must be filled immediately

d. cannot be filled, as it is over six months old

Rationale Keflex is not a controlled substance. The prescription is therefore good for a year.
It does, however,have to be presented for filling within six months depending on the state.

76. A prescription for Demerol:

a. Must be received on a triplicate form, according to federal law


b. May be called or faxed in
c. may be obtained by telephone in an emergency
d. Both a and c are correct

Rationale Demerol (meperidine) is a C-II. It must therefore be submitted on a triplicate form. It can,
however,be obtained by a telephone order in an emergency, provided the telephone order is
followed by a hard copy within three days.

77. You receive a prescription for meperidine (Classified as a C-II). It contains a corrected date.
Youshould:

a. Alert the pharmacist


b. Ask the patient why the date was corrected
c. Ignore the change in date and fill the prescription
d. Refuse to fill the prescription and give it back to the patient
Rationale Prescriptions for C-II drugs are not allowed to have extra writing or corrections on the
form. Do notreturn the form, as the person will simply attempt to have the prescription filled at
another pharmacy. Alert thepharmacist.

Arif Pharma Clinic Copyright@arifpharma.com arifpharma.com


Page 277

78. Which of the following is true regarding a prescription for Valium?


a. It requires a triplicate form
b. It is only valid for six months
c. No more than five refills are allowed
d. both b and c

Rationale Valium is a C-IV (or C-III, depending on the state). It therefore does not have to be on the
triplicateform. Prescriptions for controlled substances other than C-II are valid for six months, and
five refills are allowed.

79. A patient brings in a prescription, dated 2/25/2010, to Hill Road Pharmacy on that same day.
Theprescription is for Ceclor (a very pricey drug) and has five refills authorized. He receives a
refill on 4/15/2010.On 5/12/2010, the patient discovers that Dale pharmacy is cheaper and
has the prescriptiontransferred there. Dale pharmacy:

a. Will only fill two of the remaining four refills


b. Will be able to fill refills on the prescription until 2/25/2011
c. Will have to get a refill authorization in order to fill the remaining refills
d. Cannot fi ll this prescription
Rationale The prescription is still valid for one year, so no changes would be made in the fi lling or
expirationdate

80. Which of the following medications is in schedule II?

a. Diazepam
b. Buprenorphine
c. Midazolam
d. Methadone
e. Zolpidem

81. The Pure Food and Drug Act of 1906 required which of the following?

a. That drugs not be mislabeled


b. Industry purity and strength standards
c. Therapeutic benefit documentation
d. Both a and b

Rationale The Pure Food and Drug Act of 1906 required drugs not be mislabeled and set standardsfor
strength and purity.

Arif Pharma Clinic Copyright@arifpharma.com arifpharma.com


Page 278

82. Which of the following medications is in schedule II?

a. Diazepam

b. Buprenorphine

c. Midazolam

d. Methadone

e. Zolpidem

83. The Pure Food and Drug Act of 1906 required which of the following?

a. That drugs not be mislabeled


b. Industry purity and strength standards
c. Therapeutic benefit documentation
d. Both a and b
Rationale The Pure Food and Drug Act of 1906 required drugs not be mislabeled and set standards
for strength and purity.

84. Drugs are intended for use in which of the following?

a. Prevention of disease
b. Treatment of disease
c. Diagnosis of disease
d. All of the above
Ratioale Drugs are intended for the use of prevention, treatment, and diagnosis of disease.

85. Which of the following medications is with a low abuse potential and acceptable medical uses?

a. Schedule II
b. Schedule III
c. Schedule IV
d. OTC
Rationale Medications with a low abuse potential and acceptable medication use are in scheduleIV.

86. The Durham-Humphrey Amendment created which of the following?

a. Scheduled medications
b. OTC medication class and Rx medication class
c. Safety regulations
d. Purity laws
Rationale The Durham-Humphrey Amendment created 2 medication classes: OTC and prescription.

Arif Pharma Clinic Copyright@arifpharma.com arifpharma.com


Page 279

87. The Drug Listing Act required which of the following?

a. Tax incentives for orphan drugs


b. Labeling of dietary supplements
c. Is on all prescription medications
c. All of the above

Rationale The Drug Listing Act required each drug to be assigned a NDC number.

88. Pharmacies must make a reasonable effort to obtain which of the following?

a. Patient’s name
b. Patient’s physical address
c. NDC numbers for medications
d. Patient counseling
Rationale To accurately dispense a prescription, the pharmacy should obtain the patient’s name,
address, and age.

89. The Poison Prevention Act created which pharmacy dispensing standard?

a. Patient counseling
b. Child-resistant container use
c. Counting tray use
d. All of the above
Rationale The Poison Prevention Act created the standard requirement of child resistant containers.

90. Which of the following medications is in schedule III?

a. Anabolic steroids
b. Lorazepam
c. Temazepam
d. Morphine

Rationale Anabolic steroids are in DEA schedule III.

Arif Pharma Clinic Copyright@arifpharma.com arifpharma.com


Page 280

Arif Pharma Clinic Copyright@arifpharma.com arifpharma.com


Page 281

Pregnancy & Lactation

Updated DHA exam study Material

Www.arifpharma.com

Drx Arif Khan


ARIF PHARMA CLINIC COPYRIGHT@ARIFPHARMA.COM
Page 282

Drugs used in pregnancy


Hypertension ◆
:
v
Methyldopa
Labetalol
Hydralazine
HTN + Hypoproteinemia + Seizures MgSO4
Asthma Cromolyn (Category B)
Gestational diabetes Insulin
Metformin
Headache Paracetamol
DVT Heparin not Warfarin (Category X)
Tonic-clonic seizures Valproic acid + Folic acid

Epilepsy v
: Lamotrigine or levetiracetam
Depression (Mild) Fluoxetine
Hypothyroidism Levothyroxine
Hyperthyroidism Propylthiouracil (PTU)
Methimazole
Nausea and vomiting Cyclizine

UTI:
v Nitrofurantoin

UTI + G6PD deficiency :
v Cefuroxime
Otitis media Azithromycin
Chlamydia
Syphilis Benzathine penicillin - Penicillin (Ampicillin)
Vaginal discharge + Candida albicans Clotrimazole
Herpes (Genital infection) Acyclovir 400 bid for 7 days
Hepatitis B Tenofovir
HIV Abacavir and Lamivudine
Malaria Chloroquine, Hydroxychloroquine
Trichomoniasis Metronidazole

 Contraindications
• Misoprostol category X causes abortion
• Vitamin A is contraindicated in high doses
• Castor oil is contraindicated as a laxative
• ACEi causes renal dysfunction in the fetus
• Naproxen causes uncontrolled bleeding in the baby
• Finasteride cause genital malformation in an infant
• Warfarin (category X) causes cranial facial abnormalities → Nasal bone hypoplasia
❖ Notes
• Inactivated influenza vaccine can be given during pregnancy
• Glargine (long-acting) is category B
• Paracetamol oral and rectal is category B – when taken IV is category C
• Sensitive to ampicillin give Erythromycin
❖ Doses
Dose of Thyroid Hormone in Pregnancy? 1.0 - 2.0 microgram/kg/day

ARIF PHARMA CLINIC COPYRIGHT@ARIFPHARMA.COM


Page 283

Pregnancy & Lactation

1. The dose of folic acid in nonpregnant women is

A. 400 mcg/day

B. 700 mcg/day

C. . 1200 mcg/day

D. 900mcg/day

Note:

Nonpregnant 400 mcg/day


Pregnant 600 mcg/day
Lactating 500 mcg/day

2. Which of the following vaccines should be administered for everypregnancy?

A. Tdap (Tetaunes. Diaphtheri. Pertussis)

B. Dap

C. Dtap

D. Mmr

Answer: A - Pregnant women should receive a dose of Tdap during each pregnancy,ideally
between 27 & 36 weeks gestation.

Tdap & Dtap are the same vaccines but Tdap is used for adults andpregnant, and Dtap is
used for (0-6) years old children

3. What is the antidote for methotrexate toxicity:

A. Leucovorin

B. Naproxen

C. Vit k

D. Naloxone

ARIF PHARMA CLINIC COPYRIGHT@ARIFPHARMA.COM


Page 284

5. In the ovulation phase hormones which Predominant.

A- FSH

B- LH

C- PROGESTERONE

D- Progesterone and Lh

6. Human chorionic gonadotropin is used to:

A- Induce ovulation and treatment of infertility

B- Induce ovulation

C- Infertilityd- Hormon

7. Daily dose in case of iron deficiency anemia

A. 100mg three times daily

B. 1200

C. 200

D. 300

8. Dose in iron deficiency anemia is:-

A- 100mg tid
B- 212,5 mg tid
C- 275 mg tid
D- 325 mg bid

If in the question of elemental iron: the recommended oral daily dose forthe treatment of iron
deficiency in adults is in the range of 150 to 200mg/day of elemental iron"50-65 mg 3 times per
day”. If in the question (ferrous sulfate): it is in the range of (1000 – 1200mg/day) of ferrous sulfate
in 3 or 4 divided doses, 1000mg divided into 3 doses and 1200mg divided into 4 doses

ARIF PHARMA CLINIC COPYRIGHT@ARIFPHARMA.COM


Page 285

9. Combination contraceptive act on:

A- Ovulation
B- Implantation
C- Follicle development

10. Mechanism of action of clomiphene

A- Inhibit negative feedback of estrogen


B- Increase release of gonadotropin hormonesC- Decrease
estrogen

11. Estrogen antagonist and used first line in treatment of breast cancer

A- Anastrazole
B – Tamoxifen

12. Which of the following is more prone to influenza symptoms andproblems

A- Geriatric patient

B- Pregnant

C- Children older than 12years old

13. When taking oral contraceptives with erythromycin:

A- The erythromycin increases the contraceptive effect


B- the contraceptive increase the erythromycin effect
C- the erythromycin decreases the contraceptive effect
D- the contraceptive decreases the erythromycin effect

14. Vit c for females:

Answer: 75mg

15. Captopril is contraindicated with:

A- Pregnancy
B- Hypertension
C- Diabetes
D- CHF

ARIF PHARMA CLINIC COPYRIGHT@ARIFPHARMA.COM


Page 286

16. Which of the following is used as Anti-estrogen when used to causeabortion

A- Anastrozole

B- Tamoxifen

17. Anti-diabetic drug used in pregnancy?

A- Rosiglitazone
B- Metformin

N: B large Glargine is long-acting insulin

NOTE:

• Metformin is category B in pregnancy & can be used.


• Insulin is the drug of choice in gestational diabetes (diabetic pregnancy) but
glargine (long-acting insulin) is category C in pregnancy.
• If there is no insulin in choices choose metformin NOT glargine.

18. Pregnant woman taking valproic acid to go to a physician with tonic- Clonic
seizures. Which of the following is true?

A. Stop valporic and use another drug


B. Use valporic with iron supplementation
C. Use valporic with folic acid
D. Use valporic with multivitamins
E. Go to the doctor

The answer is C … If going to the doctor is in option so the answer is E

19. Pregnant woman with G6PD deficiency has G-ve. M.o. and UTI, Which is the drug of
choice to treat her UTI:

A- vancomycin

B- Nitrofurantoin
C- Cefuroxime
D- Sulphamethoxazole
ARIF PHARMA CLINIC COPYRIGHT@ARIFPHARMA.COM
Page 287

N.B:
Nitrofurantoin & Sulpha drugs cannot be used in GDPD deficiencypatients because it has
high risk to them.

20. Pregnant woman has a UTI (Urinary Tract Infection) which is the drugof
The choice to treat her UTI:

A- Ciprofloxacin
B- Tetracyclin
C- Sulphamethoxazole
D- Nitrofurantoin

• If there is no cefuroxime as an option …choose nitrofurantoin in this


case
• Nitrofurantoin is pregnancy category B. It is one of the few drugs commonly used in
pregnancy to treat UTI & contraindicated only atterm (during labor & delivery).
• vancomycin is the last choice
• Sulphamethoxazole & tetracycline cannot be used in pregnancy

21. About side effects of oral contraceptives except:

A- Depression

B- Hypertension

C- Constipation

22. DAILY intake of vitamin A for an adult female is:

A- 500 mcg
B- 700 mcg
C- 1500 mcg

Accurate doses (if mentioned in other questions)

adult male 1000 mcg


adult female 800 mcg
pregnant 900 mcg
breastfeed 1200-1300

ARIF PHARMA CLINIC COPYRIGHT@ARIFPHARMA.COM


Page 288

23. A pregnant woman at 43 weeks of pregnancy She began labor actions for12 hrs. The
contractions were strong at first but after some while became very weak. Which of the
following could be useful in her case:

A- Ergotamine
B- Saline infusion
C- Oxytocin

24. Pregnant women have hepatitis B when deliver we must give for a baby?

A. Hepatitis B vaccine only


B. Hepatitis B vaccine with interferon
C. Hepatitis B vaccine with immunoglobulin hepatitis.
D. Hepatitis B immunoglobulin with lamivudine

25. Pregnant has DVT (Deep Vein Thrombosis take

A- warfarin
B- Aspirin
C- Heparin

(warfarin is contraindicated in pregnancy) Heparin (unfractionated and low molecular weight) is the
preferred drug formanaging VTE " venous thromboembolism " in pregnancy.

26. A pregnant woman at Her 35 weeks has a headache she should take

A- Ergotamine
B- Acetaminophen (Paracetamol)

Acetaminophen is the drug of choice Ergotamine is contraindicated in pregnancy category X

27. Paracetamol category b in pregnancy means

A- Absolutely safe

B- Used with caution

Used with caution = generally safe in short-term use with a least possibleeffective dose

ARIF PHARMA CLINIC COPYRIGHT@ARIFPHARMA.COM


Page 289

28. A Pregnant Woman with syphilis should take

A- Ampicillin
B- Amoxicillin
C- Clavulanate
D- D- Ciprofloxacin

Note: Penicillin is the drug of choice for syphilis during pregnancy So ifthere is
(Penicillin eg: Benethamine penicillin (Benzylpenicillin) in choices will be better.

29. Which the following hormone suppress due to take oral contraception:

A. FSH (Follicle-stimulating hormone)


B. LH (Luteinizing hormone) (in the phase of ovulation)
C. GnRH (Gonadotropin-releasing hormone)

30. In the ovulation phase, Which hormone predominates?

A. FSH
B. LH
C. GnRH

Lh hormone is predominant in the ovulation phase. 2.Contraceptives make feedback


inhibition to (GNRH) hormone

31. MOA of Chorionic gonadotrophic during of treatment of infertility?

A. Follicle
B. Inhibition of reduced ovulation
C. Follice and induceovulation

32. Sevelamer is to treat

Answer: Hyperphosphatemia (Renagel)

ARIF PHARMA CLINIC COPYRIGHT@ARIFPHARMA.COM


Page 290

33. A pregnant woman has deep vein thrombosis (DVT) is hospitalizedtreated by:

Unfractionated heparin

Doses questions

Vitamin D Calcium Folic acid

Geriatrics Adult. ........1000mg Male. ........ 400mcg


<70years ..........600Iu Pregnant Female. ....... 400-
....1200mg 800mcg
> 70years. 800iu
Geriatric .......... 1200mg Pregnant. ......... 600mcg
Pregnant. ........ 600iu
Adult...600iu

34. Which of the following prevents Neural tube defects (birth defects)

A. Folic acid (vit. B9)

35. A woman had DVT and was treated a year ago, now she wants touse oral contraceptive
pills the most suitable ocs for her

A- Levonorgestrel (or l-norgestrel or D-norgestrel)


B- Ethinyle estradiol

Note: Estrogen increases the risk of DVT Progestin-only OCs are commonly prescribed when women
wish to takeOCs but estrogen is contraindicated.
Levo =progesterone=synthetic progesterone=second generationprogestin

36. Pregnant hypertensive women take

A- Prazosine
B- Methyldopa
C- Propanolo

ARIF PHARMA CLINIC COPYRIGHT@ARIFPHARMA.COM


Page 291

37. Iron deficiency anemia treated with…………》Iron

Megaloblastic anemia treated with ..................... 》Folic acid and vit b12

Pernicous anemiea treated with ......................... 》t b12

Anemia of chronic renal failure treated with…….》Epoetin


but if there is
iron deficiency it will be treated with ....................... 》Derbepoetin

38. Woman taking oxycarbamazipine, after 2 weeks of administration, redness, and flushing
appear….

A. Shift to phenytoin
B. Shift to carbamazepine
C. Take clozapine
D. Keep using oxycarbamazipine

It is a normal side effect but in the other question (young girl)answer is


shifted to ethosuximide

39. A Postmenopause old woman suffering from facial flushing and vaginal drying has done a
hysterectomy procedure. Which drug ofthe following should she use:

A. Estrogen

B. Progesterone

40. A 22 years woman wants to become pregnant, she is taking metformin and pioglitazone. she has
a history of hypoglycemia and sheprefers oral therapy. What will be the best approach in her
case?

A. Stop pioglitazone and titrate metformin

B. Stop both if them and start insulin therapy

C. Stop metformin and titrate pioglitazone d.do nothing

ARIF PHARMA CLINIC COPYRIGHT@ARIFPHARMA.COM


Page 292

41. Influenza vaccine in pregnancy is?

Answer: Safe

(IIV) Inactivated influenza vaccine is safe for pregnant

42. Pregnant woman with G6PD deficiency has G-ve. M.o. and UTI,which is the drug of choice
to treat her UTI:

A- Vancomycine

B- Nitrofurantoin

C- cefuroxime

D- Sulphamethoxazole

Nitrofurantoin & Sulpha drugs cannot be used in GDPD deficiencypatients because it has
high risk to them.

43. CASE WOMAN Hyprttention diabetic and sensitive of sulph. whichis contraindicated with
sulpha

A- Glyburide

B- Metformine

C- Amidrone

D- Enalopril

Glipride as it is sulphonylurea derivative and she is allergic to any sulf acontaining drugs.

44. A pregnant women senstive to amoxicillin which is the Drug Of Choice for her disease. what
will be the suitable altrenative anti-bioticfor her

A- Erythromycin
B- Sulfacetamide
C- Cefixime

ARIF PHARMA CLINIC COPYRIGHT@ARIFPHARMA.COM


Page 293

45. A 39-year-old what is oral birth control pills appropriate for her

A- Levonorgestrel

B- Ethinyl estradiol/lnestrenol

C- Lynestrenol / Norethisterone

D- Medroxyprogesterone (depot contraceptive)

This ques. may be incomplete … if there is any of the following


contraindications of COCP the answer will be LevonorgestrelTake the minipills of
progestin if
• Older than age 35 + smoke
• Older than age 35 + migraine headache
• Older than age 35 + obese
• Older than age 50
• Breast feeding
• Diabetes mellitus with vascular disease
• Risk of DVT or history of thromboembolism "blood clots"
• History of uncontrolled HTN or heart problems
• Breast or endometrial cancer Need to get pregnant.

46. 14years-obese girl comes to the clinic with a severe rash. She was initiated on oxcarbazepine
about 3 weeks ago for the management of partialseizures. Her medical history is significant
only for seizures. She has recently become sexually active and admits to inconsistent
contraceptive use.
Which one of the following interventions is best for her?

A. Change to carbamazepine.
B. Change to topiramate.
C. Change to valproic acid.
D. No change in therapy is necessary

47. Pregnant woman has vaginal discharge and has Candida albicanswhat's the drug of choice?
A- Acyclovir
B- Clotrimazole
C- Tetracycline
D- Fluconazole

ARIF PHARMA CLINIC COPYRIGHT@ARIFPHARMA.COM


Page 294

48. Pregnant woman has vaginal discharge and has Candida albicanswhat's the drug of choice?

A- Acyclovir
B- Clotrimazole
C- Tetracycline
D- Fluconazole

49. A Woman takes estrogen and she made a hysterectomy should take it?

A- Testosterone
B- Hydroxyprogestrone

When you take estrogen, you should always take progesterone tocounteract the negative effect of
estrogen

50. Case has pain and solid tumor

Answer: Oral contraceptives

N.B:
- Hormone therapy "e.g. oral contraceptive pills" are used to treatendometriosis-associated pain and
they are effective.
- Danazol can be used but it isn't the first choice because it can cause serious side effects and can
be harmful to the baby if the patient becomes pregnant while taking this medication.
- Progestins have a more favorable side effect profile than danazol.
- Surgery is the last resort and is recommended if the patient planning forpregnancy

51. A pregnant woman in her third week she takes levothyroxine 100 mcg of, you advise her:

A. Stop the medication immediately


B. Increase the dose of levothyroxine
C. Ask the prescriber to change the medication
D. Continue using the medication as prescribed

• Levothyroxine is considered the treatment of choice for the control of


hypothyroidism during pregnancy.
• Due to alterations of endogenous maternal thyroid hormones, the levothyroxine dose may need to be
increased during pregnancy and thedose usually needs to be decreased after delivery.

ARIF PHARMA CLINIC COPYRIGHT@ARIFPHARMA.COM


Page 295

52. Vitamine is contraindicated in pregnancy in highly doses

A- Vit. A
B- Vit. D
C- Vit. C
D- Vit. B12

53. Which is category X?

A- Ciprofloxacin
B- Rituximab

N.B:
FDA pregnancy category: C use rituximab only if clearly needed and the benefit outweighs risk
FDA pregnancy category: C Ciprofloxacin should not be used during pregnancy unless the potential benefits
justifies the potential risk to bothfetus and mother.

54. Misoprostol is used for:

A- Prevent ulcer

B- Prevent and treat gastric ulcer

C- Treatment of ulcer

55. Woman with a seizure is on valproic acid shes planning to get pregnant what is the best choice?

A- continue on valproic acid


B- Hange to levetiracetam
C- Stop valproic acid

56. What is the meaning of pregnancy category B:


Animal reproduction studies have failed to demonstrate a risk to the fetus
and there are no adequate and well-controlled studies in pregnant women.

ARIF PHARMA CLINIC COPYRIGHT@ARIFPHARMA.COM


Page 296

57. 35years BMI35, (Oral contraceptive)?

Only Progesterone pill

58. Selective estrogen receptor modulator (SERM):


Tamoxifen

59. Anti hypertensive drug... can affect fetal growth:


A- Amlodipine
B- Hydrochlorothiazide
C- Lisinopril

60. Anti hypertension drug safe in pregnant?

Answer: Labetalol

61. When Women start folic acid:


Answer: Before pregnancy with one month and 1st trimester.

62. (Case) Girl has epilepsy taking valproic acid and it's control the disease she planned to
be pregnant and she already takes folic acid what to do:

A- Shift from valproic acid to phenytoin


B- Keep her using valproic acid
C- Give her leviteracetam and decrease valproic acid till epilepsy is well Controlled
then discontinue

ARIF PHARMA CLINIC COPYRIGHT@ARIFPHARMA.COM


Page 297

63. Anti hypertensive drug... can affect fetal growth:


A- Amlodipine
B- Hydrochlorothiazide
C- lisinopril

Drug Mechanism Indication Side Effects


Captopril ACEInh HTN, Angina, Cough
Arrhythmia Hypertension
Proteinuria
Lisinopril ACEInh HTN, Angina,
Fetal renal damage
Arrhythmia (C.I.in Pregnancy)
Enalapril ACEInh HTN, Angina,
Arrhythmia Hyperkalemia
Losartan (AT-1) Blocker HTN, Angina, As ACEInh without
Arrhythmia cough

64. (case) woman take levothyroxine 50 and she become pregnant and she feel fatigue:

A- Don’t change dose coz fatigue is normal sign in pregnant.


B- Increase levothyroxine dose

65. Category A drug in pregnancy means:


A- No evidence of any risk on human
B- No evidence of any risk on human and animals

66. Infant with croup what virus u suggest he has:


A- Rotavirus
B- Parainfluenza
C- Pneumonia

Viral croup is caused by any virus that infects the voice box (larynx) and windpipe
(trachea). The virus that most often causes croup is parainfluenza. It can start like
a cold. But over time, your child will develop a "barky" cough. He may also make a
high-pitched, wheezing sound in his lower airways when he breathes. Croup sound
is a harsh, rattling sound in the upper airway called a "stridor". There may also be
a mild fever.

ARIF PHARMA CLINIC COPYRIGHT@ARIFPHARMA.COM


Page 298

67. which antiemetic we can use it for pregnant:

Answer: Cyclizine

o Cyclizine
o Diphenhydramine (Benadryl)
o Dimenhydrinate (Gravol, Dramamine)
o Doxylamine
o Meclizine (Bonine, Antivert)
o Promethazine (Pentazine, Phenergan, Promacot) can be administered
o via a rectal suppository for adults and children over 2 years of age.
o Hydroxyzine

68. A nursing women was taking a drug and caused decrease in the milk what drug can decrease
the milk:

Drugs inhibit lactation:

-Ethinylestradiol (Estrogens)
-Ergometrine (Oxytocics)
-Chlorphenamine (ANTIALLERGICS)
-Levodopa + carbidopa (ANTIPARKINSONISM DRUGS)
-Amiloride (Diuretics)
-Furosemide (Diuretics)
-Hydrochlorothiazide (Diuretics)
-Ethinylestradiol + levonorgestrel (Hormonal contraceptives)
-Hthinylestradiol + norethisterone (Hormonal contraceptives)

69. Stop in lactation:


A- Anti-cholinergic
B- Anti-muscarinic
C- Antihistamine
Page 299

70. Vaccine contra indicated in pregnancy:

71. What is the category of Lisinopril in pregnancy:

A- Category B
B- Category C
C- Category D
D- Category X

72. Patient take Mycophenolate and she want to be pregnant, What is your advise:-

A- Continues use Mycophenolate


B- Stop Mycophenolate before 2 weeks of trying to be pregnant
C- Stop Mycophenolate before 6 weeks of trying to be pregnant

It is known that mycophenolate products can cause birth defects in the unborn baby
of mothers who are, or have recently been, taking mycophenolate. It is forthis
reason women are advised not to become pregnant whilst taking mycophenolate
and for 6 weeks after stopping.
Source: Oxford Transplant Centre/Oxford Kidney Unit

73. TTT open angle glaucoma in pregnant:


Answer: Brimonidine
Page 300

74. The mother has infection when she delivered her baby he come with fever etc which antibiotic should
baby take:

A- Gentamicin & ampicillin


B- Gentamicin
C- Ampicillin
D- Clindamycin

75. Effect of finesteride on pregnancy:


Abnormalities of the external genitalia

76. Mother went to pharmacist her baby 1 month oldsuffer from mild fever:
A- Paracetamol nasal metered dose 15 mg / kg
B- Ibuprofen syrup dose 12 mg /kg

77. Lactating woman took pseudoephdrine what effect on baby:

Mothers reported irritability was reported in 20% of infants exposed


to pseudoephedrine.

78. Which drug is dangerous & CI in third trimester:

A- Methotrexate
B- Lithium

1-Methotrexate is category X and it mean never use with mother and its danger to her
live.
2-Lithium is category D and it mean that it is danger to fetal
*Risks of Using Lithium in Pregnancy
First trimester – There is a small risk of congenital problems, including serious heart
malformation.
Second trimester – There continues to be a risk of congenital problems, but not to the
same degree as in the first trimester.
Page 301

79. Pregnant women admitted to hospital suffering from upper respiratory disease, She took ceftriaxone
IV dose; and stayed in the hospital for 3 days. So what the vaccine should take?
A- Influenza.
B- Influenza + pneumonia
C- Meningitis + influenza.
D- Pneumonia + meningitis
80. Pregnancy woman take naproxen what harm could to baby?
A- Uncontrolled bleeding.
B- GI reflux disease.
81. Pregnancy with otitis media?
A- Azithromycin
B- Cifexime

82. Which drug suppresses milk production in breastfeeding women?

A- Estrogen.
B- Reserpine.
C- Metoclopramide.

* Estrogen has been shown to decrease the quantity and quality of human milk;
use only if clearly needed; monitor the growth of the infant closely.

83. Which drug contraindicated in breastfeeding:


A- Lithium.
B- Warfarin.
C- Methyldopa.

84. Drugs that contraindicated during lactation (all answers true)

A- Anticancer
B- Aspirin
C- Iodine
D- Androgen
E- Bromocriptin

**I think Tetracycline is also contraindicated in lactation


Page 302

85. Pregnant woman has infection. I don’t remember the m.o. and she is sensitive to penicillin.
And the m.o resistant to erythromycin and clindamycin. Which antibiotic is suitable for her?

A- Ciprofloxacin.
B- Vancomycin.

86. 20 years girl comes to take contraceptive piles which of the following can make oral contraceptive fail?

A- Rifampicin
B- Amoxicillin
C- Floxacilline
87. Role of nicotine in abortion?
By FDA. Cigarette smoking is known to cause: information

A- Spontaneous abortion,
B- Low birth weight
C- Increased perinatal mortality.
D- Potent vasoconstriction... Decrease uterine & placenta blood flow... Risk of abortion

88. Pregnant woman has syphilis can be treated by:


A- Ciprofloxacin
B- Amoxicillin + Clavulanic acid
C- Trimethoprim-sulfamethoxazole

*If there is no Benzathine penicillin as an option …choose Amoxicillin + Clavulanic acid.

89. Breastfeeding women need contraceptives which of these:

A- Estrogen.
B- Progesterone.
C- Monophasic pill.
D- Biphasic pills.

* Mono and bi phasic are techniques for administration of combined oral contraceptives(Estrogen
and progesterone).
Page 303

90. Breast-feeding woman has hypertension which antihypertensive drug has hazard on baby.

A- Metoprolol

B- Propranolol

C- Atenolol

D- Verapamil

91. A pregnant woman is allergic to penicillin. Which antibiotic should be given to her as
prophylaxis against streptococci b4 delivery?

A- Cefazolin.

B- Vancomycin.

C- Clindamycin.

92. Why should finasteride be stopped if a woman wants to be pregnant?

A- Cause abortion.

B- Teratogenic effects.

C- Genital malformation to infant.

D- Cranial facial abnormality.

E- Uterine vascular resistance

93. Frist baby milk name?

Answer: Colostrum

** Colostrum: a yellowish liquid, especially rich in immune factors, secreted by the mammary gland of
female mammals a few days before and after the birth of their young.

94. Estrogen receptor positive cause breast cancer which can be treated by

Answer: Tamoxifen

95. What is the maximum prescription time for narcotics for patients admitted in an emergency?

A- 24 hours

B- 72 hours

C- 45 hours

D- 30 hours
Page 304

** 24 hrs. In case of emergency. 3 days (72hr) in case of outpatient.

96. If a lactating mother has less milk production, what are the effects on nursing infants?

A- Constipation

B- Loss of appetite

C- Diminished weight gain

D- Increased sleep

97. Pregnancy with renal failure which is contraindicated ---

Answer: ACEI
98. Pregnant women must take which vaccine:
A- Influenza type 2.
B- Hepatitis B
C- Pneumococcal

99. Which drug can be used for long time treatment asthma in pregnant women:
A- Albuterol.
B- Cromolyn.
C- Theophylline.

100. Pregnant women with osteoarthritis, which is the last choice?


A- Acetaminophen
B- Glucosamine
C- Aspirin.
Page 305

101. Progesterone is added to estrogens in HRT to achieve which of the following


effects?

A. Decrease the estrogen action on the breast


B. Decrease the occurrence of endometrial cancers
C. Increase the effectiveness of the estrogens
D. Inhibit bone resorption

Rationale Progestins are added to HRT regimens to decrease the risk of endometrial
cancer

102. Which of the following would be the best agent to use in a patient with
PCOS?

A. Pioglitazone
B. Metformin
C. Regular insulin
D. Repaglinide

Rationale Pioglitozone, as an insulin sensitizer, might be efficacious in the


treatment of PCOS but metformin has fewer and less severe side effects.
Repaglinide stimulates insulin secretion which would be detrimental in PCOS.

DRX ARIF KHAN


Page 306

NOTE: CERTAIN PRODUCTS ON THIS LIST MAY HAVE QUANTITY


LIMITS (QLL), STEP THERAPY, OR PRIOR
AUTHORIZATION REQUIREMENTS. THESE REQUIREMENTS ARE
PROVIDED NEXT TO THE COVERED PRODUCT.
Drx Arif khan
Covered OTC Drugs REFERENCE DRUG NAME REQUIREMENTS/LIMITS
TOPICAL ANTIBACTERIAL/ANTIFUNGAL OTC DRUGS
OTC bacitracin topical ointment
OTC clotrimazole (vaginal use) Mycelex
OTC clotrimazole (topical use) Lotrimin
OTC miconazole 2% ointment
OTC miconazole vaginal suppositories, cream
OTC triple antibiotic cream Neosporin
PAIN RELIEVER OTC DRUGS
OTC acetaminophen tablets, capsules, Tylenol QLL= up to 4 grams APAP/day
suppositories, liquids, drops
OTC EC aspirin 81 mg, 325 mg, Ecotrin
OTC aspirin 325 mg
OTC ibuprofen Motrin
OTC naproxen Aleve
SMOKING CESSATION OTC DRUGS
OTC nicotine patch Nicoderm Duration for all formulary smoking
cessation meds=84 days/year
TOPICAL DERMATOLOGICAL (“SKIN/SCALP”) OTC DRUGS
OTC capsaicin
OTC DOAK TAR DISTILLATE, OIL
hydrocortisone Ala-Cort/Cetacort/Hytone
(prescription and OTC forms covered)

Arif Pharma clinic Copyright@arifpharma.com arifpharma.com


Page 307

OTC permethrin 1% lotion Nix


OTC Pyrethrin 0.33%
OTC zinc oxide ointment Desitin

DIABETES OTC DRUGS


INSULIN OTC VIALS
OTC HUMULIN 50/50 VIAL
OTC HUMULIN N
OTC HUMULIN R (100 U/ML VIAL)
OTC HUMULIN 70/30 VIAL
OTC NOVOLIN 70/30 VIAL
OTC NOVOLIN R VIAL
OTC NOVOLIN N VIAL
OTHER OTC DRUGS FOR DIABETES
OTC glucose chewable tablets

COVERED OTC DRUG REFERENCE DRUG NAME REQUIREMENTS/LIMITS


DIGESTION OTC DRUGS
ANTIDIARRHEAL DRUGS
OTC loperamide Imodium
ANTI-HEMORRHOIDAL DRUGS
OTC hemorrhoidal cream preparation
ANTIULCER DRUGS
OTC cimetidine Tagamet
OTC famotidine Pepcid
OTC nizatidine Axid
OTC ranitidine Zantac
PROTON PUMP INHIBITORS (“PPI”)
OTC omeprazole QLL=120 tabs /30 days
LAXATIVES AND CATHARTICS
OTC bisacodyl Dulcolax

Arif Pharma clinic Copyright@arifpharma.com arifpharma.com


Page 308

OTC docusate Colace


OTC glycerin Fleet
OTC MIRALAX QLL=510 g/30 days
OTC psyllium Metamucil
OTC SENOKOT (brand and generic dosage
forms covered)
OTC SORBITOL 70% ORAL SOLN
OTHER DIGESTION DRUGS
OTC aluminum hydroxide gel Alternagel
OTC antacid liquid, suspension
OTC calcium antacid tablet Tums
OTC effervescent pain relief Alka Seltzer
OTC simethicone drops
OTC VITAMINS
OTC calciferol, OTC ergocalciferol drops Drisdol
OTC calcium carbonate, Caltrate
OTC calcium carbonate 600 mg + D
OTC calcium citrate Citracal
OTC ferrous fumarate
OTC ferrous gluconate
OTC ferrous sulfate Iron
OTC magnesium oxide
OTC multivitamins (generic, adult
multivitamins)

COVERED OTC DRUG REFERENCE DRUG NAME REQUIREMENTS/LIMITS


OTC niacin
OTC pyridoxine (vitamin B6)
OTC SLO NIACIN
OTC sodium bicarbonate
OTC vitamin C 500, 1000 mg

Arif Pharma clinic Copyright@arifpharma.com arifpharma.com


Page 309

OTC vitamin D
OTC thiamine (vitamin B1)
FAMILY PLANNING OTC DRUGS
OTC NEXT CHOICE QLL=2 tabs (1 pkg)/1 month;
QLL=6 tabs (3 pkg)/year
OTC PLAN B ONE STEP QLL=1 tab (1 pkg)/1 month;
QLL=3 tabs (3 pkg)/year
EYE CARE OTC DRUGS
OTC artificial tears Tears Again
OTC REFRESH TEARS, LIQUIGEL
(15 ML AND 30 ML BOTTLE ONLY)
OTC sodium chloride 5% drops, ointment
OTC SYSTANE
(15 ML AND 30 ML BOTTLE ONLY)
OTC ZADITOR
COUGH/COLD/ALLERGY OTC DRUGS
ANTIHISTAMINES
cetirizine OTC tablets, OTC Zyrtec cetirizine-D QLL=60 tabs/30 days
cetirizine-D OTC tablets, cetirizine soln QLL=150 ml/30 days
cetirizine solution
OTC diphenhydramine Benadryl
OTC loratadine, OTC Claritin, Claritin-D OTC loratadine-D=30 tabs/30 days
OTC loratadine-D
ANTIHISTAMINE/DECONGESTANT COMBINATIONS
OTC brompheniramine-pseudoephedrine elixir
ANTITUSSIVE AND EXPECTORANT DRUGS
OTC CHERATUSSIN AC
OTC MUCINEX, DM
OTC tussin DM Robitussin DM
OTHER DRUGS FOR COUGH/COLD ALLERGY
OTC nasal spray Afrin
OTC pseudoephedrine (all generic dosage Sudafed
forms covered)

Arif Pharma clinic Copyright@arifpharma.com arifpharma.com


Page 310

COVERED OTC DRUG REFERENCE DRUG NAME REQUIREMENTS/LIMITS

DIABETIC OTC SUPPLIES (GLUCOMETERS, INSULIN SYRINGES, TEST STRIPS, LANCETS)


ACCU-CHEK ACTIVE GLUCOMETER/TEST STRIPS Combined QLL for test
strips=204/month
ACCU-CHEK ADVANTAGE GLUCOMETER/TEST Combined QLL for test
STRIPS strips=204/month
ACCU-CHEK AVIVA GLUCOMETER/TEST STRIPS Combined QLL for test
strips=204/month
ACCU-CHEK COMPACT GLUCOMETER/TEST Combined QLL for test
STRIPS strips=204/month
ACCU-CHEK COMPLETE GLUCOMETER
ACCU-CHEK SIMPLICITY
ACCU-CHEK COMFORT CURVE TEST STRIPS Combined QLL for test
strips=204/month
ACCU-CHEK MULTICLIX LANCET
DEVICE/LANCETS
ACCU-CHEK SOFTCLIX LANCET
DEVICE/LANCETS
MICROLET LANCING DEVICE/LANCETS
AUTOJECT 2 INJECTION DEVICE
insulin syringes
ONE TOUCH ULTRA2, UKTRALINK, ULTRAMINI,
ULTRASMART, VERIO, VERIO IQ
ONE TOUCH SELECT
ONE-TOUCH TEST STRIPS, CONTROL SOLUTION Combined QLL for test
strips=204/month
SOFT TOUCH
SOFTCLIX
CHEMSTRIP
KETOSTIX

Arif Pharma clinic Copyright@arifpharma.com arifpharma.com


Page 311

ASTHMA OTC SUPPLIES (PEAK FLOW METERS, SPACERS)


AEROCHAMBER, MICROCHAMBER QLL=#1/YEAR
ASSESS PEAK FLOW METER QLL=#1/YEAR
MICROCHAMBER PEAK FLOW METER QLL=#1/YEAR
PERSONAL BEST PEAK FLOW METER QLL=#1/YEAR
OTHER OTC DRUGS/SUPPLIES
sodium chloride 0.9%
nebulizer solution OTC

Arif Pharma clinic Copyright@arifpharma.com arifpharma.com


Page 312

❖ Prescription & OTC Drugs

➢ Prescription drugs are available only by recommendation of an authorized healthprofessional, such as a physician.
➢ Nonprescription (over-the-counter, or OTC) drugs are available on request anddo not require approval by a health
professional.

❖ Prescription & OTC Drugs

➢ Prescription and OTC drugs have been viewed differently by the public sine the classifications were established by
the Durham-Humphrey Amendment of 1951.
➢ In general, the public views OTC drugs asminimally effective and safe and prescription drugs as more potent
and frequently dangerous, However, these distinctions are

❖ OTC Drugs Interesting Facts

➢ Each year the U.S. spends over $14 billionon OTC drugs

➢ More than 300,000 different OTC productsare available on the market

➢ OTC expenditures comprise 60% of theannual drug purchase in the U.S.

➢ An estimated 3 out of 4 people routinely self-medicate with these drug products

Arif Pharma clinic Copyright@arifpharma.com arifpharma.com


Page 313

❖ Abuse of OTC products

➢ OTC products generally have a greater margin of safety than their prescription counterparts, but issues of abuse
need tobe considered.
➢ Physical dependence
➢ Psychological dependence

❖ Abuse of OTC products

➢ Nonprescription products that can be severely habit-forming: decongestants,laxatives, antihistamines, sleep


aids, antacids, and ephedrine.
➢ The active ingredients in OTC drugs havebeen classified and placed in category I(considered safe and effective)
➢ However, as recently as 1992, the FDA hasbanned over 400 ingredients from 7 categories of OTC products.

❖ “Switching” policy of the FDA

➢ The FDA is attempting to make more drugsavailable to the general public by switching some frequently used and
safeprescription medications to OTC status.
➢ This policy is in response to public demandto have access to effective drugs for self-medication and has resulted in
over 63 switched ingredients, such as ulcer and hair-growing medications

Arif Pharma clinic Copyright@arifpharma.com arifpharma.com


Page 314

❖ OTC drugs and self-care

➢ More than one-third of the time people treat their routine health problems with OTC medications to receive
symptomatic relieffrom their ailments.
➢ If done correctly, self-care with OTC medications can provide significant relieffrom minor, self-limiting health
problemsat a minimal cost.

❖ OTC Labels
Required label information includes:

➢ Approved uses of the product


➢ Detailed instructions on safe and effective use

➢ Cautions or warnings to those at greatest riskwhen taking the medication.

❖ Label information controlled by the FDA

When to use
Product name How to use
What to watch for
Identity
OTC Indication Possible instruction
Antacid Direction
Active ingredients Ingredients 12 fl Warning
OZ Precautions The whan drug should no
GOTCHA, INC. Expiration date
Quantity Manufacturers longer be use

Arif Pharma clinic Copyright@arifpharma.com arifpharma.com


Page 315

❖ Rules for proper OTC drug use

➢ Always know what you are taking.

➢ Know the effects.


➢ Read and heed the warnings and cautions.
➢ Don’t use anything for more than 1 to 2 wks.

➢ Be particularly cautious if also takingprescription drugs.


➢ If you have questions, ask a pharmacist.
If you don’t need it, don’t

❖ Types of OTC drugs

➢ Internal analgesics
▪ Analgesics
▪ Salicylates
➢ Therapeutic considerations
▪ Analgesic actions
▪ Anti-inflammatory effects

Arif Pharma clinic Copyright@arifpharma.com arifpharma.com


Page 316

▪ Antipyretic effects
▪ Side effects

❖ Types of OTC drugs


➢ Cold, allergy, and cough remedies
▪ Decongestants
▪ Antitussives
▪ Expectorants
▪ Vitamin C
➢ Sleep aids
▪ Melatonin
➢ Stimulants
▪ “Look-alike” and “act-alike
➢ Gastrointestinal medication
▪ Antacids and anti-heartburn medication
➢ Diet aids
➢ Skin products
• Acne medications
• Sun products

Arif Pharma clinic Copyright@arifpharma.com arifpharma.com


Page 317

➢ Skin first-aid products

▪ OTC herbal products

❖ Prescription drugs

➢ There are currently more than 10,000 prescription products sold in the UnitedStates, representing
▪ Approximately 1500 different drugs
▪ With 20 to 50 new medications approved eachyear by the FDA
➢ According to the Durham-Humphrey Amendment of 1951, drugs are controlledwith prescription if they are:
▪ Habit-forming
▪ Not safe for self-medication
▪ Intended to treat ailments that require the supervision of a health professional New and
without an established safe track
▪ Record

❖ Doctor-patient communication

➢ When a physician prescribes a drug, a patientshould insist on answers to the following questions:
➢ What is the desired outcome?
➢ What are the possible side effects of thedrug?
➢ How should the drug be taken to minimizeproblems and maximize benefits?

Arif Pharma clinic Copyright@arifpharma.com arifpharma.com


Page 318

❖ Generic and proprietary drugs

➢ Generic is the official, non-patented, nonproprietary name of a drug. The term generic is used by the
public to refer to thecommon name of a drug that is not subjectto trademark rights.

➢ Proprietary is a brand or trademark name that isregistered with the U.S. Patent Office. Proprietary
denoted medications marketedunder specific brand names

❖ Common categories of prescription drugs

➢ Analgesics
▪ Low-potency (Darvon)
▪ Moderate potency (Percodan)
▪ High-potency (Demerol)

➢ Antibiotics
▪ Antibacterials

➢ Antidepressants

➢ Antidiabetic drugs
➢ Antiepileptic drugs

➢ Antiulcer drugs

Arif Pharma clinic Copyright@arifpharma.com arifpharma.com


Page 319

➢ Bronchodilators

➢ Cardiovascular drugs

▪ Antihypertensive agents
▪ Antianginal agents
▪ Drugs to treat congestive heart failure

▪ Cholesterol and lipid-lowering drugs

➢ Hormone-related drugs
➢ Sedative-hypnotic agents Drugs to treat HIV

Arif Pharma clinic Copyright@arifpharma.com arifpharma.com


Page 320

✓ Who may recommend an OTC medication to a patient?

A- Pharmacy technician
B- Certified pharmacy technician
C- Certified pharmacy technician with OTC training
D- None of the above

Rationale Only a pharmacist may make recommendations to a patient.

✓ All of the following agents represent an approved OTC treatment for acne vulgaris except

A- PROPA PH.
B- Liquimat.
C- Rezamid.
D- Carmol-HC.
E- Loroxide.

Rationale Products containing benzoyl peroxide, sulfur, salicylic acid (3% to 6%), and resorcinol (1% to 2%) have been
shown to be effective agents in the treatment of acne vulgaris. Benzoyl peroxide (Loroxide), salicylic acid (PROPA pH),
sulfur (Liquimat), and resorcinol and sulfur (Rezamid Lotion) are available OTC anti-acne products. CarmolHC, a urea-
containing product that also contains hydrocortisone, is effective in treating dry skin

Arif Pharma clinic Copyright@arifpharma.com arifpharma.com


Page 321

Arif Pharma clinic Copyright@arifpharma.com arifpharma.com


Page 322

Geriatrics

1. Old patient with end-stage colorectal cancer takes 60 mg of morphine twice. Suffer pain when he
goes to the bathroom Add?

A - Mg
B - morphine

2. Recommended calcium dose for geriatric

A- 800mg

B- B- 600mg

C- C- 500mg

D- 1200mg

3. Recommended vit D3 dose for geriatric

A- 200 iu

B- 400iu

C- 600 iu

D- 300 iu

For Vit D dose-.

Younger than 70 years old 600 iu

Older than 70 years old 800 iu

[Type here] [Type here] [Type here]


Page 323

4. A Geriatric patient with osteoarthritis and mild pain in the hip. He has hypertension and coronary artery
disease what is the best drug for pain

A- Glucosamine

B- Paracetamol(acetaminophen)
C- Ibrufen
D- D- Aspirin

5. Nitroprusside in congestive heart failure patients is administered by:

A- Sublingual
B- Transdermal
C- Subcutaneous
D- Slow i.v.infusion

6. A diabetic old patient with hyperlipidemia complains of erectiledysfunction so DOC

A- Sildenafil
B- Testosterone patch

7. 85 years man has pain in the joint .. what is the most dangerous abbreviationput in a prescription?

A- Qid
B- P.R.N
C- Pe
D- OD

OD means Once daily Mistaken as “right eye” (OD-oculus dexter), leadingto oral liquid medications
administered in the eye SO it can be fatal QD (Every day), QOD (Every other day), Q1d (Daily) If any
of them in choices choose them. Because these abbreviations can be mistaken for QID (Four times daily).

8. Dose of calcium for woman 65year


A- 600 mg
B- 800 mg
C- 1200mg

[Type here] [Type here] [Type here]


Page 324

9. An elderly man around 60 years old, complains of polyuria, and dry mouth. There is no family history

of diabetes. he has done lab tests and the results were positive for diabetes initial treatment should be!

Answer: Metformin

10. Old man has rheumatoid arthritis taking (methotrexate - ibuprofen -Losec) and these drugs were not
effective, so the next step we use:

A- Hydroxy chloroquine
B- Lefulonamide
C- Alendronate Na

11. PQ is a 75-year-old patient who has just been diagnosed with hypothyroidism. Her past medical history
is significant for congestive heart failure, type 2 diabetes mellitus, osteoporosis, and chronic stable
angina, all of which are well-controlled. Her medications include:

Metoprolol 25 mg bid
Calcium carbonate 1250 mg bidVitamin D 1000 IU
daily Glyburide 2.5 mg bid
Enalapril 10 mg bid Furosemide 40 mg daily
Nitroglycerin SL spray prn

*PQ should be started on a low dose of levothyroxine because of her:

A. Age.
B. Gender.
C. Diabetes.
D. Metoprolol use.
E. Nitroglycerin use.

12. A old man will be on simvastatin and he is on verapamil what dose of simvastatin will be appropriate:
A- 10 mg/d
B- 20 mg/d
C- 30 mg/d
D- 40 mg/d

[Type here] [Type here] [Type here]


Page 325

13. Drug treatment extrapyramidal for 70-year-old:


A- Benztropin
B- Haloperidol
C- Memantin

14. Which is the pt with multiple prescriptions not take OTC:

A- Adult
B- Pediatric
C- Children
D- Geriatric

15. case Geriatric pt + back pain + high LDL+ DM which analgesic can give to him?

Answer: Acetaminophen

[Type here] [Type here] [Type here]


Page 326

16. Levothyroxine dose in geriatric:

Mild Hypothyroidism

1.7 mcg/kg or 100-125 mcg PO qDay: not to exceed 300 mcg/day


>50 years (or <50 yr with CV disease)
• Usual initial dose: 25-50 mcg/day
• May adjust dose by 12.5-25 mcg q6-8Week
>50 years with CV disease
• Usual initial dose: 12.5-25 mcg PO qDay
• May adjust dose by 12.5-25 mcg q4-6weeks until patient becomes euthyroid and
serumTSH concentration normalized; adjustments q6-8weeks also used
• Dose range: 100-125 mcg PO qDay

Severe Hypothyroidism
Initial: 12.5-25 mcg PO qDay
Adjust dose by 25 mcg/day q2-4Week PRN
Subclinical Hypothyroidism
Initial: 1 mcg/kg PO qDay may be adequate, OR
If replacement therapy is not initiated, monitor the patient annually for clinical status

17. Drug treatment extrapyramidal for 70-year-old:


A- Benztropin and Diphenhydramine,Trihexyphenidyl
B- Haloperidol.
C- Memantin

18. 65 years old man suffering from heartburn and peptic ulcer. Which drug is not recommended
because of its unwanted effect on the central nervous system?

A- Famotidine
B- Cimetidine.
C- Nizaditine

19. What is the best hypnotic for geriatric?


A- Alprazolam.
B- Diazepam.
C- Phenobarbitone.

** If there is no Zolpidem or zaleplon in the other choices the answer Alprazolam will becorrect.

[Type here] [Type here] [Type here]


Page 327

20. Side effects and adverse drug reaction in geriatrics due to.
A- Decrease elimination of the drug.
B- Polypharmacy.
C- Geriatric less sensitive to drug effect.
D- Good relation with health providers.
21. The same question, case of 75 years old have HTN, DM, hyperglycemia what his score
according to CHADS
A- 1
B- 2
C- 3
D- 4 Score 4
**Age 2 HTN 1 DM 1.

22. A 62-year-old woman is noted to have open-angle glaucoma. She


inadvertently applies excessive pilocarpine to her eyes. This may result in
which of the following?

A. Bronchial smooth muscle dilation


B. Decreased gastrointestinal motility
C. Dilation of blood vessels
D. Mydriasis

Rationale Excessive pilocarpine may initially result in the dilation of blood vessels with a drop
in blood pressure and a compensatory reflex stimulation of heart rate. Higher levels will
directly inhibit the heart rate. In addition, pilocarpine stimulation of muscarinic
cholinoreceptors can result in miosis, bronchial smooth muscle dilation, and increased GI
motility.

23. A 55-year-old man with congestive heart failure is noted to be taking furosemide each day.
Which of the following is most likely to be found in the serum?

A. Decreased potassium level


B. Decreased uric acid level
C. Elevated magnesium level
D. Low bicarbonate level

[Type here] [Type here] [Type here]


Page 328

N.B:- In addition, pilocarpine stimulation of muscarinic cholinoreceptors can


result in miosis, bronchial smooth muscle dilation, and increased GI motility.

24. A 55-year-old woman is being treated for RA. Her disease has
become much worse, and a new medication is added. After 6 months,
she notes night sweats, weight loss, chronic cough, and a chest
radiograph that indicates a cavitary lesion. Which of the following
medications was most likely prescribed for RA?

A- Gold salts
B- Infliximab
C- Methotrexate
D- Naprosyn

Rationale The anti-TNF-α immunoglobulin agents are usually well tolerated and
modify the disease process of RA; however, they tend to predispose patients to
infections, particularly tuberculosis. The patient in question has a typical clinical
presentation of tuberculosis. Diagnosis would be confirmed by sputum culture
and acid-fast smear, and therapy started with multiple antituberculosis agents.

25. A 74-year-old woman with a history of atrial fibrillation presents to the


emergency department after a bowel movement with bright red blood.
Her blood pressure is 88/56 mm Hg with a pulse of 118 beats/minute. She
is on warfarin for anticoagulation and a stat INR is 7.2. The decision is
made to start transfusing blood. What is the most appropriate treatment
to reverse warfarin?

(A) Fresh frozen plasma


(B) Platelet transfusion
(C) Protamine sulfate
(D) Vitamin K
(E) Whole blood transfusion

Rationale Fresh frozen plasma. The best way to convert warfarin in an


emergency is with fresh frozen plasma. This patient’s vital signs show that she is
probably hypovolemic. Fresh frozen plasma will replace the coagulation factors
deficient (II, VII, IX, and X) from the use of warfarin and the lack of vitamin K. (B)
A transfusion of platelets would not replenish the deficient clotting factors II, VII,
IX, and X. (C) Protamine sulfate is used for the reversal of heparin, not warfarin.
(D) Vitamin K is used in the reversal of warfarin, but it takes a couple of days for
the vitamin K to replenish the deficient clotting factors. This patient needs
immediate reversal. (E) A whole blood transfusion is rarely necessary. Fres frozen

[Type here] [Type here] [Type here]


Page 329

plasma has a greater concentration of clotting factors, which is needed in the


reversal of warfarin.

26. A 64-year-old man is brought to the emergency department


unconscious. He undergoes a CT ofthe chest, which reveals a pulmonary
embolism. He is considered for immediate therapy with heparin. Because
the man is unconscious, history cannot be obtained from him. Which of the
following would represent a contraindication to heparin therapy?

(A) Alcoholism
(B) Drug abuse
(C) Hypertension
(D) Immune deficiency state
(E) Recent surgery to remove genital warts

Rationale Alcoholism. Heparin is contraindicated for patients who are


hypersensitive to it; have bleeding disorders; are alcoholics; or are having or
have had recent surgery of the brain, eye, or spinal cord. (B) Drugabuse is not a
contraindication to heparin. (C) Hypertension is not a contraindication to
heparin. (D) An immunedeficiency state is not a contraindication to heparin. (E)
Recent surgery to the brain, eye, or spinal cord represents a contraindication to
heparin therapy.

27. case geriatric pt + back pain + high LDL+ DM which analgesic can give
to him?

Answer: Acetominaphine

28. A 63-year-old man with debilitating Parkinson’s disease is currently


taking levodopa. His primary care physician adds carbidopa to his
treatment regimen. One week later, the patient presents to the
emergency department complaining of anorexia, nausea, and
vomiting. What is the most likely explanation for these findings?
A- Drug toxicity
B- Idiosyncratic drug reaction
C- Stimulation of the chemoreceptor trigger zone
D- Underlying infection
E- Undiagnosed malignancy

Rationale Stimulation of the chemoreceptor trigger zone. One of the adverse


effects of carbidopa is nausea and vomiting. This can occur because of
stimulation of the chemoreceptor trigger zone of the medulla. This is not a
drug toxicity nor is it an unexpected idiosyncratic drug reaction. (A) There is no
evidence to suggest drug toxicity in this case. (B) This is not an unexpected
reaction. It is an important side effect of carbidopa to realize. (D) There is no

[Type here] [Type here] [Type here]


Page 330

evidence in the history of this patient to suggest infection.


(F) There is no evidence in the history of this patient to suggest underlying
malignancy.

29. A 69-year-old man with a history of hypertension on a thiazide diuretic


once daily is planning to have a hip replacement. He sees the
anesthesiologist preoperatively and is found to have serum potassium
of 2.9 mEq/L on routine laboratory studies. His blood pressure is
110/86 mm Hg. What is the best course of action for this patient at this
time?

A- Begin exercise regimen immediately


B- Cardiac enzyme evaluation for possible myocardial infarction
C- Discontinue thiazide diuretic and begin ACE inhibitor therapy
D- Increase the thiazide diuretic dose to be administered twice daily
E- Supplement potassium by increasing the intake of fruits and bananas

Rationale Supplement potassium by increasing the intake of fruits and


bananas. Hypokalemia is the most frequent problem encountered with
thiazide diuretics, and it can predispose patients who are taking digoxin to
ventricular arrhythmias. Often, K+ can be supplemented by diet alone such as
by increasing the intake of citrus fruits, bananas, and prunes. In some cases, K+
salt supplementation may be necessary. (A) This patient must have his
hypokalemia managed immediately. (B) There is no indication that this patient
is having a myocardial infarction. (C) This patient has good blood pressure
control on the thiazide diuretic. (D) The thiazide is working nicely on a daily
dose regimen. Blood pressure is well controlled
30. A 67-year-old man with long-standing hypertension presents to his
primary care physician for a follow-up. He is managed with
spironolactone. Physical examination reveals bilateral
gynecomastia. He states that he has recently undergone an
endoscopy, which revealed gastric ulcers. What is the most likely
explanation for these findings?
A- Chronic stress
B- Drug toxicity
C- Pituitary tumor
D- Thyroid disease
E- Thyroid storm
Rationale Drug toxicity. Spironolactone frequently causes gastritis and can
cause peptic ulcers. Because it chemically resembles some of the sex steroids,

[Type here] [Type here] [Type here]


Page 331

spironolactone may act at receptors in other organs to induce gynecomastia


in male patients and menstrual irregularities in female patients. Therefore,
the drug should not be given at high doses on a chronic basis. (A) Chronic
stress can produce gastric ulcers but not gynecomastia. (C) A pituitary tumor
would not be likely to produce gynecomastia and gastric ulcers. (D) Thyroid
disease does not usually produce gynecomastia. (E) This patient has no
clinical findings to suggest a thyroid storm.

31. A 63-year-old woman is hospitalized in the intensive care unit with


overwhelming sepsis. She is on multiple intravenous medications
and is now begun on imipenem/ Cilastatin. The treating physician
must be concerned about which of the following side effects of this
therapy?

A- Cardiotoxicity
B- Gastrointestinal ischemia
C- Pulmonary fibrosis
D- Renal failure
E- Seizures

Rationale Seizures. Imipenem/cilastatin can cause nausea, vomiting, and


diarrhea. Eosinophilia and neutropenia are less common than with other
lactams. High levels of imipenem may provoke seizures, but meropenem is
possibly less likely to do so. Doripenem has not demonstrated any potential to
cause seizures in animal studies. (A) Imipenem/cilastatin is not cardiotoxic. (B)
Imipenem/cilastatin is not implicated in causing gastrointestinal ischemia. (C)
Pulmonary fibrosis is not a side effect of this antibiotic. It is common with an
antineoplastic medication, bleomycin. (D) Renal failure is certainly possible
with antibiotics; however, the physician must be concerned about new-onset
seizures from imipenem/cilastatin

32. A 59-year-old man with multiple organ system failure and sepsis is
treated with multiple medications, Including gentamicin. Unfortunately,
because of an error in the pharmacy, the patient is given a dose of
daptomycin. Over the next few hours, the patient becomes tachypneic and
develops a fever of 103°F. What is the most likely rationale for this
response?

A- Atrial fibrillation
B- Cardiac arrest

[Type here] [Type here] [Type here]


Page 332

C- Inactivation of surfactant
D- Pulmonary embolus
E- Pulmonary infarct
Rationale Inactivation of surfactant. Daptomycin is indicated for the
treatment of complicated skin and skin structure infections and bacteremia
caused by Staphylococcus aureus, including those with right-sided infective
endocarditis. The efficacy of treatment with daptomycin in left-sided
endocarditis has not been demonstrated. Additionally, daptomycin is
inactivated by pulmonary surfactants; thus, it should never be used in the
treatment of pneumonia. (A) Daptomycin does not cause atrial fibrillation.
(B) Daptomycin does not cause cardiac arrest. (D) Daptomycin does not
cause thromboembolic diseases such as pulmonary embolism. (E) A
pulmonary infarct is unlikely to develop in this patient.

33. A 57-year-old man who has chronic leukemia with brain


metastasis has begun on high-dose intrathecal cytarabine. The
treating physician must be aware of which of the following
significant adverse effects?

A- Myocardial infarction

B- Paralysis

C- Pulmonary embolism

D- Renal cast formation

E- Uremic pericarditis
Rationale Paralysis. Cytarabine has several toxicities. Nausea, vomiting,
diarrhea, and severe myelosuppression (primarily granulocytopenia) are the
major toxicities associated with ara-C. Hepatic dysfunction is also occasionally
encountered. At high doses or with intrathecal injection, ara-C may cause
leukoencephalopathy or paralysis. (A) High- dose cytarabine can cause
leukoencephalopathy, not cardiac disease. (C) High-dose cytarabine does not
typically cause venous stasis or embolism. (D) High-dose cytarabine can cause
paralysis, not renal cast formation. (E) Cytarabine at lower doses can cause
gastrointestinal and hematologic toxicities. Uremic pericarditis is an unlikely
effect.

[Type here] [Type here] [Type here]


Page 333

34. An 83-year-old woman nursing home resident complains of trouble


falling asleep at night. Her daughter also mentions that her mother’s
mood appears depressed, and the nursing home staff state that she is
not eating well. Which of the following medications could best help
this patient?

A- Amitriptyline
B- Buspirone
C- Mirtazapine
D- Olanzapine
E- Venlafaxine

Rationale Mirtazapine. Many drugs carry the side effect of appetite


suppression, which is usually seen as an adverse effect. Elderly patients with
depression may have a problem with the appetite suppression associated with
this medication because of their already poor nutritional status. In addition,
many of these patients report problems sleeping at night. Of course, a single
drug that addresses all these issues is preferable to multiple drugs.
Mirtazapine is a tetracyclic antidepressant that can cause both appetite
stimulation and drowsiness; it would probably be the best choice for this
patient. (A) Amitriptyline is an antidepressant that can cause drowsiness but is
not known to cause appetite stimulation as mirtazapine. Mirtazapine would
probably be a better choice to treat this patient. (B) Buspirone is an
antidepressant that can cause drowsiness but is not known to cause appetite
stimulation as mirtazapine. Mirtazapine would probably be a better choice to
treat this patient. (D) Olanzapine causes the desired side effects of drowsiness
and appetite stimulation but is not an antidepressant. Olanzapine is an
atypical antipsychotic but should not be used to treat psychosis related to
dementia in the elderly. (E) Venlafaxine is an antidepressant that can cause
drowsiness but is not known to cause appetite stimulation as mirtazapine.
Mirtazapine would probably be a better choice to treat this patient

35. A 69-year-old man with a history of squamous cell carcinoma of the larynx
underwent treatment with surgical resection and postoperative radiation therapy.
He currently has significant xerostomia. Which of the following would be the best
course of treatment for this patient?

A- Bethanechol
B- Carbachol
C- Oral liquid intake
D- Resection of the parotid gland
E- Pilocarpine

[Type here] [Type here] [Type here]


Page 334

Rationale Pilocarpine. Pilocarpine is one of the most potent stimulators of


secretions (secretagogues) such as sweat, tears, and saliva, but its use for
producing these effects has been limited because of its lack of selectivity. The
drug is beneficial in promoting salivation in patients with xerostomia resulting
from irradiation of the head and neck. Sjögren syndrome, which is
characterized by dry mouth and lack of tears, is treated with oral pilocarpine
tablets and cevimeline, a cholinergic drug that also has the drawback of being
nonspecific. (A) Bethanechol is useful for patients with postoperative urinary
retention and neurogenic bowel. (B) Carbachol is useful for patients with
ophthalmologic issues such as glaucoma. (C) Although oral liquid intake would
be helpful, additional pharmacologic treatment would be more beneficial for
this patient. (D) Resection of the parotid gland would unlikely change the
symptoms exhibited by this patient.

[Type here] [Type here] [Type here]


Page 335

Pediatrics
Drx Arif Khan
1. According to KSA cold preparation; drugs not given to children less than:

A- 2 years
B- 4 years
C- 6 years
D- 5 years

2. Phenytoin suspension dose?

Answer: 15ml/day or 5 mL (125 mg/5 mL) three times daily, or one teaspoonful,
by mouth 3 times daily. Adjust the dosage to suit individual requirements,
up to a maximum of 25 mL daily

3. According to the Saudia Food and drug authority cough medications are contraindicated in
children:

A- less than one year


B- less than two years
C- less than three years
D- less than six years

4. Which of the following diabetic drugs are approved by FDA for pediatric use

A- Pioglitazone

E- Metformin

F- Glimepiride

G- Rastinone

5. A 2-year-old child come to the clinic for taking the hepatitis vaccine, we know that he took the
pneumonia vaccine a month ago, so we should:

A- give him the vaccine immediately

B- Wait for 3 monthsc- wait for 6 months

C- Wait for 1 year


Page 336

5. For a 4-year-old child, the maximum daily dose of paracetamol is


A- 240mg
B- 480 mg
C- 1 g
D- 2 g
N.B. Dose for children of paracetamol:

• Four years old child should take 240 mg paracetamol per dose, repeated every four hours, Maximum:
5 doses daily>>>>> so, 5 x240mg= 1200 mg =1.2g choice: 1g (as age-based dose)
• For (age-based dose): 10 to 15 mg/kg/dose every 4 to 6 hours as needed; do not exceed 5 doses in 24
hours; maximum daily dose: 75mg/kg/day
• Adult maximum dose of paracetamol: 4000mg = 4g

6. A Child with otitis media,

Answer: high dose amoxicillin

7. A Child born to a hepatitis B-positive mother must take

A- First dose of the hepatitis B vaccine


B- One dose of the Hepatitis B Immune Globulin (HBIG).
C- Both of them

8. Child is on oxcarbazepine for epilepsy and suffers from


a rash
Change to ethosuximide

9. Month-old baby with a history of premature birth and chronic lung disease is admitted to the pediatric
intensive care unit with respiratory distress requiring intubation; fever; and a 3-day history of cold-like
symptoms. A nasal swab is positive for RSV. Which one of the followingis the best intervention?

A. Palivizumab.
B. Corticosteroids.
C. Cefuroxime.
D. Intravenous fluids and supportive care
Page 337

10. Phenytoin suspension dose:

15ml/day
or 5 mL (125 mg/5 mL) three times daily,
or One teaspoonful, by mouth 3 times daily. Adjust the dosage to suit individual requirements, up to a
maximum of 25 mL daily

11. According to the Saudia Food and drug authority cough medications are contraindicated in
children:

A- less than one year


B- Less than two years
C- less than three years
D- Less than six years

12. At what age the tuberculosis vaccines (BCG) should be given:

A- At birth
B- 2months_4monthd_6months
C- 6weeks_2months_4monthsd- 1 year

13. A child is taking high dose amoxicillin (99mg/kg) for otitis media came to the clinic for pneumonia
and influenza vaccine:

A- Cancel the vaccines


B- Proceed to give the vaccines
C- Delay the vaccines for one year

14. MMR vaccine is given at what age?

The first dose is 12 months and the second dose 4years

FULL FORM OF MMR VACCINE:- Measles, Mumps & Rubella (MMR) Vaccine

MMRV vaccine protects against four diseases: measles,


Mumps, rubella, and varicella (chickenpox).
This vaccine is only licensed for use in children 12 months through 12 years of age.
CDC recommends that children get one dose of MMRV vaccine at 12 through 15 months of age,
and the second dose at 4 through 6 years of age.
Page 338

Children can receive the second dose of MMRV vaccine earlier than 4 through 6 years.
This second dose of MMRV vaccine can be given 3 months after the first dose.
A doctor can help parents decide whether to use this vaccine or the MMR vaccine

15. In which muscle do we give vaccines for infants:


The vastus lateralis muscle in the anterolateral thigh

16. The child receives a high dose of Amoxicillin for otitis and he has- an appointment for a vaccine what
we do shall we postpone the vaccination ?:

Answer:- NO

17. The child received a short-acting b agonist without improvement then he develop a cough with
weeping what is the treatment:

A- Amoxicillin
B- Azithromycin
C- Steroid

18. Baby with immune deficiency syndrome took immunoglobulin which vaccine is C.I:

A- Mmr
B- Rubella
C- Meningitis
D- Rotavirus (RV)

Babies with “Severe combined immunodeficiency” (SCID) should not get


rotavirus vaccine.
19. Child’s weight is 16 kg and length 92 cm calculate the body surface area.

So BSA(m²)= SQRT (92x16)/3600

= SQRT (.48555)

=.69681 m²
Page 339

20. Anti metabolite cause bone marrow suppression:


A- Azathioprine
B- Methotrexate

21. Drug and dose of Infant with mild fever:


Paracetamol 1 gm per day

22. Chlild with otitis media with QT prolongation, which abx is CI:
Answer: Azithromycin

23. Dosage form preferred for children:


Answer: Oral

24. one drop has a volume of:


A- 0.5 ml
B- 1 ml
C- 1.5 ml
D- variable, Depending on the liquid and the dropper

25. one drop has a volume of:


A- 0.5 ml
B- 1 ml
C- 1.5 ml
D. variable, depending on the liquid and the dropper

26. Case) A child swallowed 100 capsules of iron each capsule contains 18 mg no signs and symptoms
so the ttt is:

A- Activated charcoal
B- Activated charcoal double dose
C- Gastric lavage

27. Neonate came to the clinic with fever, persisting cough, and nasal discharge which
diagnosis:

A- Pneumonia
B- Laryngitis
C- Pharyngitis
Page 340

28. Child has Neisseria what is the precautions:

Isolate to prevent transmission of infectious agents and prevent transmission of


pathogens spread through close respiratory or mucousmembrane contact with
respiratory secretions

29. Child patient starts a course of antibiotics he takes the drug for seven days and still one dose
and he has to take a vaccine:

Answer: Take the vaccine immediately

30. Most common cause for Pediatric dose error than adult:
A- Most medication is weight dependant
B- Most medication is liquid
C- Greater surface area

31. 12 years baby weight: 30 kg & length 120 cm should receive medicine 5 mg/m2 What is the total
dose:

32. Newborn with fever, dyspnea, and jaundiceHe can take:

A- Ampicillin and gentamicin


B- Ceftazidime and gentamicin
C- Ampicillin and ceftriaxone
Page 341

33. Babie infected with (I think with tuberculosis):


A- The family should take rifampicin
B- Should take rifampicin within
C- Rifampicin has no role in the protection

34. For a Newborn baby prescribed for morphine, what is the best route of administration:
A- IV
B- IM
C- SC
D- Suppository.

35. In pediatric analgesics for tooth growth pain contraindications have a bad effect on the brain.
A- Aspirin
B- Acetic acid
C- Antiseptic
36. Teething gel for children contains:
A- Lidocaine
B- Aspirin
C- Acetic acid
D- Antiseptic
37. Child with BMI of 85% of his age so he is
A- Weight loss
B- Obese
C- Highly obese
D- Very obese.
37. Child 85% BMI of his age he had muscle and skin thickness test for...
A- Edema
B- Skin test
C- Body fat
D- Vasculature.
Page 342

38. Five-year-old child and a weight 80 % of his age, skin thickness and arm muscles are normal,
this child is:
A- Risk of overweight
B- Overweight
C- Under normal.
D- Normal range

BMI percentile & definitional categories childhood weight


Underweight ˂ 10 %
Normal 10 % to 84 %
Overweight 85 % to 94 %
Obese > 95 %
Extreme obese > 120 %

39. Side effects of ciprofloxacin in children:

A- Phototoxicity.
B- Seizures.
C- Vertigo.

** If Phototoxicity is not an option, choose seizures.

40. Levocetirizine can be used after:


a. Over 6 months
b. Over 8 years
c. over 12 years
d. Over 16 years.

*Levocetirizine: Pediatric drug information:


Page 343

*Is used in the relief of symptoms associated with perennial allergic rhinitis (FDA approved in ages ≥6
months and adults), seasonal allergic rhinitis (FDA approved in ages ≥2 years and adults), and
treatment of the uncomplicated skin manifestations of chronic idiopathicurticaria (FDA approved in
ages ≥6 months and adults). Dose in Children 6 months to 5 years – 1.25 mg (1/2 tsp oral solution)
once daily in theevening. (tsp = teaspoon).

We will choose the youngest age in the choices & if there are 6 months in choices it willbe the answer.

41.A 9-year-old patient whose main complaint is shooting arm and leg pain is seen in your hospital.
After careful study, you make the diagnosis of Anderson-Fabry disease. Which of the following
would be the best course of treatment for this patient?

A. α-Galactosidase A
B. High-dose glucocorticoids
C. Indomethacin
D. Sacrosidase

Rationale Anderson-Fabry disease can be treated successfully with α-galactosidase

42. A 6-year-old boy presents to the emergency department with an altered mental status. He is
hyperventilating, has a rash on his hands, and has a high fever. His mother has been giving him
an antipyretic for his fever for the past 2 days. The patient’s liver enzymes are elevated. His
mental status continues to decline. What is the mechanism of action of the most likely
medication given tothis child by his mother

A- Inhibits phospholipase A2
B-Irreversibly inhibits cyclooxygenases 1 and 2
C- Reversibly inhibits cyclooxygenases 1 and 2
D- Reversibly inhibits cyclooxygenase 2
E-Reversibly inhibits H1 histamine receptors
Page 344

Rationale Irreversibly inhibits cyclooxygenases 1 and 2. The boy is suffering from Reye’s
syndrome causedby taking aspirin for a viral infection. The mechanism of action of aspirin is the
irreversible inhibition of cyclooxygenases 1 and 2. The exact reason for aspirin causing Reye’s
syndrome has not been discovered.
(A) Inhibiting phospholipase A2 is the mechanism of action of glucocorticoids, which does not
cause Reye’s syndrome. (C) The mechanism of action of NSAIDs is the reversible inhibition of
cyclooxygenases 1 and 2. NSAIDs do not cause Reye’s syndrome. (D) The mechanism of action of
celecoxib is the reversible inhibition of cyclooxygenase 2. Celecoxib does not cause Reye’s
syndrome. (E) The diphenhydramine’s action mechanism is the reversible inhibition of H1
histamine receptors. Antihistamines do not cause Reye’s syndrome

43. Would this be the best course of treatment for this patient?

A. α-Galactosidase A
B. High-dose glucocorticoids
C. Indomethacin
D. Sacrosidase

Rationale Anderson-Fabry disease can be treated successfully with α-galactosidase

44. Most common cause for Pediatric dose error than adult:
A- Most medication is weight dependant
B- Most medication is liquid
C- Greater surface area

45. Mother went to the pharmacist her baby 1 month oldsuffer from mild fever:
A- Paracetamol nasal metered dose 15 mg/kg
B- Ibuprofen syrup dose 12 mg /kg

46. Newborn with fever, dyspnea, and jaundiceHe can take:

A- Ampicillin and gentamicin


B- Ceftazidime and gentamicin
C- Ampicillin and ceftriaxone

47. Babie infected with (I think with tuberculosis):


A- The family should take rifampicin
B- Should take rifampicin within
C- Rifampicin has no role in the protection

48. Which of the following is an example of a medication that does not require
child-resistant packaging?
Page 345

A- Birth control pills


B- Lisinopril
C- Hydrochlorothiazide
D- Atorvastatin
Rationale Birth control pills do not require child-resistant packaging.

49. A 12-year-old male with Type-1 diabetes mellitus takes an insulin preparation before his
meals to avoid hyperglycemia. Once glucose molecules enter his cells, they are
phosphorylated. Phosphorylated glucose is unable to leave the cell. Two enzymes that can
phosphorylate glucose are glucokinase and hexokinase. Glucokinase is found primarily in
the liver and has a lower affinity for glucose but a greater capacity to phosphorylate glucose
than hexokinase, which is found in other body tissues. Compared to glucokinase, which of
the following differences in enzyme kinetics will hexokinase display?

A- Higher Km and higher Vmax


B- Higher Km and lower Vmax
C- Lower Km and higher Vmax
D- Lower Km and lower Vmax

Rationale Lower Km and lower Vmax. Hexokinase has a lower Km and lower Vmax than
glucokinase. Km refers to the concentration of substrate (glucose, in this case) needed for the
reaction rate to reach 1/2 Vmax. Glucokinase and hexokinase both carry out the same reaction on
glucose; but because hexokinase has a higher affinity, a lower concentration of glucose is needed
for it to reach 1/2 Vmax. However, glucokinase has a higher capacity. At high concentrations of
glucose, glucokinase actually works faster than hexokinase so glucokinase has a higher Vmax. (A)
Hexokinase has a lower Km and a lower Vmax than glucokinase. (B) Hexokinase has a lower Km
and a lower Vmax than glucokinase. (C) Hexokinase has a lower Km and a lower Vmax than
glucokinase. (E) Hexokinase has a lower Km and a lower Vmax than glucokinase.

50. A 13-year-old girl with abnormal menses presents to her primary care physician for
treatment. She has a history of inguinal hernia repairs in the past. Her physician begins
therapy with oral micronized estradiol in order to regulate menses. Which of the following
is true regarding this therapy?

A- Limited bioavailability
B- Limited first-pass metabolism
C- Minimally available
D- Nephrotoxicity at low doses
E- Neuromuscular blockade likely
Page 346

Rationale Limited first-pass metabolism. These agents and their esterified or conjugated derivatives
are readily absorbed through the gastrointestinal tract, skin, and mucous membranes. Taken orally,
estradiol is rapidly metabolized (and partially inactivated) by the microsomal enzymes of the liver.
Micronized estradiol is available and has better bioavailability. Although there is some first-pass
metabolism, it is not sufficient to lessen the effectiveness when taken orally. (A) Micronized
estradiol has better bioavailability. (C) These agents are maximally available. (D) Nephrotoxicity
occurs at high doses. (E) Neuromuscular blockade is highly unlikely.

51. A 6-year-old boy returns home from his last day of school before Thanksgiving break.
Over the break, he develops a cough, stuffy nose, headache, and fever. His mother
administers a cough syrup containing guaifenesin. Which of the following effects is likely
caused by guaifenesin?

A- Cough becomes more productive


B- Cough stops altogether
C- Fever diminishes
D- Headache resolves
E- Headache worsens
Rationale Cough becomes more productive. This patient’s presentation is consistent with an upper
Respiratory tract (URT) viral infection. Cough syrups are often preparations containing multiple
drugs that each target a different symptom commonly associated with URT infections such as
cough, headache, fever, and nasal congestion. Guaifenesin is an expectorant; its effect is to thin
secreted mucus to make it more easily removed by ciliary action and coughing. In this way,
guaifenesin will initially make a cough become more productive. (B) Cough syrups often contain an
opioid derivative such as dextromethorphan. It appears to work somewhat differently than other
opioids but works just as well for cough suppression. (C) Acetaminophen is a common ingredient in
cough syrup preparations. Acetaminophen relieves fever apparently by inhibiting cyclooxygenase
enzymes in the CNS to prevent production of profebrile prostaglandins. (D) Acetaminophen is a
common ingredient in cough syrup preparations. Acetaminophen relieves pain by inhibiting
cyclooxygenase enzymes in the CNS to prevent the production of pain-stimulating prostaglandins.
(E) Guaifenesin is an expectorant. Headache is a rare side effect; and although guaifenesin does not
directly lessen the pain of a headache, it will probably not make the pain worse.

52. The parents of an 8-year-old boy complained of his inability to concentrate and focus and
his impulsiveness. Following multiple visits and tests, he is given a diagnosis and
prescribed a mixture of amphetamine salts. The parents are initially pleased with the
results but soon remark that they find him excessively preoccupied with tasks and
“zombielike.” How can the physician best address this concern?
Page 347

A- Apologies for making a hasty and incorrect diagnosis

B- Change to another drug of this class

C- Decrease the dose

D- Explain that this is an unfortunate but inevitable side effect

E- Increase the dose


Rationale Decrease the dose. This boy’s presentation is consistent with a diagnosis of ADHD. There
is nothing wrong with the physician’s choice of treatment of amphetamine salts. The reaction
exhibited by this patient is called supra normalization and reflects a higher-than-necessary dose.
Decreasing the amphetamine salts dose should correct this effect. (A) The physician’s diagnosis is
most likely correct, and his treatment choice is appropriate. ADHD can be efficaciously treated
with amphetamine salts. (B) Because the boy’s behavior improved with administration of
amphetamine salts, switching to another drug is not necessary. The issue can likely be corrected
simply by reducing his dose. (D) This side effect is not inevitable. Simply decreasing the dose will
probably solve the problem while still treating the ADHD. (E) This patient is exhibiting
supranormalization, which occurs when the amphetamine salts dose is too high. The way to
address this problem is to decrease the dose, not increase it.
Page 348

Antidotes

Poison/Drug Antidote
Anticholinergics (Atropine, Benztropine  Physostigmine
Xss CNS depression from Diazepam
Isoniazid (INH) ◆
v
:  Vitamin B6 (Pyridoxine)
Valproic acid  L-carnitine
Methotrexate (MTX)  Leucovorin
Methemoglobinemia inducing agents: ◆
v
:  Methylene blue
Nitrites, Nitrates
Digoxin :
v
◆  Digibind, Digifab
Beta-blockers  Glucagon
Calcium channel blockers  Calcium - Glucagon
Opioids (Morphine, Heroin, Codeine, Fentanyl  Naloxone – Naltrexone - Nalmefene
Benzodiazepine (Diazepam, Zolpidem  Flumazenil
TCA (Imipramine, Amitriptyline)  Sodium bicarbonate
Warfarin  Vitamin K1 (Phyto-menadione)
Heparin  Protamine sulfate
Dabigatran  Idarucizumab
Thrombolytics  Aminocaproic acid
(Streptokinase, urokinase, alteplase, Reteplase)  Tranexamic acid
Nerve gases  Atropine followed by Pralidoxime (2-PAM)
Organophosphorus insecticides
Carbamate insecticides  Atropine
Cyanide gas  Sodium thiosulfate – Amyl nitrite – Na nitrite
Chlorine gas  Sodium bicarbonate
Carbon monoxide  100% O2
Hydrogen sulfide  Sodium nitrite
Methanol  Ethanol – Fomepizole – Folic acid - Leucovorin
Ethylene glycol  Ethanol – Fomepizole - Pyridoxine
Iron (Fe) ◆
:
v  Deferoxamine
Lead (Pb)  D-Penicillamine – CaEDTA – Dimercaprol – DMSA - BAL
Arsenic  BAL
Thallium  Prussian blue
Mercury  BAL – DMSA
Cupper  D-penicillamine
Lithium (Li)  Sodium bicarbonate - Polystyrene sulfonate
Paracetamol/Acetaminophen ◆ :
v  N-Acetyl cysteine (NAC)
Aspirin (Salicylic acid)  Alkalinization (NaHCO3)
Sulfonylurea  Octreotide
Insulin  Dextrose 50%
Crotaline snake bites (e.g. Rattlesnakes)  Crotalidae anti-venom (CroFab)
Black widow spider venom  Latrodectus anti-venom
Brown recluse spider bite  Loxosceles anti-venom
Scorpion sting  Scorpion anti-venom
Clostridium botulinum  Botulinum anti-toxin
Sevelamer controls high blood levels of phosphorus in people with chronic kidney disease who are on dialysis.
ARIF PHARMA CLINIC Copyright@arifpharma.com Www.arifpharma.com
Page 349

Important Mechanism Of Action

1 Digoxin
inhibits sodium-potassium ATPaseNa+/K+
ATPase inhibitor
2 Donepezil
reversible acetylcholinesterase (ache) inhibitor

3 Tyramine

inhibits enterohepatic reuptake of intestinal bile salts and thereby increases


the fecal loss of bile salt-bound low-density lipoprotein cholesterol
4 Minoxidil
Vasodilator act by producing relaxation of vascular smooth muscle, primarily in arteriesand
arterioles
5 Amantadine
Anti-Parkinson Agent inhibiting the N-methyl-D- aspartate (NMDA) type of glutamate receptors.

6 Chylomicron
Transfer lipids or fat from the intestine to peripheral tissues.

7 Cyproheptadine
Antihistamine and serotonin antagonist

8 clonidine
Stimulate alpha and decrease sympathetic responses

9 Dantrolene
Dantrolene, a direct-acting skeletal muscle relaxant, inhibits the release of Ca ions from the
sarcoplasmic reticulum leading to decreased response to an action potential and decreased
muscle contraction.
10 Atropine
is an anticholinergic agent (antimuscarinic agent) which competitively blocks
the muscarinic receptors in peripheral tissues
11 TCAs
The TCAs block norepinephrine and serotonin reuptake into the presynaptic neuron
Blocking of receptors: TCAs also block serotonergic, α-adrenergic, histaminic, and
muscarinic receptors
12 Duloxetine
SSI

13 Fingolimod
Is used for the treatment of multiple sclerosis (MS) it’s an oral drug that alters lymphocyte
migration resulting in fewer lymphocytes in the CNS
14 Benzodiazepine
Page 350

15 Aspirin
Aspirin is a weak organic acid that irreversibly acetylates (and, thus, inactivates)
Cyclooxygenase
16 abciximab
abciximab inhibits the GP IIb/IIIa receptor complex. By binding to GP IIb/IIIa, abciximab
blocks the binding of fibrinogen and von Willebrand factor and, consequently, aggregation
does not occur

17 L-dopa
Levodopa increases dopamine levels in the brain leading to the stimulation of dopamine
receptors.

18 Clomifene
Clomifene is a nonsteroidal compound that has both oestrogenic and anti-oestrogenic
effects. It stimulates ovulation by inhibiting the negative feedback effect of estrogens at
receptor sites in the hypothalamus and pituitary, thereby increasing hypothalamic GnRH
secretion w/ subsequent release of pituitary FSH and LH

19 Coenzyme Q 10
Coenzyme Q10 (CoQ10) is an antioxidant that your body produces naturally. Your cells use
CoQ10 for growth and maintenance.

20 carbon monoxide
Carbon monoxide poisoning occurs when carbon monoxide builds up in your bloodstream.
When too much carbon monoxide is in the air, your body replaces the oxygen in your red
blood cells with carbon monoxide

21 clopidogrel
inhibit ADP that causes platelet aggregation

22 Cromolyn
mast cell stabilizer Released based on inflammatory substance

23 Salbutamol
salbutamol is a short-acting, selective beta2-adrenergic receptor agonist

24 Calcium channel blockers


Calcium channel antagonists block the inward movement of calcium by binding to L-type
calcium channels in the heart and in the smooth muscle of the coronary and peripheral
arteriolar vasculature. This causes vascular smooth muscle to relax, dilating mainly
arterioles. Calcium channel blockers do not dilate veins.

25 Minoxidil
Vasodilator

26 Zyrtec
h1_blocker (Zyrtec (cetirizine)

27 bisacodyl
Stimulation of enteric nerves causes colonic contraction
Page 351

28 bulk laxatives
Add water and bulky to stool and soften stool like a jelly

29 cyclosporin?
immunosuppressant after organ transplantation to reduce the possibility of rejection of new organ
by theimmune system

30 both of cimetidine(h2blochker) and spironolactone?


Anti-androgenic effect

31 chloroquine?
chloroquine binds to heme and prevents its polymerization to hemozoin

32 benzodiazepine action?
Inhances GABA inhibitory effect (gamma-Aminobutyric acid GABA agonist)

33 Diltiazem action?
CCB calcium channel blocker

34 MOA of. spironolactone (Adverse effect of Spironolactone)


Hyperkalemia

35 MOA of beta - carotene?


Precursor for vit A (retinol) Antioxidant

36 MOA of antiarrhythmic Class 1A (quinidine)?


decrease rate of phase o depolarization

37 MOA of cromolyn (cromolyn sodium )?


mast cell stabilizers

38 Atropine is?
Muscarinic antagonist

39 Mechanism of action of Dopamine?


Dopamine b1 agonist

40 The drug is most commonly used in the selective COX2 inhibitors?


Celecoxib

41 Anti-inflammatory MOC?
decrease prostaglandin

42 Clonidine MOA in the treatment of Hypertension:


Alpha2 Agoinst decreases sympathy outflow

43 Allopurinol?

Anti-inflammatory
44 Amphetamine pharmacological action?
Indirect-acting adrenergic agonist.
Page 352

45 Calcium-channel blocker’s mode of action?


decrease the inward calcium to cells

46 About minoxidil?

dilate arteries only


47 dantrolene?
cause skeletal muscle relaxant by binding to the ryanodine receptor decreasing intracellular calcium
concentration

48 Has 5-HT antagonist and H1 antihistamine effects?


Cyproheptadine

49 phentolamine?
alpha antagonist

50 fluorouracil?
is Pyrimidine derivative

51 loperamide?
opioid agonist anti-diarrheal

52 clomiphene?
inhibit negative feedback of estrogen

53 amphetamine?
Indirect-acting adrenergic agonist.

54 Cephalosporin act?
Inhibitors of cell wall synthesis

55 digoxin?
+ve inotropic effect

56 prazosin?
is a postsynaptic alpha1 blocker

57 aminoglycosides?
protein synthesis inhibitor

58 Rofecoxib is?
selective cox2 inhibitor

59 Clonidine and methyldopa are?


alpha 2 agonists

60 Does loperamide stimulate?


MU receptors MI

61 Atenolol is?

a
selective beta 1 blocker
Page 353

62 Does morphine act on?


mu receptors

63 clomiphene?
Inhibit negative feedback of estrogen

64 Aminoglycosides?
protein synthesis inhibitor

65 bethanechol?
Selective muscarinic agoinst

66 Flutamide?
Used in t treatment of prostate cancer which acts as Nonsteroidal
antiandrogen that inhibits androgen uptake and/or inhibits binding of
androgen in target tissues

67 domperidone & doxazocin


Domperidone has peripheral dopamine receptor-blocking properties (dopamine antagonist) and
does not readily cross the blood-brain barrier.
Doxazosin Competitively inhibits postsynaptic alpha1-adrenergic
receptors

68 Benzodiazepine?
GABA agonist

69 Statin
Decrease cholesterol by HMG CoA reductase enzyme inhibitor

70 Fluoxetine
selective serotonin reuptake inhibitor (SSRI)

71 Benzodiazepine?
Anxiolytic

72 Diazepam action is
increase glutamate secretion

73 Labetalol
is a selective alpha-1 and non-selective beta-adrenergic blocker

74 (Isoprenaline)
Non-selective β stimulant

75 Amlodipine
Calcium channel blockers (CCBs)

76 Phenylephrine act on
alpha agonist
77 Amantadine
antiviral and antiparkinsonian
Page 354

78 Ranitidine mechanism of action:


H2 antagonist

79 Cyproheptadine acts as:


Antihistamine
80 The mechanism of action of TCAs is
decrease reuptake of amine at synaptic

81 Nitroglycerin
coronary vasodilation

82 dantrolene
Postsynaptic muscle relaxant inhibitors ca²⁺ ions release

83 Fluoroquinolone
Fluoroquinolones act by inhibiting two enzymes involved in bacterial DNA synthesis, both of which are
DNA topoisomerases that human cells lack and that is essential for bacterial DNA replication,
thereby enabling these agents to be both

84 Cromolyn
Cromolyn is a mast cell stabilizer that inhibits the Type I immediate hypersensitivity reaction by
preventing the antigen-stimulated release of histamine. also prevents the release of leukotrienes
and inhibits eosinophil chemotaxis

85 Asprin Antiplatlet Mechanism?


irreversibly inhibits prostaglandin H synthase (cyclooxygenase-1) in platelets and megakaryocytes, and
thereby blocks the formation of thromboxane A2 (TXA2; a potent vasoconstrictor and platelet
aggregate).

86 Abciximab
Glycoprotein IIb/IIIa receptor antagonist

Best Of Luck
Important Drugs MOA
arifpharma.com

Drx Arif Khan


Page 355

All The Best

Intrinsic Sympathomimetic Activity Drugs & Abbreviation


Sing
Important Abbreviation Sing

Intrinsic sympathomimetic activity DRUGS:

✓ Pin= Pindolol
✓ Ox= Oxprenololac= Acebutelol
✓ Electromagnetic rays include (radio, micro, ultraviolet, x-ray & Gama)
✓ Terbutaline is used as …>> Antiasthmatic
✓ Freeze drying is done by … >> Sublimation.
✓ Converting big fragments into small fragments is … >>Reduction
✓ Q.d.s = Q.i.d = Four Times Per Day
✓ P.r = for the rectum
✓ qqh means every 4 hours
✓ Bretylium class3 anti antiarrhythmic is an adrenergic neuronalblocking agent
✓ Beftazidime is 3rd generation cephalosporin
✓ Amantadine is dopamine agonist
✓ Diphtheria is An upper respiratory tract illness (makes toughpharyngeal
membrane) caused by Corynebacterium diphtheria
✓ Indamide use ------ Sulphonyl urea "Diabetic treatment"indapamide
✓ (Natrilix) -------------------------------- Thiazide diuretic
✓ Drug used in absence seizure (petit mal seizures)
ethosuximide
✓ ANGIOTEC is the trade name of Enalapril
✓ The parameter describing dissociation in solution: PKA
✓ Reserpine mechanism: deplete catecholamines fromsympathetic nerve
endings
✓ MAOI work through increased availability of monoamineneurotransmitter
✓ All the following side effects of atropine:
urine retention, constipation, blurred vision, dyspnea
✓ Mechanism of action hydrochlorothiazide: inhibition of NA reabsorption in the
distal tubules causing increased excretionof sodium and water as well as potassium
and hydrogen ions.
✓ Phase 1 of Arrhythmia: Transient early repolarization, due topotassium efflux
✓ Phase 2 of Arrhythmia: Plateau Phase Calcium influx continues and is
balanced to some degree by potassiumefflux
✓ Main side effect of nitrites is…............................. Headache
✓ Volatile liquid drug is given as inhaler…Amyl nitrite
✓ About tonic-clonic seizure: Unconsciousness,
involuntary movement seizures usually involve (tonic
phase) muscle rigidity, followed by violent muscle
contractions (clonic phase), and loss of alertness
(consciousness) --- it’s also called grand-mal epilepsy
✓ Phentolamine (Rogitine) is a nonselective alpha-
adrenergicantagonist.

ARIF PHARMA CLINIC COPYRIGHT@ARIFPHARMA.COM


Page 356

❖ Considerable variation occurs in the use of capitalization, italicization,


and punctuation in abbreviations. The following list shows the
abbreviations that are not often encountered by pharmacists:

A, aa., or aa = of eachdil =
dilute
Ad = to, up to
D.C., dc, or disc. = Discontinue
A.d. = Right ear
A.s. = left ear
A.u. = Each ear, both ears
O.d. = right eye
O.I. or o.s. = left eye
O.u. = each eye, both eyedisp. =
Dispense
Ad lib = At pleasure freely div. =
Divide, to be divided
D.t.d. = give of such dosesaq. =
water
DW= Distilled water D5W=
Dextrose% in waterasa = aspirin
E.m.p. = As directedet =
nd
Ex aq. = in water
BP = British Pharmacopeia BSA =
body surface area c.or c = with
Ft = Make
Amp. = Ampoule
Cap or INJ = Capsule OR
Injection
IM = Intramuscular
IV = Intervenes Pulv.
= powder
Gtt or gtt = Drop drops elix =
elixir
Comp= Compound, compoundedg or GM
=gram
Gr or gr = Graingal =
gallon
Cc or cc. cubic centimeteroz. =
ounce
M2 or m2 = Square meter
μl or μL = Mmicrolitre
Mcg, mcg, or μg = Microgram
I or L = liter Lb
= pound

mEq = Milliequivalentmg =
Milligram

ARIF PHARMA CLINIC COPYRIGHT@ARIFPHARMA.COM


Page 357

ml or Ml = Milliliterfl or
fld = fluid
Fl oz = fluid ouncedl or dL
= decilitercp = chest pain
D.A.W. described as writtenGI =
Gastrointestinal
Non-rep. = do not repeatH =
hypodermic
NPO = Nothing by mouthN.S., NS
= normal saline
½ NS = half-strength normal salineO = pint
IVP = Intravenous push
IVPB = Intravenous piggybackOTC = over
the counter
K = POTASSIUM
PDR = Physicians, Desk Reference
μ = Greek mu
M = mix
Ppt = precipitated
P.o. = by mouth Pr =
for rectum Pv = Vaginal
use
Prn or p.r.n. = As needed Pt. =
pint
N & V = Nausea & vomiting
N.F. = National formulary
Ante = Before
A.c. = Before meals
P.c. = After mealsh or
Hr = Hour
Q. = Every
Q.d. = Every day
Q.h. = Every hour
Q.4 hr. = Every 4 hours
B.i.d. = Twice daily
A.m. = Morning
Noct. = Night, in the night
H.s. = At bed time

Copyright@arifpharma.com

Drx Arif Khan ARIF PHARMA CLINIC


ARIF PHARMA CLINIC COPYRIGHT@ARIFPHARMA.COM
Page 358

DHA Important MCQ (Set-1)

Q.1 An Elderly patient suffering from depression was given St. John’s wort.
Which of the following drugs if administered concomitantly will have a
clinically significant interaction?

A. Simvastatin
B. Salbutamol
C. *Sertraline
D. Gliclazide

Q.2 The high prevalence of osteoporosis in the elderly results in which


incidence the in geriatric population?

A. Hyperkalemia
B. Bone fractures
C. * Urinary retention
D. Memory impairment

Q.3 An elderly man presented to the Emergency Room with dizziness, altered
consciousness, respiratory depression, and rhabdomyolysis. He has recently
been prescribed medication for his inability to sleep. Which is the most likely
drug toxicity?

A. * Benzodiazepine
B. Acetaminophen
C. Salicylate
D. Opioid

Q. 4 A man weighing 78 kg is to be administered one Gram of vancomycin


intravenously every 12 hours for seven days. The half-life of vancomycin is
approximately eight hours and volume of distribution is approximately 1
L/kg.

A. 5
B. 7
C. *13
D. 19

Arif Pharma clinic Copyright@arifpharma.com arifpharma.com


Page 359

A. 1.75 L/kg
B. * 2.25 L/hrs
C. 3.55 L/hrs
D. 4.25 L/hrs

Q.6 A 45-year-old patient of rheumatoid arthritis has been prescribed


naproxen 250 mg enteric-coated tablets twice daily. The elimination half-
life of naproxen is 12 hours. How much time it would take to reach 95% of
steady plasma concentration?

A. 22.3 hrs
B. 38.7 hrs
C. * 51.6 hrs
D. 66.5 hrs

Q. 7 A 35 year- old individual is receiving 20 mg of propranolol hydrochloride


four times daily as a prophylaxis to migraine attacks. If the drug has only 25%
bioavailability due to extensive first-pass effect. What amount of the drug
reaches the systemic circulation after each dosing?

A. 2.5 mg
B. * 5.0 mg
C. 7.5 mg
D. 10 mg

Q. 8 An NSAID1 has an oral dose of 200 mg, bioavailability of 80%, and an


elimination half-life of four hours. How much drug will be remaining in the
body after 12 hours?

A. * 20 mg
B. 40 mg
C. 60 mg
D. 80 mg

Q. 9 Which one of the medications can be dispensed without a prescription?

A. Insulin vials
B. Metoprolol 50 mg tablet
C. * Ibuprofen 400 mg tablet

Arif Pharma clinic Copyright@arifpharma.com arifpharma.com


Page 360

D. Aripiprazole 10 mg tablet

Q. 10 What is the indication of pseudoephedrine?

A. * Nasal congestion
B. Pyrexia
C. Allergy
D. Pain

Q. 11. What is the appropriate recommendation for diclofenac oral tablet


use?

A. * With food or milk to decrease GI upset


B. Take with fatty meals to enhance absorption
C. Two hours after meals to avoid delay in absorption
D. Four hours before meals to prevent drug-food interaction

Q. 12 In which one of the following conditions estrogen-containing


contraceptives contraindicated?

A. Depression
B. Uncomplicated diabetes
C. Controlled hypertension
D. * 40-year-old women who smoke

Q. 13 A 31-year-old three months pregnant woman, develops a urinary tract


infection. What is the most appropriate antimicrobial therapy?

A. Tetracycline
B. Ciprofloxacin
C. * Nitrofurantoin
D. Sulfamethoxazole and trimethoprim

Q. 14 Which of the following disease states warrant dose adjustment?

A. Colon cancer
B. * Renal failure
C. Hepatic cirrhosis
D. No dose adjustments are needed for voriconazole

Q. 15 Which of the following substance is classified as a weak electrolyte?

Arif Pharma clinic Copyright@arifpharma.com arifpharma.com


Page 361

A. Urea
B. Dextrose
C. Creatinine
D. * Calcium chloride

Q. 16 Which of the following bases can be used to prepare rectal


suppositories?

A. Sucrose
B. *Cocoa butter
C. Methylcellulose
D. Propylene glycol

Q. 17 Which of the following replacement fluids is an isotonic solution?

A. 3% saline
B. 5% dextrose
C. * Ringers lactate
D. ½ strength normal saline

Q. 18 The pharmacy received a call from the intensive care unit nurse who
has a patient receiving an intravenous solution containing amoxicillin. The
doctor containing ordered morphine to be injected into the Y site of the
intravenous solution every 4 hours. The nurse wants to know if these drugs
will be compatible at the y site. From which references the pharmacist
should provide the required information?

A. Red book
B. Merck manual
C. Review of natural products
D. * Handbook on injectable drugs

Q. 19 Which of the following parenteral anticoagulants require routine


monitoring of coagulation lab parameters?

A. * UFH intravenously
B. UFH subcutaneously
C. Enoxaparin subcutaneously
D. Fondaparinux subcutaneously

Arif Pharma clinic Copyright@arifpharma.com arifpharma.com


Page 362

Q. 20 What is the possible serious side effect that is common amongst all
anticoagulant agents?

A. * Major bleeding
B. Hypokalemia
C. Liver dysfunction
D. Renal dysfunction

Q. 21 Stroke or previous TIA is a contraindication to the use of which


antiplatelet agent?

A. Aspirin
B. * Prasugrel
C. Ticagrelor
D. Clopidogrel

Q. 22 A 61-year-old woman with deep vein thrombosis is on heparin 1200


units/hour. The nurse asks for a heparin dosing recommendation after
receiving the hematology test this morning (see image and lab results).

Test results from normal values

APTT 240 30-40 sec, INT 1.1 0.8-1.2

Prothrombin time 12 10-13sec

Q. 23 What is the best recommendation based on hospital heparin protocol?

A. * Keep the same dose


B. Decrease the infusion rate to 1100 units/hour
C. Hold the infusion for 60 minutes then resume with 1000 units/hour
D. Hold the infusion for 60 minutes then resume with 900 units/hour

Q. 24 Following is the prescription for a six-year-old boy:

Rx, Levothyroxine 12 micrograms, Syrup tolu 5 ml, Syrup wild cherry qs ad 5


ml, d. t. p. how many micrograms of levothyroxine should be used to
dispense this prescription?

A. 12 mcg
B. 24 mcg

Arif Pharma clinic Copyright@arifpharma.com arifpharma.com


Page 363

C. * 28. 8 cg
D. 34.8 mcg

Q. 25 An ophthalmic drop of drug x has a 0.1% concentration.

Express this concentration of drug x in parts per million.

A. 10 ppm
B. 100 ppm
C. * 1000 ppm
D. 10000 ppm

Q. 26 A patient, who is on warfarin therapy, is using garlic supplements, is


using garlic supplements In order to see any interaction, which reference
would be most appropriate?

A. Red book
B. Merck manual
C. *Review of natural products
D. Handbook on injectable drugs

Q. 27 Which of the following drugs should be avoided in patients with


reduced ejection fraction heart failure?

A. Hydralazine
B. Paracetamol
C. *Pioglitazone
D. Spironolacton

Q. 28 Which of the following P2Y12 receptor inhibitors exhibit the most


variable anti-platelet effects?

A. Prasugrel
B. Ticagrelor
C. Ticlopidine
D. * Clopidogrel

Q. 29 Which of the following is a high-intensity statin therapy?

A. Lovastatin 40 mg daily
B. Pravastatin 40 mg daily

Arif Pharma clinic Copyright@arifpharma.com arifpharma.com


Page 364

C. Simvastatin 40 mg daily
D. *Atorvastatin 40 mg daily

Q. 30 Which NSAID permanently inactivates thrombin-A2 synthesis in


platelets?

A. *Aspirin
B. Naproxen
C. Celecoxib
D. Ibuprofen

Q. 31 How many grams of 5% diclofenac cream should be mixed with 100 g of


1% cream to make a 2.5% diclofenac cream?

A. 30 g
B. *60 g
C. 990 g
D. 120 g

Q. 32 A 25-year-old woman, on thyroxine for hypothyroidism becomes


pregnant. She now complains of constant fatigued. What is the most proper
recommendation?

A. Increase the dietary iodine


B. Double the dose of thyroxine
C. Do nothing, fatigue is normal during pregnancy
D. * Adjust thyroxine dose based on serum TSH during the first trimester

Q. 33 Which of the following anti-anginal agents reduce myocardial ischemia


by dilating coronary arteries?

A. Atenolol
B. Amlodipine
C. Hydrochlorothiazide
D. *Isosorbidedinitrate

Q. 34 Which diuretic listed below would have the greatest blood pressure
lowering effect when given in patients with an egfr below 60 ml/min?

A. * Chlorthalidone 25 mg daily

Arif Pharma clinic Copyright@arifpharma.com arifpharma.com


Page 365

B. Spironolactone 25 mg daily
C. Triamterene 25 mg daily
D. Furosemide 40 mg daily

Q. 35 What is the best time to take simvastatin for its maximal effect?

A.* At bedtime

B. before lunch

C. Before breakfast

D. Hour after breakfast

Q. 36 The pharmacy received the following prescription for to dispense:

Rx

Menthol 0.6%

Simple syrup as ad 100 ml

How many grams of menthol should be used to prepare this prescription?

A. 6
B. 60
C. * 0.6
D. 1.2

Q. 37 The pharmacy has received a prescription for 100ml of 10% w/v


calcium chloride solution. How many milliosmoles of calcium chloride are
needed to prepare this prescription (calcium chloride molecular weight =
147)?

A. 30
B. 180
C. * 300
D. 500

Q. 38 Which of the following side effect is very common with dihydropyridine


calcium-channels blockers?

A. Bradycardia

Arif Pharma clinic Copyright@arifpharma.com arifpharma.com


Page 366

B. Pericarditis
C. Pulmonary edema
D. *Swelling of the ankles

Q. 39 A 58-year-old man is admitted to the hospital for DVT treatment. The


physician is initiating unfractionated heparin and warfarin therapy. The
physician is asking the pharmacist about the duration of anticoagulants
overlapping in DVT treatment.

What should be the pharmacist’s recommendation?

A. Minimum of 2 days of heparin and INR greater than 1.5


B. * Minimum of 5 days of heparin and INR greater than 2.0
C. Minimum of 7 days of heparin and INR greater than 2.5
D. Minimum of 10 days of heparin and INR greater than 3.0

Q. 40 Which of the following could lead to decreased PT/INR in a patient on


warfarin therapy?

A. Decrease in consumption of dietary vitamin k


B. *Initiation of carbamazepine or rifampin
C. Hypermetabolic states like fever, illness, hyperthyroidism
D. Initiation of amiodarone, sulfamethoxazole or metronidazole

Q. 41 Which of the following points should be emphasized when counseling a


patient on the proper use of sublingual nitroglycerin tables?

A. Chew and swallow the tablet


B. Stand up before taking a dose since dizziness can occur
C. * Store tablets in the original glass container and keep them with you at
all times
D. If chest pain does not improve at all within 2 minutes of taking a dose,
go to the emergency department immediately

Q. 42 A 24-year-old man is diagnosed with psychosis. He agrees to start drug


therapy, however, he is asking for the least sedating medication.

A. Clozapine
B. Quetiapine
C. Olanzapine

Arif Pharma clinic Copyright@arifpharma.com arifpharma.com


Page 367

D. * Risperidone

Q. 43 Which of the following medications is most appropriate to use in a 71-


year-old man to treat extrapyramidal symptoms?

A. *Benztropine
B. Haloperidol
C. Donepezil
D. Ibuprofen

Q. 44 What is the maximum day supply a pharmacist can dispense for a


diazepam prescription?

A. 7 days
B. *14 days
C. 21 days
D. 30 days

Q. 45 Which of the following medications can be used for narcotic addiction?

A. Codeine
B. Morphine
C. *Methadone
D. Hydrocodone

Q. 46 Which of the following resources should be considered when


promoting pharmaceutical products in Saudi Arabia?

A. Code of Ethics for Pharmacists


B. Ministry of health pharmacy code
C. Unethical behavior of pharmacists
D. *Saudi Code of pharmaceutical promotional practices in KSA

Q. 47 An order for amoxicillin 500 mg three times daily for 7 days is


prescribed. The pharmacy only stocks amoxicillin 250 mg capsules.

How many capsules the pharmacist should dispense?

A. 21
B. * 42
C. 64

Arif Pharma clinic Copyright@arifpharma.com arifpharma.com


Page 368

D. 80

Q. 48 What is the total maximum daily dose of over-the-counter


paracetamol?

A. 1g
B. 2g
C. 4g
D. 8g

Q. 49 What is the total maximum daily dose of over-the-counter Ibuprofen?

A. 800 mg
B. *1200 mg
C. 2000 mg
D. 3000 mg

Q. 50 Which of the following over-the-counter medications can be dispensed


to a 31-year-old man for mild dry cough?

A. Guaifenesin
B. Phenylephrine
C. Pseudoephedrine
D. Dextromethorphan

Q. 51 Which of the following anti-diabetic medications should be used as a


first-line agent in patients with pre-diabetes?

A. Insulin
B. Glyburide
C. *Metformin
D. Pioglitazone

Q. 52 What is the preferred anticoagulant therapy during hemodialysis


procedure?

A. * Unfractionated heparin
B. Rivaroxaban
C. Enoxaparin
D. Dabigatran

Arif Pharma clinic Copyright@arifpharma.com arifpharma.com


Page 369

Q. 53 Which of the following medications can be used to lower the elevated


serum potassium concentration?

A. Dextrose
B. Sevelamer
C. * Kayexalate
D. Sodium bicarbonate

Q. 54 A 44-year-old man presents to his doctor for a blood pressure check.


His blood pressure reading is 169/95 mm hg. He has been seeing his doctor
several times in the last three months for blood pressure checks and lab
tests. Although his lab tests are fine, the doctor recommended he start blood
pressure-reducing medicines.

Which of the following pharmacotherapy regimens would be most


appropriate?

A. * Amlodipine 5 mg lisinopril 10 mg in one pill, daily


B. Lisinopril 10 mg with atenolol in one pill, twice daily
C. Chlorthalidone 25 mg with spironolactone 25 mg daily
D. Furosemide 40 mg twice daily and doxazosin 2 mg daily

Q. 55 A patient comes to the pharmacy with two cold medications asking if


both can be used together. Drug A contains desloratadine 5 mg and
pseudoephedrine 30 mg per tablet and drug B contains diphenhydramine 25
mg and pseudoephedrine 30 mg per tablet.

What should be the pharmacist’s response?

A. Both can be used together as they have a synergistic effect


B. Both should be avoided as they have unsafe ingredients
C. * Use one of them as they are from the same drug class
D. Use drug A as it has a better ingredient

Q. 56 What is the appropriate mode of taking a chewable aspirin tablet set?

A. Should be chewed
B. Should be chewed
C. * Can be chewed or swallowed
D. Should crush and dissolved in 30 ml of water

Arif Pharma clinic Copyright@arifpharma.com arifpharma.com


Page 370

Q. 57 How many milliliters of 0.9% normal saline can be prepared from 40


grams of NaCl?

A. 1000
B. * 2222
C. 4444
D. 6666

Q. 58 A 42-year-old woman with breast cancer is prescribed vincristine at a


dose of 1.5 mg/meter square. Her weight is 72 kg and her height is 170 cm.

What is the dose the pharmacy should prepare?

A. 2 mg
B. 2.25 mg
C. 2.5 mg
D. * 2.75 mg

Q. 59 A content uniformity test for tablets is endorsed by the United stated


pharmacopeia to ensure which quality?

A. Bioavailability
B. Stability
C. Solubility
D. * Potency

Q. 60 What is the bioavailability (F) of a diclofenac intramuscular injection

A. 25%
B. 50%
C. 75%
D. * Approximately 100%

Q. 61 At which ph aspirin (pka 3.5) will be most soluble?

A. 2
B. 3
C. 5
D. *6

Arif Pharma clinic Copyright@arifpharma.com arifpharma.com


Page 371

Q. 62 A two-year-old boy who weighs 10 kg is prescribed drug x as 1


mg/kg/hour infusion. The pharmacy prepares drug x using a 5 ml vial of 25
mg/ml concentration diluted in normal saline to make a 100 ml solution.

What should be the rate of infusion (in ml/hour)?

A. 2
B. 4
C. 6
D. *8

Q. 63 A 54-year-old man is diagnosed with stage 1 hypertension. His left


ventricular ejection fraction is less than 40%.

What would be the most appropriate drug for him?

A. Amlodipine
B. * Lisinopril
C. Spironolactone
D. Hydrochlorothiazide

Q. 64 A 34-year-old woman presents to the physician with an upper


respiratory infection. Several years ago she experienced an episode of
hypotension and bronchospasm following ampicillin IV administration.

What would be the most appropriate empirical therapy?

A. Cefaclor
B. Nafcillin
C. Amoxicillin
D. * Erythromycin

Q. 65 Which antidepressant drug must be avoided in a 21-year-old female


with a history of seizure disorder?

A. Amitriptyline
B. Citalopram
C. Paroxetine
D. * Bupropion

Arif Pharma clinic Copyright@arifpharma.com arifpharma.com


Page 372

Q. 66 Which of the following therapies is considered therapeutic equivalence


for omeprazole 20 mg daily in a peptic ulcer patient?

A. Esomeprazole 40 mg daily
B. *Pantoprazole 40 mg daily
C. Cimetidine 800 mg daily
D. Ranitidine 300 mg daily

Q. 67 What is the side effect associated with calcium carbonate-containing


antacids?

A. Weight gain
B. * Constipation
C. Hypercalcemia
D. Increase salivation

Q. 68 Which one of the following medications can cause weight loss?

A. * Orlistat
B. Olanzapine
C. Atorvastatin
D. Glibenclamide

Q. 69 A mother presents to the pharmacy requesting an antipyretic for her


one-month-old boy who has a low-grade fever.

A. * Take the boy to the hospital immediately


B. Paracetamol drop dosed at 20 mg/kg
C. Ibuprofen syrup dosed at 12 mg/kg
D. Baby aspirin suppository

Q. 70 How do the proton pump inhibitors exert their pharmacological action?

A. Increases gastrointestinal motility


B. Inhibits epithelial growth factor in the stomach
C. Stimulates histamine-2 receptors in the gastric parietal cells
D. * Inhibiting H/K- adenosine triphosphate in gastric parietal cells

Q. 71 What is the most important ingredient in maternal multivitamin


products?

Arif Pharma clinic Copyright@arifpharma.com arifpharma.com


Page 373

A. Iron
B. Copper
C. * Folic acid
D. Beta-carotene

Q. 72 A patient presents to the pharmacy complaining that a few days after


starting benzoyl peroxide cream and soap the acne got worse. What should
be the pharmacist’s advice?

A. Continue the soap


B. Discontinue the soap
C. * continue the cream and soap
D. Discontinue the cream and soap

Q. 73 A 54-year-old man with reduced ejection fraction heart failure


presented to the pharmacy with mild pain in his knees and requested an OTC
painkiller. Which of the following is the best OTC medication?

A. Naproxen
B. Celecoxib
C. Ibuprofen
D. * Paracetamol

Q. 74 Which of the following is a benefit of the formulary system?

A. Decreased cost
B. * Increased safety
C. Less patient confusion
D. Physician satisfaction

Q. 75 Which of the following laxatives has the slowest onset of action?

A. * Psyllium
B. Glycerin
C. Bisacodyl
D. Mike of magnesia

Q. 76 Why should pharmacists be aware of look-alike, sound-alike


medications during the media medication cement process?

Arif Pharma clinic Copyright@arifpharma.com arifpharma.com


Page 374

A. They have a narrow therapeutic index


B. They have similar therapeutic action
C. * They can be mistaken for one another
D. They are controlled medications and should be dispensed in accordance
with MOH rules and regulations

Q. 77 Which one of these items is a function of the hospital pharmacy and


therapeutics committee?

A. Develops hospital guidelines on hand hygiene


B. Meets with manufacturer representatives to discuss new drugs
C. * Develop policies on the use of medications in the institution
D. Discusses the most recent articles published in the New England Journal
of Medicine and JAMA

Q. 78 Which of the following counseling points is very important to discuss


with patients presenting with constipation?

A. Stay in the bathroom for at least 30 minutes in the morning


B. * Increase water intake throughout the day
C. Decrease fiber intake in your diet
D. Increase fat in your diet

Q. 79 What is the best time to take simvastatin for its maximal effect?

A. * At bedtime
B. Before lunch
C. Before breakfast
D. Hour after breakfast

Q. 80 What is the most common side effect(s) of niacin that should be


mentioned to patients during counseling?

A. Chills
B. Tinnitus
C. Dry cough
D. * Flushing and dyspepsia

Q. 81 Which of the following antiarrhythmic drugs may cause iodine-induced


hypothyroidism?

Arif Pharma clinic Copyright@arifpharma.com arifpharma.com


Page 375

A. Digoxin
B. Quinidine
C. * Amiodarone
D. Propranolol

Q. 82 Which of the following is the most suitable method of drug delivery for
infants?

A. Inhalers
B. Evohaler
C. Turbohalers
D. * Neutralizers

Q. 83 What information should be provided to patients starting on


diphenhydramine?

A. May cause diarrhea


B. * May cause drowsiness
C. Should be taken in the morning
D. Should be taken with at least 200 ml of orange juice

Q. 84 A 65-year-old man with a history of hypertension, hyperlipidemia,


erectile dysfunction, and newly diagnosed diabetes mellitus type II, is on
metformin 10 mg daily, atorvastatin 40 mg once daily, and sildenafil 25 mg as
needed. He comes prescription for metoprolol tartrate 25 mg twice daily.

Which one of the following advice should be given?

A. Use of metoprolol tartrate may worsen erectile dysfunction


B. * Metoprolol may mask symptoms of hypoglycemia except for sweating
C. Recovery time from hypoglycemia will be shorter with metoprolol
tartrate.
D. Since metoprolol tartrate is a B-1 selective blocker, it will not mask
symptoms of hypoglycemia
Q. 85 Which of the following NSAIDs has gastrointestinal side effects?
A. Naproxen
B. Meloxicam
C. * Ibuprofen

Arif Pharma clinic Copyright@arifpharma.com arifpharma.com


Page 376

D. Diclofenac
Q. 86 A patient asks the pharmacist to verify the bottle of his medicine. The
label was attached to the wrong medicine and dispensed to the patient by
another pharmacist.
What should be the most appropriate response?
A. Apologize and give the correct medication
B. Apologize and give the correct medication and report the error to
Ministry of the health
C. * Apologize and give the correct medication and report the error to the
medication safety officer
D. Apologize and give the correct medication and ask to report the error to
the hospital administration
Q. 87 Which of the following medications can be used to lower the elevated
serum potassium concentration?
A. Dextrose
B. Sevelamer
C. * Kayexalate
D. Sodium bicarbonate
Q. 88 In patients who require dual antiplatelet therapy, what is the
recommended maintenance dose of aspirin when used in combination with
ticagrelor?
A. 325 mg
B. 162 mg
C. * 81 mg
D. 50 mg
Q. 89 Where can a pharmacist find out the rules and regulations for
pharmacy practice in the Kingdom of Saudi Arabia?
A. * Ministry of Health
B. Saudi Food and Drug Authority
C. Saudi Pharmaceutical Association
D. Saudi Commission for health specialist
Q. 90 How many grams of sodium chloride is present in 500 ml of ½ strength
normal saline?
A. 2.00 g

Arif Pharma clinic Copyright@arifpharma.com arifpharma.com


Page 377

B. * 2.25 g
C. 2.5 g
D. 5.00 g
Q. 91 How many milliliters of amoxicillin 250 mg/5 ml suspension should be
administered to a three-year-old boy amoxicillin?
A. 2
B. 2.2
C. 2.4
D. * 2.6
Q. 91 A 78-year-old man recently had an acute myocardial stent placed. He
has a history of hypertension and hypertriglyceridemia. and asthma. What
would be the recommendation for anti-platelet therapy in addition to
clopidogrel 75 mg daily?
A. * Chewable aspirin 81 mg daily
B. Chewable aspirin 325 mg daily
C. Enteric-coated aspirin 81 mg
D. Enteric-coated aspirin 325 mg daily
Q. 92 An 88-year-old frail woman with osteoarthritis has difficulty in medical
history and has failed numerous aids to help her with the pain.
Which of the following drugs is the best recommendation?
A. * Acetaminophen
B. Glucosamine D
C. Celecoxib
D. Ibuprofen
Q. 93 An 88-year-old frail woman with osteoarthritis has difficulty in walking
and pain. She has no other medical history and has failed numerous aids to
help her with the pain.
Which of the following drugs is the best recommendation?
A. * Acetaminophen
B. Glucosamine D
C. Celecoxib
D. Ibuprofen
Q. 94 A 48-year-old woman with rheumatoid arthritis and on ibuprofen and
ranitidine for three months has been advised to start DMARD therapy.

Arif Pharma clinic Copyright@arifpharma.com arifpharma.com


Page 378

Which of the following would be the recommended DMARD therapy?


A. methotrexate 7.5 mg daily with folic acid 5 mg daily
B. Methotrexate 7.5 mg daily with folic acid 5 mg monthly
C. * Methotrexate 7.5 mg weekly with folic acid 5 mg weekly on any other
day
D. Methotrexate 7.5 mg weekly with folic acid 5 mg weekly on the same
day
Q. 95 A 58-year-old man is diagnosed with deep vein thrombosis. He has a
history of type-2 diabetes, hypertension, and renal impairment with a GFR of
29 ml/min. his weight is 70 kg. What would be the recommended dose of
the low molecular weight heparin?
A. * 1 mg/kg subcutaneously once daily
B. 1.5 mg/kg subcutaneously once daily
C. 1 mg/kg subcutaneously every twelve hours
D. 1.5 mg/kg subcutaneously every twelve hours
Q. 96 A 78-year-old woman with a history of atrial fibrillation and heart
failure is on amiodarone, ACEI, and B- blocker. She has been started on
digoxin to improve symptoms and exercise intolerance. She has normal renal
function. What would be the most appropriate dose of digoxin per day?
A. 250 mcg
B. 125 mcg
C. * 62.5 mcg
D. 3.25 mcg
Q. 97 A 72-year-old man with a history of hypertension and COPD has EKG
findings suggestive of irregular rhythm (see vitals).
Blood pressure 135/80 mmHg
Heart rate 80/min
What is the most appropriate drug to manage his atrial fibrillation?
A. Digoxin
B. * Verapamil
C. Metoprolol
D. Warfarin (target INR of 2-3)
Q. 98 A 52 year-old woman with NYHA class III heart failure was started on
25 mg BID. She is recently complaining of dizziness.

Arif Pharma clinic Copyright@arifpharma.com arifpharma.com


Page 379

Blood pressure 100/70 mmHg heart rate 70/min


What is the best approach for heart failure and her symptoms?
A. Decrease carvedilol dose
B. * Decrease lisinopril dose
C. Stop lisinopril
D. Stop carvedilol
Q. 99 A 26-year-old man is coughing up greenish-yellow sputum. The doctor
thinks it is of viral origin and decides not to prescribe antibiotics. The doctor
also recommends that he sees the pharmacist for some cough mixture.
Which of the following active ingredients would be best in the cough
mixture?
A. Codeine
B. * Guaifenesin
C. Pholcodine
D. Dextromethorphan
Q. 100 A 56-year-old man with a history of epilepsy well controlled with
phenytoin 100 mg TID, has a nasogastric tube placed for a recent stroke. The
nurse requested phenytoin suspension. She is confused with the labeling on
the bottle which reads phenytoin base 30 mg/5 ml and his original tablets
read phenytoin sodium 100 mg.
What would be the daily dose of phenytoin suspension?
A. 100 mg
B. * 270 mg
C. 300 mg
D. 900 mg

Q. 101 An agitated woman calls the pharmacy and tells that she has burned
her hand while retrieving a baking tray from hot oven.
What would be the best first aid?
A. Apply topical NSAID cream
B. Apply antimicrobial wound dressing
C. * Wash the hand with tap water for 20-30 mints
D. Immerse the hand in ice cold water for 30 mints

Arif Pharma clinic Copyright@arifpharma.com arifpharma.com


Page 380

Q. 102 A 59-year-old woman with a history of bronchial asthma has been


diagnosed with open-angle glaucoma. The doctor is thinking about
prescribing a topical eye drop.
What would be the best topical eye drops?
A. Timolol
B. Pilocarpine
C. Brimonidine
D. * Latanoprost
Q. 103 A 75-year-old man has recently been diagnosed with wopen-anglengle
glaucoma. He is prescribed topical eye drops once daily. He presents to the
pharmacy with the prescription for eye drops labeled “Instill one drop daily
in both eyes”.
What would be the best time to administer the eye drops?
A. At night
B. In the morning
C. In the afternoon
D. Any time of the day
Q. 104 A 55-year-old woman presents with recurrent symptoms of
helicobacter pylori infections. She was previously treated with the classical
amoxicillin, clarithromycin, and omeprazole regimen. The clinic is unable to
obtain susceptibility testing for helicobacter pylori but the pattern of local
antimicrobial resistance is well known.
What would be the optimal treatment for her?
A. Amoxicillin (1g), clarithromycin (500 mg), metronidazole (500 mg) plu,
omeprazole (20 mg) BID for 14 days
B. Amoxicillin (1 g), clarithromycin (500 mg) and metronidazole (500 mg),
and omeprazole (40 mg) BID for 14 days
C. * Bismuth subsalicylate (2) and tetracycline (500 mg) QID plus
metronidazole (500 mg) and omeprazole ( 40 mg) BID for days
D. Bismuth subsalicylate (2) and doxycycline (100 mg) QID plus
metronidazole (500 mg) TID and Omeprazole (40 mg) BID for 14 days
Q. 105 A 20-year-old woman with a history of periodic aura is having another
acute attack.
What would be the first choice drug?

Arif Pharma clinic Copyright@arifpharma.com arifpharma.com


Page 381

A. Codeine 30 mg four times daily


B. Ibuprofen 400 mg four times daily
C. * Paracetamol 1000mg four times daily
D. Sumatriptan 50 mg at the onset of migraine
Q. 106 A 25-year-old woman with a history of migraine associated with aura
has taken tablet ergotamine 2 mg sublingual at the first sign of migraine,
then 2 mg every 30 minutes as needed.
What is the maximum number of ergotamine tablets she can use in one
week?
A. 4
B. 5
C. * 6
D. 7
Q. 107 A 21-year-old woman with a history of chronic migraine associated
with aura is asking to for medication to prevent another attack.
What would be the recommended prophylaxis?
A. Sumatriptan 50 mg daily
B. Amitriptyline 50 mg at night
C. * Propranolol 20 mg twice daily
D. Topiramate 25 mg twice daily
Q. 108 A 70-year-old man with a history of osteoarthritis is admitted for knee
replacement surgery. Postoperative follow-up is smooth with no
complications. What would be the recommended prophylaxis for venous
thromboembolism?
A. Aspirin 81 mg PO every 24 hours
B. * Enoxaparin 40 mg SC every 24 hours
C. Clopidogrel 75 mg PO every 24 hours
D. Enoxaparin 4 mg/kg SC every 12 hours
Q. 109 A 60 year-old man with chronic kidney disease for five years,
hypertension, coronary artery disease and diabetes mellitus is due for
routine follow up (see lab result).
Test result normal values
Sodium 140 134-146 mmol/L

Arif Pharma clinic Copyright@arifpharma.com arifpharma.com


Page 382

Potassium 4 3.5-5.1 mmol/L


Chloride 100 97-108 mmol/L
Bicarbonate 22 21-25 mmol/L
Calcium 2.5 2.15-2.62 mmol/L
Calcium ionised1.2 1.1-1.3 mmol/L
Phosphate, inorganic 2.5 0.82-1.51 mmol/L
Magnesium 1.0 0.7-1.2 mmol/L
Copper 1.8 1.72-3.54 mmol/L
Which of the following drugs is most appropriate?
A. * Sevelamer
B. Calcium acetate
C. Calcium carbonate
D. Aluminium carbonate
Q. 110 An 18-year-old boy, weighing 60 kg, is diagnosed with type I diabetes.
He has been prescribed a basal/bolus insulin regime of 0.5 u/kg to mimic
physiological levels of insulin as closely as possible.
Which would be the most appropriate initial basal insulin regimen?
A. 30 units of insulin aspart once daily
B. 30 units of insulin glargine once daily
C. 20 units of insulin Levemir once daily
D. * 15 units of insulin glargine once daily
Q. 111 A 73 year-old woman has a three week history of weight loss, heat
intolerance, palpitations and tachycardia. On examination she is anxious and
has warm extremities. She is not fit for ablative therapy and the doctor
wants to try medications instead (see lab results).
Test result normal values thyroid-stimulating hormone 0.12 0.4-5.0 uU/ml
Thyroxine (T4 free serum)25 8.5-15.2 pmol/L
What would be the best initial choice of drug?
A. Propranolol
B. * Methimazole

Arif Pharma clinic Copyright@arifpharma.com arifpharma.com


Page 383

C. Levothyroxine
D. Ipodate sodium
Q.112 At what time should the routine plasma samples for digoxin
monitoring be drawn?
A. 2 hours post-dose
B. 4 hours post-dose
C. 5 hours post-dose
D. * 6 hours post-dose
Q. 113 What is the best time to do lithium levels after the dose is
administered?
A. 4 hours
B. 6 hours
C. 8 hours
D. * 12 hours
Q. 114 What is the most appropriate time to do peak levels of gentamicin?
A. * One hour post-dose
B. Two hours post-dose
C. Three hours post-dose
D. Four hours post-dose
Q. 115 A 48 year-old man is on phenytoin 100 mg TID for epilepsy. His dose is
adjusted to 150 mg TID.
When will it be appropriate to do plasma phenytoin concentration at the
new steady state?
A. One day
B. Three days
C. * Seven days
D. 30 days
Q. 116 What would be the most suitable time to do the peak levels of
vancomycin?
A. Just after the 4th dose
B. 1 hour after the 1st dose
C. 2 hours after the 2nd dose
D. * 30 minutes after the 3rd dose

Arif Pharma clinic Copyright@arifpharma.com arifpharma.com


Page 384

Q. 117 In which of the following patient categories the gentamicin extended


interval regimes (once daily dosing) is most suitable?
A. Pregnancy
B. Burns patient
C. * Gram-negative infections
D. Enterococcal endocarditis
Q. 118 A 70-year-old man long-term resident in a care home has a long-term
catheter in situ. His routine urine culture shows 10000 colony forming units
(CFU) per mm3. He is asymptomatic.
What would be the best antibiotic recommendation?
A. Cephalexin
B. Trimethoprim
C. Ciprofloxacin
D. * Not indicated
Q. 119 An 18-year-old girl is diagnosed with seizures. She is prescribed low-
dose lamotrigine, the dose to be increased slowly to reach the target dose.
She is asking why she cannot take the full dose immediately.
What is the reason?
A. SIADHI is a dose-related
B. * Rash is a dose-related side effect
C. Leukopenia is a dose related side effect
D. Sever aplastic anemia is a dose related effect
Q.120 1syndrome of inappropriate ADH secretion
A 79 year-old man with alzheimer’s disease has mild to moderate dementia
and is on oral rivastigmine 6 mg twice daily. He is not able to tolerate the
medication due to nausea and vomiting.
What would be the best recommendation
A. Suppositories of prochlorperazine 25 mg 60 mints before each dose of
rivastigmine
B. Discontinue rivastigmine and substitute with galantamine 4 mg twice
daily
C. Metoclopramide 10 mg 30 mints before each dose of rivastigmine
D. * Change to transdermal application of rivastigmine 4.6 mg/24 hours

Arif Pharma clinic Copyright@arifpharma.com arifpharma.com


Page 385

Q. 121 A 69-year-old man routine follow-up is complaining of erectile


dysfunction. He has a history of angina, hypertension, and diabetes mellitus.
His medication, include daily aspirin, meroprolol, metformin and nitrates as
required for his angina pain.
Which of the following recommendations would be best?
A. Avanafil
B. Bupropion
C. Sildenafil
D. * Non- pharmacological treatment
Q. 122 A 28-year-old man who was well-controlled on phenytoin is brought
to the emergency department in a state of status epilepticus.
What would be the best recommendation?
A. Oral clonazepam and phenytoin
B. Intravenous lorazepam and stop phenytoin
C. Rectal diazepam and intravenous lorazepam
D. * Increase the dose of phenytoin and give it intravenously
Q. 123 89-year-old man with benign prostatic hypertrophy (BPH) has been on
a1-blocked for eight months. On his routine follow up he is complaining of
lower urinary tract symptoms of difficulty voiding. The doctor has decided to
start him on a phosphodiesterase-5 inhibitor in addition to the a1-blocked.
Which of the following combination is most appropriate?
A. * Tadalafil 4 hours after tamsulosin
B. Sildenafil 4 hours after tamsulosin
C. Tadalafil at the same time as the tamsulosin
D. Sildenafil at the same time as the tamsulosin
Q. 124 A 28 year-old woman with a history of seizures that is well controlled
on valproic acid is asking for advice regarding her medications. She is
planning to start a family and now taking folic acid. She wants to optimize
the medications before her pregnancy.
What would be the best recommendation?
A. Introduce phenytoin to valproic acid
B. Discontinue valproic acid since immediately
C. Continue on valproic acid since she is well controlled

Arif Pharma clinic Copyright@arifpharma.com arifpharma.com


Page 386

D. * Introduce levetiracetam slowly and titrate the dose of valproic acid to


discontinue before pregnancy
Q. 125 A 15-month-old boy is brought to the emergency department by his
parents with high-grade fever, vomiting, and a non-blanching rash. He is
started immediately on cefotaxime. The cerebral spinal fluid culture reveals
Neisseria meningitides.
What would be the recommendations regarding prophylaxis of his family and
close contacts?
A. His parents should receive rifampicin
B. Antibiotic prophylaxis is not indicated
C. * Patient’s close contacts should receive rifampicin
D. Siblings under the age of 18 should receive rifampicin
Q. 126 A 79-year-old woman has urinary urgency, frequency, and bed wetting
at night. She has a history of osteoarthritis and hyperthyroidism.
Which of the following interventions is most appropriate?
A. Pelvic floor exercises
B. Darifenacin
C. Tolterodine
D. * Oxybutynin
Q. 127 A 63 year-old woman has recently been started on levothyroxine 50
mcg/day. The dose is to be titrated based on T4/TSH and symptoms.
What would be the optimal time to repeat her thyroid function tests?
A. 1 week
B. 2 weeks
C. 3 weeks
D. * 4 weeks
Q. 128 A 28 year-old expectant mother who is diagnosed with Group B B-
haemoltic streptococcus and has received three doses of ampicillin during
labour, delivery is through natural means. Routine examinations of the baby
shows high grade fever, jaundice, and signs of respiratory distress.
What would be the optimal emiric antibiotic treatment
A. Not required
B. Ampicillin and ceftriaxone

Arif Pharma clinic Copyright@arifpharma.com arifpharma.com


Page 387

C. * Ampicillin and gentamicin


D. Ceftazidime and gentamicin
Q.129 An 18 year-old boy with a history of illicit drug use is suspected to have
an overdose of benzodiazepines and is experiencing ataxia, drowsiness and
nystagmus.
What would be the best antidote?
A. Activated charcoal 50 grams
B. * Flumazenil 300 mcg intravenously
C. Naloxone 400 mcg intramuscularly
D. Haloperidol 10 mg intramuscularly
Q. 130 A 79 year-old woman with a history of open angle glaucoma presents
to the pharmacy asking for advice on how long to wait between instilling her
second eye drops. She has been prescribed to different eye drops and she
prefers to instill the eye drops when she goes to sleep.
How long should she wait to administer the second eye drops?
A. 3 seconds
B. 3 mints
C. * 5 mints
D. 30 second
Q. 131 The following lab results are from a 35 year-old woman who is being
treated for reflux esophagitis (see lab results).
Test result normal values
Sodium 138 134-146 mmol/L
Potassium 4 3.5-5.1 mmol/L
Chloride 100 97-108 mmol/L
Bicarbonate 22 21-28 mmol/L
Calcium 2.2 2.15-2.62 mmol/L
Calcium ionised 1.1 1.1-1.3 mmol/L
Phosphate, inorganic 1 0.82-1.51 mmol/L
Magnesium 0.4 0.7-1.2 mmol/L

Arif Pharma clinic Copyright@arifpharma.com arifpharma.com


Page 388

Which of the following drugs would have contributed to the abnormality/ies


in lab results?
A. * Omeprazole
B. Cimetidine
C. Ranitidine
D. Gaviscon
Q. 132 A 58 year-old man, recently diagnosed with stage-I hypertension, was
started on a low dose ACET a week ago. The aim is to titrate the dose to
target within four weeks. On routine follow-up, he is complaining of
persistent cough.
What would be the best action?
A. Stop the ACEI1
B. * Substitute the ACEI with an ARB2
C. Continue titrating the ACEI and ignore the cough
D. Substitute the ACEI with a calcium channel blocker
Q. 133 1Angiotensin converting enzyme inhibitor
2Angiotensin receptor blocker
A 16 year-old girl with asthma recently had a routine laboratory follow up
(see lab results).
Test result normal values
Sodium 140 134-146 mmol/L
Potassium 3.0 3.5-5.1 mmol/L
Chloride 8 97-108 mmol/L
Bicarbonate 22 21-28 mmol/L
Calcium 2.2 2.15-2.62 mmol/L
Calcium ionised 1.2 1.1-1.3 mmol/L
Phosphate, inorganic 1.0 0.82-1.51 mmol/L
Magnesium 0.9 0.7-1.7 mmol/L
Copper 1.8 1.72-3.76 mmol/L
Which medication is most likely to have contributed to her results?

Arif Pharma clinic Copyright@arifpharma.com arifpharma.com


Page 389

A. Leukotriene receptor antagonist


B. * Inhaled short-acting B2 agonist
C. Inhaled corticosteroids
D. Inhaled short-acting antimuscarinic
Q. 134 Which one of the following drugs is considered first line treatment for
alzheimer’s disease?
A. * Donepezil
B. Amantadine
C. Pramipexole
D. Trihexyphenidyl
Q. 135 A 70 year-old woman is taking multiple drugs.
According to beers criteria, which one of the following drugs is potentially
inappropriate since it might be associated with increased cognitive decline ?
A. Aspirin
B. Warfarin
C. Enalapril
D. * Amitriptyline
Q. 136 A 55 year-old man is prescribed simvastatin for hyperlipidemia. He is
already taking verapamil for hypertension.
What should be to the maximum dose of simvastatin to be prescribed ?
A. * 10 mg
B. 20 mg
C. 40 mg
D. 80 mg
Q. 137 Which of the following is considered a high intensity statin therapy ?
A. * Rosuvastatin 20 mg once daily
B. Atorvastatin 20 mg once daily
C. Simvastatin 20 mg once daily
D. Pravastatin 40 mg once daily
Q. 138 Which one of the following medications for osteoporosis will
significantly reduce the risk of hip fracture?
A. Ibandronate
B. * Risedronate

Arif Pharma clinic Copyright@arifpharma.com arifpharma.com


Page 390

C. Raloxifene
D. Calcitonin
Q. 139 Which one of the following medications is known to induce
pulmonary toxicity?
A. * Amiodarone
B. Metoprolol
C. Carbamazepine
D. Glibenclamide
Q. 140 Which of the following medications is known to be a powerful enzyme
inducer?
A. Warfarin
B. * Rifampicin
C. Itraconazole
D. Metronidazole
Q. 141 Which of the following is an important counselling point for patients
starting on warfarin?
A. Avoid green leafy vegetables
B. * Keep intake of green leafy vegetables constant
C. Green leafy vegetables should be taken at least 4 hours after warfarin
D. Green leafy vegetables should be taken at least 2 hours before warfarin
Q. 142 What is the recommendation for administering live vaccine and an
antibody containing product?
A. * Separate the two administrations by two weeks
B. Administrations both at the same visit but different sites
C. Separate the two administrations by one month and administer at
different site
D. Administer the live vaccine first then antibody containing product one
week after
Q. 143 What is the study design called when subjects act as both control and
the active arm?
A. Cohort
B. * Cross-over
C. Case series
D. Cross-sectional

Arif Pharma clinic Copyright@arifpharma.com arifpharma.com


Page 391

Q. 144 In clinical trials what is the acceptable power of a study?


A. 50
B. 65
C. 75
D. * 85
Q. 145 What is considered the lowest level of evidence when reviewing
published medical literature?
A. Case series
B. Case reports
C. Clinical trials
D. * Expert opinion
Q. 146 A patient is asking the pharmacist about sharing the insulin pen with
his brother to save some money.
What should be the most appropriate response?
A. Insulin pens can be shared among siblings
B. Insulin pens can be shared if the needle is changed
C. * Insulin pens should not be shared even if the needle changed
D. Insulin pens can be shared if 0.5 ml is removed between injections
Q. 147 What is the recommendation regarding air bubble in the syringe for a
patient starting on enoxaparin subcutaneous injection?
A. * Inject the dose with the air bubble
B. Inject the air bubble if the volume is small
C. Remove the air bubble to avoid wastage of the dose
D. Remove air bubble and adjust volume before injection
Q. 148 Which one of the following is a side effect to chronic use of
corticosteroids ?
A. Hypotension
B. Hyperthyroidism
C. Hyperparathyroidism
D. * Sodium and water retention
Q. 149 A 55 year-old man with myocardial infarction present to the clinic
with dyspnea interrupting his daily activity.
Which one of the following medications can cause it?

Arif Pharma clinic Copyright@arifpharma.com arifpharma.com


Page 392

A. Aspirin
B. Enalapril
C. * Ticagrelor
D. Carvedilol
Q. 150 The pharmacy receives a prescription for labetalol 800 mg twice daily
for three days. The available strength of labetalol in the pharmacy is 200 mg.
How many tablets should the pharmacy dispense for the total duration?
A. 8
B. 12
C. 16
D. * 24
Q. 151 What is the role of ascorbic acid when added to extemporaneously
prepared formulations?
A. Solvent
B. * Preservative
C. Colouring agent
D. Flavoring agent
Q. 152 What is the effect of smoking on serum olanzapine levels?
A. Has no effect
B. * Decrease in serum olanzapine levels
C. Increase in serum olanzapine levels
D. Decrease in olanzapine levels initially then increase in it
Q. 153 A 75 year-old woman comes to the pharmacy with a new prescription
for zolpidem.
What is the maximum dose she can receive?
A. 2.5 mg
B. * 5 mg
C. 10 mg
D. 15 mg
Q. 154 What is the recommended haemoglobin target when treating
anaemia due to chronic kindly disease with epoetinalfa?
A. * 11 g/dl
B. 13 g/dl

Arif Pharma clinic Copyright@arifpharma.com arifpharma.com


Page 393

C. 14 g/dl
D. 16 g/dl
Q. 155 Which one of the following parameters should be monitored regularly
when starting a patient on clozapine?
A. Haemoglobin
B. Platelet count
C. Red blood cell count
D. * Absolute neutrophil count
Q. 156 Which one of the following medications is considered a high alert drug
?
A. * Insulin
B. Finasteride
C. Ceftriaxone
D. Ciprofloxacin
Q. 157 Which one of the following pair represents the characteristic of an
ophthalmic preparation?
A. * Sterile and isotonic
B. Sterile and hypotonic
C. Sterile and hypertonic
D. Pyrogen-free and hypertonic
Q. 158 Which one of the following is a property of well-formulated
suspension?
A. * Re-suspend upon moderate shaking
B. Require small amount of preservative
C. Form cake at the bottom of the bottle
D. Separate the powder from the solution easily upon shaking
Q. 159 The nurse is asking the pharmacist recommended to reduce the
inadvertent intrathecal administration of vincristine.
Which one of the following is recommended?
A. Dispense the syringe without needle
B. * Dispense vincristine in mini bag instead of syringe
C. Dispense vincristine in 60-ml syringe
D. Dispense the vial for the nurse to prepare at the bedside

Arif Pharma clinic Copyright@arifpharma.com arifpharma.com


Page 394

Q. 160 Which of the following strategies can reduce chemotherapeutic


medications errors?
A. Permit STAT order for chemotherapy
B. Accept verbal orders for chemotherapy
C. * Pharmacist who verify chemo order should calculate dose
D. Perform thorough check by one pharmacist to avoid mix up
Q. 161 What is the relationship between childhood vaccines and autism?
A. MMR vaccines caused autism
B. Any vaccine can cause autism
C. Hepatitis B vaccine causes autism
D. * None of the vaccines causes autism
Q. 162 What is the suitable dosage form for infants ?
A. * Oral syrup
B. Capsule
C. Tablet
D. Caplet
Q. 163 Which medication can cause orthostatic hypotension?
A. * Prazosin
B. Donepezil
C. Rivastigmine
D. Spironolactone
Q. 164 What is the half-life of drug A if its elimination rate constant is 0.02
per hour ?
A. 20
B. 28
C. * 35
D. 48
Q.165 What would be the expected change in the clearance of medications in
acute renal failure ?
A. Clearance of drugs eliminated mainly by the kindly increases
B. * Clarence of drugs eliminated mainly by the kindly decreases
C. Clearance is a constant parameter and is affected by the drug dose
D. Clearance is a constant parameter and is affected by the drug
concentration

Arif Pharma clinic Copyright@arifpharma.com arifpharma.com


Page 395

Q. 166 A patient is receiving intravenous 0.45% normal saline at rate of 75


ml/hour.
What will be the amount of sodium chloride (in grams) present in one liter of
this solution ?
A. * 0.45
B. 4.5
C. 45
D. 90
Q. 167 The pharmacist received a prescription to extemporaneously prepare
moxifloxacin oral suspension from the 400 mg tablet in a concentration of 20
mg/ml with total volume of 60 ml.
How many tablets of moxifloxacin 400 mg will be required ?
A. 2
B. *3
C. 4
D. 6
Q. 168 In a hemodynamically stable patient, when is it recommended to
draw gentamicin trough level?
A. * At steady state before fourth dose
B. At steady state after the first dose
C. At steady state after the sixth dose
D. At steady state after the second dose
Q. 169 What is medication formulated in XL formulations?
A. To easily crush tablet
B. To mask the bitter taste
C. To prevent medication errors
D. * To release the active ingredient slowly
Q. 170 Which one of the following is considered a dangerous abbreviation ?
A. Cap for capsule
B. Q6hr for every six hours
C. IVP for intravenous push
D. * IU for international unit

Arif Pharma clinic Copyright@arifpharma.com arifpharma.com


Page 396

Q. 171 Which of the following pairs is considered as acceptable patient


identifiers?
A. Bed and birth-date
B. First name and birth-date
C. Gender and medical record number
D. * Medical record number and full name
Q. 172 What does high volume of distribution indicate?
A. Drug concentrates in the blood
B. * Drug accumulates in the tissues
C. Drug crosses blood brain barrier freely
D. Drug distribution equally between the blood and the tissues
Q. 173 The nurse is asking the pharmacist about recommendation for
administering inactivated and live vaccine
What should be the appropriate recommendation ?
A. * Both vaccines can be administered simultaneously or at any interval
B. Administer the live vaccine first then the inactivated vaccine one week
after
C. Administer the inactivated vaccine first then the live vaccine one week
after
D. Separate between the two vaccines by one month and administer at
different site
Q. 174 Avoidance of consuming tyramine containing food is recommended
with which of the following medications
A. Warfarin
B. * Linezolid
C. Prednisone
D. Cyclosporine
Q. 175 What information should be provided to a patient on fentanyl
transdermal patches ?
A. * Avoid increase in body core temperature
B. Rotate the site of application to avoid tolerance
C. Store patches in the refrigerator to decrease stinging sensation
D. Apply heating pads on the transdermal patch to improve absorption
When dosing

Arif Pharma clinic Copyright@arifpharma.com arifpharma.com


Page 397

Q. 176 Trimethoprim/sulfamethoxazole combination, the dosage


recommendation is based on which component
A. Either one
B. * Trimethoprim
C. Sulfamethoxazole
D. Dose is fixed at 5 mg/kg/day in divided doses
Q. 177 A physician would like to replace patient’s valproic acid with
immediate release lamotrigine.
What is the recommended maximum initial dose?
A. * 200 mg
B. 300 mg
C. 400 mg
D. 500 mg
Q. 178 Which of the following strategy can be recommended to reduce
metformin associated gastrointestinal side effects?
A. Administer 2 hours after main meals
B. * Administer in divided doses with meals
C. Administer with H2 receptor antagonist
D. Administer before meals on empty stomach
Q. 179 Which of the following is an appropriate instruction for oral
suspension?

A. For external use


B. * Shake well before use
C. Only administer if clear solution
D. Shake then wait for suspension to separate then administer
Q. 180 Which routes of administration result in faster absorption
A. Oral
B. * Rectal
C. Topical
D. Subcutaneous
Q. 181 Which dosage form is appropriate when maximum moisturization is
required?

Arif Pharma clinic Copyright@arifpharma.com arifpharma.com


Page 398

A. Gel
B. Cream
C. Lotion
D. * Ointment
Q. 182 Which of the following indications is appropriate for misoprostol?
A. Misoprostol is ineffective for prevention or treatment
B. Misoprostol is effective for both prevention and treatment
C. * Misoprostol is effective for preventing NSAIDs induced ulcer
D. Misoprostol is effective for treatment of NSAIDs induced ulcer
Q. 183 Which one of the following is considered first line therapy for the
management of partial seizures?
A. * Carbamazepine
B. Phenobarbital
C. Gabapentin
D. Primidone
Q. 184 The physician is asking pharmacist about the effects of low albumin
and acute renal failure on phenytoin serum concentration. The patient is
receiving phenytoin 300 mg once daily.
What should be the most appropriate suggestion?
A. * In hypoalbuminemia and acute renal failure obtain free phenytoin
serum concentration
B. In hypoalbuminemia and acute renal failure obtain peak phenytoin
serum concentration
C. In hypoalbuminemia and acute renal failure obtain total phenytoin
serum concentration
D. In hyoalbuminemia and acute renal failure obtain tough phenytoin
serum concentration
Q. 185 Which of the following medications require observing the patient for
six hours with hourly heart rate monitoring after the first dose ?
A. Verapamil
B. Bisoprolol
C. Amlodipine
D. * Fingolimod
Q. 186 What is the monitoring parameter for warfarin therapy

Arif Pharma clinic Copyright@arifpharma.com arifpharma.com


Page 399

A. CT
B. * INR
C. Aptt
D. PT: a PTT Ratio
Q 187 A 29 year-old pregnant woman been recently diagnosed with open
angle glaucoma.
What would be the treatment of choice?
A. * Topical timolol 0.1 gel once daily
B. Oral acetazolamide 250 mg once daily
C. Topical latanoprost 50 mcg/ml once daily
D. Topical bimatorost 300 mcg/ml once daily
Q. 188 A 27 year-old pregnant woman has been recently diagnosed with iron
deficiency anaemia. Her haemoglobin is less than 11 g/dl.
What would be the best recommendation ?
A. Ferrous sulphate tablets with a cup of coffee or tea
B. * Ferrous sulphate tablets on an empty stomach
C. Ferrous sulphate tablets with antacids
D. Ferrous sulphate slow release tablets
Q. 189 A patient is on warfarin for mechanical mitral valve replacement .
What should be the INR1 goal range ?
A. 2-2.5
B. 2-3
C. * 2.5-3.5
D. 3.5-4
1international Normalization Ratio
Q. 190 Which one of the following is a known side effect of unfractionated
heparin ?
A. Hypokalemia
B. * Hyperkalemia
C. Hypocalcemia
D. Hypercalcemia
Q. 191 What is the pharmacological category for insulin lispro?

Arif Pharma clinic Copyright@arifpharma.com arifpharma.com


Page 400

A. Long acting insulin


B. * Rapid acting insulin
C. Short acting insulin
D. Intermediate acting insulin
Q. 192 What is the antidote of acetaminophen ?
A. Ethanol
B. Naloxone
C. Methylene blue
D. * N-Acetylcyseine
Q. 193 Which of the following signs correlate with phenytoin level of 200
micromole/L ?
A. * Coma
B. Ataxia
C. Nystagmus
D. Decreased mental status
Q. 194 A 70 year-old woman is diagnosed with sustained ventricular
tachycardia. The resident doctor wants to initiate one of the class III
potassium channel blockers. She has a history of torsades de pointes
arrhythmia and has renal impairment .
Which of the following medications is best to initiate ?
A. * Amiodarone
B. Dofetilide
C. Ibutilide
D. Sotalol
Q. 196 A 28 year-old woman has been taking fluoxetine for depression and
now has worsening depressive episodes. The resident doctor on the floor has
been asked by the consultant to stop fluoxetine and initiate phenelzine.
How long should the washout period be before starting phenelzine
A. 2 weeks
B. 3 weeks
C. 4 weeks
D. * 5 weeks
Q. 197 A 58 year-old woman presents to the emergency department with
severe right flank pain, nausea and vomiting. She had urinary frequency and

Arif Pharma clinic Copyright@arifpharma.com arifpharma.com


Page 401

dysuria for the last three days. Her past medical history includes glucose-6-
phosphate dehydrogenase deficiency.
What would be the best treatment option?
A. Nitrofurantoin for 5-7 days
B. * Cephalexin for 5-7 days
C. Ciprofloxacin for 5-7 days
D. Co-trimoxazole for 5-7 days
Q. 198 A 68 year-old woman with a history of type-2 diabetes and
dyslipidemia is on atenolol and lisinopril. The doctor has started simvastatin
as her estimated 10-year ASCVD risk is 7.5%
What would be the recommended dose for simvastatin
A. 10 mg
B. * 20 mg
C. 40 mg
D. 80 mg
Q. 199 A 60 year-old woman has been recently diagnosed with stage II
hypertension.
Which of the following medications would be suitable for her?
A. * Losartan/hydrochlorothiazide
B. Atenolol/nifedipine
C. Valsartan/furosemide
D. Amlodipine/triamterene
Q. 200 A 40 year-old man is prescribed 20 mg methylprednisolone
intravenously four times a day on the ward. The medical team would like to
discharge him on oral prednisolone tables .
What should be the oral prednisolone dose ?
A. 20 mg daily
B. 40 mg daily
C. 80 mg daily
D. * 100 mg daily
Q. 201 A 28 year-old woman with seizure is planning to start a family. She is
taking folic acid 400 microgram per day. She is asking about safe epilepsy
medications during pregnancy.

Arif Pharma clinic Copyright@arifpharma.com arifpharma.com


Page 402

Which of the following would be the best medication?


A. Carbamazepine
B. Valproic acid
C. * Lamotrigine
D. Phenytoin
Q. 202 Which of the following is an irreversible side effect of an
aminoglycoside ?
A. Stomatitis
B. * Ototoxicity
C. Nephrotoxicity
D. Pseudomembranous colitis
Q.203 A 75 year-old man with NYHA-IV heart failure has been started on
furosemide 120 mg intravenous infusion to improve pulmonary edema.
What would be the recommended duration for furosemide infusion ?
A. 10 minutes
B. 20 minutes
C. * 30 minutes
D. 40 minutes
Q. 204 A 21 year-old woman is worried about her weight and is inquiring
about medication to reduce it. Her height is 165 cm and he weight is 69 kg.
What is her body mass index?
A. 10
B. 20
C. * 25
D. 27
Q. 205 The physician is asking the pharmacist about when to discontinue
clopidogrel before the open heart surgery.
What should be the pharmacist suggestion?
A. Discontinue clopidogrel 3 days before surgery
B. * Discontinue clopidogrel 5 days befor surgery
C. Discontinue clopidogrel 2 weeks before surgery
D. Risk of bleeding is minimal continue clopidogrel

Arif Pharma clinic Copyright@arifpharma.com arifpharma.com


Page 403

Q. 206 What is the appropriate treatment of poisoning by organophosphate


nerve agents ?
A. * Pralidoxime
B. Hemodialysis
C. Gastric lavage
D. Activated charcoal
Q. 207 Which medication requires negative pregnancy test before dispensing
to woman of child bearing age ?
A. Labetalol
B. Prednisone
C. Ceftriaxone
D. * Isotretinoin
Q. 208 Which of the following medications can be teratogenic if a pregnant
woman handles crushes or breaks the tablet ?
A. * Bosentan
B. Alfuzosin
C. Topiramate
D. Didanosine
Q. 209 A 75 year-old obese man has been recently diagnosed with
osteoarthritis of the knee. He has tried non-pharmacological measures such
as weight loss and various aids to improve symptoms. But he still has pain in
the knee during walking
What would be the best initial treatment ?
A. * Topical NSAIDs
B. Topical glucosamine
C. Oral COX-2 inhibitors
D. Oral non-selective NSAIDs
Q.210 A 21 year-old man with a history of illicit drug use is suspected to have
overdose of an apioid and is experiencing severe toxicity with respiratory
depression and pin-point pupils.
What would be the best antidote ?
A. Activated charcoal 50 grams
B. Diazepam 10 mg intramuscularly
C. Flumazenil 300 mcg intravenously

Arif Pharma clinic Copyright@arifpharma.com arifpharma.com


Page 404

D. * Naloxone 400 mcg intramuscularly


Q. 211 A nurse on the ward is being asked by the doctor to administer normal
saline 250 ml set running at 5 drops/ml.
What will be the infusion rate in drops /minute?

A. 2
B. * 10
C. 21
D. 100
Q.212 A pharmacy intern is asked to prepare a solution containing 8.4 g of
drug, from available solution labeled as 20% w/v.
What will be the volume he must take from 20% w/v solution ?
A. 4.2 ml
B. 8.4 ml
C. * 42 ml
D. 84 ml
Q.213 A 55 year-old man with an African descent is diagnosed with
hypertension with a BP of 140/80.
Which of the following is the most appropriate first line treatment ?
A. Ramipril
B. * Amlodipine
C. Carvedilol
D. Candesartan
Q. 214 A 65 year-old man with NYHA class-IV heart failure has been started
on digoxin to improve his symptoms.
Which of the following electrolyte imbalance will predispose him to digoxin
toxicity?
A. Hyperkalemia
B. * Hypercalcemia
C. Hypernatremia
D. Hypermagnesemia
Q. 215 A patient is receiving intravenous fluid at a rate of 75 ml/hour.

Arif Pharma clinic Copyright@arifpharma.com arifpharma.com


Page 405

How much time will it take to infuse 1.5 litres ?


A. 5 hours
B. 10 hours
C. 15 hours
D. * 20 hours
Q. 216 A patient is receiving 5% dextrose saline at a rate of 100 ml/hour.
How many litters will he receive in 20 hours?
A. *2L
B. 3L
C. 4 L
D. 6L
Q. 217 A 55 year-old man with bipolar disorder is stable on lithium for the
last 12 months.
Which of the following tests should be ordered on routine follow up?
A. Renal profile
B. Liver function test
C. Complete blood count
D. * Thyroid function tests
Q. 218 A 55 year-old man with a BMI of 37 is diagnosed with type II diabetes
three months ago. He failed to adhere to prescribed diet and exercise to
prescribed diet and exercise regimen.
Which of the following medications would be best indicated ?
A. * Metformin
B. Gliclazide
C. Tolbutamide
D. Glibenclamide
Q. 219 An eight month-old boy with recurrent otitis media has been started
on high dose amoxicillin (90 mg/kg) for seven days. There is no loss. He is due
his 3rd dose of pneumococcal vaccination and the influenza vaccination. The
nurse is asking about due vaccination.
What would be the best action?
A. Cancel pneumococcal and influenza vaccination
B. * Proceed with pneumococcal and influenza vaccination

Arif Pharma clinic Copyright@arifpharma.com arifpharma.com


Page 406

C. Postpone pneumococcal and influenza vaccination for one year


D. Delay pneumococcal and influenza vaccination till antibiotic use
Q. 220 How many grams of dextrose is present in 100 ml of 10% dextrose
solution ?
A. 1G
B. * 10 G
C. 100 G
D. 1000 g
Q. 221 A patient is started on dopamine 5 mcg/kg/minute. The patient
weight is 60 kg.
How many milligram of dopamine will be administered in 24 hours ?
A. 43 mg
B. 180 mg
C. * 432 mg
D. 1800 mg
Q. 223 What dose “ scored tablet in quarters” indicate ?
A. Tablet can be crushed
B. Tablet can be broken in half
C. Tablet is immediate release formulation
D. * tablet can be broken in half or quarters
Q. 224 A 10 year-old boy is prescribed a drug at a dose of 10 mg/kg every 8
hours. His weight is 30 kg.
What will be the total daily dose he receives ?
A. 30 mg
B. 100 mg
C. 300 mg
D. * 900 mg
Q. 225 A 25 year-old man presents to the emergency room after ingesting 30
tablets of alprazolam.
What should be the appropriate antidote ?
A. Naloxone
B. Ethanol
C. * Flumazenil

Arif Pharma clinic Copyright@arifpharma.com arifpharma.com


Page 407

D. Physostigmine
Q. 226 A patient on warfarin presents with nose bleeding since yesterday. His
INR is 10 .
What should be the pharmacist advice ?
A. Administer charcoal
B. Perform hemodialysis
C. * Hold warfarin and administer vitamin k
D. Continue same dose and monitor INR closely
Q. 227 A 65 year-old man was diagnosed with type II diabetes five years ago
and is in an oral biguanide. His routine follows up and laboratory work up is
scheduled.
Which of the following additional tests is recommended ?
A. Vitamin C
B. Vitamins D
C. * Vitamin B12
D. Electrolytes
Q. 228 A 48 year-old man with myocardial infarction is started on an ACEI. He
is started on a low dose and the aim is to titrate dose within four weeks. On
routine follow up of labs one week from starting the drug, his serum
creatinine is increased by 20% from the base line.
What would be the most appropriate action ?
A. Stop the ACEI1
B. * Continue titrating the ACEI
C. Keep the current dose of ACEI
D. Substitute the ACEI with an ARB2 1Angiotensin converting enzyme
Q. 229 A 52 year-old man has been admitted in the intensive care unit (ICU)
for seven days and suspected of having endocarditis with unstable
hemodynamics. He is started on vancomycin 2000 mg IV as a loading dose
followed by 1500 mg every 12 hours. The concentration of vancomycin is 5
mg/ml in 0.9% normal saline.
What should be the maximum rate of infusion for loading dose of
vancomycin ?
A. 1 ml/min

Arif Pharma clinic Copyright@arifpharma.com arifpharma.com


Page 408

B. * 2 ml/min
C. 3 ml/min
D. 4 ml/min
Q. 230 A 65 year-old man with a history of chronic NSAIDs use presents with
melena. The endoscopy shows erosive esophagitis.
Which of the following drugs will be the most appropriate treatment ?
A. Ranitidine
B. Sucralfate
C. * Lansoprazole
D. Metoclopramide
Q. 231 A 45 year-old man with bipolar disorder has been stable on lithium for
12 months. He is due for routine laboratory work up.
Which of the following electrolyte imbalances will predispose him to lithium
toxicity ?
A. Magnesium
B. Potassium
C. Calcium
D. * Sodium
Q. 232 A 60 year-old man with a history of heart failure presents with an
acute attack of gout in the left toe with excruciating pain. He rates his pain in
his toe as 8/10
Which is the best treatment option ?
A. Aspirin 300 mg daily
B. Diclofenac 75 mg daily
C. Allopurinol 300 mg daily
D. * Colchicine 600 mcg daily
Q. 233 A 60 year-old man with a history of heart failure presents with an
acute attack of gout in the left toe with excruciating pain. He rates his pain as
8/10.
Which of the following medications could have contributed to his acute
gout?
A. Losartan 20 mg daily
B. * Furosemide 40 mg daily

Arif Pharma clinic Copyright@arifpharma.com arifpharma.com


Page 409

C. Hydrochlorothiazide 25 mg
D. Carvedilol 25 mg twice daily
Q. 234 Which of the following is an important counselling point for woman
starting on fingolimod ?
A. * Use effective contraception to avoid pregnancy during and 2 month
after discontinuing treatment
B. Pregnancy should be avoided for six months after discontinuing
treatment
C. Use effective contraception to avoid pregnancy during treatment
D. Fingolimod is safe during pregnancy
Q. 235 A 60 year-old man with a history of angina pectoris is complaining of
shoulder pain.
Which of the following analgesics can inversely affect his disease outcome ?
A. Acetaminophen
B. * Celecoxib
C. Morphine
D. Codeine
Q. 236 Which one of the following calcium channel blockers is non-
dihydropyridine drug ?
A. Nicardipine
B. Amlodipine
C. Nifedipine
D. * Diltiazem
Q. 237 What is the content of a monophasic contraceptives pill?
A. Constant dose of estrogen
B. Variable dose of progesterone
C. * constant dose estrogen and progesterone
D. Variable doses of estrogen and progesterone
Q. 238 Which of the following vachines is contraindicated during pregnancy ?
A. Hepatitis B vaccine
B. Influenza virus vaccine
C. * Varicella zoster vaccine
D. Meningococcal polysaccharide vaccine

Arif Pharma clinic Copyright@arifpharma.com arifpharma.com


Page 410

Q. 239 A woman on mycophenolate is planning for pregnancy she is asking


for pharmacist advice.
What should be the most appropriate advice ?
A. Mycophenolate should be continued during pregnancy
B. Mycophenolate should be continued for the first trimester then
discontinued
C. * mycophenolate should be discontinued at least 6 weeks prior to trying
to conceive
D. Mycophenolate should be continued at least 12 weeks prior to trying to
conceive
Q. 240 Which of the following cardiac enzymes have the highest specificity
and sensitivity to aid the diagnosis of an acute coronary syndrome ?
A. BNP and NT-proBNP
B. * Troponins T and I
C. C-reactive protein (CRP)
D. Creatinine kinase (CK-MB)
Q. 241 A 54 year-old man with a history of cirrhosis and alcoholic liver
disease presents with hepatic encephalopathy.
What is best treatment option?
A. Senna
B. * Lactulose
C. Rifaximin
D. Ceftriaxone
Q. 242 Which one of the following side effects is known to bisphosphonates ?
A. Headache
B. Muscle pain
C. Hypotension
D. * Osteonecrosis of the jaw
Q. 243 When dosing levodopa/carbidopa,on which component should the
dose be based on?
A. * Dose should be bases on levodopa
B. Dose should be based on carbidopa
C. Dose should be based on both components
D. Either one since the ratio of the combination is 1:1

Arif Pharma clinic Copyright@arifpharma.com arifpharma.com


Page 411

Q. 244 Which of the following anti-neoplastic medications can be given intra-


thecal ?
A. Vindesine
B. Vincristine
C. Vinblastine
D. * Cytosine arabinose
Q. 245 Which of the following anti-neoplastic medications is known to cause
bone marrow suppression ?
A. Bleomycin
B. Vincristine
C. * Daunorubicin
D. L-asparaginase
Q. 246 A 65 year-old man with acute decompensated heart failure has been
started on dobutamine 5 mcg/kg/minute. The infusion bag of dobutamine is
150 mg in 100 ml of 0.9% sodium chloride. He has normal renal function and
his weight is 70 kg.
What would be the rate of infusion of dobutamine?
A. 0.25 ml/hour
B. 1.3 ml/hour
C. * 14 ml/hour
D. 26 ml/hour
Q. 247 A 62 year-old man has been admitted in the surgical ward for 14 days
and is suspected to have methicillin-resistant S. aureus (MRSA) infection in
his wound. He is started on vancomycin 1500 mg IV as a loading dose
followed by 1000 mg every 12 hours. He has normal renal function and his
weight is 70 kg.
What is the recommended time to take vancomycin serum levels?
A. After the 1st dose
B. After the 2nd dose
C. * After the 3rd dose
D. After the 4th dose

Arif Pharma clinic Copyright@arifpharma.com arifpharma.com


Page 412

All The Best Copyright@arifpharma.com

Drx Arif Khan


Through our YouTube channels Arif Pharma Clinic and Arif Pharma
Academy, we provide study material for pharmacy and nursing staff for
Prometric exams such as DHA Exam, MOH Exam, and HAAD Exam, so that
students who dream of getting a job abroad We try to make things easy. And
also provide GPAT exam study material. Through our team of highly
experienced and qualified faculty headed by Pharmacist Arif Khan, Director of
the Arif Pharma Clinic YouTube Channel, we have managed to touch the
minds and lives of many students. And if you're with me, I'll keep trying.

Arif Pharma Clinic


Subscribe Now & Press The Bell Icon

Arif Pharma clinic Copyright@arifpharma.com arifpharma.com


Page 413

DHA Important One liner Question Set (2)

1. Long-action analgesic and anti-inflammatory drugs? {NSAID has a more Enterogenic


effect? (Piroxicam)}

2. MOA of anti-inflammatory? (Decrease Prostaglandin)

3. Most likely drug causes Peptic ulcer? Piroxicam Or Ketorolac Or Indomethacin

4. Man takes an overdose of drugs and then takes the antidote but still no effect, why?
(antidote dose was small)

5. Biological substance can be stored at … C? (2 – 8 C)

6. What pharmacist should do when the wrong medication is prescribed by Dr? (Inform the
prescriber doctor)

7. What should do with an elderly man with hypertension? (Start with a small initial dose
then increase)

8. What should be done with the girl who has seizures and tremors
when using Oxcarbazepine "anti-convulsant"? (shift to
Topiramate) "Topamax"

9. TTT of trigeminal neuralgia? (Carbamazepine) "Tegretol"

10. Child taking Oxcarbazepine suffers from rash and redness? shift to Topiramate Or
Ethosuximide
N.B; this shifting is due to the child because this S.E is a normal side effect, of the use of
adult; keep using Oxcarbazepine

11. Medication is used to treat absence seizures? (Ethosuximide)

12. Pt suffered before from a migraine attack with aura, he needs a drug as
prophylaxis? (Topiramate 25 mg twice)

Arif Pharma Clinic Copyright@arifpharma.com arifpharma.com


Page 414

13.Middle age man has HbA1c= 9 and he takes Glipizide + Lisinopril + Pioglitazone Dr
add? (insulin)

14. Example of drug-disease interaction? (Pioglitazone & heart failure) (Celecoxib &
Cardiac toxicity )

15. Woman wants to be pregnant, she takes metformin and pioglitazone, she has a history
of hypoglycemia and prefers oral therapy, best TTT? (Stop pioglitazone and titrate
metformin )

N.B: side effect of pioglitazone is Hypoglycemic

16. Normal range of HbA1c is, non-diabetic 4 – 5.6

17. Bioavailability of Diclofenac injection in muscle; 100%

18. Bioavailability refers to the extent and rate at which the active moiety enters…..?
(systemic circulation )"site of action"
19. Heparin & Warfarin are contra-indicated with? ( Aspirin & Diclofenac & ibuprofen )

20. Heparin or Warfarin if added to Aspirin will lead to? (Bleeding )

21. Interaction between warfarin and ibuprofen cause risk of bleeding, why? (warfarin
will be replaced by ibuprofen )

22. Warfarin oral anti-coagulant is determined or calculated by? (INR international


normalized ratio)

23. Heparin injection anti-coagulant is determined by? (APTT activated partial


thromboplastin time)

24. Heparin and warfarin were prescribed for the patient, how long he will take heparin, and
how much INR?
(take heparin for 2 days with warfarin then stop heparin, warfarin biological T1/2 is 48
hours

25. Best counseling point for patients taking warfarin? (Vit K consistent)

Arif Pharma Clinic Copyright@arifpharma.com arifpharma.com


Page 415

26. Anti-dote of warfarin? (Vitamin K other name Phytomenadion, phylloquinone K1,


menaquinone K2, menadione K3)

27. Anti-dote of Heparin? Protamine sulfate

28. Anti-dote of Paracetamol? N-Acetyl Cysteine Anti-dote of iron? Deferoxamine

29. Drug acts on non-receptor mechanisms? The drug is used to remove excess iron by binding
to free iron. Deferoxamine

30. Which decreases warfarin actions? Multivitamins

31. Which drug causes nasal bone hypoplasia? Warfarin

32. Drug that increases the action of Warfarin? None of the above

33. Why should warfarin be stopped if a woman wants to be pregnant? Cranial facial
abnormality

34. Warfarin affects? the vitamin K-dependent clotting factors II, VII, IX, X

35. Heparin increases? increases the action of anti-thrombin on thrombin and factor Xa

36. Decreaseor increase the dose to the patient by a technician is? Omission error Or
(improper dose error)

37. Types of error in Pharmacy? dispensing error

37. Nurse gives patient dose 2 hours later, what is the type of error? wrong time error
administration error
38. Example of error of deteriorating drug? giving expired drugs storing drug out of
Refrigerator

39. Most occurring error in prescription? wrong drug order

Arif Pharma Clinic Copyright@arifpharma.com arifpharma.com


Page 416

40. Mosterrors that may occur when the patient is transferred from one unit of a facility to
another? missing drug he was taking

41. Drprescribes Chlorpheniramine for the driver but the pharmacist discovers a problem.?
the error occur and drug won't reach Pt
42. Barbiturates are Sedatives, Analgesics, and anti-convulsant Ex / barbital ultra short
barbiturates are used mainly? Anesthesia Ex/methohexital - thiopental - thiamylal

43. Sideeffects of Barbiturates in an elderly patient? Agitation Or Excitement


confusion

44. Drug with slow metabolism? Phenobarbital; which is a short-acting barbiturate

45. Anti-fungal Amphotericin B is used for what infection? Systemic & Subcutaneous

46. Acyclovir is used for? Herpes simplex virus

47. MOA of anti-emetic Metoclopramide? dopamine D2 antagonist "dopamine blocker" ,


agonist on serotonin 5-HT4 receptor weak inhibition of 5-HT3 receptors

48.Metoclopramide action on which area? Chemo trigger zone

49.Which drug increase milk production by increasing prolactin level? Metoclopramide,


Domperidone

50.Which drug decrease milk supply? Ergotamine, Estrogen, bromoceptrin

51.TTT of nausea and vomiting in Breastfeeding? Domperidone "anti-emetic without CNS


effect", Metoclopramide

52.Duration of using Domperidone? 3 – 5 Days not be used for more than 7 days

53.MOA of domperidone? Dopamine antagonist & can't cross BBB

54.MOA of Doxazosin? inhibits postsynaptic alpha 1 adrenergic receptor

Arif Pharma Clinic Copyright@arifpharma.com arifpharma.com


Page 417

55.Duration of Esomeprazole in gastroesophageal disease? up to 24 hours

56.Esomeprazole for esophageal injury? 30 days

57.Which committee rules out a drug from the hospital? Therapeutic evaluation committee

58.Non-formulary drug order reviewed by? = Management of the drug? Pharmacy &
therapeutic committee

59.Formula of the drug can be adjusted by? = Removing a drug from the formulary?
Pharmacy & therapeutic committee

60. MOA of statin? Decrease cholesterol by inhibiting HMG-CoA reductase

61.Which drug with statins can cause myopathy Gemfibrozil

N.B; Fibrates such as Gemfibrozil are used for hyperlipidemia and act on triglycerides

62.Most likely to interact with statins? Daptomycin

63.Drug likely to interact with blood glucose level? Statin or Hydrochlorothiazide

64. What Drug can cause the short onset of diabetes? Drug decrease synthesis of
cholesterol in the liver? Simvastatin

65. Diabeticpatient needs a drug for hyper-triglyceridemia? Statin


as atorvastatin

66. Patient needs a drug for hyper-triglyceridemia? Gemfibrozil

67. Maximum dose of simvastatin in geriatric? = dose of simvastatin


in hypercholesteremia? 40 mg

68. Old
man is on Verapamil "Anti-HTN", but he will start to take
simvastatin with the dose. 10 mg/day

Arif Pharma Clinic Copyright@arifpharma.com arifpharma.com


Page 418

69. Which is contra-indicted with a statin? Ketoconazole–


Gemfibrozil

70. BUN, Creatinine, Potassium, and Sodium are high so the patient
should stop?
(Gemfibrozil) or (Metformin)

71. To
determine the severity of myopathy caused by Statins?
Serum creatinine kinase

72. TTT
of H.Pylori? Clarithromycin + Amoxicillin + PPI as
omeprazole or pantoprazole

73. 20 mg of omeprazole equal to what of pantoprazole or


ranitidine in H.Pylori ttt ? 40 mg pantoprazole

74. Sensitivity
test of Penicillin done after? 30 days or more or
more than 20 days

75. Penicillin
sensitivity test for the intradermal stage which is a
must? Skin scratch test

76. Penicillin allergy test is completed in about? 1 Hour

77. Viascratch and intradermal test in Penicillin skin test should


wait for? 15 – 20 minutes

78. patient
with pneumonia went to the hospital to get a vaccine,
which should give? both Pneumonia and Influenza only

79. Pregnant woman suffering from upper respiratory disease took


ceftriaxone IV, Which vaccines hospital should give her?

80. Patient suffers from lung obstruction, which vaccine should he


take? //

81.Rhabdomyolysis can be detected by? Creatine kinase level,


serum Ca, myoglobin K, urine analysis

Arif Pharma Clinic Copyright@arifpharma.com arifpharma.com


Page 419

82. Which anti-psychotic patient has involuntary movements,


gains weight, can't sleep, isolated from people must take?
Clozapine then Haloperidol

83. Ptsuffers from psychosis, he takes medications, he developed


involuntary movements, what drug he takes? Haloperidol

84. Clozapine cause? Propylthiouracil cause? Agranulocytosis


lower white blood cells

85. Pt
on Clozapine, what should be checked during ttt RBCs?
Neutrophils

86. What will happen if we break down sustained-release drugs


instead of using them as tablets? Toxicity

87. What is the toxicity when breakup sustained release drug?


multi-dose toxicity

88. Laxative is contra-indicated in pregnancy? Castor oil

89. MOA of Bisacodyl is? Stimulation of enteric nerves causes


colonic contractions

90. What is the lowest potency laxative? Bisacodyl

91. What has the lowest onset of action in laxatives? Psyllium

92. Newborn baby is prescribed for him Morphine, what is the best
route of administration? IV

93. Neonatesuffers from cardio coagulopathy, so Vit K should be


administered through? IM

94. Whois the reasonable for determining the best dosage form?
Pharmacist/Clinical pharmacist

Arif Pharma Clinic Copyright@arifpharma.com arifpharma.com


Page 420

95. Patient
entered the hospital suffering from cramps, and gave
him Diazepam "Skeletal Muscle relaxant" but there is no effect
becausethis patient is addicted to a kind of? Opioid

96. Pt came to emergency with respiratory depression. Opioid

97. Usesof opioids? analgesic - antidiarrheal - antitussive - aids


to anesthesia Not anti-inflammatory

98. TCA; Amitriptyline - Amoxapine - Desipramine - Doxepin -


Imipramine - Nortriptyline - Protriptyline – Trimipramine MOA;
inhibit the reuptake of the biogenic amines at synaptic, mostly
norepinephrine (NE), as well as serotonin (5HT)

99. Tetracyclic antidepressant (TeCA)? Maprotiline - Mirtazapine –


Amoxapine

100.Patient with severe HTN & tremors and he is addicted, what is the best anti-
depressant? Amitriptyline

101.The dose of Voriconazole should be adjusted in the? (Liver cirrhosis)

102. Aliskiren? is the first in a class of drugs called direct renin inhibitors, indication is
essential = primary hypertension

103.Aliskiren contraindications? (furosemide, ketoconazole, sildenafil, celecoxib, aspirin,


atorvastatin, nifedipene)

104.By using Lamotrigine, we start with small doses and then increase them. (to prevent
the development of serious rash)

105.Lamotrigine " Lamictal "? anti-epileptic medication, also called an anticonvulsant

106.Pt takes valproic acid, and has to start taking Lamotrigine, what is the maximum initial
dose of Lamotrigine? 200 mg

107.A pregnant woman should watch out for? vitamin A daily intake of vitamin A for an
adult female is 700 mcg, for adult male is 900 mcg, for breastfeeding is 1200-1300

Arif Pharma Clinic Copyright@arifpharma.com arifpharma.com


Page 421

108.Terazosin? anti-HTN, MOA? selective alpha 1 antagonist N.B.; which drug treats BPH by
relaxation of the bladder neck? Terazosin

109.Maximum dose of Phenytoin? 300mg per day phenytoin dose is 100; 300 per day N.B;
each 100 mg of phenytoin sodium contains 92 mg of phenytoin base

110.Dr prescribes phenytoin 100mg TID, then change the dose, after how many days need
monitoring? 30 days

111.Side effects of Phenytoin? (congenital anomalies,hepatic necrosis – confusion –


tremor – irritability …N.B: Alcohol decreases t h e effect of phenytoin by increasing
its metabolism

112. Uses of phenytoin other than anti-convulsant? TTT of irregular heartbeat & painful
nerve conduction "neuralgia

113.Sevelamer "Renagel" $ decreases serum uric acid and phosphorus concentration in


chronic kidney disease $ treat hyperphosphatemia

114.Vincristine " anti-cancer for Leukemia " administration? (by minibag "IV push" or
infusion "IV" )

115.Is dosage form preferred for children? (Oral)

116.Montelukast "singulair"? is a leukotriene antagonist, is administered orally TTT of


asthma, relieves symptoms of seasonal allergy

117.Biosimilar? is a biological medicinal product that is an identical copy of an original


product that is manufactured by a different company, are officially approved version of
the original product, and can be manufactured when the original products expire

118.MOA of PPI proton pump inhibitor? (irreversible block hydrogen potassium adenosine
triphosphatase enzyme) inhibitor H+/K+-ATP ase proton pump

119.Receptor response in milliseconds and other in seconds? (ligand-gated receptors and G


– protein receptors)

120.Example of the isotonic solution? (Ringer solution)

121.Example of the high alert solution? (dextrose 20%)

122.The patient with acne takes Doxycycline for systemic use & adapalene for topical use,

Arif Pharma Clinic Copyright@arifpharma.com arifpharma.com


Page 422

he suffers from heartburn. (take omeprazole with doxycycline) N.B; Doxycycline can't be
dispensed without a prescription

123.When using Doxycycline, how to decrease GIT side effects? (drink a large amount of
water)

124.A smoker man drink coffee, take steroid, and is married to 2 wives, how many factors
affect ulcer? (3)

125.Monoamine oxidase inhibitors (MAOIs) inhibited with cheese? (Tranylcypromine)

126.TTT of malignant hypertension? (IV sodium nitroprusside) Nitroprusside for congestive


heart failure patients is administered by? (slow iv infusion) but in less urgent cases Oral
drugs such as captopril, clonidine, labetalol, and prazosin " all have delayed onset of
action "

127.Anti-pseudomonas drug? (meropenem)

128.Myasthenia gravis (MG)? is a long-term neuromuscular disease that leads to skeletal


muscle weakness TTT of MG. (Pyridostigmine) or neostigmine

129.Rivastigmine "Exelon patch"? treatment of Alzheimer’s and Parkinsonism "


MOA? (reversible cholinesterase inhibitor) Side effects? (nausea and vomiting)

130. Phentolamine? is a synthetic imidazoline with alpha-adrenergic antagonist activity

131. Addictive drivers challenge police, talkative, aggressive … addiction of? (amphetamine)

132. MOA of Amphetamine? (sympathomimetic amine that promotes the release of


dopamine and norephedrine from their storage sites in presynaptic nerve terminals)
(indirect adrenergic agonist)

133. The woman wants the prescription of her drug because her case is like his case as
she said, what will you do? (give her the telephone of the prescriber doctor)

134. Does the car driver want an antihistamine? (Desloratadine)

135. What is the time interval between 2 drugs if the half-life is 4 hours? (8 hours)

136. The base is used in the suppository? (Cocoa butter)

Arif Pharma Clinic Copyright@arifpharma.com arifpharma.com


Page 423

137.Dose of Zaleplon? sedative, tt of insomnia Normal: oral 10 mg before bedtime, the


maximum dose is 20 mg Geriatric: oral 5 mg before bedtime, the maximum dose is 10
mg

138. Pt with a Cold, take drug A (pseudoephedrine and chlorpheniramine) and drug B
(pseudoephedrine and loratadine) you advised him for sensitivity. (take drug B
because it has more benefits)

139. More important mentalic multivitamin? = which is found in vit B12? (Cobalt)

140. MOA of Hydralazine? (Relaxation for smooth muscle, Dilation for arteries more
than veins Vasodilator)

141. The drug can't be used as monotherapy in mild hypertension? Hydralazine

142.Pt with depression takes Fluoxetine but has no effect so Dr wants to change it to
Phenelzine, when start to take it? (4 weeks)

143. Avanafil? (is a phosphodiesterase-5 PDE5 inhibitor, approved for erectile


dysfunction)

144. Pt with psychosis, he started to take drug that has lowest hypnotic effect, this is?
(Risperidone)

145. What is the best combination for secondary HTN? (ACE-inhibitor + CCB calcium
channel blockers thiazide)

146. Tablet must be uniform to ensure? (Bioavailability)

147. Infant with croup, what is the virus that cause this? (Para influenza)

148. TTT of Extrapyramidal for 70 years old? (Benztropine) or


(Benztropine and diphenhydramine and trihexyphenidyl)

Drx Arif Khan

New Updated DHA Exam Study Material For Pharmacists


Arif Pharma Clinic Copyright@arifpharma.com arifpharma.com
Page 424

Important One-Liner Question Set (3)

1. Palivizumab used in prophylaxis of RSV virus


N.B: it is a monoclonal antibody that binds to the f protein on RSV to prevent it from
infecting the cell
2. Ace inhibitors used in case of a hypertensive patient with Proteinurea
3. Woman suffers from Asthma she is on salbutamol but she came and investigation
show mild obstruction What to give her next I choose a button-side inhaler
Line of asthma: 1-saba 2-inhaled steroid 3-laba 4- leukotriene antagonist 5-oral
steroid 6- omalizumab
4. Tuberculosis Vaccine (BCG) taken at birth
5. MMR vaccine is given at 12 months and four years
6. Man with HBA1C 9 – increase the dose of metformin
N.B: if HBA1C is between 7-10 additional action should be taken
7. woman is taking combined oralcontraceptives she is smoking 20 cigarettes per day
and Her BMI is 37 (obese)
Answer: shift to levonorgestrel (progesterone only)
N.B: BMI below 18.5 is underweight

18.5-25 Normal

25-30 obese

Above 30 obese

8. Pregnant woman has UTI gram VE microorganism what DOC

Ciprofloxacin

Tetracycline

Sulphamethoxazole

Nitrofurantoin

9. Pioglitazone added to therapy should make liver function test

10. metoprolol masks the hypoglycemic effect

Arif Pharma Clinic Copyright@arifpharma.com arifpharma.com


Page 425

11. most important vitamin in case of pregnancy: folic acid

12. a woman with penicillin sensitivity (he said she had hypotension when she take
penicillin) if she needs an antibiotic for prophylaxis choose erythromycin

13. antidote for iron toxicity is deferoxamine

14. Patient with lung obstruction …. Shouldvaccination with influenza and pneumococcal

15. sustained release if broken will cause multidose toxicity

16. a large amount of Parenteral medicationSterile water for injection

17. child’s BMI o f 85% had a muscle thickness testbody fat

18. maximum prescription time for controlled drugs for one time is 90 days

19. which is antiprogestin: mifepristone

20. rivastigmine is: a cholinesterase inhibitor

21. antihypertensive is harmful to infants during breastfeeding: atenolol Information: risk of


hypoglycemia and bradycardia in baby

22. the patient has more alow limb and trunk pain and used antipsychotics for 11 years
Diagnose: tardive dyskinesia

23. laxative with the slowest onset of action: psyllium

24. Enalapril MOA: ACE inhibitor

25. Best antihistaminic for a driver : loratidine

26. Increase osteoporosis in geriatric will increase: bone fractures

27. Patient has Parkinson's disease anddevelop mydriasis and closed angle glaucoma
Drug which cause this: trihexyphenidyl

28. Case asks about the required dose of simvastatin I choose 40 mg Of his body weight:
Subcutaneous fat

29. forget to dispense a patient dose until the next scheduled dose: omission error

30. : Zanamavir used in case of acute influenza

N.B: it's a neuraminidase inhibitor IInhibitsviral uncoating

31. Alendronate is used to reduce the risk of hipfractures

Arif Pharma Clinic Copyright@arifpharma.com arifpharma.com


Page 426

32. antiplatelet work on ADP (adenosine diphosphate) of platelet: clopidogrel

33. maximum OTC daily dose of ibuprofen:1200 mg

Line of Treating H.PYLORI


34. For treatment of amoeba we can add to metronidazole ……: diloxanide
N.B: it is used in cyst form of amoeba

35. Nilotinib used in Philadelphial myelogenous leukemia

36. Patient takes warfarin to advise him to take consistent green vegetable Vitamin

37. validity of pharmacy 5 years

38. if the child receives a high dose of amoxicillin and Has an Appointment for a
vaccine: vaccine Proceed to take a vaccine

39. Antiarrhythmic cause iodine hypothyroidism: Amiodarone

40. Antineoplastic cause bone marrow suppression: doxorubicin

41. MOA of Montelukast: leukotriene antagonist

42. Diabetic with high triglycerides and cholesterol: Statin

43. Example of deteriorating toxicity is given a drug that is expired

44. Which base used in the suppository: cocoa butter

45. Topiramate first line In the prophylaxis of migraine; dose 25-100 daily

46. which is a controlled drug: Fentanyl

47. Category C in pregnancy: no available studies on human or animal

48. Drug changes mental status; diazepam

49. Drug cause water retention: indomethacin

50. What is the fast absorption: Oral, Rectal, Topical, Subcutaneous

51. Ca carbonate SE: Constipation

52. Case there is elevated t3 and t4.. normal TSH drug used: Methamizol Information:
Antithyroid drug inhibits thyroid Peroxidase inhibits oxidation of iodine

Arif Pharma Clinic Copyright@arifpharma.com arifpharma.com


Page 427

53. PO: taken by mouth

54. the best way to take NSAID: is with food and milk After the meal wasn't in the
choices

55. Mycophenolate should stop 6 weeks before pregnancy

56. Codein used as a cough suppressant

57. Pain killer in green line: oxycodone

58. Patient identification by name and medical record

59. Narcotic used to treat narcotic addiction: methadone

60. best drug delivery in infants: neutralizer

61. Case: BPH: tadalafil after 4 hours from tamsulosin There was another choice but
sildenafil Tadalafil is the correct choice

62. Iron deficiency is the most common cause of failure of treatment with epoetin

63. Iong term with cortisone need calcium

64. Nurse who received an extra vial should return it to the pharmacy

65. Drug cause pulmonary toxicity: Amiodarone

66. Antibiotic need in community-acquired pneumonia need hospitalization:


ceftriaxone + azithromycin

67. Antidote leucovorin of methotrexate toxicity: Antidotes of methotrexate: 1-


leucovorin 2-thymidine 3-glucarpidase

68. Females with a history of DVT should take levonorgestrel (progesterone only)

70. Study in which different studies are used to make a conclusive answer was a
systematic review If meta-analysis is present. choose it

71. Drug with distribution volume 35L Highly distributed in plasma or highly dissolved
in adipose tissue I chose highly dissolved in adipose tissue I am not completely sure
about the answer

72. Used nas antispasmodic: baclofen

73. Drug increase paracetamol toxicity: Ethanol

74. Drug decrease warfarin action: Multivitamins

75. Need to be monitored high alert medication IV potassium chloride or oral digoxin I

Arif Pharma Clinic Copyright@arifpharma.com arifpharma.com


Page 428

chose potassium chloride iv as it may cause arrhythmia answer please check it

76. Lithium toxicity is increased by disturbances in which electrolyte sodium as low


sodium levels increase lithium toxicity

77. Tell a patient about SE of diphenhydramine: Drowsiness

78. Heparin toxicity we use: Protamine

79. Cockroft Gault equation method used: Chronic stable renal failure

80. Drug is dose-dependent in pharmacokinetics: Phenytoin

81. Case pain with endometrial cancer: The answer may be Danazol as the second
choice was combined oral contraceptive which contains estrogen so it can't be used

New Updated DHA Exam Study


Material
Copyright@arifpharma.com

All The Best


Through our YouTube channels Arif Pharma Clinic and Arif Pharma
Academy, we provide study material for pharmacy and nursing staff for
Prometric exams such as DHA Exam, MOH Exam, and HAAD Exam, so that
students who dream of getting a job abroad We try to make things easy. And also
provide GPAT exam study material. Through our team of highly experienced
and qualified faculty headed by Pharmacist Arif Khan, Director of the Arif
Pharma Clinic YouTube Channel, we have managed to touch the minds and lives
of many students. And if you're with me, I'll keep trying.

Arif Pharma Clinic


Subscribe Now & Press The Bell Icon

Arif Pharma Clinic Copyright@arifpharma.com arifpharma.com


Page 429

DHA Practice Set-01


Answer Sheet is below

1- Which of the following NSAIDs is available in a Parenteral form?

A. Ibuprofen
B. Ketorolac
C. Tolmetin
D. Piroxicam
E. None of the above

2- Muscle relaxant are seldom used for more


Than

A. 2 to 3 weeks because Of toxicity


B. Of instability
C. Of short duration of need
D. Of tolerance being developed
E. They are used longer commonly

3- Linezolid is used against infections caused by antibiotic resistant

A. Gram- positive cocci.


B. Gram- negative bacilli
C. HIV virus
D. Gram-positive bacilli
E. None of the above

4-Nateglinide stimulates the release of

A. Insulin
B. Epinephrine
C. Glucose
D. Glucagon
E. Norepinephrine

5- Mifoprestone is a receptor antagonist of which hormone

A. Estrogen
B. Thyroid
C. Insulin

ARIF PHARMA CLINIC Copyright@arifpharma.com arifpharma.com


Page 430

D. Cortisol
E. Progesterone

6- Rivastigmine has the primary action of inhibiting

A. Acetyl cholinesterase
B. Epinephrine
C. Gastric acid
D. Calcium influx
E. Histamine release

7- Which of the following are low molecular- weight heparin?

A. Tinzaparin
B. Enoxaparin
C. Dalteparin
D.B and C
E. A.B and C

8- Ocular timolo is used primary in

A. Glaucoma
B. Pink eye
C. conjunctivitis
D. Eye infections
E. Eye cleaners

9- Sucralfate is used for short term theraoy of

A. Ulcers
B. Hypertension
C. Carcinomas
D. Calcium depletion
E. Dental caries

10- Lorazepam is classified as

A. Loop diuretic
B. MAO inhibitor
C. Thiazide diuretic

ARIF PHARMA CLINIC Copyright@arifpharma.com arifpharma.com


Page 431

D. Dibenzazepine
E. Polycyclic amine

11- Probanthaline is contraindicated in patients with

A. Glaucoma
B. Myasthenia gravis
C. Obstructive disease of Gl tract
D. Ulcerative colitis
E. All of the above

12- Antiepileptics as a group are noted for developing or causing

A. Rashes
B. Atrial tachycardias
C. Tolerance
D. Spasms
E. Headache

13- Selenium sulfide is used

A. Orally
B. Tolerance
C. By injection
D. By none of the a bove
E. By all of the a bove

14-Zonisamide is considered a broad spaectrum

A. Antibiotic
B. Anticonvulsant
C. Antiviral
D. NSAID
E. Antihistamine

15- Baclofen is used primarily as a(an)

A. Sympathomimetic
B. Antianxiety agent
C. Muscle relaxant
D. Antispasmodic

ARIF PHARMA CLINIC Copyright@arifpharma.com arifpharma.com


Page 432

E. Tranquilizer

16- Beta interferon is used to treat

A. Adult- onset diabetes


B. Hypertension
C. cardiac insufficiency
D. Multiple sclerosis
E. Ulcers

17-. Aprevalent side effect of propylthiouracil is

A. Hearing loss
B. Visual impairment
C. Acidosis
D. Leukopenia
E. Muscular spasm

18- Candassartan is used as a(an)

A. Diuretic
B. Antihypertensive
C. Hypnotic
D. Sedative
E. Antidepressant

19- The pharmacologic actions of sulindac include

1.Anti-inflammatory properties
11.Analgesic properties
111.Antipyretic properties

F. 1only
G.11only
H.1and11only
I. 11and111only
J. 1,11and111

20- Mirtazapine is used to treat symptoms of

A. Depression

ARIF PHARMA CLINIC Copyright@arifpharma.com arifpharma.com


Page 433

B. Parkinsonism
C. Gout
D. Petit mal epilepsy
E. None of the above

21 - Ofloxacin is classified as a (an)

A. Antifungal
B. Antiviral
C. Broad-spectrum antibiotic
D. Narrow spectrum antibiotic
E. Antirickettsial

22- Zidovudine may commonly cause which of


The following as a side effect?

A. Vasoconstriction
B. Dryness of the mouth
C. Hyperactivity
D. Anemia
E. Anuria

23- Mupicorin ointment is used topically to treat

A. Conjunctivitis
B. All types of eye infections
C. Impetigo
D. Carcinomas of the skin
E. Hemorrhoids

24- Pentoxifylline acts primarily by

A. Dissolving cholesterol
B. Decreasing viscosity of the blood
C. Increasing biotransformation
D. Oxidative mechanisms
E. Preventing formation of cholesterol

ARIF PHARMA CLINIC Copyright@arifpharma.com arifpharma.com


Page 434

25- Which of the following is most useful in treating hypertensive crisis?

A. Sodium nitorprusside
B. Serpasil
C. Chlorothiazide
D. Spironolactone
E. Triamterene

26- Alloprinol differs from most other agents used to treat gouty conditions
because

A. Does not decrease uric acid levels


B. Prevents formation of uric acid
C. Increase elimination of uric acid
D. Causes rapid biotransformation of uric acid
E. Has analgesic properties

27- Estrogen tend to increase the risk of

A. Endometrial carcinoma
B. Hirsutism
C. Hearing loss
D. Visual problems
E. pregnancy

28- Which of the following drugs is contraindicated when used with sildenafil?

A. Fluroquinolones
B. Organic nitrates
C. Penicillins
D. Calcium channel blockers
E. none of the above

29- Gentamicin exhibits

A. Significant hepatotoxicity
B. Significant cardiotoxicity
C. Significant dermal toxicity
D. Significant nephrotoxicity
E. All of the above

ARIF PHARMA CLINIC Copyright@arifpharma.com arifpharma.com


Page 435

30- Risendronate is classified as a (an)

A. Cardio suppressant
B. Bisphosphonate
C. Diuretic
D. Immunosuppressant
E. Sympathomimetic

31- Carbamazepine is used as an anticonvulsant as well as to treat pain From

A. Kidney infections
B. Burns
C. Muscle injuries
D. Sinus headache
E. Trigeminal neuralgia

32- Methylphenidate is used to treat

A. Fatigue
B. Hyperkinesis disorders
C. Anxiety
D. Depression
E. Obesity

33- Doxorubicin is used to treat

A. A wide variety of infections


B. Gram- positive infections
C. Gram- negative infections
D. Viral infections
E. Carcinomas

34- A drug is useful in treating potentially fatal fungal infections is

A. Nystatin
B. Propionic acid
C. Amphotericin
D. Nystatin
E. Griseofulvin

ARIF PHARMA CLINIC Copyright@arifpharma.com arifpharma.com


Page 436

35- The use of thyroid hormones in the treatment of obesity is

A. Widely accepted
B. Unjustified
C. Acceptable in combination with other drugs
D. Only acceptable if other treatments fail
E. Acceptable if the patient is closely monitored

36- An advantage of dextromethorphan ovecodeine as an antitussive is that it

A. Is twice as affective
B. Is more stable
C. Has no side effects
D. Produces very little depression of the CNS
E. Has better analgesic properties

37- Nonselective 2- adrenoreceptor antagonists are used primarily in

A. Peripheral vascular disorder


B. Tachycardia
C. Migraine headache
D. Atherosclerosis
E. Renal insufficiency

38- Antimuscarinic drugs are contraindicated in

A. Narrow angle glaucoma


B. Paralytic ilens
C. Pyloric or intestinal obstruction
D. A and B
E. A, B and C

39- Zanamivir and ostelamivir are both FDA approved for

A. Treatment of influenza A infection


B. Treatment of influenza B infection
C. Prophylaxis of influenza infections
D. A and B
E. A, B and C

ARIF PHARMA CLINIC Copyright@arifpharma.com arifpharma.com


Page 437

40- Which of the following drugs is not a proton pump inhibitor?

A. Pantoprazole
B. RabeprazolE
C. Lansoprazole
D. Sulfisoxazole
E. Omeprazole

41- A danger of prolonged use of pilocarpine salt as a miotic is

A. Tearning
B. Glaucoma
C. Conjunctivitis
D. Detached retina
E. Lens opacity

42- A serious side effect of furosemide in treating heart patients is that it

A. Interact with digitalis glycosides


B. Causes arterial blockade
C. May cause anuria
D. Causes hypertension
E. May lose its effects

43- Which of the following drugs require electrocardiographic monitorizing


Prior to infection of therapy?

A. Ziprasidone
B. Arsenic trioxide
C. Cisapride
D. A and B
A, B and C

44- Lumivudine is usually classified as a(an)

A. H2 antagonist
B. Ulcer protectant
C. Antiviral

ARIF PHARMA CLINIC Copyright@arifpharma.com arifpharma.com


Page 438

D. Oral antidiabetic
E. Anthelmintic

45- Prolonged usages of sublingual nitrates is likely to cause

A. Ulcers
B. Anuria
C. Rashes
D. Development of tolerance
E. Persistent headache

46- Streptokinase is used to

A. Dissolve blood clots


B. Treat digestive disorders
C. Promote carbohydrate degradation
D. Treat muscle injuries
E. Replace pepsin

47- Gold compounds have been used to teat

A. Worm infestations
B. Ulcers
C. Kideny failure
D. Rheumatoid arthritis
E. Psoriasis

48- Tamoxifen is classified as a(an)

A. Estrogen
B. Antiestrogen
C. Androsterone
D. Testosterone
E. Thyroid hormone

49-Beriberi is associated with a deficiency of

A. Vitamin D

ARIF PHARMA CLINIC Copyright@arifpharma.com arifpharma.com


Page 439

B. Thiamine
C. Vitamin C
D. Niacin
E. Riboflavin

50- A drug is used to treat delirium tremens is

A. Chlordiazepoxide
B. Haloperidol
C. Disulfiram
D. Methadone
E. None of the above

51- Cromolyn sodium acts by

A. Destroying histamine
B. Releasing histamine
C. Bio-transforming histamine
D. Preventing the release of histamine
E. None of the above

52- The anti-inflammatory action of aspirin is due to

A. Analgesia
B. Inhibition of clotting
C. Antipyretic effect
D. Degradation of prostaglandins
E. Inhibition of prostaglandins synthesis

53- Castor oil is classified as which type of laxative?

A. Lubricating
B. Anthraquinone
C. Irritant
D. Stool softening
E. Bulk producing

ARIF PHARMA CLINIC Copyright@arifpharma.com arifpharma.com


Page 440

54- Penicillamine is most commonly used to treat

A. Parkinsonism
B. Wilson's disease
C. Neoplasms
D. Raynaud's disease
E. Gram- positive infections

55- Which of the following durg are withdrawn from the US market
In 2001 due to increased risk of myopathy and rhabdomyolysis?

A. Cerevastatin
B. Cisapride
C. Dexfenfluramine
D. Terfenadine
E. Astemizole

56- Doxycycline oral gel is used to treat

A. Periodontitis
B. Aphthous ulcers
C. Acne
D. Lyme disease
E. Arthritis

57- Which of the following drugs requires a dosage adjustment in patients with
renal impairment?

A. Famotidine
B. Capecitabine
C. Gentamicin
D. A and C
E. A, B and C

58- Aprevalent side effect of norethindrone is

ARIF PHARMA CLINIC Copyright@arifpharma.com arifpharma.com


Page 441

A. Diarrhea
B. Breakthrough bleeding
C. Blood Dyscrasiency
D. Cardiac insufficiency
E. Abortion

59- the preferred way to offset hypokalemia is to

A. Eat citrus fruit


B.Eat sea food or fish
C. Administer potassium salt
D. Administer IV electrolytes
E. Dimnish urination

60- Persons receiving MAO inhibitors should control their intake of

A. Some fermented and beverages


B. Carbohydrates
C. Fats and lipids
D. Water
E. salicylate analgesics

61- Which of the following drug's may inhibit the metabolism of ziprasidone?

I. Erythromycin
II. Ketoconazole
III. Phenobarbital

A. I only
B. II only
C. I and II only
D. II and III only
E. I, II and III

62- Flumazenal is a specific antagonist for which of the following drugs?

I. Meperidine

ARIF PHARMA CLINIC Copyright@arifpharma.com arifpharma.com


Page 442

II. Propoxyphene
III. Diazepam

A. I only
B. III only
C. I and II only
D. II and III only
E. I, II and III

63- Which of the following antidepressant also had an indication for smoking
cessation?

A. Haloperidol
B. Bupropion
C. Citalopram
D. Mirtazapine
E. Paroxetine

64- Docosanole is a (an)

A. Diuretic
B. Nonnarcotic analgesic
C. Anti- inflammatory
D. Antiviral
E. Anthelmintic

65- Which of the following statements is true regarding the


Administration of alendronate?

I. Take more than 30 minutes before first food or beverage of the day
II. Take with 6 to 8 ounces of plain water
III. Remain fully upright (sitting or standing) for about 30 minutes

A. I only
B. III only
C. I and II only
D. II and III only
E. I, II and III

ARIF PHARMA CLINIC Copyright@arifpharma.com arifpharma.com


Page 443

66- Testosterone is available as which of the following dosage form?


I. Ointments
II. Injection
III. Transdermal patch

A. I only
B. III only
C. I and II only
D. II and III only
E. I, II and III

67- Which of the following is an oral hypoglycemic agent?

A. Ketorolac
B. Pioglitazone
C. Doconasol
D. Riluzole
E. Terbinafine

68- The concurrent administration (within 1 to 2 hours of dosing) of nislopidine


should be avoided with which of the following?
I. High _ fat meal
II. Grapefruit juice
III. Cocoa

A. I only
B. III only
C. I and II only
D. II and III only
E. I, II and III

69- Which of the following drugs may be administered without regard to meals?

I. Valaciclovir
II. Cetirizine
III. Acarbose

A. I only
B. II only

ARIF PHARMA CLINIC Copyright@arifpharma.com arifpharma.com


Page 444

C. I and II only
D. II and III only
E. I, II and III

70- Dexfenfluramine, an agent approved for the treatment of obesity, was


removed from the U.S market due to

A. Cardiac valvular dysfunction


B. Gastric ulcerations
C. Renal toxicity
D. Ototoxicity
E. Hepaic impairment

71- The terminal half-life of alendronate is

A. 10 days
B. 10 months
C. 10 years
D. 10 hours
E. 10 minutes

72- Serotonin syndrome is characterized by which of the following syndrome?

I. Mental states changes


II. Tremor
III. Diaphoresis

A. I only
B. III only
C. I and II only
D. II and III only
E. I, II and III

73- the FDA has recommended the removal of which ingredient found in
cough/cold products due to an association with hemorrhagic strike?

A. Dextromethorphan
B. Psedoephedrine
C. Guanfenisen
D. Phenylpropanolamine
E. Ephedrine

ARIF PHARMA CLINIC Copyright@arifpharma.com arifpharma.com


Page 445

74- Which of the following antacids should not be used in dialysis patients?

l. Aluminum hydroxide
ll. Magnnesium hydroxide
lll. Calcium carbonate

A. l only
B. ll only
C. l and lll only
D. ll and lll only
E. l,ll and lll

75- Melatonin regulates which of the following functions?

l. Sleep
ll. Circadian
lll. Respiration

A. l only
B. ll only
C. l and ll only
D. ll and lll only
E. l, ll and lll

76- Tacrolimus ointment is used primarily to treat

A. Pain
B. Atopic dermatitis
C. Sunburns
D. Skin grafts
E. Poison ivy

77- In 2001, Which analgesic experienced significant misuses and diversion,


requiring revisions in the warning section of the product labeling?

A. Oxy contain
B. Demerol
C. Tylenol #3
D. Toradol

ARIF PHARMA CLINIC Copyright@arifpharma.com arifpharma.com


Page 446

E. Roxanol

78- Which of the following drug combination has (have) been effective therapy for
the treatment of Helicobacter pylori?

l. Clarithromycin/ ranitidine bismuth citrate


ll. Omeprazole/ clarithromycin
lll. Ciprofloxacin/ erythromycin

A. l only
B. ll only
C. l and ll only
D. ll and lll only
E. l ,ll and lll

79- Ziprasiodne, an agent used to treat schizophrenia , exerts its pharmacological


effects by

A. Inhibiting histamine reuptake


B. Stimulating serotonin synthesis
C. Blocking dopamine
D. Stimulating dopamine release
E. None of the above

80- Ocular timolol is used primarily to treat

A. Mydriasis
B. Glaucoma
C. Cataracts
D. Conjunctivitis
E. None of the above

81- Cetirizine is a

A. Nonspecific histamine antagonist


B. Histamine 1 antagonist
C. Histamine 2 antagonist
D. Histamine 1 agonist
E. Histamine 2 ahonist

ARIF PHARMA CLINIC Copyright@arifpharma.com arifpharma.com


Page 447

82- Bivalirudin is a (an)


A. Anticoagulant
B. Anti- infective
C. NSAID
D. Antiviral
E. Antihistamine

83- Cevimeline is used to treat

A. Dermatitis
B. Xerostomia
C. Allergic rhinitis
D. Tinea versicolor
E. Eczema

84- Crotalide is used to treat the bites of


A. Fleas
B. North American vipers
C. Ticks
D. Rabid animals
E. Black widow spiders

85- Which of the following statements regarding fosphenytoin are true?

A. Fosphenytoin is completely converted to phenytion after intravenous or I


ntramuscular administration
B.The administration of 1 mmol of Phenytion
C. Monitoring serum phenytion leveis is not necessary

A. l only
B. ll only
C. l and ll only
D. ll and lll only
E. l, ll and lll

86- The principal adverse effect of acarbose is

A. Hematologic
B. Gastrointestinal

ARIF PHARMA CLINIC Copyright@arifpharma.com arifpharma.com


Page 448

C. Renal
D. Hepatic
E. D. ermal

87- Unoprostone is FDA approved to treat

A. Ovarian cancer
B. Hodgkin's disease
C. Open angel glaucoma
D. Conjunctivitis
E. Tinnitus

88- Oral parenteral ketorolac therapy is limited to 5 days of therapy due to an


increased risk of

A. Renal impairment
B. Gastro intestinal bleeding/ perforation
C. Liver failure
D. CNS disturbances
E. A and B

89- Which of the following HMG-CoA reductase inhibitors causes the greatest
percentage increase in HDL?

A. Simvastatin
B. Pravastatin
C. Lovastatin
D. Fluvastatin
E. All produce *****alent reductions

90- Sibutramine should not be used in patients

l. With a body mass index greater than 30 kg/m2


ll. Taking MAO inhibitors
lll. With poorly controlled hypertension

A. l only
B. lll only
C. l and ll only

ARIF PHARMA CLINIC Copyright@arifpharma.com arifpharma.com


Page 449

D. ll and lll only


E. l, ll and lll

91- Diuretics tend to enhance lithium salt toxicity due to

A. Sodium depletion
B. Potassium depletion
C. Direct drug interaction
D. Increased absorption
E. Increased solubility of the lithium salts

92- Leflunamide is used to treat

A. Crohn's disease
B. Rheumatoid arthritis
C. Psoriasis
D. Photoallergic reactions
E. None of the above

93- Sumatriptan use is contraindicated in patients

l. With a history of ischemic heart disease


ll. Taking phenelzine
lll. With uncontrolled hypertension

A. l only
B. lll only
C. l and ll only
D. ll and lll only
E. l ,ll and lll

94- Eptifibatide is classifide as a (an)

A. Fluoroquinolone
B. Glycoprotein llb/ llla receptor inhibitor
C. Diuretic
D. Antihypertensive
E. None of the above

ARIF PHARMA CLINIC Copyright@arifpharma.com arifpharma.com


Page 450

95- Finasteride's action in the treatment of benign prostatic hypertrophy is a


(an)

A. HMG- CoA reductase inhibitor


B. Alpha-5reductase inhibitor
C. Serotonin reuptake inhibitor
D. P450 hepatic enzyme inhibitor
E. None of the above

96- A useful agent in the treatment of bladder incontinence is

A. Nalmefene
B. Olterodine
C. Tolcapone
D. Valrubicin
E. None of the above

97- Which of the following agents are approved for treatment of narcolepsy?

A. Modafinil
B. Methylphenidate
C. Dextroamphetamine

A. l only
B. lll only
C.l and ll only
D. ll and lll only
E. l , ll and lll

98- Misoprostol is used to

A. Treat gastric ulcers


B. Prevent NSAID-induced gastric ulcrrs
C. Prevent osteoporosis
D. Prevent renal toxicity
E. Treat Paget's disease
99- Acarbose, a hypoglycemic agent, exerts which of the following mechanisms of
action?

ARIF PHARMA CLINIC Copyright@arifpharma.com arifpharma.com


Page 451

A. Increased insulin secretion


B. Increased insulin receptor sensitivity
C. Decreases circulating insulin antibiotics
D. Delays digestion of carbohydrates
E. None of the above

100- Which of the following drugs interferes with the metabolism of indinavir?

A. Ketoconazole
B. Ciprofloxacin
C. Digoxin
D. Lorazepam
E. None of the above

MCQ Answer Sheet

ARIF PHARMA CLINIC

हम आपके साथ है तो DHA आपकी मुठ्ठी में है

arifpharma.com

All The Best


DHA Practice Set With Answer Sheet Set

ARIF PHARMA CLINIC Copyright@arifpharma.com arifpharma.com


Page 452

Q. 1 Sodium chromoglycate is an.

• A. Capillary constriction

• B. Bronchodilator

• C. Mast cell stabilization

• D. Leukotrienes modifier

Q. 2 Anaphylactic shock is treated using.

• A. Salbutamol

• B. Allopurinol

• C. Epinephrine

• D. All the above

Q. 3 Respiratory acidosis is caused due to.

• A. CO2 retention

• B. CO2 removal

• C. O2 removal

• D. O2 retention

Q. 4 Indomethacin is the ........... choice for acute gout.

• A. Last

• B. First

• C. Second

• D. All the above

Q. 5 Which is true for insulin dosing.

• A. Initial dose 0.6 u/kg/day split 2/3 A.M and 1/3 P.M

• B. Regular NPH ratio has to be followed i.e 1: 1 or 1: 2

• C. dosage can be increased during acute illness
• D. All the above

Copyright @Arifpharma.com Arif Pharma Clinic


Page 453

Q. 6 Along with CNS depressant which of the following cannot be taken.

• A. Diphenhydramine

• B. Analgesic

• C. Aspirin

• D. Verapamil

Q. 7 which of the following cannot be used while treating hypertension using drug therapy
?.

• A. For asthmatic patient do not use beta-blockers



• B. For asthmatic patient use beta-blockers

• C. For pregnant women use ACEIs

• D. None of the above

Q. 8 Choose the characteristic of Erythrosine.

• A. Can be used for pregnant woman



• B. Cannot be used for pregnant woman

• C. decrease GIT motility

• D. All the above

Q. 9 Select the antiarrythmic class? a.

• A. Lidocaine

• B. Quinidine

• C. Amiodarone

• D. Captopril

Q. 10 Identify the antifungal element.

• A. Gamciclovir

• B. ampicillin

• C. amoxicillin

• D. All the above

Copyright @Arifpharma.com Arif Pharma Clinic


Page 454

Q. 11 Histamine performs.

• A. Elevation of blood pressure


• B. Raise in heart rate

• C. Stimulates gastric secretion

• D. Prostaglandins modifier

Q. 12 Select the hormone that is predominant during ovulation phase.

• A. FH

• B. SH

• C. LH

• D. FSH

Q. 13 Spinal anesthesia is done using.

• A. Tetracaine

• B. Pindolol

• C. Atenolol

• D. Hydralazine

Q. 14 Which of the following is found in Viagra.

• A. Albumin

• B. Sildenafil

• C. Dextra

• D. None of the above

Q. 15 if one part of molecule is soluble in a 10,000 Part of water, then it is called.

• A. Insoluble

• B. Immiscible

• C. Slightly soluble

• D. All the above

Q. 16 Picosulphate can be used in the form of.

Copyright @Arifpharma.com Arif Pharma Clinic


Page 455

• A. ttt
• B. Laxative
• C. Morphine
• D. All the above

Q. 17 G+ve, G-Ye is present in.

• A. 3rd generation cephalosporin



• B. 1st generation cephalosporin

• C. 2nd generation cephalosporin

• D. None of the above

Q. 18 A pound is equal to.

• A. 453 kg

• B. 0.453 kg

• C. 4.53 kg

• D. 45.3 kg

Q. 19 Shrinking of blood cell is caused due to.

• A. Infusion of hypertonic solution in blood



• B. Extraction of hypertonic solution in blood

• C. infusion of hypotonic solution in blood

• D. None of the above

Q. 20 Select the metal that can be used in rheumatoid arthritis.

• A. Silver

• B. Gold

• C. Bronce

• D. All the above

Q. 21 Choose the antibiotics that is resistant to penicillinase.

• A. Penicillin V

Copyright @Arifpharma.com Arif Pharma Clinic


Page 456


• B. Floxapen

• C. Amoxicillin

• D. None of the above

Q. 22 Growth hormone is.

• A. Excreted from adrenal cortex



• B. Not excreted from adrenal cortex

• C. Is a mineral corticoids

• D. None of the above

Q. 23 Epinephrine - find the false one.

• A. Increaser B.P

• B. Vasoconstrictive

• C. Effective orally

• D. None of the above

Q. 24 Expand GMP.

• A. Good Manufacture Product



• B. General Manufacturing Protocol

• C. General Manufacturing Product

• D. Good Manufacturing Protocol

Q. 25 Electromagnetic rays are.

• A. Gamma rays

• B. X-rays

• C. UV rays

• D. A & B

Q. 26 Select the drug that generates lupus syndrome.

• A. Procainamide

Copyright @Arifpharma.com Arif Pharma Clinic


Page 457

• B. Lidocaine

• C. Cloramphinicol

• D. Amikacine

Q. 27 Choose the meaning for - Store in cool place.

• A. Store in refrigerator

• B. Store in freezer

• C. Store in room temperature

• D. Store at 10-20c

Q. 28 Ibuprofen has.

• A. Anti inflammatory effect



• B. Inflammatory effect

• C. Both A & B

• D. None of the above

Q. 29 ISONIAZIDE is used for treating.

• A. Tuberculosis

• B. Peptic ulcer

• C. Thiazides

• D. None of the above

Q. 30 Sodium Lauryl Sulphate is an.

• A. Surfactant that cannot be used orally



• B. Surfactant that can be used orally

• C. is an antidote

• D. All the above

Q. 31 Select the action performed by Inderal.

• A. b -adrenergic receptor blocker



• B. Similar to ergotamine

Copyright @Arifpharma.com Arif Pharma Clinic


Page 458

• C. Similar to tubocurarine

• D. Similar to antihistamine

Q. 32 Prazocin acts as an.

• A. Alpha blocker

• B. Alpha 1 blocker

• C. Alpha 2 blocker

• D. Alpha 3 blocker

Q. 33 Ascobic acid belongs to.

• A. Vitamin A

• B. Vitamin B

• C. Vitamin C

• D. Vitamin D

Q. 34 Postural Hypotension is the side effect caused due to.

• A. Cefixime

• B. Alpha blocker

• C. Ventamix

• D. None of the above

Q. 35 Erythromycin cab be used for the patients allergic to.

• A. Penicillins

• B. Claforan

• C. Verapamil

• D. All the above

Q. 36 Raloxifine is a selective estrogen modulator that can be contraindicated in -------- --.

• A. LDL Patient

• B. DVT Patient

• C. CHF Patient

Copyright @Arifpharma.com Arif Pharma Clinic


Page 459


• D. None of the above

Q. 37 Parkinsonism is treated using.

• A. Nicotine

• B. Dopamine

• C. Verapamil

• D. All the above

Q. 38 Select the antiviral for herpes simplex.

• A. Idoxrubicin

• B. Dipivefrene

• C. Danocratic

• D. None of the above

Q. 39 Bio availability means.

• A. The drug reaches the blood circulation



• B. The drug reaches the area of action

• C. The drug reaches the lungs

• D. All the above

Q. 40 Select the side effect caused by phenytoin.

• A. Hypoglycemia

• B. Gingival hyperplasia

• C. Acne

• D. All the above

Q. 41 Rifampicin treats.

• A. Psoriasis

• B. Tuberculosis

• C. Head Ache

Copyright @Arifpharma.com Arif Pharma Clinic


Page 460

• D. None of the above

Q. 42 Which of the following drug cannot treat acute pain.

• A. Prednisolone

• B. Colchicine

• C. Naproxen

• D. Codeine

Q. 43 Allopurinol is an.

• A. Renal tubular reabsorption agent



• B. Antiinflammatory agent

• C. Inflammatory agent

• D. All the above

Q. 44 Identify the colloid solution in the following.

• A. Albumin 5%

• B. Beta starch

• C. Albumin 20%

• D. All the above

Q. 45 Select the first line agent for Obese type 2.

• A. Metformin

• B. Glyburinid

• C. Phenytoin

• D. All the above

Q. 46 If 4mg of lorazepam is dosed to an adult will lead to.

• A. Diuretic

• B. Hypnotic

• C. Head ache

• D. None of the above

Copyright @Arifpharma.com Arif Pharma Clinic


Page 461

Q. 47 Select the one that is used in theophylline toxicity management.

• A. Methylphenidate to reduce excessive sedation



• B. Activated charcoal to enhance elimination

• C. Reducing proteinurea.

• D. All the above

Q. 48 Choose the anti inflammatory having analgesic effect.

• A. Ibuprofen

• B. Aspirin

• C. Naprosyn

• D. All the above

Q. 49 Select the optimal size of an ophthalmic drops.

• A. <100 microgram

• B. <1 microgram

• C. <10 microgram

• D. <0.10 microgram

Q. 50 Class 3 Antiarrythmic drugs works through.

• A. Suppressing phase 4 depolarization and by adrenergic receptor blocking



• B. Raising phase 4 depolarization

• C. adrenergic receptor release

• D. None of the above

Q. 51 ceftazidime belongs to.

• A. Third generation cephalosporin



• B. First generation cephalosporin

• C. Second generation cephalosporin

• D. None of the above

Q. 52 Choose the drug that acts as a skeletal muscle relaxant.

Copyright @Arifpharma.com Arif Pharma Clinic


Page 462

• A. Digoxin

• B. Vecuronium

• C. Diazoxide

• D. All the above

Q. 53 Select the level for Half Life of first order kinetic.

• A. 0.54k

• B. 0.693k

• C. 1k

• D. .692k

Q. 54 Select the colloid in the following.

• A. albumin 20%

• B. Beta starch

• C. Dextran

• D. All the above

Q. 55 Choose the daily dosage of the folic acid.

• A. 400-500 microgram

• B. 100-150 microgram

• C. 200-300 microgram

• D. None of the above

Q. 56 Digoxin toxicity causes.

• A. Asthmatic

• B. Hepatic

• C. Renal failure

• D. Heart failure

Q. 57 Reducing the particle size and increasing the surface area enhances.

• A. Solubility

Copyright @Arifpharma.com Arif Pharma Clinic


Page 463


• B. Insolubility

• C. Levigation

• D. All the above

Q. 58 If a drug is metabolized in liver it become.

• A. Polar

• B. Soluble

• C. Insoluble

• D. Non Polar

Q. 59 Choose the component that denaturants in the body.

• A. Carbohydrate

• B. Fat

• C. Proteins

• D. All the Above

Q. 60 Generally the drugs belong to.

• A. Strong electrolyte

• B. Weak electrolyte

• C. Non electrolyte

• D. Electrolyte

Q. 61 Dopamine is used for.

• A. Hypodynamic shock treatment



• B. Hyperdynamic shock treatment

• C. Shock treatment

• D. All the above

Q. 62 Pharmacokinetic constant belongs to.

• A. First order

Copyright @Arifpharma.com Arif Pharma Clinic


Page 464

• B. Zeroth order

• C. Second order
• D. All the above

.Q. 63 Expand GMP .

• A. Good Manufacturing Practice



• B. Good Manufacturing Protocol

• C. General Manufacturing Protocol

• D. All the above

Q. 64 Choose the side effects of Cyclophosphamide.

• A. Skin rash

• B. Vomiting

• C. Hair loss

• D. All the above

Q. 65 Over dosage of Digoxine is used for.

• A. anti dote FAB Fragment



• B. increase FAB Fragment

• C. Decrease FAB Fragment

• D. All the above

Q. 66 During ECG what is the effect caused by Digitoxine.

• A. Shorten P.R

• B. Prolong P.R

• C. Maintain P.R

• D. None of the above

Q. 67 Buspirone acts on.

• A. Serotonin (5HT2a) receptor



• B. Hypernatrimia

Copyright @Arifpharma.com Arif Pharma Clinic


Page 465

• C. Hypokalaemia

• D. Hyeruriceamia

Q. 68 Passive diffusion means.

• A. Drug transfers through cell membrane



• B. Drug transfers through blood

• C. Drug induced orally

• D. None of the above

Q. 69 Short acting B- blocker for Acute arrhythmia as IV administration.

• A. Cyclizine

• B. Esmolo

• C. Penicillin

• D. All the above

Q. 70 Isopretrenol is an.

• A. Alpha & Beta agonist



• B. Beta agonist

• C. Alpha agonist

• D. All the above

Q. 71 Select the dosage of cefixime.

• A. 800mg

• B. 1500mg

• C. 1000mg

• D. 400mg

Q. 72 Tetracaine is treated for.

• A. Spinal anesthesia

• B. Allergic

Copyright @Arifpharma.com Arif Pharma Clinic


Page 466


• C. Opioid analgesic

• D. All the above

Q. 73 Promethasine treats.

• A. Antihistamic

• B. Chlorodiazepoxide

• C. Diazepam

• D. All the above

Q. 74 Verapamil is a.

• A. Liver enzyme inhibitor



• B. Lungs enzyme inhibitor

• C. Heart enzyme inhibitor

• D. All the above

Q. 75 Diloxamide furoate drug is an.

• A. Systemic amoebicide

• B. Schistosomiasis

• C. Influenza A infection

• D. All the above

Q. 76 Choose the side effect that is not due to betamethasone.

• A. Hypoglycemia

• B. Muscle weakness

• C. Lowered resistance

• D. None of the above

Q. 77 Choose the natural Estrogens.

• A. Oestrone

• B. Oestriol

Copyright @Arifpharma.com Arif Pharma Clinic


Page 467

• C. Oestradiol

• D. All the above

Q. 78 Select the dosage of Hepatitis Vaccine.

• A. Only once in An year



• B. Twice in An year

• C. Thrice in An year

• D. All the above

Q. 79 Select the antibiotic that resist penicillinase.

• A. Floxapen

• B. ampicillin

• C. penicillin

• D. All the above

Q. 80 Choose the one that affects adsorption is.

• A. Physical Rx

• B. Phemical Rx

• C. Irreversible

• D. All the above

Q. 81 the term shock refers to.

• A. hypoperfusion

• B. hypotension

• C. hypertension

• D. None of the above

Q. 82 Select the initial symptoms of aspirin poisoning.

• A. Ringing in the ears



• B. Blurred vision

• C. Fatigue

Copyright @Arifpharma.com Arif Pharma Clinic


Page 468


• D. All the above

Q. 83 Inorder to control reflex tachycardia,--------------- is prescribed along with Propranolol.

• A. Hydralazine

• B. Phenytoin

• C. Quinidine

• D. None of the above

Q. 84 Morphine controls.

• A. Acute myocardial infarction pain



• B. Joint pains

• C. Discolouration of urine

• D. All the above

Q. 85 Side effect caused byACEI's.

• A. Reducing proteinurea

• B. Chronic dry cough

• C. Vomiting

• D. All the above

Q. 86 Suspension.

• A. Needs filtration before every process



• B. Needs filtration before IV infusion

• C. Does not need any filteration

• D. None of the above

Q. 87 Gastric acid secretion is reduced using.

• A. Omeprazole

• B. Dopaminergic

• C. Amiloride

Copyright @Arifpharma.com Arif Pharma Clinic


Page 469

• D. All the above

Q. 88 Chiral molecule performs.

• A. Optical activity

• B. Solubility

• C. InSoluble

• D. All the above

Q. 89 Bambuterol treats.

• A. Asthma

• B. Head ache

• C. INR

• D. All the above

Q. 90 Choose the disease due to the viral infection.

• A. Rabies

• B. Herpes

• C. Chicken pox

• D. All the above

Q. 91 Fibrinolytic is manufactured using.

• A. Alteblase

• B. Salbutamol

• C. Podocodeine

• D. None of the above

Q. 92 Select the one that belongs to phenothiazines group.

• A. Prochlorperazine

• B. Primidone

• C. Alkaloids

• D. None of the above

Copyright @Arifpharma.com Arif Pharma Clinic


Page 470

Q. 93 Choose the side effect caused due to oral contraceptive.

• A. Hypertension

• B. Dizziness

• C. Weight gain

• D. All the above

Q. 94 Nature of the solute and the type of the solution is defined as.

• A. Insolubility analysis

• B. Solubility analysis

• C. Determination analysis

• D. All the above

Q. 95 Hemolytic is due to the.

• A. Infusion of hypotonic solution in blood



• B. Infusion of hypertonic solution in blood

• C. Infusion of hypotonic solution in tissue

• D. All the above

Q. 96 homogenous means.

• A. True solutions

• B. Soluble solutions

• C. Insoluble solutions

• D. None of the above

Q. 97 Choose the side effect of Methotrixate.

• A. Bone marrow depression



• B. Alopecia

• C. Nausea

• D. All the above

Q. 98 Triazolam results.

Copyright @Arifpharma.com Arif Pharma Clinic


Page 471

• A. less sedative action



• B. More sedative action

• C. Medium sedative action

• D. None of the above

Q. 99 Choose the side effect of predinsolone.

• A. Cataract

• B. Sodium retention

• C. Skeletal weakness

• D. All the above

Q. 100 Cephalosporin is an.

• A. Inhibitors of cell membrane function



• B. Inhibitors of metabolism

• C. Inhibitors of protein synthesis

• D. Inhibitors of cell wall synthesis

Q. 101 Inderal (propranolol) is used to treat.

• A. Angina

• B. High blood pressure

• C. Heart rhythm disorders

• D. All the above

Q. 102 Viral infection leads to.

• A. Chicken pox

• B. Poliomyelitis

• C. Rabies

• D. All the above

Q. 103 airway control, IV of crystalloid solution, monitor heart rhythm and dopamine to
support blood pressure has to be performed during the treatment of.

Copyright @Arifpharma.com Arif Pharma Clinic


Page 472

• A. Shock

• B. Bacterial Infection

• C. Heart attack

• D. None of the above

Q. 104 Choose the odd one about Infliximab.

• A. Anti TNF

• B. Reduce inflammation of Crohn's disease

• C. Rheumatoid arthritis

• D. IL-1 blocker

Q. 105 Carbamazepine is an.

• A. Dose Dependant pharmaceutical therapeutic doses



• B. Dose Independant pharmaceutical therapeutic doses

• C. Pharmaceutical therapeutic doses

• D. None of the above

Q. 106 Contraindication may be.

• A. Absolute

• B. Relative

• C. Both A & B

• D. None of the above

Q. 107 Hypertension is caused due to.

• A. Contraindication of OTC drug



• B. Indication of OTC drug

• C. Contraindication of OTC sympathomimetic drug

• D. None of the above

Q. 108 Digoxin toxicity is measured regularly.

• A. checking Mg levels

Copyright @Arifpharma.com Arif Pharma Clinic


Page 473


• B. checking K levels

• C. Monitor ECG

• D. All the above

Q. 109 Azaleic acid is used to cure.

• A. Acne

• B. Clear the bumps & lesions

• C. Swelling caused by rosacea

• D. All the above

Q. 110 Expiry - January 2015 refers to ...It can be used till.

• A. 31 January

• B. 31 December

• C. 1 January

• D. 1 January

Q. 111 A molecule is said to be Chiral.

• A. If it is superimposable on its mirror image



• B. If it is not superimposable on its mirror image

• C. Solubility

• D. None of the above

Q. 112 Traveler diarrhea is caused by the ................ Bacteria.

• A. E-coli

• B. Salmonella

• C. Campylobacter

• D. Listeria

Q. 113 Indications for procainamide and NAPA serum level monitoring includes.

• A. ethanol

Copyright @Arifpharma.com Arif Pharma Clinic


Page 474

• B. ofloxacin

• C. quinidine, ranitidine, trimethoprim

• D. All the above

Q. 114 Charcoal acts due to.

• A. Absorption

• B. Neutralization

• C. Adsorption

• D. None of the above

Q. 115 Alteblase cures.

• A. Heart attacks

• B. Strokes

• C. Blood clots

• D. All the above

Q. 116 Primidone is a minor first-generation.

• A. Antiepileptic drug

• B. Drug

• C. Prodrug

• D. None of the above

Q. 117 Select the Narcotics that is banned today.

• A. Heroin

• B. Codeine

• C. Nicotine

• D. Morphine

Q. 118 Levigation is a process of.

• A. An insoluble substance is grinded to a fine powder when moist



• B. An insoluble substance is grinded to a fine powder when dry

Copyright @Arifpharma.com Arif Pharma Clinic


Page 475


• C. An soluble substance is grinded to a fine powder when moist

• D. An soluble substance is grinded to a fine powder when dry

Q. 119 loading dose means.

• A. Large initial dose or a series of doses given to achieve a therapeutic


concentration

• B. Small initial dose or a series of doses given to achieve a therapeutic
concentration

• C. Acheives therapeutic failure

• D. All the above

Q. 120 Denaturation results when combined with.

• A. Proteins

• B. Carbohydrates

• C. Fat

• D. None of the above

Q. 121 Desmopressin is used for treating.

• A. Antidiuretic in central diabetes insipidus



• B. Nocturnal enuresis

• C. Antihemorrhagic in hemophilia and von will ebrand's disease.

• D. All the above

Q. 122 Solutions reatcs.

• A. Faster

• B. Accurate

• C. Stable

• D. None of the above

Q. 123 Massive PE causes.

• A. Hemodynamic instability

Copyright @Arifpharma.com Arif Pharma Clinic


Page 476

• B. Congestive heart failure



• C. Chronic lung disease

• D. All the above

Q. 124 Deltiazem acts.

• A. Increases metabolism

• B. Inhibitor of the CYP3A4 enzyme

• C. A & B

• D. None of the above

Q. 125 Monooxygenases are enzymes that.

• A. Catalysed by cytochrome P250



• B. Catalysed by cytochrome P450

• C. Catalysed by cytochrome P150

• D. None of the above

Q. 126 Pseudomonal infections can be treated using the antibiotic.

• A. Meropenem

• B. Imipenem

• C. Both A & B

• D. Amoxicillin

Q. 127 Choose the side effect of cyclophosphamide.

• A. Bone marrow depression



• B. Blared vision

• C. Decrease blood Pressure

• D. Drowsiness

Q. 128 Cephalexin treats.

• A. Bacteria infection

• B. Respiratory infections

Copyright @Arifpharma.com Arif Pharma Clinic


Page 477


• C. Ear infections

• D. All the above

Q. 129 Daily dose of Vitamin A during deficiency is.

• A. 30,000-50,000 IU

• B. 3000-5000 IU

• C. 300-500 IU

• D. 3,00,000-5,00,000 IU

Q. 130 Choose the Vasodilator from the given list.

• A. Nitroglycerin

• B. Zidovudine

• C. Hypercalcemia

• D. None of the above

Q. 131 Labitolol is a.

• A. Non selective b1 blocker



• B. Non selective a1 blocker

• C. Non selective b blocker

• D. None of the above

Q. 132 P.c refers to ................ in Latin abbreviation.

• A. After Meals

• B. Before Meals

• C. Before Sleep

• D. None of the above

Q. 133 Choose the heart`s dominant pacemaker.

• A. SA node

• B. AV node

Copyright @Arifpharma.com Arif Pharma Clinic


Page 478

• C. PV node

• D. None of the above

Q. 134 Cardiac muscle cells or cardiomyocytes send out electrical impulses due to the
property.

• A. Automaticity or spontaneous depolarization



• B. Inotropy

• C. Depolarization

• D. None of the above

Q. 135 Bacterial infection releases.

• A. Exotoxin

• B. Endotoxin

• C. Vomiting Sensation

• D. Cytotoxin

Q. 136 Rheumatoid arthritis (RA) is a.

• A. Autoimmune disorder

• B. Warm, swollen, and painful joints

• C. Causes pain in the feet, hips, knees, and hands

• D. All the above

Q. 137 Propranolol is used along with hydralazine.

• A. Increase the reflex tachycardia



• B. Reduce the reflex tachycardia

• C. Increases Oedema

• D. None of the above

Q. 138 ................control pain during acute myocardial infarction.

• A. Morphine

• B. Zetzine

Copyright @Arifpharma.com Arif Pharma Clinic


Page 479

• C. Bethidine

• D. All the above

Q. 139 Primaquine has to be avoided for.

• A. Deficiency of Glucose-6-phosphate dehydrogenase



• B. Strong Glucose-6-phosphate dehydrogenase

• C. Glucose-3-phosphate dehydrogenase

• D. None of the above

Q. 140 Select the hormone that are secreted from posterior pituitary gland.

• A. Oxytocin and vasopressin



• B. Only Oxytocin

• C. Only Vasopressin

• D. None of the above

Q.141 .... drugs are referred to as laxatives.

• A. Cathartic drugs

• B. Laxative drugs

• C. Purgative drugs

• D. All the above

Q. 142 Choose the correct one.

• A. Parts per million (ppm)



• B. Parts per billion (ppb)

• C. Parts per trillion (ppt)

• D. All the above

Q. 143 Podocodeine is used to cure.

• A. Dry cough

• B. Cold

• C. Vomiting

Copyright @Arifpharma.com Arif Pharma Clinic


Page 480


• D. None of the above

Q. 144 Select the recommended dosage of vitamin C for female.

• A. 30mg/day

• B. 50mg/day

• C. 75mg/day

• D. 300mg/day

Q. 145 Heparin is generally monitored using.

• A. APTT

• B. APTF

• C. PT

• D. TF

Q. 146 NSAID expand.

• A. Non-steroidal anti-inflammatory drugs



• B. Non-soluble anti-inflammatory drugs

• C. Non-steroidal anti-injection drugs

• D. All the above

Q. 147 The biological half-life of water in a human is about.

• A. 1 to 14 days

• B. 7 to 14 days

• C. 1 to 7 days

• D. 7 to 21 days

Q. 148 Nitroglycren is used in the form of................... for malignant hypertension.

• A. Infusion

• B. IV

• C. Injection

Copyright @Arifpharma.com Arif Pharma Clinic


Page 481

• D. Tablet

Q. 149 Antipseudomonal drug is used in the combination therapy consisting of.

• A. Beta-lactam antibiotic with an aminoglycoside



• B. Alpha-lactam antibiotic with an aminoglycoside

• C. beta-lactam antibiotic

• D. None of the above

Q. 150 Choose that factors that affect the drug distribution.

• A. Tissue solubility

• B. Protein binding

• C. Molecular weight

• D. All the above

Q. 151 If a solution Contain 8ppm of NaCl then calculate the concentration of the compound
in the solution.

• A. 0.8

• B. 0.08

• C. 0.008

• D. 0.0008

Q. 152 --- is used to treat trigeminal neuralgia.

• A. Baclofen can be used alone



• B. Baclofen can be used in combination with carbamazepine

• C. A and B

• D. No it cannot be used

Q. 153 ----- is a measurement of the concentration of a solution.

• A. Parts Per Million (ppm)



• B. Piece Per Million

• C. Weights Per Million

Copyright @Arifpharma.com Arif Pharma Clinic


Page 482

• D. None of the above

Q. 154 1 ppm is one part by weight, or volume, of solute in 1 million parts by weight, or
volume, of solution. True or False.

• A. True

• B. False

• C.

• D.

Q. 155 ---Process require CYP450:

• A. Oxidation

• B. Hydrolysis

• C. Glucoronidation

• D. Deamination

Q. 156 Which of the following reduces the rate of gastric emptying.

• A. Metoclopramide

• B. Atropin

• C. Hypothyrodism

• D. Atropin and Hypothyrodism

Q. 157 Identify the one that is not an alpha 2 agonist.

• A. Guanethidine

• B. Guanfacine

• C. Clonidine alpha

• D. None of the above

Q. 158 Quinidine can also produce------- , a symptom complex that includes headache and
tinnitus.

• A. Cinchonism

• B. Guanethidine

• C. Alpha

Copyright @Arifpharma.com Arif Pharma Clinic


Page 483


• D. All the above

Q. 159 With respect to digitoxin, which is wrong:

• A. Half-life of this is seven days



• B. Hepatic metabolism by microsomal

• C. 100% Oral bioavailability

• D. None of the above

Q. 160 Halothane is an.

• A. Inhaled anesthetic

• B. Volatile substance given by inhalation

• C. Non Volatile substance

• D. A and B

Q. 161 Pharmacodynamic parameters.

• A. Describe the relationship between drug concentration and the pharmacologic


effect

• B. Describe about drug concentration

• C. Describe about pharmacologic effect

• D. None of the above

Q. 162 ---describe the action of the drug on the body.

• A. Pharmacodynamic parameters

• B. Physical appearance

• C. Pharmacokinetic parameters

• D. All the above

Q. 163 Procainamide is a class ---- antiarrhythmic used for VT that is refractory to


defibrillation and epinephrine.

• A. Class Ia

• B. Class 1b

Copyright @Arifpharma.com Arif Pharma Clinic


Page 484

• C. None of the above



• D. All the above

Q. 164 Phenytoin is a class ----- antiarrhythmic agent that has been successfully utilised for
over half a century for treatment of Ventricular tachycardia.

• A. Class Ib

• B. Class 1a

• C. None of the above

• D. All the above

Q. 165 ---- is a medication that acts as a class I antiarrhythmic agent (Ia) in the heart.

• A. Quinidine

• B. Lidocaine

• C. Encainide

• D. Carvedilol

Q. 166 Class 1a is a Fast Channel blockers

• A. True

• B. False

• C.

• D.

Q. 167 Class II is a beta blockers

• A. True

• B. False

• C.

• D.

Q. 168 Class IV is said to be a calcium Channel Blockers

• A. True

• B. False

Copyright @Arifpharma.com Arif Pharma Clinic


Page 485

• C.

• D.

Q. 169 Class III agents predominantly block the

• A. Potassium channel

• B. Sodium Channel

• C. Beta channel

• D. None of the above

Q. 170 Hypothyroidism may be due

• A. Iodine deficiency

• B. Low hypothulmus

• C. Pituitary hormons

• D. All the above

Q. 171 Arrhythmia due to

• A. Dysfunction in heart pulse or conduction disorder



• B. Head ache

• C. Body Pain

• D. None of the above

Q. 172 Hydralazine is a medication used to treat

• A. High blood pressure



• B. Low Blood Pressure

• C. None of the above

• D. All the above

Q. 173 Fludrocortisone is a medication used to treat

• A. High blood pressure



• B. Low Blood Pressure

• C. None of the above

Copyright @Arifpharma.com Arif Pharma Clinic


Page 486


• D. All the above

Q. 174 Hydralazine dilates

• A. Arterioles more than Veins



• B. Veins more than Arterioles

• C. Vessel relaxation

• D. A and C

Q. 175 Side Effects of Guan the dine does not include

• A. Delayed or retrograde ejaculation



• B. Orthostatic hypotension

• C. Diarrhea

• D. Hypersensitivity reactions

Q. 176 WBC is also Call as leukocyte: Say True or False.

• A. True

• B. False

• C.

• D.

Q. 177 Red Blood Cells (RBC) are also called Erythrocytes: Say True or False.

• A. True

• B. False

• C.

• D.

Q. 178 Lymphocyte is a form of small leukocyte: Say True or False.

• A. True

• B. False

• C.

Copyright @Arifpharma.com Arif Pharma Clinic


Page 487

• D.

Q. 179 Identify the odd one out.

• A. Reiculocyte

• B. Leukocyte

• C. lymphocyte

• D. None

Q. 180 Reticulocytes develop and mature in the bone marrow and then circulate for about --
----- in the blood stream before developing into mature red blood cells. .

• A. A day

• B. An hour

• C. A Week

• D. All the above

Q. 181 Pharmacist responsibility is to.

• A. Take care of patients



• B. Administration

• C. Taking care of the effect of the drug

• D. Drug interaction

Q. 182 Minoxidil refers to.

• A. Dilate arteries only



• B. Decreasing myocardium contractility

• C. Resulting in bradycardia

• D. All the above

Q. 183 Minoxidil is an arterial vasodilator: Say True or False.

• A. True

• B. FALSE

• C.

Copyright @Arifpharma.com Arif Pharma Clinic


Page 488

• D.

Q. 184 Bradycardia can be caused by.

• A. Heart tissue damage related to aging.



• B. Harm in heart tissues from heart disease or heart attack

• C. A and B

• D. None of the above

Q. 185 Mizolastine is an.

• A. non-sedating antihistamine

• B. Antibiotic

• C. Anti platelet

• D. None of the above

Q. 186 Mizolastine is an anti-allergy medicine: Say True or False.

• A. FALSE

• B. TRUE

• C.

• D

Q. 187 Cisplatin is an anti-metabolite drug: Say True or False.

• A. TRUE

• B. FALSE

• C.

• D.

Q. 188 Cisplatin, Cisplatinum or cis-diamminedichloroplatinum (II) (CDDP) is a .

• A. Platinum-based chemotherapy drug



• B. Cures Allergies

• C. Cures Pain

• D. None of the above

Copyright @Arifpharma.com Arif Pharma Clinic


Page 489

Q. 189 Identify the odd one.

• A. carboplatin

• B. oxaliplatin.

• C. Cisplatin

• D. None of the above

Q. 190 Digioxin is taken only orally.

• A. TRUE

• B. False

• C.

• D.

Q. 191 Heroin is derived from the morphine alkaloid found in opium.

• A. TRUE

• B. FALSE

• C.

• D.

Q. 192 Opiates cannot be used in the form of an Anti-inflammatory.

• A. TRUE

• B. FALSE

• C.

• D.

Q. 193 Anti-inflammatory (or antiinflammatory) is the property of a substance that treats.

• A. Inflammation

• B. Swelling

• C. Both A and B

• D. None of the above

Q. 194 NSAIDs refers to.

Copyright @Arifpharma.com Arif Pharma Clinic


Page 490

• A. Nonsteroidal anti-inflammatory drugs



• B. Notsteroidal anti-inflammatory drugs

• C. Nonsubstance anti-inflammatory drugs

• D. Nonsteroidal anti-inflammation drugs

Q. 195 Sympathomimetic mimic the effects as.

• A. Heart rate increase



• B. Bronchiole dilation

• C. Nasal decongestants.

• D. All the above

Q. 196 Identify the false action of beta-adrenergic agents.

• A. Asthma

• B. Short-acting

• C. Used in inhalers

• D. have significant systemic effects.

Q. 197 Identify the beta-adrenergic agents that is not short acting?

• A. Pirbuterol

• B. Albuterol

• C. Terbutaline

• D. Salmetrol

Q. 198 Symptoms of bronchospasm include.

• A. Shortness of breath

• B. Wheezing

• C. Coughing and Chest tightness

• D. All the above

Q. 199 Opiate is a term classically used in pharmacology to mean a Drug derived from
opium.

Copyright @Arifpharma.com Arif Pharma Clinic


Page 491

• A. TRUE

• B. FALSE

• C.

• D

Q. 200. Opiates can be used in the form of all of the following.

• A. Analgesics

• B. Antidiarrheal

• C. Antitussive

• D. All the above

Copyright @Arifpharma.com Arif Pharma Clinic


Page 492

DHA Interview Questions in Dubai, UAE


Speaking Test Interview
– What is your full name?
– Can I see your ID?
– Where are you from?
– Do you work or study?
– What subject are you studying?
– Where do you live now?
– What kind of transportation do you use?
– Do you like walking? Why?
– Are there specific places for walking in your area?
– On what occasions do you give presents?
– Do you think it’s difficult to choose a gift for somebody?
– Do we give rich people expensive gifts? Why?

Cue Card

Talk about a situation that made you feel angry. Please say:
– Describe the situation.
– When and where did it occur?
– Why did you feel angry about it?

Discussion

– Do you still feel angry about it?


– How often does it happen?
– Do you think people should express their feelings?
– Who can express feelings more freely, men or women? Why?

Best Of Luck
Arif Pharma Clinic

You might also like